Anda di halaman 1dari 420

AMA-Notes

Contents
0. INTRODCUCITON TO AMA....................................................................................................................... 6
1. LEARNING CURVE....................................................................................................................................... 7
1.1. Logarithm and Anti-Logarithm ................................................................................................................ 7
1.2. Introduction to Learning Curve ............................................................................................................... 7
1.3. Learning Curve formula ............................................................................................................................ 8
2. LINEAR PROGRAMMING......................................................................................................................... 25
2.1. Learning Objectives .................................................................................................................................. 25
2.2. Introduction ............................................................................................................................................... 25
2.3. Maximization Linear Programming Problem (LPP)............................................................................ 26
2.3.1. Graphical Approach ............................................................................................................................. 26
2.3.2. Simplex Method .................................................................................................................................... 27
2.3.3. Steps in solving a maximization problem using Simplex Method ................................................ 31
2.4. Minimization Linear Programming Problem (LPP) ............................................................................ 32
2.4.1. Graphical Approach ............................................................................................................................. 33
2.4.2. Simplex Method .................................................................................................................................... 34
2.5. Infeasible Solution .................................................................................................................................... 36
2.6. Unbounded Solution ................................................................................................................................ 37
2.7. Multiple Optimal Solution ...................................................................................................................... 39
2.8. Degeneracy ................................................................................................................................................ 40
2.9. Interpretation of final simplex table....................................................................................................... 42
2.10. Primal and Dual .................................................................................................................................... 42
2.10.1. Conversion of Primal to Dual ............................................................................................................. 42
2.10.2. Interpretation of dual problem and Comparison of simplex tables .............................................. 46
2.11. Formulation types................................................................................................................................. 50
3. ASSIGNMENT PROBLEMS ........................................................................................................................ 56
3.1. Introduction ............................................................................................................................................... 56
3.2. Minimization balanced assignment problem – Hungarian Method ................................................. 56
3.3. Maximization balanced assignment problem – Hungarian Method ................................................ 58
3.4. Minimization unbalanced assignment problem – Hungarian Method & Degeneracy .................. 59
3.5. Maximization unbalanced assignment problem – Hungarian Method ............................................ 63
3.6. Multiple Optimal Solution ...................................................................................................................... 69
4. TRANSPORTATION ................................................................................................................................... 76

E M Reddy Page | 1
AMA-Notes

4.1. Introduction ............................................................................................................................................... 76


4.2. Minimization Balanced ............................................................................................................................ 76
4.2.1. Northwest Corner Method .................................................................................................................. 76
4.2.2. Least Cost Method ................................................................................................................................ 77
4.2.3. Vogel’s approximation method .......................................................................................................... 77
4.3. Minimization Balanced - Degeneracy .................................................................................................... 79
4.4. Maximization Unbalanced and Improvement through Looping ...................................................... 81
4.5. Multiple Optimal Solution ...................................................................................................................... 87
4.6. Formulation ............................................................................................................................................... 92
5. STANDARD COSTING or VARIANCE ANALYSIS: ........................................................................... 101
5.1. Learning Objectives ................................................................................................................................ 101
5.2. Introduction ............................................................................................................................................. 101
5.3. Understanding Standard Cost .............................................................................................................. 102
5.4. Cost Variances ......................................................................................................................................... 103
5.4.1. Material Cost Variances – Single Raw Material Input .................................................................. 103
5.4.2. Material Cost Variances – Mix of Raw Materials ........................................................................... 105
5.4.3. Labour Variances – without mix ...................................................................................................... 109
5.4.4. Labour Variances – with mix ............................................................................................................ 109
5.4.5. Labour Variance – Idle time .............................................................................................................. 111
5.4.6. Variable Overhead Variances ........................................................................................................... 112
5.4.7. Fixed Overhead Variances – With Calendar .................................................................................. 114
5.4.8. Fixed Overhead Variances – Without Calendar............................................................................. 118
5.4.9. Cost Variances Reconciliation ........................................................................................................... 120
5.5. Marginal Costing vs. Absorption Costing .......................................................................................... 124
5.6. Sales Variances ........................................................................................................................................ 127
5.7. Types of Reconciliation Problems ........................................................................................................ 132
5.7.1. Standard Reconciliation Statement .................................................................................................. 132
5.8. Concept of Standard Profit .................................................................................................................... 138
5.9. Reconciliation with Work in Progress ................................................................................................. 140
5.10. Opportunity Cost Method of Reconciliation .................................................................................. 146
5.11. Planning Variance vs. Operating Variance ..................................................................................... 154
5.12. Planning vs. Operating Variance – Market Size and Market Share Variance ............................ 163
5.13. Balance Score Card Method of Reconciliation ................................................................................ 166
5.14. Miscellaneous Concepts – Standard Costing Ratios ...................................................................... 171

E M Reddy Page | 2
AMA-Notes

5.15. Partial Plan vs. Single Plan ................................................................................................................ 175


5.16. Reverse Working Problems ............................................................................................................... 191
5.17. Investigation of Variances ................................................................................................................. 196
6. RELEVANT COSTING .............................................................................................................................. 199
6.1. Learning Objectives ................................................................................................................................ 199
6.2. Relevant Cost of Materials .................................................................................................................... 199
6.3. Relevant Cost of Labour ........................................................................................................................ 201
6.4. Relevant Cost of Overheads .................................................................................................................. 202
6.5. Comprehensive Problems ..................................................................................................................... 203
6.6. Limiting Factor and Relevant Costing ................................................................................................. 208
6.7. Joint product and Relevant Costing ..................................................................................................... 211
6.8. Relevant Costing under Uncertainty ................................................................................................... 214
7. MARGINAL COSTING ............................................................................................................................. 220
7.1. Learning Objectives ................................................................................................................................ 220
7.2. Basics in Marginal Costing .................................................................................................................... 220
7.2.1. Income Statement ............................................................................................................................... 221
7.2.2. PV Ratio ............................................................................................................................................... 221
7.2.3. Breakeven Point .................................................................................................................................. 221
7.2.4. Margin of Safety .................................................................................................................................. 221
7.2.5. Profit ..................................................................................................................................................... 222
7.3. Issues in the concept of Break-even point ........................................................................................... 222
7.3.1. Multiple break-even points (Step fixed cost) .................................................................................. 222
7.3.2. Break-even point with semi-variable cost ....................................................................................... 227
7.3.3. Marginal Costing (vs.) Absorption Costing Break-even point..................................................... 229
7.3.4. Break-even point with two products ............................................................................................... 234
7.4. Indifference Point ................................................................................................................................... 235
7.4.1. Introduction to Indifference point .................................................................................................... 235
7.4.2. Indifference point as a state of demand .......................................................................................... 237
7.4.3. Limiting Factor and Indifference Point ........................................................................................... 240
7.4.4. Understanding how to analyze a semi-variable cost ..................................................................... 244
7.5. Limiting Factor Problems ...................................................................................................................... 249
7.5.1. Basic Limiting factor allocation problems ....................................................................................... 249
7.5.2. Limiting factor in Make or Buy Situation........................................................................................ 251
7.5.3. Multiple Limiting factors in Make or Buy Situation ...................................................................... 258

E M Reddy Page | 3
AMA-Notes

7.5.4. Limiting factor with Specific Fixed Cost ......................................................................................... 262


7.6. Shut down (or) Continue decisions...................................................................................................... 268
7.7. Differential Costing ................................................................................................................................ 270
8. SIMULATION ............................................................................................................................................. 279
9. MATERIALS REQUIREMENT PLANNING (MRP) ............................................................................. 289
9.1. Introduction ............................................................................................................................................. 289
9.2. Planning Order Release ......................................................................................................................... 289
9.3. Construction of Product Tree ................................................................................................................ 291
9.4. Preparation of MRP ................................................................................................................................ 292
9.5. Preparation of MRP with safety stock ................................................................................................. 292
9.6. Material Purchase Budget and Economic Order Quantity [EOQ]................................................... 293
10. NETWORK ANALYSIS ......................................................................................................................... 296
10.1. Learning Objectives ............................................................................................................................ 296
10.2. Introduction ......................................................................................................................................... 296
10.3. Understanding some basic terms used in Network ...................................................................... 296
10.3.1. Activities & Events ............................................................................................................................. 296
10.3.2. Errors in Networking ......................................................................................................................... 297
10.3.2.1. Looping Error .................................................................................................................................. 297
10.3.2.2. Dangling Error ................................................................................................................................ 298
10.3.2.3. Mistake in Preceding, Succeeding relationship .......................................................................... 298
10.3.3. Conventions in Network Drawing................................................................................................... 299
10.4. Floats, Forward Pass, Backward Pass and Critical Path ............................................................... 299
10.5. Network Crashing .............................................................................................................................. 303
10.6. Program Evaluation Review Technique .......................................................................................... 310
10.7. Resource Allocation ............................................................................................................................ 314
10.8. Resource Leveling (or) Resource Smoothing .................................................................................. 316
10.9. Understanding to draw a network & use dummy activities ........................................................ 320
10.10. Network Updation ............................................................................................................................. 330
11. TRANSFER PRICING ............................................................................................................................ 332
11.1. Introduction ......................................................................................................................................... 332
11.2. Transfer Price - Variable Cost ........................................................................................................... 332
11.3. Transfer Price – Specific Fixed Cost ................................................................................................. 339
11.4. Linear Programming Method of Transfer Price fixation .............................................................. 342
11.5. Limiting Factor and Transfer Pricing............................................................................................... 347

E M Reddy Page | 4
AMA-Notes

11.6. Multi-Division range fixations .......................................................................................................... 352


11.7. Lump Sum Consideration Method .................................................................................................. 354
11.8. Transfer Pricing Fixation Chart ........................................................................................................ 358
12. PRICING .................................................................................................................................................. 360
12.1. Pricing Using Calculus....................................................................................................................... 360
12.2. Pricing Under Uncertainty and Expected Value of Perfect Information (EVPI) ....................... 364
12.3. ROI (Return on Investment) Pricing ................................................................................................ 367
13. DEVELOPMENTS IN COST ACCOUNTING (STRAGETIC COST MANAGEMENT ................ 368
13.1. Learning Objectives ............................................................................................................................ 368
13.2. Through Put Costing .......................................................................................................................... 368
13.3. Theory of Constraints......................................................................................................................... 370
13.3.1. Theory of Constraints Measures (TOC Measures)......................................................................... 373
13.4. Just-in-Time System (JIT)................................................................................................................... 377
13.5. Backflush Costing ............................................................................................................................... 381
13.5.1. Backflush Costing – Version 1 .......................................................................................................... 381
13.5.2. Backflush Costing – Version 2 .......................................................................................................... 382
13.5.3. Backflush Costing – Version 3 .......................................................................................................... 383
13.6. Total Quality Management (TQM) .................................................................................................. 387
13.7. Life Cycle Costing (LCC) ................................................................................................................... 393
13.8. Target Costing ..................................................................................................................................... 395
13.9. Activity Based Costing (ABC)........................................................................................................... 400
13.9.1. Activity Based Costing (ABC) - Introduction ................................................................................. 400
13.9.2. Activity Based Costing (ABC) & Target Costing ........................................................................... 412
13.9.3. Activity Based Costing (ABC) with Variance Analysis................................................................. 417

E M Reddy Page | 5
AMA-Notes

0. INTRODCUCITON TO AMA

1) Source for Preparation:


i. Revision Test Papers
ii. Practice Manual
iii. Steady Material
iv. Advance Management Accounting -CA-Final – By Saxena and Vasisht
v. Callen Drury
vi. Hongren
2) Examination Pattern
i. 7 Questions
a) Q1 – 20 Marks (4QX5M) – Compulsory question
b) Q2 – Q7 – 96 Marks (6QX16M) – Should write 6 questions
 Q7 – Theory question in most of the cases (5QX4M – 1Q Choice) – one
theory from QT
ii. 27 Marks – 33 Marks theory will be asked. Everyone has to compulsorily write 16 Marks of
theory
3) Total Subject can be divided into two types broadly:
i. Costing – 79 Marks
ii. QT - Quantitative Techniques (Operations Research) – 37 Marks – 4 Marks theory
(compulsory)

E M Reddy Page | 6
AMA-Notes

1. LEARNING CURVE

1.1. Logarithm and Anti-Logarithm

1) Generally, we used to express numbers in decimal systems


2) In decimal system we generally do addition, subtraction, multiplication and division. In which
multiplication and division are difficult to calculate in case of big numbers.
3) The another way to do multiplications (exponential -26 ) and divisions is using “logarithms”.
4) For every decimal number there exists a corresponding number in the log system and vice versa.
This can be found using the log tables and anti-log tables respectively.
5) Log Computation – Converting a number in decimal system into Log system:
Number No. of digits No. of digits – 1 Log Table Value Log Value
4000 4 4–1=3 0.6021 3.6021
400 3 3–1=2 0.6021 2.6021
40 2 2–1=1 0.6021 1.6021
4 1 1 – 1 =0 0.6021 0.6021
4568 4 4 – 1= 3 0.6598 3.6598
6) Anti-log Computation – Converting a number in Log system into a decimal number:
Log Decimal Table Integer Integer+1 𝟏𝟎(𝐈𝐧𝐭𝐞𝐠𝐞𝐫+𝟏) Value
Value Portion Value
3.6021 0.6021 0.4000 3 3+1 = 4 10000 4000
2.6021 0.6021 0.4000 2 2+1 = 3 1000 400
1.6021 0.6021 0.4000 1 1+1 = 2 100 40
0.6021 0.6021 0.4000 0 0+1 = 1 10 4
3.6598 0.6598 0.4568 3 3+1 = 4 10000 4568
7) Decimal function vs. Log function:
Decimal Function Log Function
AXB Log A + Log B
A/B Log A − Log B
B B Log A
A
A+B Log(A + B)
A–B Log(A − B)

1.2. Introduction to Learning Curve

1) When a person performs the task repeatedly, the time taken to do it gradually reduces.
2) The above reduction in time happens because of learning effect. The learning effect occurs because:
a) He becomes more familiar with the Job
b) He develops better tooling methods to perform it
c) He identifies and eliminates unwanted activities in performing the Job
3) The learning curve looks as follows:

E M Reddy Page | 7
AMA-Notes

As the number of units increases, the average time per unit decreases. The rate at which it decreases
is called “Learning Rate” and is always expresses as percentage (%).
4) The other names given to learning curve are “Experience Curve”, “Improvement Curve” and
“Progress Curve”.
5) A learning of 80% means, every time the production doubles the average time per unit becomes
80% of the previous average. Example can be understood as follows:
No. of Units Cumulative Average time per unit (Hours)
1 100
2 80
4 64
8 51.20
6) The learning curve can be applied by the management accountant in the following three areas:
a) Price fixation
b) Fixing Labour Standards for Variance analysis
c) Volume determination for a given capacity
7) Learning curve will not have impact in following situations:
a) Where the production is automated involving lesser human element
b) Where the Jobs are non-repetitive or User Specific or Customer Specific
c) Where the job is performed by highly experienced persons who has reached saturation point
or a state of steady point.

1.3. Learning Curve formula

Question no 1: Direct labour hours to assemble the first unit of new equipment were 400. Assuming
that this type of assemble will experience a learning effect of 90%. Compute the average direct
labour for the 3rd & the 4th units as also for the 5th to 8th units.
Find also the average labour for the 6th & 7th units. And also calculate the time taken from the 1st
unit to 8th Unit. B= -0.1520

Solution:
Units Cumulative Total time in Incremental Increme Average time
(X) Average time per hours Hours ntal for incremental
unit (Y) Units unit

E M Reddy Page | 8
AMA-Notes

1 400 400 x 1 = 400 400 1 400/1 = 400


2 400 x 90% = 360 360 x 2 = 720 720 – 400 = 320 1 320/1 = 320
4 360 x 90% = 324 324 x 4 = 1296 1296 – 720 = 576 2 576/2 = 288
8 324 x 90% = 291.6 x 8 = 2332.8 2332.8 – 1296 = 4 1036.8/4 =
291.6 1036.8 259.2

Conclusion:
1) Average direct labour for 3rd and 4th units = 288 Hours
2) Average direct labour for 5th to 8th units = 259.2 Hours

A re-look into average time of 3rd and 4th units:

TT(4)−TT(2) Time taken for 4 units−Time taken for 2 units 1296−720 576
= = = 2 Units = 288
2 Units 2 Units 2 Units

[Y4 x 4]−[Y2 x 2] Cumulative average after producing 4 units X 4 −Cumulative average after producing 2 units X 2
=
2 Units 2 Units
[324 x 4]−[360 x 2] 1296−720 576
= = = 2 Units = 288
2 Units 2 Units

A re-look into average time of 5th to 8th units:

TT(8)−TT(4) Time taken for 8 units−Time taken for 4 units 2332.8−1296 1036.8
= = = 4 Units = 259.2
4 Units 4 Units 4 Units

[Y8 x 8]−[Y4 x 4] Cumulative average after producing 8 units X 8 −Cumulative average after producing 4 units X 4
=
4 Units 4 Units
[291.6 x 8]−[324 x 4] 2332.8−1296 1036.8
= = = 4 Units = 259.2
4 Units 4 Units

Average time of 6th and 7th units:

TT(7)−TT(5) [Y7 x 7]−[Y5 x 5]


=
2 Units 2 Units

Is Y5 and Y7 is available in the table?


If average time is asked in between doublings (twice), the problem cannot be solved in the form of table as
attempted above. One has to use the following formula:

YX = a(X)b ; where
YX → Cumulative Average time per unit for ‘X’ units
a → time required for the first unit
X →target units
Log of learning rate Log P
b→ = Log 2
Log 2

Note: Learning rates should always be expresses in decimals. For example, if learning rate is 90%, it should
be return as 0.90.

Calculation ofY7 :

E M Reddy Page | 9
AMA-Notes

Y7 =a(X)b ; where
Y7 → Cumulative Average time per unit for ‘7’ units
a → 400
Log of learning rate Log 0.90
b→ =
Log 2 Log 2
X → 7 units

Log Value Computations:

Function Values No. of Digits Base (No. of Digits – 1) Log Table Value Log Value
A 400 3 2 0.6021 2.6021
X 7 1 0 0.8451 0.8451
P 0.90 0 -1 0.9542 -0.0458
Constant Base 2 1 0 0.3010 0.3010
Log P −0.0458
b = Log 2 = = - 0.1522
0.3010

Y7 = 400 x (7)−0.1522 → Apply log on both Sides


Log Y7 = Log 400 + Log 7−0.1522
Log Y7 = Log 400 – 0.1522 Log 7
= 2.6021 – (0.1522 X 0.8451)
= 2.6021 – 0.1286
= 2.4735
Y7 = Antilog of 2.4735
= 297.50 Hours

Calculation ofY5 :

Y5 = 400 x (5)−0.1522 → Apply log on both Sides


Log Y5 = Log 400 + Log 5−0.1522
Log Y5 = Log 400 – 0.1522 Log 5
= 2.6021 – (0.1522 X 0.6990)
= 2.6021 – 0.1064
= 2.4957
Y5 = Antilog of 2.4957
= 313.10 Hours

Average time of 6th and 7th units:

TT(7)−TT(5) [Y7 x 7]−[Y5 x 5]


=
2 Units 2 Units

[297.50 X 7]−[313.50 X 5]
= 2 units

2082.5−1565.5
= 2 Units

E M Reddy Page | 10
AMA-Notes

= 258.8

Alternatively, the solution can be presented as follows:


Unit Cumulative Total time in Incremental Hours Increme Average time
s Average time hours ntal for incremental
(X) per unit (Y) Units unit
5 313.10 313.1 x 5 = 1565.5 - - -
7 297.50 297.5 x 7 = 2082.5 2082.5 – 1565.5 = 517 2 517/2 = 258.58

Calculation of time taken for each unit individually:

Calculation of 𝑌3

Y3 = 400 x (3)−0.1522 → Apply log on both Sides


Log Y3 = Log 400 + Log 3−0.1522
Log Y3 = Log 400 – 0.1522 Log 3
= 2.6021 – (0.1522 X 0.4771)
= 2.6021 – 0.0726
= 2.5295
Y3 = Antilog of 2.5295
= 0.3385 X 10(2+1)
= 338.50 Hours

TT(3)−TT(2)
Time taken for 3rd unit = 1 Unit

[Y3 x 2]−[Y2 x 2]
= 1 Unit

[338.50 X 3]−[360 X 2]
= 1 Unit

1015.5−720
= 1 Unit

= 295.5 Hours

Calculation of 𝑌6

Y6 = 400 x (6)−0.1522 → Apply log on both Sides


Log Y6 = Log 400 + Log 6−0.1522
Log Y6 = Log 400 – 0.1522 Log 6
= 2.6021 – (0.1522 X 0.7781)
= 2.6021 – 0.1184
= 2.4837
Y6 = Antilog of 2.4837
= 0.3046 X 10(2+1)
= 304.60 Hours

E M Reddy Page | 11
AMA-Notes

Calculation of time taken for each unit:


Units Cumulative average time per unit Total Time (Hours) Time taken for the Unit
(Hours)
1 400 400 x 1 = 400 400
2 360 360 x 2 = 720 720 – 400 = 320
3 338.50 338.50 x 2 = 1015.5 1015.5 – 720 = 295.5
4 324 324 x 2 = 1296 1296 – 1015.5 = 280.5
5 313.10 313.10 x 2 = 1565.5 1565.5 – 1015.5 = 269.5
6 304.60 304.60 x 2 = 1827.6 1827.6 – 1565.5 = 262.1
7 297.50 297.50 x 2 = 2082.5 2082.5 – 1827.6 = 254.9
8 291.60 291.60 x 2 = 2332.8 2332.8 – 2082.5 = 250.3

Question no 2: First batch of 25 transistor radios took a total of 250 direct labour hours. It is
proposed to assemble another 40 units. What will be the average labour per unit in this lot?
Assuming that there is 85% learning rate. B=-0.23455

Solution:

Facts:
1st Batch 25 Units 1 Batch
New Order 40 Units 1.6 Batches
Cumulative Production 65 Units 2.6 Batches

Log Computation:
Function Values No. of Digits Base (No. of Digits – 1) Log Table Value Log Value
a 250 3 2 0.3979 2.3979
X 2.6 1 0 0.4150 0.4150

Calculation ofY2.6:

YX = a(X)b
Y2.6 = 250 x (2.6)−0.23455 → Apply log on both Sides
Log Y2.6 = Log 250 + Log 2.6−0.23455
Log Y2.6 = Log 250 – 0.23455 Log 2.6
= 2.3979 – (0.23455 X 0.4150)
= 2.3979 – 0.9733
= 2.3006
Y5 = Antilog of 2.3006
= 0.1998 X 10(2+1)
= 0.1998 X 1000
= 199.80 Hours

Average time of 1.6 batches or 40 units:


TT(2.6)−TT(1) [Y2.6 x 2.6]−[Y1 x 1]
=
1.6 Batches or 40 Units 40 Units

E M Reddy Page | 12
AMA-Notes

[199.8 X 2.6]−[250 X 1]
= 40 units

519.48−250
= 40 Units

= 6.74 Hours per unit

Alternatively, can be presented as follows:


Batc Cumulative Total time in Incremental Hours Increme Average time
hes Average time hours ntal for incremental
(X) per batch (Y) Units unit
1 250 250 X 1 = 250 - - -
2.6 199.80 199.8 x 2.6 = 519.48 – 250 = 269.48 40 269.48/40 =
519.48 6.73

Notes:

1) While solving a problem, we should observe how “a-time per first unit” is given. If it is given as the
time taken for the first unit, do the calculations in units but if it is given as the time taken for 1st
batch, then convert the problem into batches and solve.

Additional Calculation for learning:

Calculation of “a-time taken for the first unit”:

250 Hours
YX = Y25 = = 10 Hours
25 units

Y25 = a X 250.23455
10 = a X 250.23455 → Apply log on both Sides
Log 10 = Log a -0.23455 x Log 25
1 = Log a – (0.23455 x 1.3979)
1 = Log a – 0.3279
Log a = 1+0.3279
Log a = 1.3279 → Apply anti log on both Sides
a = Anti-log of 1.3279
= 0.2127 X 10(1+1)
= 0.2127 X 100
= 21.27 Hours

Calculation ofY65 :

YX = a(X)b
Y65 = 21.27 x (65)−0.23455 → Apply log on both Sides
Log Y65 = Log 21.27 + Log 65−0.23455
Log Y65 = Log 21.27 – 0.23455 Log 65
= 1.3278 – (0.23455 X 1.8129)

E M Reddy Page | 13
AMA-Notes

= 1.3278 – 0.4252
= 0.9026
Y65 = Antilog of 0.9026
= 0.79909 X 10(0+1)
= 0.79909 X 10
= 7.9909 Hours or 8 Hours

TT(65)−TT(25)
Average Time taken for 40 units = 40 Units

[Y65 x 65]−[Y25 x 25]


= 40 Units

[8 x 65]−[10 x 25]
= 40 Units

520−250
= 40

270
= 40

= 6.75 Hours

Question no 3: A company has found that the average direct labour just after completion of X units
was 26.4 hours. The average at the end of the first unit was 52 hours. If there is learning curve effect
of 85%. What would have been the total output to date?

Solution:
Units Cumulative average time per unit
1 52 Hours
X 26.4 Hours

Learning Curve rate 85%.

YX = a(X)b
26.4 = 52 x (X)−0.23455 → Apply log on both Sides
Log 26.4 = Log 52 + Log X −0.23455
Log 26.4 = Log 52 – 0.23455 Log X
1.4216 = 1.7160 – (0.23455 x Log X)
(0.23455 x Log X) = 1.7160 – 1.4216
1.7160 – 1.4216
Log X = 0.23455
Log X = 1.2549 → Apply anti log on both sides
X = Antilog of 1.2549
= 0.1799 X 10(1+1)
= 0.1799 X 1000
= 17.99 Units or 18 Units

E M Reddy Page | 14
AMA-Notes

Question no 4: Suraksha an electronic firm has designed a new model of fire lock system and
assembled the first unit as a prototype for demonstration. The direct labour expended on this unit
was 260 hours and the direct material cost was Rs. 37,000. The direct labour rate is Rs.30 per hour.
Following successful demonstration to potential customers, confirm orders has been received for
supply of 50 units during the first 6 months and the supply of 75 units during the following 6
months. The company wishes to set competitive prices for the supplies in each of the periods by
passing on the benefits of learning curve effect of 80% i.e. normally applicable to this type of
industry. Further the variable overhead in regular production runs is estimated to be 125% of the
direct labour cost and the fixed overheads are charged at 75% of the direct labour cost. In view of
the large production volumes it is expected that 5% discount can be got on the materials use for the
first 6 months and 10% discount for the 2nd 6 months. The company sets selling prices with a 40%
mark-up on cost. Determine the selling price per unit that should be set for the offer in each of the
6 months. B=-0.3220

Solution:

Step 1: Calculation of Y51

YX = a(X)b
Y51 = 260 x (51)−0.3220 → Apply log on both Sides
Log Y51 = Log 260 + Log 51−0.3220
Log Y51 = Log 260 – 0.3220 Log 51
= 2.4150 – (0.3220 X 1.7076)
= 2.4150 – 0.5498
= 1.8652
Y51 = Antilog of 1.8652
= 0.7333 X 10(1+1)
= 0.7333 X 1000
= 73.33 Hours

Step 2: Calculation of Y126

YX = a(X)b
Y126 = 260 x (126)−0.3220 → Apply log on both Sides
Log Y126 = Log 260 + Log 126−0.3220
Log Y126 = Log 260 – 0.3220 Log 126
= 2.4150 – (0.3220 X 2.1004)
= 2.4150 – 0.6763
= 1.7386
Y51 = Antilog of 1.7386
= 0.5481 X 10(1+1)
= 0.5481 X 1000
= 54.81 Hours

Step 3: Average time per unit for each order

E M Reddy Page | 15
AMA-Notes

Units Cumulative Total time in Incremental Hours Increme Average time


(X) Average time hours ntal for incremental
per unit (Y) Units unit
1 260 260 x 1 = 260 260 1 260/1 = 260
51 73.33 73.33 x 51 = 3739.83 – 260 = 50 3479.83/50 =
3739.83 3479.83 69.60
126 54.81 54.81 x 126 = 6906.06 – 3739.83 = 75 3166.23/75 =
6906.06 3166.23 42.22

Step 4: Fixation of selling price


Particulars 1st Order 2nd Order
Units 50 75
Material Cost per unit 37,000 x 95% = Rs. 35,150 37,000 x 90% = Rs. 33,300
Labour Cost per unit 69.6 X 30 = Rs. 2088 42.2 X 30 = Rs. 1266
Variable Overhead per unit 2088 X 125% = Rs. 2,610 1266 X 125% = Rs. 1,583
Fixed Overhead per unit 2088 X 75% = Rs. 1,566 1266 X 75% = Rs. 950
Total Cost per unit Rs. 41,414 Rs. 37,099
Margin per unit (40% of cost) 41,414 x 40% = Rs. 16,567 37,099 x 40% = Rs. 14,840
Selling price per unit Rs. 57,981 Rs. 51,939

**Question no 5: EGM manufactures electrical goods on behalf of various clients as per their
requirements. Currently having lost one major client, EGM is left with a large surplus of skilled
labour. This labour cannot be retrenched nor can additional be recruited. EGM located HHDG a
marketing firm in household goods, for whom it can offer manufacturing facilities to find gainful
work for the skilled labour that may otherwise idle. EGM has compiled the following information
so as to take decision whether to undertake manufacture on behalf of HHDG.
Capital outlay on the special machinery Rs.2 lakhs (machine having no salvage value). Incremental
overheads Rs.1 lakh per annum. Cost of Material Rs.180 per unit. Skilled labour rate Rs.30 per hour.
The contract if entered into must be for a period of 3 years and HHDG will offer a unit price of
Rs.260 valid for all the three years. A first unit trail run took 10 hours of direct labour of a skilled
workman. It is expected that on repetitive production there will be learning effect of 82%. HHDG
will accept all the production that EGM is capable of. It was also assured that the surplus skilled
labour available will be adequate to manufacture of 3,000 units in the first year. The cost of capital
for EGM is 18%. You may assume that all cash flows occur at the year-end. Except for the capital
outlay that has to be at the start of year 1.
What decision should EGM take with regard to acceptance of the contract for HHDG? B=-0.2864

Solution:

Basic Details:

Selling Price = Rs.260


Material Cost = Rs.180
Labour Cost = Nil (As the labour cost is payable with order or without order also).
Total Variable Cost = Rs.180
Contribution per unit = Rs.80
Fixed Cost = Rs.100000

E M Reddy Page | 16
AMA-Notes

Step 1: Calculation of the annual labour capacity in hours

a = 10 Hours
p = 82% or 0.82
X = 3000 Units

Log Values Computation:

Function Values No. of digits Base Table Value Log Value


a 10 2 1 0 1
X 3000 4 3 0.4771 3.4771

Finding out no.of hours available to produce 3000 units in the first year:

Y3000 = 10 x (3000)−0.2864 → Apply log on both Sides


Log Y3000 = Log 10 + Log 3000−0.2864
Log Y3000 = Log 10 – 0.2864 Log 3000
= 1 – (0.2864 X 3.4771)
= 1 – 0.9958
= 0.0042
Y3000 = Antilog of 0.0042
= 0.10099 X 10(0+1)
= 0.10099 X 10
= 1.0099 Hours per unit
Total time for 3000 units = 1.0099 X 3000 = 3030 hours (approx.)
In a given year 3030 hours are available for production. In 1st year with this capacity, we can produce 3000
units and in the subsequent years with the same capacity the production shall experience increase due to
learning effect.

Step 2: Total time for 3 years


Year Total Time (Hours)
1 3030
2 6060
3 9090

Step 3: Calculation of Year 2 production

Let cumulative units produced at the end of year 2 be ‘X’.


Units x Cumulative average time per unit = Total time
X * [a * X b ] = Total Time
−0.2864
X * [10 * X ] = 6060 Hours
1 −0.2864
X *X * 10 = 6060 Hours
1−0.2864
X * 10 = 6060 Hours
0.7136
X * 10 = 6060 Hours
6060 Hours
X 0.7136 = 10

E M Reddy Page | 17
AMA-Notes

X 0.7136 = 606 Hours → Apply log on both sides


Log X 0.7136 = Log 606
0.7136 Log X = 2.7825
2.7825
Log X = 0.7136
Log X = 3.8992 → Apply anti log on both sides
X = Antilog of 3.8992
= 0.7929 X 10(3+1)
= 0.7929 X 10000
= 7929 or 7300 (approx.)
Cumulative Units = 7930 Units
Year 2 Production = 7930 – 3000
= 4930

Step 4: Calculation of Year 3 production

Let cumulative units produced at the end of year 3 be ‘X’.


Units x Cumulative average time per unit = Total time
X * [a * X b ] = Total Time
−0.2864
X * [10 * X ] = 9090 Hours
1 −0.2864
X *X * 10 = 9090 Hours
1−0.2864
X * 10 = 9090 Hours
0.7136
X * 10 = 9090 Hours
9090 Hours
X 0.7136 = 10
X 0.7136 = 909 Hours → Apply log on both sides
Log X 0.7136 = Log 909
0.7136 Log X = 2.9590
2.9590
Log X = 0.7136
Log X = 4.1466 → Apply anti log on both sides
X = Antilog of 4.1466
= 0.1402 X 10(4+1)
= 0.1402 X 100000
= 14020 or 14000 (approx.)
Cumulative Units = 14000 Units
Year 3 Production = 14000 – 7930
= 6090

Step 5: Operating Cash inflows


Year No. of Units Contribution @ Incremental Profit = Contribution –
Rs.80 (Rs.) Overhead Overhead
1 3000 3000 x 80 = 2,40,000 1,00,000 1,40,000
2 4930 4930 x 80 = 3,94,400 1,00,000 2,94,400
3 6070 6070 x 80 = 4,85,600 1,00,000 3,85,600

E M Reddy Page | 18
AMA-Notes

Step 6: Calculation of Net Present Value


Year Cash Flow Present Value Factor @18% Discounted Cash Flow
1 1,40,000 0.8475 1,18,650
2 2,94,400 0.7182 2,11,438
3 3,85,600 0.6086 2,34,676
Present Value of Cash inflows 5,64,764
Less: Fixed Asset cost 2,00,000
Net Present Value 3,64,764

The HHDG order can be accepted since the Net Present Value is positive.

Question no 6: The following information is provided by a firm. The factory manager wants to use
appropriate average learning rate on activities, so that he may forecast costs and prices for certain
levels of activity.
(i) A set of very experienced people feed data into the computer for processing inventory records in
the factory. The manager wishes to apply 80% learning rate on data entry and calculation of
inventory.
(ii) A new type of machinery is to be installed in the factory. This is a patented process and the
output may take a year for full-fledged production. The factor manager wants to use a learning rate
on the workers at the new machine.
(iii) An operation uses contract labour. The contractor shifts people among various jobs once in
two days. The labour force performs on task in 3 days. The manager wants to apply an average
learning rate for these workers.
You are required to advice to the manager with reasons on the applicability of the learning curve
theory on the above information.

Solution:

The learning curve does not apply to very experienced people for the same job, since time taken can never
tend to become zero or reduce very considerably after a certain range of output. This is the limitation of the
learning curve.
(i) Data entry is a manual job so learning rate theory may be applied. Calculation of inventory is a
computerized job. Learning rate applies only to manual labour.
(ii) Learning rate should not be applied to a new process which the firm has never tried before.
(iii) The workers are shifted even before completion of one unit of work. Hence learning rate will not apply.

Question no 7: Bandookwala & Co., A firearms manufacturer has designed a new type of gun and a
first lot of 25 guns assembled for test purposes had the following costs:
Direct Materials Rs. 24,500
Direct Labour Rs. 22,500
Variable Overheads Rs. 16,875
Fixed Overheads Rs. 11,250
Total Costs Rs. 75,125
The Variable overheads and fixed overheads are charged to products in the proposition of direct
labour cost.
BSF being satisfied with this gun have asked the lowest bid for supply of 1,000 guns. The company

E M Reddy Page | 19
AMA-Notes

will pass on the benefit of learning of 85% to the client in setting the bid. The company will set a
selling price to earn 40% gross profit margin. Determine the unit price that should be bid.

Solution:
1 Lot 25 guns
40 lots 1000 guns
41 lots 1025 guns

Step 1: Calculation of “b”


Log of Learning ratio
b= Log 2
Log 0.85
= Log 2
−0.0706
= 0,3010
= -0.2346
Step 2: Calculation of Y41

YX = a(X)b
Y41 = 22,500 x (41)−0.2346 → Apply log on both Sides
Log Y41 = Log 22,500 + Log 41−0.2346
Log Y41 = Log 22,500 – 0.2346 Log 41
= 4.3522 – (0.2346 X 1.6128)
= 4.3522 – 0.3784
= 3.9738
Y41 = Antilog of 3.9738
= 0.9414 X 10(3+1)
= 0.9414 X 10000
= Rs. 9,414

Step 3: Calculation of Labour Cost per unit


Lots Cumulative Total cost in Incremental Rs. Increme Average cost
(X) Average time Rs. ntal Lots for incremental
per lot (Y) lot
1 22,500 22,500 x 1 = 22,500 1 22,500/1 =
260 22,500
41 9,414 9,414 x 41 = 3,95,474 – 22,500 = 40 3,63,474/40 =
3,85,74 3,63,474 9087

Step 4: Calculation of selling price per lot


Particulars Computation Amount (Rs.)
Material Cost Given 24,500
Labour Cost Given 9,087
Variable Overhead 16,875
x 9,087 6,815
22,500

E M Reddy Page | 20
AMA-Notes

Fixed Overhead 11,250


x 9,087 4,544
22,500
Total Cost per lot 44,946
44,946
Margin ( 60 x40) 29,964
Selling price per lot 74,910
Selling price per gun (74,910/25) 2,996

Notes:

1) “a” in the problem can be given in four ways:


a) Time for 1st Unit - YX means ‘cumulative average time per unit’
b) Time for 1st Batch/Lot - YX means ‘cumulative average time per batch/lot’
c) Cost of 1st Unit - YX means ‘cumulative average cost per unit’
d) Cost of 1st Batch/Lot - YX means ‘cumulative average cost per batch/lot’

Question no 8: An electronics firm which has developed a new type of fire alarm system has been
asked to quote for a prospective contract. The customer requires separate price quotations for each
of the following possible orders.
1st Order 100 fire alarm Systems
nd
2 Order 60 fire alarm Systems
rd
3 Order 40 fire alarm Systems
The firm estimates the following cost per unit for the first order.
Direct Material Rs.500
Direct Labour
Department A (Highly automatic) 20 hours @ Rs.10/Hour
Department B (Skilled labour) 40 hours @ Rs.15/Hour
Variable overheads 20% of direct labour
Fixed Overheads absorbed
Department A Rs.8 per Hour
Department B Rs.5 per Hour
Determine a price per unit for each of the three orders, assuming the firms uses a mark-up of 25%
on total cost and allows for an 80% learning curve. Extract from 80% learning curve tables is given:
X 1.0 1.3 1.4 1.5 1.6 1.7 1.8 1.9 2.0
Y (%) 100.0 91.7 89.5 87.6 86.1 84.4 83.0 81.5 80.0
‘X’ represents the cumulative volume produced to date expressed as a multiple of initial order and
Y is the learning curve factor for a given ‘X’ value expressed as a percentage of the cost of initial
order.
Solution:
Calculation of average time per unit for each order:
Batches 𝐘𝐗 % Cumulative Cumulative Total Increm Incremen Average time
(X) average time average Time ental tal Units per incremental
per batch time per time Unit
(𝐘𝐗 ) unit
1 100% 4000 40 4000 4000 100 4000/100=40

E M Reddy Page | 21
AMA-Notes

1.6 86.1% 4000 x 86.1% 3444/100 = 3444 x 1.6 1510.4 60 1510.4/60=25.17


= 3444 34.44 = 5510.4
2 80% 4000 x 80.0% 3200/100 = 3200 x 2 = 889.6 40 889.6/40=22.24
= 3444 32 6400

Calculation of selling price per unit for each order:


Particulars First Order Second Order Third Order
(Rs.) (Rs.) (Rs.)
Material Cost 500 500 500
Labour cost: Automated 200 200 200
Labour cost: Skilled @ Rs.15 40 x 15 = 600 25.17 x 15 = 22.24 x 15 =
377.55 333.60
Variable overhead@20% of labour cost [(200+600)] x [(200+377.55)] x [(200+333.60)] x
20% = 160 20% = 115.51 20% = 106.72
Fixed overheads Department A @ Rs.8 per 20 x 8 = 160 20 x 8 = 160 20 x 8 = 160
hour
Fixed overheads Department B @ Rs.5 per 40 x 5 = 200 25.17 x 5 = 22.24 x 5 =
hour 125.85 111.2
Total Cost 1820 1478.91 1411.52
Add: Margin 25% of cost 455 369.73 352.88
Selling price per unit 2275 1848.63 1764.4

Question no 9: Sundaram products Ltd manufactures complex electronic measuring instruments


for which highly skilled labour required. Conventional standard costing has been used for some
time but problems have been experienced in setting nearest standards for labour costs.
Analysis of production times has shown that there is a learning effect of 90%. During period 11 the
following data were recorded:
Cumulative production at start of period 526 Units
Production in period 86 Units
Wages Paid Rs. 71,823 for 6,861 actual hours
Material actual cost Rs. 20,850
Actual overheads for period Rs. 1,52,600
Budget and standard cost for electronic meters.
Budgeted Production 86 Units
Budgeted Overhead Rs. 1,50,903
Standard labour cost Rs.10 per hour
Standard Material cost per unit Rs.250
Hours for the first unit 200 Hours
You are required to:
(a) Calculate and analyze where possible the materials, labour and variable overhead cost
variances;
(b) Calculate a total standard cost for electrometers.
Solution:
Step 1: Computation of Y612

E M Reddy Page | 22
AMA-Notes

YX = a(X)b
Y612 = 200 x (612)−0.1520 → Apply log on both Sides
Log Y612 = Log 200 + Log 612−0.1520
Log Y612 = Log 200 – 0.1520 Log 612
= 2.3010 – (0.1520 X 2.7868)
= 2.3010 – 0.4236
= 1.8774
Y612 = Antilog of 1.8774
= 0.7541 X 10(1+1)
= 0.7541 X 100
= 75.41 Hours
Step 1: Computation of Y612

YX = a(X)b
Y526 = 200 x (526)−0.1520 → Apply log on both Sides
Log Y526 = Log 200 + Log 526−0.1520
Log Y526 = Log 200 – 0.1520 Log 526
= 2.3010 – (0.1520 X 2.7210)
= 2.3010 – 0.4136
= 1.8874
Y526 = Antilog of 1.8874
= 0.7711 X 10(1+1)
= 0.7716 X 100
= 77.16 Hours

Step 3: Calculation of Standard Hours

Time allowed for 612 Units = Y612 x 612 Units + [Y526 x 526 Units]
= 75.41 Hours x 612 Units + [77.16 Hours x 526 Units]
= 46,151 Hours
Time allowed for 526 Units = Y526 x 526 Units
= 77.16 Hours x 526 Units
= 40,586 Hours
Time allowed for 86 Units = Time allowed for 612 Units – Time allowed for 526 units
(Standard Hours) = 46,151 Hours – 40.586 Hours
(527th Unit – 612th Unit) = 5,565 Hours

Step 4: Computation table


[1] [2] [3]
SH x SR AH x AR AH x SR
5,565 Hours x Rs.10 Rs. 71,823 6,861 Hours x Rs.10
Rs. 55,650 Rs. 71,823 Rs. 68,610

E M Reddy Page | 23
AMA-Notes

Step 5: Variances Calculation

Labour Cost Variance


(1-2) = Rs.55,650 –
Rs.71,823 = Rs.16,173
(Adverse)
Labour Rate Variance Labour Efficiency
(3-2) = Rs.68,610 – Variance (1-3) =
Rs.71,823 = Rs.3,213 Rs.55,650 – Rs.68,610
(Adverse) = Rs.12,960 (Adverse)

E M Reddy Page | 24
AMA-Notes

2. LINEAR PROGRAMMING

2.1. Learning Objectives

1) Maximization linear programming problem


a) Graphical Method
b) Simplex Method
2) Minimization linear programming problem
a) Graphical Method
b) Simplex Method
3) Infeasible Solution
4) Unbounded Solution
5) Multiple Optimal Solution
6) Primal Dual Issues
7) Interpretation of a dual Problem
8) Interpretation of a final simplex table
9) Shortcut Substitution
10) Formulation time

2.2. Introduction

1) Example:
Products X Y
Contribution Rs. 4 6
Raw Material Kgs per unit 1 Kg. 2 Kg.
Contribution /Kg. 4 3
Rank based I II
Labour Hours per unit 2 Hours 2 hours
Contribution per hour 2 3
Rank II I

2) When something limits our production it is called “Limiting Factor” which may be raw material
resource availability or labour hour capacity etc.,
3) When there exists a limiting factor it should be allocated to the most profitable product and the
profitability is ranked using the contribution per limiting factor.
4) In the above example, there are two limiting factors (i) Raw Material – 100 Kgs (ii) Labour Hours –
100 Hours
5) Raw Material as a limiting factor selects Product X and labour hour Product Y. Thus there exists a
conflict in ranking. In such a case the problem should be solved using LPP technique (Linear
Programming Problem) technique.
6) This chapter will be applied when we have an objective to be achieved with multiple constraints.
7) The above situation can be formulated into a Linear Programming Problem as follows:
Let X1 be number of units of Product ‘X’ and X2 be number of units of Product ‘Y’.
Max Z = 4X1 + 6X2
Subject to
X1 + 2X2 < 100

E M Reddy Page | 25
AMA-Notes

2X1 + 2X2 < 100


Where X1 & X2 > 0
8) The first equation (Max Z = 4X1 + 6X2 ) is called “Objective Function” which can be of two
types:
a) Maximization – Maximize production, profit etc.,
b) Minimization – Minimize cost, time etc.,
9) The objective function should be achieved subject to constraints which can be of 3 types:
a) Not more than (<)
b) At least (>)
c) Exactly (=)
10) The items for which we should calculate values are called “Variables” which are represented as
variables (Z, X1 , X2 etc.,)
11) The numbers prefixed to the variables are called “Coefficients”. For example, coefficient of X2 in
objective function is ‘6’.
12) We have to find out the value of variables to achieve our object and this can be done by solving the
LPP (Linear Programming Problems) under two approaches
a) Graphical Approach
b) Simplex Method
Graphical approach can be used only for two variable problems and that to it can give only solution
and not wealth of information which simplex can do.

2.3. Maximization Linear Programming Problem (LPP)

Question no 1:
Maximize Z = 3𝐗 𝟏 + 4𝐗 𝟐
Subject to 2𝐗 𝟏 + 3𝐗 𝟐 < 16 (Raw Materials Kgs)
Subject to 4𝐗 𝟏 + 2𝐗 𝟐 < 16 (Labour Hours)
Subject to 𝐗 𝟏 >0, 𝐗 𝟐 >0

Solution:

2.3.1. Graphical Approach

Step 1: Identification of two points for each line (constraint functions)


2X1 + 3X2 = 16
16
Let X1 =0, X2 = = 5.33
3
16
Let X2 =0, X1 = =8
2

4X1 + 2X2 = 16
16
Let X1 =0, X2 = =8
2
16
Let X2 =0, X1 = =4
4

Step 2: Identifying feasible and extreme points

E M Reddy Page | 26
AMA-Notes

Notes:

1) All the points on this straight line satisfies equality. Points above the straight line represents > to
constraint and below the straight line < to constraint.
2) Since the both constraints are <, the feasible region should be below both the straight lines. However, it
should not go below the origin because of non-negativity constraint (X1 >0, X2 >0)
3) All points on the feasible region satisfies all the constraints but there exists one point that gives
maximum profit which is called as “Optimum Solution”.
4) Extreme point theorem states that optimum solution lies at the extreme points (corner points) of the
feasible region.
5) One of the extreme is lying at the intersection of the two lines which can be found out by solving the
two equations.

Step 3: Optimum Solution


Points Calculation Profit
(0,0) 3x0+4x0 0
(0,5.33) 3 x 0 + 4 x 5.33 21.33
(2,4) 3x2+4x4 22
(4,0) 3x4+4x0 12

Produce 2 units of X1 and 4 units of X2 to produce a maximum profit of 22.

2.3.2. Simplex Method

Step 1: Conversion of Inequalities into Equalities

E M Reddy Page | 27
AMA-Notes

Subject to
2X1 + 3X2 + S1 = 16
4X1 + 2X2 + S2 = 16
Objective function → Max Z = 3X1 + 4X2 + 0S1 + 0S2 ; where S1 & S2 are slack variables

Notes:

1) The first step in any LPP (Linear Programming Problem) is to convert Inequalities into equalities.
2) The variables added in LHS (Left Hand Side) to make it equal to RHS (Right Hand Side) are called
“Slack Variables”.
3) Slack Variable represents unused or idle resources. Here S1 represent unused raw material and S2
represents idle labour hours.
4) For example, say the production plan is (X1 , X2 ) = (2, 2). It consumes 10 kg [(2 kg x 2) + (3 kg x 2)] of
raw materials and 12 hours [(4 Hours x 2) + (2 Hours x 2)] of labour time leaving 6 Kgs unused (S1)
and 4 hours idle (S2 ).
5) Since unused or idle resources generates no profit the objective function co-efficient of slack variables is
“0”.

Step 2: First Simplex table


Fixed Program Profit Quantity 𝐗𝟏 𝐗𝟐 𝐒𝟏 𝐒𝟐 Replacement
Ratio (FR) Ratio (RR)
- S1 0 16 2 3 1 0 16/3
2/3 S2 0 16 4 2 0 1 16/2=8
Cj 3 4 0 0 Outgoing
Incoming Variable (I) = X2 Zj 0 0 0 0 Variable
Outgoing Variable (O) = S1 Cj - Zj 3 4 0 0
NER (Net
Evaluation Row)
Incoming Variable

Step 3: Second Simplex table


Fixed Program Profit Quantity 𝐗𝟏 𝐗𝟐 𝐒𝟏 𝐒𝟐 Replacement
Ratio (FR) Ratio (RR)
1/4 X2 4 16/3 2/3 1 1/3 0 8
- S2 0 16/3 8/3 0 -2/3 1 2
Cj 3 4 0 0 Outgoing
Incoming Variable (I) = X1 Zj 0 0 0 0 Variable
Outgoing Variable (O) = S2 Cj - Zj 3 4 0 0
NER (Net
Evaluation Row)
Incoming Variable
A 16 4 2 0 1
B (Fixed Ratio X Key Row) 32/3 4/3 2 2/3 0
A–B 16/3 8/3 0 -2/3 1

E M Reddy Page | 28
AMA-Notes

Step 4: Third Simplex table


Fixed Program Profit Quantity 𝐗𝟏 𝐗𝟐 𝐒𝟏 𝐒𝟐 Replacement
Ratio (FR) Ratio (RR)
X2 4 4 0 1 1/2 -1/4
X1 3 2 1 0 -1/4 3/8
Cj 3 4 0 0
Zj 3 4 5/4 1/8
Cj - Zj 0 0 -5/4 -1/8
NER (Net Evaluation Row)

A 16/3 2/3 1 1/3 0


B (Fixed Ratio X Key Row) 4/3 2/3 0 -1/6 1/4
A–B 4 0 1 1/2 -1/4

Notes:
1) From the above simplex tables, we can make the following two observations:
a. Basic Variables in any simplex table will form Unit Matrix.
b. The NER (Net Evaluation Row) values of basic variables will always be zero ‘0’.
2) Final Solution: Produce 2 units of X1 and 4 units of X 2 giving a profit of Rs.22 [(Rs.3 X 2 Units) + (Rs.4
X 4 Units)] which was the answer we got in graphical approach also.
3) Like Graphical Approach, the Simplex Method also moves from one extreme point to another in search
of Optimum solution. This can be understood as follows:
Table Solution Remarks
First Simplex Table S1 = 16 & S2 = 16 This table suggest that produce ‘0’ units of X1 &
‘0’ units of X2 and keep the entire raw material
(S1 ) unused & labour time (S2 ) idle. This is the
first extreme point (X1 , X2 ) = (0, 0).
Second Simplex Table 16
X 2 = 3 & S2 = 3
16
X1 = 0 – as not in the program
16
X2 = 3 = 5.33
S1 = 0 – as not in the program
16
S2 = 3 = 5.33
This means, produce ‘0’ units of X1 and 5.33
units of X2 consuming entire raw material (S1=0)
and leaving 16/3 hours of labour time (S2 =16/3)
as idle. This yet another extreme point (X1 , X2 ) =
(0, 5.33).
Third Simplex Table X 2 = 4 & X1 = 2 X1 = 2
X2 = 4
S1 = 0 – as not in the program
S2 = 0 – as not in the program
This means, produce ‘2’ units of X1 and ‘4’ units
of X2 consuming entire raw material (S1=0) and
the available labour time (S2 =0). This yet another

E M Reddy Page | 29
AMA-Notes

extreme point (X1 , X2) = (2, 4) as well as


optimum solution.

4) Why least replacement ratio?


a. In first table, we decided to produce X2 for improvement.
b. Every X2 requires 3 Kgs and 2 Hours. With 16 Kgs available we can produce 16/3 units and
with 16 hours available we can produce 8 units.
c. If we decide to produce ‘8’ units we have labour hours (8 Units X 2 Hours = 16 House) but we
do not have sufficient raw material (8 Units X 3 Kgs = 24 Kgs but have only 16 Kgs).
d. Thus the least RR (16/3 Units) is the possible X2 output.
e. When 16/3 units of X2 is produced
i. Raw Material
1. Available – 16 Kgs
2. Used – 3 Kgs X 16/3 Units = 16 Kgs
3. Balance – ‘0’
Replaced Raw Material (S0 )fully hence became outgoing Variable.
ii. Labour Hours
1. Available – 16 Hours
2. Used – 2 Hours X 16/3 Units = 32/3 Hours
3. Balance – 16/3 Hours
Partially replaced hence continuous in the next table with balance 16/3 hours.
5) What is the meaning of NER (Net Evaluation Row)?
a. NER in second simplex table is 1/3. This means if X1 as basic variable, for every X1 produced
the profit increases by 1/3.
b. The NER is influenced by 2 factors.
i. Cj – It shows the profit increase due to introducing X1 which is 3
ii. Zj – It shows the sacrifice of profit due to introducing of X1 which is 8/3
c. Hence, the net profit increase is Cj − Zj which is 1/3. If this is positive improve else the
solution is final.
6) Meaning of Zj ?
a. To produce 1 unit of X1 we require 2 Kgs of Raw Material 4 Hours of Labour.
b. Hours are available but the entire raw material is now consumed by X2 . Hence to produce X1
we should sacrifice some X2 .
c. The second simplex table X1 column (key column) shows X2 = 2/3 and S2 = 8/3. This means
to produce one unit of X1 we should sacrifice 2/3 units of X2 and 8/3 hours of idle time.
d. The above sacrifice generates the following resources:
X1
 Sacrifice 2/3 X2
o Raw material released – 2/3 Units X 3 Kgs. = 2 Kgs
o Labour hours released – 2/3 Units X 2 Hours = 4/3 Hours. These 4/3 hours
release with 8/3 Hours idle time gives us requires 4 hours labour time.
 Use 8/3 hours of idle time
e. By sacrificing 2/3 units of X2 we lose Rs.4 X 2/3 Units = Rs.8/3 and by using idle time 8/3
Hours we lose Rs.0 X 8/3 = Rs.0. Hence the total lose is 8/3 + 0 = Rs.8/3 which is Zj .

E M Reddy Page | 30
AMA-Notes

7) Concept of Shadow Cost or Opportunity Cost:


a. In the final simplex table, NER value of S1 is -5/4. This means if S1 is introduced as basic
variable, for every S1 introduced the profit drops by 5/4.
b. Introducing S1 as basic variable means planning to keep a Kg. of raw material unused. Hence
unused raw material Kg. or wasted raw material Kg. makes company lose 5/4 profit. This is the
opportunity cost of raw material Kg.
c. Same logic can be applied for labour hours (S2 = 1/8)

2.3.3. Steps in solving a maximization problem using Simplex Method

1) Convert inequalities in to equalities


Convert of inequalities in to equalities by adding slack variables in the constraint equations. Slack
variable represents idle or unused resources since idle resource do not generate any profit, the value of
slack variable in the objective function is zero (0).
2) Constructs the initial simplex table
The table should have the following columns.
FR Program Profit Quantity 𝐗𝟏 𝐗𝟐 𝐒𝟏 𝐒𝟐 RR
Key Column Number
FR – Fixed Ratio = Key Number
Quantity Column
RR – Replacement Ratio = Key Column Number
a. Write co-efficient of the constant functions in the first table against the respective variables.
1 0 0 1
b. Identify the variables forming the unit matrix [ ] or [ ] among them. These Variables
0 1 1 0
are the ones which should enter the program column as basic variables.
c. Assume the value of all non-basic variables to be zero and find out the value if basic variables to
be entered in quantity column.
d. Calculate the values in net-evaluation row (NER). NER = Cj − Zj
e. Cj is the co-efficient of the variable in the objective function and Zj is the product of numbers in
profit column and respective variable columns. If all the numbers in NER or either negative (-)
or zero, the solution is optimal else we have to go for improvement.
3) Step for improvement
a. Identify the variable with highest positive number in NER. This is called as incoming variable
(I). The column in which this variable is placed is called as “Key Column”.
b. Calculate the replacement ratios for the existing basic variables.
c. The variable having the least replacement ratio will be the outgoing variable (O).
d. The number lying at the inter-section of key row and key column is referred to as key number.
e. Construct the second simplex table where the incoming variable will enter the program column.
f. The value of basic variables from quantity column till the replacement ratio column should be
compared as follows:
New basic Variable Continuing basic Variable
Divide the existing values of the variable in Values are computed using A – B formula
the first table by key number. The resulting where ‘A’ is the value of the variables in the
values should be entered in new table. first table and ‘B’ is the product of fixed rate
and key number.

E M Reddy Page | 31
AMA-Notes

g. Calculate values in NER for the new table and check for optimality.
h. Repeat the above steps until the optimal solution is obtained.

Note:

Slack variables are used when there is < sign in the constraint functions. If the sign is > then surplus
variables are to be used. When surplus variables are used then artificial variables should be included in the
solution.

Note:
Steps for minimization problem are all most same as that of maximization. Wherever in ascertaining
solution is optimal there should be no negative.

2.4. Minimization Linear Programming Problem (LPP)

Question no 2: A small township of 15,000 people requires, on the average, 300,000 gallons of water
daily. The city is supplied water from a central water-works where the water is purified by such
conventional methods as filteration and chlorination. In addition, two different chemical
compounds: (i) softening chemical and (ii) health chemical are needed for softening the water and
for health purposes. The waterworks plans to purchase two popular brands that contain these
chemicals. One unit of Chemico Corporation's product gives 8 pounds of softening chemical and 3
pounds of health chemical. One unit of Indian Chemical's product contains 4 pounds and 9
pounds per unit, respectively, for the same purposes. To maintain the water at a minimum level of
softness and to meet a minimum programme of health protection, experts have decided that 150
and 100 pounds of the two chemicals that make up each product must be added to water daily. At a
cost of 8 and 10 per unit respectively for Chemico's and Indian Chemical's products, what is the
optimal quantity of each product that should be used to meet the minimum level of softness and
minimum health standard?
Solution:
Facts:
Chemical Chemico Product Indian Chemical Product Minimum Requirement
Softening Chemical 8 Pounds 4 Pounds 150 Pounds
Health Chemical 3 Pounds 9 Pounds 100 Pounds

Step 1: Formulation

Let X1 be number units of Chemico product purchased and X2 be number units of Indian Chemical
produce purchased.

Minimize Z = 8X1 + 10X2


Subject to
8X1 + 4X2 > 150 (Softening Chemical requirement)
3X1 + 9X2 > 100 (Health Chemical requirement)
Where X1 & X2 > 0

E M Reddy Page | 32
AMA-Notes

2.4.1. Graphical Approach

Step 2: Solving Using Graphical Approach

Step A: Identifying two points for each constraint equation

8X1 + 4X2 = 150


150
Let X1 =0, X2 = = 37.5
4
150
Let X2 =0, X1 = = 18.75
8

3X1 + 9X2 = 100


100
Let X1 =0, X2 = = 11.11
9
100
Let X2 =0, X1 = = 33.33
3

Step B: Identifying feasible and extreme points

Step C: Optimum Solution


Points Computation Cost
(0,37.5) 8 x 0 + 10 x 37.5 375
(15.83,5.83) 8 x 15.83 + 10 x 5.83 185
(33.33,0) 8 x 33.33 + 10 x 0 267

Purchase 15.83 units of Chemico Corporation’s product and 5.83 units of Indian Chemical 22 Product.

E M Reddy Page | 33
AMA-Notes

2.4.2. Simplex Method

Step 3: Simplex Method

Step A: Conversion of Inequalities into Equalities

Objective function → Min Z = 8X1 + 10X2 + 0S1 + 0S2 + MA1 + MA2


Subject to
8X1 + 4X2 - S1 + A1 = 150
3X1 + 9X2 - S2 + A2 = 100

Notes:

1) The variables S1 &S2 are called “Surplus Variables”.


2) Surplus variables represent resource on hand in excess of requirement. For example, if X1 =15 and
X2 =15, the no. of pounds of softening chemical in hand will be 180 pounds [(8x15) + (4x15)] but we
require only 180 pound and the excess 30 pounds is called surplus variable. Same way one can
understand for S2 .
3) Since the cost of required resource and excess resource (150 pounds & 30 pounds) is already there in
8X1 & 10X2 , we do not incur any extra cost specifically for surplus variable. Hence its objective co-
efficient is “0”.
4) If we continue into the first table with surplus variable alone we will end up with negative quantity. To
avoid this situation, we introduced artificial variable.
5) These artificial variables are imagined variables and hence will not disturb equality. Our final simplex
solution should not recommend artificial variable. To prevent that we assign a very high cost to it
(M=Infinity).

Step B: First Simplex table

FR Program Cost Quantity 𝐗𝟏 𝐗𝟐 𝐒𝟏 𝐒𝟐 𝐀𝟏 𝐀𝟐 RR


4/9 A1 M 150 8 4 -1 0 1 0 75/2
- A2 M 100 3 9 0 -1 0 1 100/9
Cj 8 10 0 0 M M Outgoing
Incoming Variable (I) = Zj 11M 13M -M -M M M Variable
X2
Outgoing Variable (O) = Cj - Zj 8- 10 - M M 0 0
A2 NER (Net 11M 13M
Evaluation
Row)
Incoming Variable

Step C: Second Simplex table

FR Program Cost Quantity 𝐗𝟏 𝐗𝟐 𝐒𝟏 𝐒𝟐 𝐀𝟏 𝐀𝟐 RR


- A1 M 950/9 20/3 0 -1 4/9 1 -4/9 95/6
1/20 X2 10 100/9 1/3 1 0 -1/9 0 1/9 100/3

E M Reddy Page | 34
AMA-Notes

Cj 8 10 0 0 M M Outgoing
Incoming Variable (I) = Zj 20M+10 10 -M 4M−10 M 10−4M Variable
3 9 9
X1
Outgoing Variable (O) = Cj - Zj 14−20M 0 M 10−4M 0 13M−10
3 9 9
A1 NER (Net
Evaluation
Row)
Incoming Variable
A 150 8 4 -1 0 1 0
B (Fixed Ratio X Key Row) 400/9 4/3 4 0 -4/9 0 4/9
A–B 950/9 20/3 0 -1 4/9 1 -4/9

Step D: Third Simplex table

FR Program Cost Quantity 𝐗𝟏 𝐗𝟐 𝐒𝟏 𝐒𝟐 𝐀𝟏 𝐀𝟐 RR


- X1 8 95/6 1 0 -3/20 1/15 3/20 -1/15 -
- X2 10 35/6 0 1 1/20 -2/15 -1/20 2/15 -
Cj 8 10 0 0 M M
Zj 8 10 −14 −12 14 12
20 15 20 15
Cj - Zj 0 0 14 12
M – 20
14 12
M – 15
20 15
NER (Net
Evaluation
Row)

A 100/9 1/3 1 0 -1/9 0 1/9


B (Fixed Ratio X Key Row) 95/18 1/3 0 -1/20 1/45 1/20 -1/45
A–B 35/6 0 1 1/20 -2/15 -1/20 2/15
95
X1 = = 15.83
6
35
X2 = = 5.83
6

Question no 3:
Maximize Z = 40𝐗 𝟏 + 35𝐗 𝟐
Subject to 2𝐗 𝟏 + 3𝐗 𝟐 < 60
Subject to 4𝐗 𝟏 + 3𝐗 𝟐 < 96
Subject to 𝐗 𝟏 >0, 𝐗 𝟐 >0

Solution:
Question no 4:
Minimize Z = 60𝐘𝟏 + 95𝐘𝟐
Subject to 2𝐘𝟏 + 4𝐘𝟐 > 40
Subject to 3𝐘𝟏 + 3𝐘𝟐 > 35
Subject to 𝐘𝟏 >0, 𝐘𝟐 >0

E M Reddy Page | 35
AMA-Notes

Solution:

2.5. Infeasible Solution

Question no 5:
Minimize Z = 20𝐗 𝟏 + 30𝐗 𝟐
Subject to 2𝐗 𝟏 + 𝐗 𝟐 < 40
Subject to 4𝐗 𝟏 - 𝐗 𝟐 < 20
Subject to 𝐗 𝟏 >30
Subject to 𝐗 𝟏 & 𝐗 𝟐 >0
Solution:

Step 1: Conversion of Inequalities into Equalities

Objective function → Max Z = 20X1 + 30X2 + 0S1 + 0S2 + 0S3 - MA1


Subject to
2X1 + X2 + S1 = 40
4X1 - X2 + S2 = 20
X1 - S3 + A1 = 30

Note: In the maximization objective function the co-efficient of artificial variable should be -M as we are
assuming infinite cost.

Step 2: Final Simplex table

Refer work book page no 267.

In the final simplex table artificial variable appears in the program column as basic variable. Thus the
problem does not have solution i.e. it is the problem with infeasible solution.

Step 3: Identifying infeasibility using Graphical Approach

Step A: Identification of two points for each constraint

2X1 + X2 = 40
Let X1 =0, X2 = 40
40
Let X2 =0, X1 = = 20
2

4X1 - X2 = 20
Let X1 =0, X2 = -20
20
Let X2 =0, X1 = =5
4

Step B: Drawing the straight lines and identifying feasible region

E M Reddy Page | 36
AMA-Notes

Notes:

1) The feasible region towards the left of the graph satisfies the first two constraints but does not satisfy
the third constraint and on the other hand the feasible region towards right satisfies the third constraint
but not the first two.
2) Thus there exists there is no single point in the entire graph that satisfies all the three constraints.
Hence, the problem does not have solution.
3) Infeasible solution arises when the constraints are conflicting.
4) The following can be observed while solving the above problem:
a. In improvement there will be one new variable and two continuous variables. Hence every time
we should do ‘A – B’ calculation two times.
b. Outgoing variable is that variable having lease positive replacement ratio (see second simplex
table).

2.6. Unbounded Solution

Question no 6:
Maximize Z = 10𝐗 𝟏 + 20𝐗 𝟐
Subject to 2𝐗 𝟏 + 4𝐗 𝟐 > 16
Subject to 𝐗 𝟏 + 5𝐗 𝟐 < 15
Subject to 𝐗 𝟏 & 𝐗 𝟐 >0

Solution:

Step 1: Conversion of inequalities into equalities

E M Reddy Page | 37
AMA-Notes

Objective function → Max Z = 10X1 + 20X2 + 0S1 + 0S2 - MA1 - MA2


Subject to
2X1 + 4X2 - S1 + A1 = 16
X1 + 5X2 - S2 + A2 = 15

Step 2: Final Simplex table

Refer page no. 260

It could be seen that there is an improvement to do but outgoing variable couldn’t be identified due to all
RR (Replacement Ratios) being negative. In this problem the problem is said to be having unbounded
solution i.e. solution is (X1 ,X2 ) = (∞, ∞)

Step 3: Understanding unboundness through Graphical Approach

Step A: Identification of two points for each constraint

2X1 + 4X2 = 16
16
Let X1 =0, X2 = =4
4
16
Let X2 =0, X1 = =8
2

X1 + 5X2 = 15
15
Let X1 =0, X2 = =3
5
Let X2 =0, X1 = 15

Step B: Drawing the straight lines and identifying feasible region

E M Reddy Page | 38
AMA-Notes

Notes:

1) A feasible region in this graph extends to infinity (∞) and the problem is maximization. Hence, the
optimum solution is (X1 ,X 2 ) = (∞, ∞) i.e. the solution is unbounded.
2) Maximization problems can have greater than or equal to (>) constraints but should at least have one
less than or equal to (<) constraint to act as limiting factor or boundary else the solution will be
unbounded.
3) Minimization problems can have less than or equal to (<) constraints but should at least have one
greater than or equal to (>) constraint to prevent the solution from being nil i.e. (X1 ,X2 ) = (0, 0)

2.7. Multiple Optimal Solution

Question no 7:
Maximize Z = 8𝐗 𝟏 + 16𝐗 𝟐
Subject to 𝐗 𝟏 + 𝐗 𝟐 < 200
Subject to 𝐗 𝟐 < 125
Subject to 3𝐗 𝟏 + 6𝐗 𝟐 < 900
Subject to 𝐗 𝟏 & 𝐗 𝟐 >0

Solution:

Step 1: Conversion of inequalities into equalities

Max Z = 8X1 + 16X2 + 0S1 + 0S2 + 0S3


Subject to
X1 + X2 + S1 = 200
X2 + S2 = 125
3X1 + 6X2 + S3 = 900

Step 2: Final Simplex table

Refer page no. 271

Notes:

1) In the final simplex table, one of the non-basic variable (S2 ) has ‘0’ value in NER.
2) This means when S2 made as incoming variable, for every S2 introduced the profit changes by ‘Rs.0’ i.e.
it does not change.
3) Thus there exists another solution that gives the same profit. Hence the problem is having multiple
optimal solution.
4) Profit for the current solution:
S1 = 25 Units x 0 = Rs.0
X2 = 125 Units x 16 = Rs.2000
X1 = 50 Units x 8 = Rs.400
Total Profit = Rs.0 + Rs.2000 + Rs.400 = Rs.2400
5) Improvement in the profit by making S2 as incoming variable:

E M Reddy Page | 39
AMA-Notes

FR Program Profit Quantity 𝐗𝟏 𝐗𝟐 𝐒𝟏 𝐒𝟐 𝐒𝟑 RR


S2 0 25 0 0 1 1 -1/3
X2 16 100 0 1 -1 0 1/3
X1 8 100 1 0 2 0 -1/3
Cj 8 16 0 0 0
Zj 8 16 0 0 8/3
Cj − Zj 0 0 0 0 -8/3
𝐗𝟐 Quantity 𝐗𝟏 𝐗𝟐 𝐒𝟏 𝐒𝟐 𝐒𝟑
A 125 0 1 0 1 0
B (Fixed Ratio X Key Row) 25 0 0 1 1 -1/3
A–B 100 0 1 -1 0 1/3
𝐗𝟏 Quantity 𝐗𝟏 𝐗𝟐 𝐒𝟏 𝐒𝟐 𝐒𝟑
A 50 1 0 0 -2 1/3
B (Fixed Ratio X Key Row) -50 0 0 -2 -2 2/3
A–B 100 1 0 2 0 -1/3

It could be seen that the above solution also gives a profit of Rs.2400 [(16 x 100) + (8 x 100)]

2.8. Degeneracy

*Question no 8: Using simplex method solve the following LPP:


Minimize Z = 2𝐗 𝟏 +𝐗 𝟐
Subject to: 3𝐗 𝟏 + 𝐗 𝟐 = 3
Subject to: 4𝐗 𝟏 + 3𝐗 𝟐 > 6
Subject to: 𝐗 𝟏 + 2𝐗 𝟐 < 3
Subject to: 𝐗 𝟏 & 𝐗 𝟐 > 0

Solution:

Step 1: Conversion of inequalities into equalities

Min Z = 2X1 + X2 + 0S1 + 0S2 + MA1 + MA2


Subject to
3X1 + X2 + A1 = 3
4X1 + 3X2 - S1 + A2 = 6
X1 + 2X2 + S2 = 3

Step 2: First Simplex table


FR Program Cost Quantity 𝐗𝟏 𝐗𝟐 𝐒𝟏 𝐒𝟐 𝐀𝟏 𝐀𝟐 RR
- A1 M 3 3 1 0 0 1 0 1
4/3 A2 M 6 4 3 -1 0 0 1 3/2
1/3 S2 0 3 1 2 0 1 0 0 3
Cj 2 1 0 0 M M
Zj 7M 4M -M 0 M M

E M Reddy Page | 40
AMA-Notes

NER = CZ − Zj 2 – 7M 1 – 4M M 0 0 0

Incoming Variable = X1 ; Outgoing Variable = A1

Step 3: Second Simplex table


FR Program Cost Quantity 𝐗𝟏 𝐗𝟐 𝐒𝟏 𝐒𝟐 𝐀𝟏 𝐀𝟐 RR
1/5 X1 2 1 1 1/3 0 0 1/3 0 3
- A2 M 2 0 5/3 -1 0 -4/3 1 6/5
1 S2 0 2 0 5/3 0 1 -1/3 0 6/5
Cj 2 1 0 0 M M
Zj 2 2+5M -M 0 2−4M M
3 3
NER = CZ − Zj 0 1−5M M 0 7M−2 0
3 3

Incoming Variable = X2 ; Outgoing Variable = A2

𝐀𝟐 Quantity 𝐗𝟏 𝐗𝟐 𝐒𝟏 𝐒𝟐 𝐀𝟏 𝐀𝟐
A 6 4 3 -1 0 0 1
B (Fixed Ratio X Key Row) 4 4 4/3 0 0 4/3 0
A–B 2 0 5/3 -1 0 -4/3 1
𝐒𝟐 Quantity 𝐗𝟏 𝐗𝟐 𝐒𝟏 𝐒𝟐 𝐀𝟏 𝐀𝟐
A 3 1 2 0 1 0 0
B (Fixed Ratio X Key Row) 1 1 1/3 0 0 1/3 0
A–B 2 0 5/3 0 1 -1/3 0

Step 4: Third Simplex table


FR Program Cost Quantity 𝐗𝟏 𝐗𝟐 𝐒𝟏 𝐒𝟐 𝐀𝟏 𝐀𝟐 RR
X1 2 3/5 1 0 1/5 0 3/5 -1/5
X2 1 6/5 0 1 -3/5 0 -4/5 3/5
S2 0 0 0 0 1 1 1 -1
Cj 2 1 0 0 M M
Zj 2 1 -1/5 0 2/5 1/5
NER = CZ − Zj 0 0 1/5 0 M – 2/5 M – 1/5

Incoming Variable = X2 ; Outgoing Variable = A2

𝐗𝟏 Quantity 𝐗𝟏 𝐗𝟐 𝐒𝟏 𝐒𝟐 𝐀𝟏 𝐀𝟐
A 1 1 1/3 0 0 1/3 0
B (Fixed Ratio X Key Row) 2/5 0 1/3 -1/5 0 -4/15 1/5
A–B 3/5 1 0 1/5 0 3/5 -1/5
𝐒𝟐 Quantity 𝐗𝟏 𝐗𝟐 𝐒𝟏 𝐒𝟐 𝐀𝟏 𝐀𝟐
A 2 0 5/3 0 1 -1/3 0
B (Fixed Ratio X Key Row) 2 0 5/3 -1 0 -4/3 1
A–B 0 0 0 1 1 1 -1

E M Reddy Page | 41
AMA-Notes

Notes:

1) In this problem, A basic variable S2 show ‘0’ quantity. Instead of being a non-basic variable it sits as
basic variable with ‘0’ quantity.
2) The above situation is called degeneracy and in this case even if improvement is possible it cannot be
done because we will be having ‘0’ replacement ratio for degenerate variable.
3) Degeneracy occurs because of tie in Replacement Ratios in the previous simplex table i.e. X2 replaces
A2 & S2 fully. While A2 is sent out, S2 continuous with ‘0’ quantity.
4) Suppose we send out S2 , in the next table A2 will continue with ‘0’ quantity. Can we call it infeasible
solution?
a. No, because really A2 is also non-basic variable. To call a solution as infeasible the artificial
variable should appear in final simplex table with some quantity.

2.9. Interpretation of final simplex table

Question no 9: The simplex table for a maximization problem of linear programming is given here:

𝐂𝐣 (Profit) 𝐗 𝐣 (Program) 𝐗𝟏 𝐗𝟐 𝐒𝟏 𝐒𝟐 Quantity (𝐛𝐣 )


5 𝐗𝟐 1 1 1 0 10
0 𝐒𝟐 1 0 -1 1 3
𝐂𝐣 4 5 0 0
𝐙𝐣 5 5 5 0
𝐂𝐣 − 𝐙𝐣 -1 0 -5 0

Answer the following questions, giving reasons in brief:


(a) Is this solution optimal?
(b) Are there more than one optimal solution?
(c) Is this solution degenerate?
(d) Is this solution feasible?
(e) If 𝐒𝟏 is slack in machine A (in hours/week) and 𝐒𝟐 is slack in machine (in hours/week), which
of these machines is being used to the full capacity when producing according to this solution?
(f) A customer would like to have one unit of product 𝐗 𝟏 and is willing to pay in excess of the
normal price in order to get it. How much should the price should be increased in order to ensure
no reduction of profits?

Solution:

2.10. Primal and Dual

2.10.1. Conversion of Primal to Dual

1. Every linear programming problem has two sides to it namely (i) Primal (Original problem) and (ii) Dual
(inverted problem).
2. The number of constraints in primal equal to number of variables in dual.
3. The RHS of primal constraints becomes the co-efficient of dual variables in objective function.
4. The rows in primal becomes in columns in dual and the columns in primal becomes in dual.

E M Reddy Page | 42
AMA-Notes

5. The RHS of dual is the co-efficient of variables in the primal objective function.
6. If primal is maximization, then dual is minimization and vice versa.
7. If primal is having greater than or equal to constraint dual should be having less than equal to constraint
and vice versa.
8. For writing a dual, the pre-condition is that all the primal constraints should be of the same type.
9. If the constraints are different types change the direction of the inequalities by multiplying by ‘-1’ on
either sides.

Question no 10: Write the dual of the following LPP:


(a) Type – 1
Maximize Z = 40𝐗 𝟏 + 35𝐗 𝟐
Subject to 2𝐗 𝟏 + 3𝐗 𝟐 < 60
4𝐗 𝟏 + 3𝐗 𝟐 < 96
𝐗𝟏, 𝐗𝟐 > 0

(b) Type – 2
Minimize Z = 10𝐗 𝟏 + 20𝐗 𝟐
Subject to 3𝐗 𝟏 + 2𝐗 𝟐 > 18
𝐗 𝟏 + 3𝐗 𝟐 > 8
2𝐗 𝟏 - 𝐗 𝟐 < 8
𝐗𝟏, 𝐗𝟐 > 0

(C) Type – 3
Maximize Z = 8𝐗 𝟏 + 10𝐗 𝟐 + 5𝐗 𝟑
Subject to 𝐗 𝟏 - 𝐗 𝟑 < 4
2𝐗 𝟏 + 4𝐗 𝟐 < 12
𝐗𝟏 + 𝐗𝟐 + 𝐗𝟑 > 2
3𝐗 𝟏 + 2𝐗 𝟐 - 𝐗 𝟑 = 8
𝐗 𝟏 ,𝐗 𝟐 , 𝐗 𝟑 > 0

(d) Type – 4
Maximize Z = 3𝐗 𝟏 + 5𝐗 𝟐 + 7𝐗 𝟑
Subject to 𝐗 𝟏 + 𝐗 𝟐 + 3𝐗 𝟑 < 10
4𝐗 𝟏 - 𝐗 𝟐 + 2𝐗 𝟑 > 15
𝐗 𝟏 , 𝐗 𝟐 > 0 and 𝐗 𝟑 : unrestricted in sign

Solution:

Type – 1:
Minimize Z’ = 60Y1 + 96Y2
Subject to 2Y1 + 4Y2 > 40
3Y1 + 3Y2 > 35
Y1 , Y2 > 0

Type – 2:

E M Reddy Page | 43
AMA-Notes

Step 1: Primal Revised

Minimize Z = 10X1 + 20X2


Subject to 3X1 + 2X2 > 18
X1 + 3X2 > 8
- 2X1 + X2 > -8
X1 , X 2 > 0

Step 2: Dual

Maximize Z’ = 18Y1 + 8Y2 - 8Y3


Subject to 3Y1 + Y2 - 2Y3 < 10
2Y1 + 3Y2 + Y3 < 20
Y1 ,Y2 , Y3 > 0

Type – 3:

Step 1: Primal Revised

Maximize Z = 8X1 + 10X2 + 5X3


Subject to 1X1 + 0X2 - 1X3 < 4
2X1 + 4X2 + 0X3 < 12
1X1 + 1X2 + 1X3 > 2
3X1 + 2X2 - 1X3 < 8
3X1 + 2X2 - 1X3 > 8
X1 ,X2 , X3 > 0

Step 2: Primal again revised

Maximize Z = 8X1 + 10X2 + 5X3


Subject to 1X1 + 0X2 - 1X3 < 4
2X1 + 4X2 + 0X3 < 12
- 1X1 - 1X2 - 1X3 < -2
3X1 + 2X2 - 1X3 < 8
- 3X1 - 2X2 + 1X3 < -8
X1 ,X2 , X3 > 0

Step 3: Dual

Minimize Z’ = 4Y1 + 12Y2 - 2Y3 + 8Y4 - 8Y5


Subject to Y1 + 2Y2 - Y3 +3Y4 - 3Y5 > 8
4Y2 - Y3 + 2Y4 - 2Y5 > 10
- Y1 - Y3 - Y4 + Y5 > 5
Y1 , Y2 , Y3 , Y4 , Y5 > 0

Step 4: Dual Revised

E M Reddy Page | 44
AMA-Notes

Let Y6 = Y4 − Y5

Minimize Z’ = 4Y1 + 12Y2 - 2Y3 + 8Y6


Subject to Y1 + 2Y2 - Y3 +3Y6 > 8
4Y2 - Y3 + 2Y6 > 10
- Y1 - Y3 - Y6 > 5
Y1 , Y2 , Y3 , Y4 , Y5 > 0 and Y6 : Unrestricted sign

Notes:

1) When we have equality in primal, first we should be splitting into two equations. For example, X1 = X2
can be written as X1 > X 2 and X1 < X2 .
2) In such case the dual will have more variables than the number of constraints in primal and variables
can be reduced through substitution.
3) In the above problem, the variables Y4 and Y5 emerges from same constraint (equality constraint) due
to splitting it into two. Hence we made a substitution Y6 = Y4 − Y5 .
4) Y1 ,Y2 ,Y3 ,Y4 , Y5 are original variables and hence should satisfy non-negativity but Y6 is a derived variable
which can take any sign depending upon Y4 and Y5 values.

Type – 4:

Step 1: Primal Revised

Substitute X3 = X4 − X5
Maximize Z = 3X1 + 5X2 + 7X4 - 7X5
Subject to X1 + X2 + 3X4 - 3X5 < 10
4X1 - X2 + 2X4 - 2X5 > 15
X1 , X2 , X4 , X5 > 0 and X3 : unrestricted in sign

Step 2: Primal again revised

Maximize Z = 3X1 + 5X2 + 7X4 - 7X5


Subject to X1 + X2 + 3X4 - 3X5 < 10
-4X1 + X2 - 2X4 + 2X5 < -15
X1 , X2 , X4 , X5 > 0 and X3 : unrestricted in sign

Step 3: Dual

Minimize Z’ = 10Y1 - 15Y2


Subject to Y1 - 4Y2 > 3
Y1 + Y2 > 5
3Y1 - 2Y2 > 7
-3Y1 + 2Y2 > -7
X1 , X2 , X4 , X5 > 0 and X3 : unrestricted in sign

E M Reddy Page | 45
AMA-Notes

Reading the last two constraints together we can understand that 3Y1 - 2Y2 = 7

Notes:

1) When primal has equality then the dual should be having variable with unrestricted sign (Type – 3) and
vice versa (Type – 4)

Question no 11: Write the dual of the following LPPs:


(i) Maximize Z = 3𝐗 𝟏 + 4𝐗 𝟐
Subject to 2𝐗 𝟏 + 3𝐗 𝟐 < 16
Subject to 4𝐗 𝟏 + 2𝐗 𝟐 < 16
Subject to 𝐗 𝟏 >0, 𝐗 𝟐 >0
(ii) Minimize Z = 8𝐗 𝟏 + 10𝐗 𝟐
Subject to
8𝐗 𝟏 + 4𝐗 𝟐 > 150
3𝐗 𝟏 + 9𝐗 𝟐 > 100
Where 𝐗 𝟏 & 𝐗 𝟐 > 0

Solution:

(i) Minimize Z’ = 16Y + 16Y2


Subject to 2Y1 + 4Y2 > 3
Subject to 3Y1 + 2Y2 < 16
Subject to Y1 >0, Y2 >0
(ii) Maximize Z’ = 150Y1 + 100Y2
Subject to 8Y1 + 3Y2 < 8
Subject to 4Y1 + 9Y2 < 10
Subject to Y1 & Y2 > 0

2.10.2. Interpretation of dual problem and Comparison of simplex tables

Primal:

Maximize Z = 40X1 + 35X2


Subject to 2X1 + 3X2 < 60
4X1 + 3X2 < 96
X1 , X 2 > 0

Dual:

Minimize Z’ = 60Y1 + 96Y2


Subject to 2Y1 + 4Y2 > 40
3Y1 + 3Y2 > 35
Y1 , Y2 > 0

1) Meaning of Primal Problem:

E M Reddy Page | 46
AMA-Notes

a. Company has two products A & B, unit profit of Product A is Rs.40 and unit profit of product
B is Rs.35.
b. The company has to manufacture and sell X1 units of ‘A’ and X2 units of ‘B’ to get maximum
profit.
c. This should be achieved within material and labour constraints (60 Kgs and 96 hours)
d. 1 unit of ‘A’ consumes 2 Kgs of raw materials and 4 hours of labour and 1 unit of ‘B’ consumes
3 Kgs of raw material and 3 hours of labour
2) Final Simplex table of Primal Problem – Relevant Portion
FR Program Profit Quantity 𝐗𝟏 𝐗𝟐 𝐒𝟏 𝐒𝟐 RR
X2 35 8 0 1 2/3 -1/3
X1 40 18 1 0 -1/2 1/2
NER 0 0 -10/3 -25/3

3) Meaning of dual problem:


a. Suppose a person asks the company to sell all its raw materials and labour hours, what should be
the minimum price that should be charged for every Kg of raw material and every hour of
labour is what the dual tries to find out.
b. The price should be so fixed that, if we give 2 Kgs of raw material and 4 hours of labour time, it
should at least fetch us a profit of Rs.40 which the company would have earned had it used
these sources to produce a unit of Product ‘A’.
c. The same way we can understand that 3 Kgs of raw material and 3 hours of labour should give
us Rs.35 profit.
d. Y1 in dual represents the minimum price to be charged for a Kg of raw material and Y1 the
minimum rate per hour of the labour time.
4) Final Simplex table of Dual Problem – Relevant Portion
FR Program Cost Quantity 𝐘𝟏 𝐘𝟐 𝐒𝟏 𝐒𝟐 RR
Y2 96 25/3 0 1 -1/2 1/3
Y1 60 10/3 1 0 1/2 -2/3
NER 0 0 18 8

5) Observations from the two simplex tables:


a.
Final Simplex table of Primal Final Simplex table of Dual
Variables Quantity NER Variables Quantity NER
X1 18 0 S1 0 18
X2 8 0 S2 0 8
S1 0 -10/3 Y1 10/3 0
S2 0 -25/3 Y2 25/3 0

The variable X1 and X2 in primal the corresponding variables S1 and S2 in dual and S1 and S2 in
primal will have corresponding Y1 and Y2 in dual.
The quantity column values of primal will become NER values of dual and similarly the NER
values of primal will be same as quantity value of dual.
b. Both primal and dual will have the same objective function value.
Primal: Max Z = 40X1 + 35X2
= 40 x 18 + 35 x 8 = 1000

E M Reddy Page | 47
AMA-Notes

Dual: Min Z’ = 60Y1 + 96Y2


10 25
= [60 x ] + [96 x ]
3 3
= 200 + 800
= 1000
6) The variables in primal represents quantities and their c- efficient represents values and in dual the
variables represent the values and the co-efficient represents the quantity.
7) Thus when you read a final simplex table of dual we should understand the cost column to be quantity
(96 Hours and 60 Kgs) and the quantity column to be price [Rs.25/3 and Rs.10/3]
8) In primal simplex table the NER value of S1 is -10/3 which means if we keep a Kg of raw material idle
it pulls down the profit by Rs.10/3. In other words, a Kg of raw material is capable of giving Rs.10/3
profit which is the price we should charge for selling that Kg. (See Y1 quantity column in final simplex
table of dual problem). Same logic for S1 also.
9) When we sell 60 Kgs of raw material and 96 hours of labour time we lose 18 units of X1 and 8 units of
X2 production which is the NER value of S1 and S2 in dual.
10) Primal earns the profit by using the resources and dual earns through selling resources.
11) Primal objective function maximizes and the boundary is fixed by the constraint. On the other hand,
dual constraints try to maximize but the limit is fixed through minimization objective function. Hence
both reports the same profit.

Question no 12: One unit of product A contributes Rs.7 and required 3 Kgs of raw material and 2
hours of labour. One unit of product B contributes Rs.5 and required one Kg of raw material and
one hour of labour. Availability of the raw material at present is 48 Kgs and there are 40 hours of
labour.
(a) Formulate it as a linear programming problem.
(b) Write it’s dual.
(c) Solve the dual with simplex method and find the optimal product mix and shadow prices of the
raw material and labour.

Solution:

Step 1: Formulating into Linear Programming Problem (Primal)

Let X1 be no. of units of product ‘A’ and X2 be no. of units of product ‘B’.

Maximize Z = 7X1 + 5X2


Subject to 3X1 + X2 < 48
2X1 + X2 < 40
Where X1 and X2 > 0

Step 2: Dual for the above primal

Minimize Z’ = 48Y1 + 40Y2


Subject to 3Y1 + 2Y2 > 7
Y2 + Y2 > 5
Where Y1 and Y2 > 0

E M Reddy Page | 48
AMA-Notes

Step 3: Final Simplex table for Dual


FR Program Cost Quantity 𝐘𝟏 𝐘𝟐 𝐒𝟏 𝐒𝟐 𝐀𝟏 𝐀𝟐 RR
Y2 40 5 1 1 0 -1 0 1
S1 0 3 -1 0 1 -2 1 2
Cj 48 40 0 0 M M
NER 8 0 0 40 M M-40

Step 4: Answering the primal questions using the dual simplex table
Final Simplex table of Dual Final Simplex table of Primal
Variables Quantity NER Variables Quantity NER
Y1 0 8 S1 8 0
Y2 5 0 S2 0 -5
S1 3 0 X1 0 -3
S2 0 40 X2 40 0

Produce ‘0’ units of X1 and ‘40’ units of X2 consuming the entire labour time available and leaving 8 Kgs of
raw material unused.
The Opportunity cost of the raw material is ‘0’ since it is remaining idle but for the labour hours it is Rs.5
(NER value of S2 in final simplex table of Primal).

Question no 13: A company produces two products 𝐗 𝟏 and 𝐗 𝟐 with respective unit contributions of
Rs.8 and Rs.6. Each product process through matching operations in two machine centers 𝐌𝟏 and
𝐌𝟐 whose capacities are limited to 60 and 48 hours respectively with corresponding slack variables
𝐒𝟏 and 𝐒𝟐 . Following table gives the value for in interaction under simplex method for
maximization of contribution.
Basic Variables 𝐗𝟏 𝐗𝟐 𝐒𝟏 𝐒𝟐
𝐗𝟏 1 0 1/3 -1/6
𝐗𝟐 0 1 -1/6 1/3
You are required to evaluate if this iteration represents the optimal solution. Find out the what will
be the optimum contribution?

Solution:

Part 1: Checking for Optimality


FR Program Profit Quantity 𝐗𝟏 𝐗 𝐒𝟏 𝐒𝟐 RR
X1 8 1 0 1/3 -1/6
X2 6 0 1 -1/6 1/3
Cj 8 6 0 0
Zj 8 6 5/3 2/3
Cj − Zj 0 0 -5/3 -2/3

The problem is maximization problem and all the numbers in the NER or either negative or ‘0’. Hence the
solution is optimal.

E M Reddy Page | 49
AMA-Notes

Part 2: Optimum Contribution

1) The Simplex table gives us the profit per unit of X1 and X2 but does not give us the quantity. Hence, we
cannot find out the profit using the production mix data.
2) However, we have NER values of S1 and S2 . S1 represents unused machine 1 time and S2 represents
unused machine 2 time.
3) The opportunity cost of machine 1 time is 5/3 hours and machine 2 time is 2/3hours which means one
hour of machine 1 time and machine 2 time is capable of giving Rs.5/3 and Rs.2/3 respectively.
5 2
Therefore, profit earned by using 60 hours and 48 hours is Rs.132 [(60x3) + (48x3)].

2.11. Formulation types

Question no 14: WELLTYPE manufacturing company produces three types of typewriters; manual
type writer, electronic typewriters and deluxe electronic typewriters. All the three models are
required to be machined first and then assembled. The time required for the various models are as
follows:
Type Machine Time (in hour) Assembly Time (in hour)
Manual Typewriter 15 4
Electronic Typewriter 12 3
Deluxe Electronic Typewriter 14 5
The total available machine time and assembly time are 3,000 hours and 1,200 hours respectively.
The data regarding the selling price and variable costs for the three types are:
Manual Electronic Deluxe Electronic
Selling Price (Rs.) 4,100 7,500 14,600
Labour, Material and other variable costs 2,500 4,500 9,000
The company sells al the three types on credit basis, but will collect the amount on the first next
month. The labour, material and other variable expenses to be paid in cash. The company has
taken a loan of Rs. 40,000 from a co-operative bank and this company will have repaid it to the
bank on 1st April, 2002. The TNC bank from whom this company has borrowed Rs. 60,000 has
expressed its approval to renew the loan.
The Balance Sheet of this company as on 31.03.02 is as follows:
Liabilities Rs. Assets Rs.
Equity Share Capital 1,50,000 Land 90,000
Capital Reserve 15,000 Building 70,000
General Reserve 1,10,000 Plant & Machinery 1,00,000
Profit & Loss a/c 25,000 Furniture & Fixtures 15,000
Long term loan 1,00,000 Vehicle 30,000
Loan from TNC Bank 60,000 Inventory 5,000
Loan from Co-operative Bank 40,000 Receivables 50,000
Cash 1,40,000
5,00,000 5,00,000
The company will have to pay a sum of Rs. 10,000 towards the salary from top management
executives and other fixed overheads for the month. Interest on long term loans is to be per every
month at 24% per annum. Interest on loans from TNC and co-operative banks may be taken to be
Rs. 1,200 for the month. Also this company has promised to deliver 2 manual typewriters and 8
deluxe electronic typewriters to one of its valued customers next month.

E M Reddy Page | 50
AMA-Notes

Company is subject to the availability of cash next month. This company will also to able to sell all
their types of typewriter in the market. The senior manager of this company desires to know as to
how many units of each typewriter must be manufactured in the factory next month so as to
maximize profits of the company. Formulate this as a linear programming problem. The
formulated problem need not be solved.

Solution:

Step 1: Defining Variables

Let X1 be number of units of manual type writer, X2 be number of units of electronic type writer and X3 be
number of units of deluxe electronic type writer.

Step 2: Formulating into a linear programming problem

Maximize Z = 1600X1 + 3000X2 + 5600X3 – 13,200


Subject to 15X1 + 12X2 + 14X3 < 3,000 (Machine time availability)
Subject to 4X1 + 3X2 + 5X3 < 1200 (Assemble time availability)
Subject to X1 > 2
Subject to X3 > 8
Subject to 2500X1 + 4500X2 + 9600X3 < 1,36,800 (Cash Availability constraint) (WN-2)
Subject to X2 > 0 (non-negativity constraint)

Working Note 1: Calculation of fixed cost per annum


Particulars Amount (Rs.)
Salary 10,000
Interest on TNC & Co-operative loan 1,200
Interest on long term loan (1,00,000 x 24% x 1/12) 2000
Total Fixed Cost 13,200

Working Note 2: Cash availability


Particulars Amount (Rs.)
Cash 1,40,000
Collection from debtors 50,000
Total 1,90,00
Less: Loan repaid (40,000)
Less: Fixed Cost (WN-1) (13,200)
Cash Available to meet variable production cost 1,36,800

*Question no 15: Consider a company that must produce two products over a production period of
three months of duration. The company can pay for materials and labour for two sources:
The firm faces three decisions:
(1) How many units should it produce of product 1?
(2) How many units should it produce of product 2?
(3) How much money should it borrow to support the production of the two products?
In making these decisions, the firm wises to maximize the profit contribution subject to the

E M Reddy Page | 51
AMA-Notes

conditions stated below:


(i) Since the company’s products are enjoying a seller’s market, it can sell as many units as it can
produce. The company would therefore like to produce as many units as possible subject to
production capacity and financial constraints. The capacity constraints, along with cost and price
data, are given in Table-1.
TABLE – 1 Capacity, Price and cost data
Product Selling Price Cost of Requirement hours per unit in department
Production A B C
1 14 10 0.5 0.3 0.2
2 11 8 0.3 0.4 0.1
Available hours per production period of three 500 400 200
months
(ii) The available company funds during the production period will be Rs.3 lakhs.
(iii) A bank will give loans up to Rs.2 lakhs per production period at an interest rate of 20% p.a.
provided the company’s acid (quick) test ratio is at least 1 to 1 while the loan is outstanding. Take
simplified acid-test ratio given by
𝐒𝐮𝐫𝐩𝐥𝐮𝐬 𝐜𝐚𝐬𝐡 𝐨𝐧 𝐡𝐚𝐧𝐝 𝐚𝐟𝐭𝐞𝐫 𝐩𝐫𝐨𝐝𝐮𝐜𝐭𝐢𝐨𝐧 + 𝐀𝐜𝐜𝐨𝐮𝐧𝐭𝐬 𝐫𝐞𝐜𝐞𝐢𝐯𝐚𝐛𝐥𝐞
𝐁𝐚𝐧𝐤 𝐁𝐨𝐫𝐫𝐨𝐰𝐢𝐧𝐠 + 𝐈𝐧𝐭𝐞𝐫𝐞𝐬𝐭 𝐚𝐜𝐜𝐫𝐮𝐞𝐝 𝐭𝐡𝐞𝐫𝐞𝐨𝐧
(iv) Also make sure that the needed funds are made available for meeting the production costs.
Formulate the above as linear programming.

Solution:

Step 1: Defining Variables

Let X1 be number of units of product 1, X2 be number of units of product 2 and X3 be the amount to be
borrowed.

Step 2: Formulating into a linear programming problem

Maximize Z = 4X1 + 3X2 – 0.05X3


Subject to 0.5X1 + 0.3X2 < 500 (Department A Hours availability)
Subject to 0.3X1 + 0.4X2 < 400 (Department B Hours availability)
Subject to 0.2X1 + 0.1X2 < 200 (Department C Hours availability)
Subject to X3 < 2,00,000 (Maximum loan amount)
Subject to 10X1 + 8X2 < X3 + 3,00,000 (Fund available to meet production cost)
X3 + 3,00,000 – (10X1 +8X2 )] + [14X1 + 11X2 ]
] Subject t [ > 1 (Acid-test ratio constraint)
X3 +0.05X3
Subject to X1 , X2 , X3 > 0 (non-negativity constraint)

Notes:

1) Do not write the constraint X3 = Rs.2,00,000 because the condition is not to borrow more than
Rs.2,00,000 and not exactly Rs.2,00,000.
2) The fund available for production expense is computed as X3 + Rs.3,00,000 and the interest is not
reduced unlike the previous problem because the interest is not paid, it is only accrued.

E M Reddy Page | 52
AMA-Notes

3) It is assumed that the entire sales are credit sales because quick ratio numerator has accounts receivables.
4) Generally, interest cost is fixed and does not vary with production but in this case the amount to be
borrowed X3 is dependent on units to be produced X1 and X2 . Hence, interest cost is linked to
production.

**Question no 16: A refinery makes 3 grades of petrol (A, B, C) from 3 crude oils (D, E, F) crude
can be used in any grade
Grade Specifications Selling Price per liter
A Not less than 50% of Crude D 8.0
Not more than 25% of Crude E
B Not less than 50% of Crude D 6.5
Not more than 25% of Crude E
C No Specifications 5.5
But the others satisfy the following specifications.
There are capacity limitations on the amount of the three crude elements that can be used.
Crude Capacity Price per liter
D 500 9.5
E 500 5.5
F 300 6.5
It is required to produce the maximum profit.

Solution:

Step 1: Analyzing facts

1) A – Selling Price =Rs.8


a. D – Cost Rs.9.5 - X1 = -1.5
b. E – Cost Rs.5.5 - X2 = 2.5
c. F – Cost Rs.6.5 - X3 = 1.5
2) B – Selling Price =Rs.6.5
a. D – Cost Rs.9.5 - Y1 = -3
b. E – Cost Rs.5.5 - Y2 = 1
c. F – Cost Rs.6.5 - Y3 = 0
3) C – Selling Price =Rs.5.5
a. D – Cost Rs.9.5 - Z1 = -4
b. E – Cost Rs.5.5 - Z2 = 0
c. F – Cost Rs.6.5 - Z3 = -1

Step 2: Defining Variables

Let X1 = Liters of crude oil D in Grade A petrol


Let X2 = Liters of crude oil E in Grade A petrol
Let X3 = Liters of crude oil F in Grade A petrol
Let Y1 = Liters of crude oil D in Grade B petrol
Let Y2 = Liters of crude oil E in Grade B petrol
Let Y3 = Liters of crude oil F in Grade B petrol

E M Reddy Page | 53
AMA-Notes

Let Z1 = Liters of crude oil D in Grade C petrol


Let Z2 = Liters of crude oil E in Grade C petrol
Let Z3 = Liters of crude oil F in Grade C petrol

Step 3: Formulation

Maximization Z = -1.5X1 + 2.5X2 + 1.5X3 - 3Y1 + Y2 + 0Y3 - 4Z1 + 0Z2 - Z3


Subject to X1 + Y1 + Z1 < 500 (Crude D availability)
Subject to X2 + Y2 + Z2 < 500 (Crude E availability)
Subject to X3 + Y3 + Z3 < 300 (Crude F availability)
X1 1
Subject toX > 2 (Proportion of D in grade A)
1 +X2 +X3
X2 1
Subject toX < 4 (Crude E proportion in grade A)
1 +X2 +X3
Y1 1
Subject toY > 4 (Crude D proportion in grade B)
1 +Y2 +Y3
Y2 1
Subject toY < 2 (Crude D proportion in grade B)
1 +Y2 +Y3
Where X1 , X2 , X3 , Y1 , Y2 , Y3 , Z1 , Z2 , Z3 > 0

Question no 17: A leading CA is attempting to determine a ‘best’ investment portfolio and is


considering six alternative investment proposals. The following table indicates point estimates for
the price per shares, the annual dividend per share and a measure of risk associated with each
investment.
Shares under consideration
Particulars A B C D E F
Current price per share (Rs.) 80 100 160 120 150 200
Projected annual growth rate 0.08 0.07 0.10 0.12 0.09 0.15
Projected annual dividend per share 4.00 4.50 7.50 5.50 5.75 0.00
Projected risk in return 0.05 0.03 0.10 0.20 0.06 0.08
The total amount available for investment is Rs.25 lakhs and the following conditions are required
to be satisfied:
(i) The maximum rupee amount to be invested in alternative F is Rs.2,50,000
(ii) No more than Rs.5,00,000 should be invested in alternative A and B combined.
(iii) Total weighted risk should not be greater than 0.10 where
𝐚𝐦𝐨𝐮𝐧𝐭 𝐢𝐧𝐯𝐞𝐬𝐭𝐞𝐝 𝐢𝐧 𝐚𝐥𝐭𝐞𝐫𝐧𝐚𝐭𝐢𝐞 𝐉 𝐱 𝐫𝐢𝐬𝐤 𝐨𝐟 𝐚𝐥𝐭𝐞𝐫𝐧𝐚𝐭𝐢𝐯𝐞 𝐉
Total weighted risk =
𝐭𝐨𝐭𝐚𝐥 𝐚𝐦𝐨𝐮𝐧𝐭 𝐢𝐧𝐯𝐞𝐬𝐭𝐞𝐝 𝐢𝐧 𝐚𝐥𝐥 𝐭𝐡𝐞 𝐚𝐥𝐭𝐞𝐫𝐧𝐚𝐭𝐢𝐯𝐞𝐬
(iv) For the sake of diversity at least 100 shares of each stock should be purchased.
(v) At least 10% of the total investment should be alternatives A and B.
(vi) Dividends for the year should be at least Rs. 10,000.
Rupee return per share of stock is defined as price per share one year hence less current price per
share plus dividend per share. If the objective is to maximize total rupee return, formulate the lines
at programming model for determining the optimal number of shares to be purchased in each of
the shares under consideration. You may assume that the time horizon for investment is one year.
The formulated LP problem is not required to be solved.

Solution:

E M Reddy Page | 54
AMA-Notes

Step 1: Defining Variables

Let X1 = Number of Shares of A


X2 = Number of Shares of B
X3 = Number of Shares of C
X4 = Number of Shares of D
X5 = Number of Shares of E
X6 = Number of Shares of F

Step 2: Calculation of return per share


Particulars A B C D E F
Dividend per share 4.00 4.50 7.50 5.50 5.75 0.00
Capital gains (Price x growth rate) 6.40 7.00 16.00 14.40 13.50 30.00
Total Return 10.40 11.50 23.50 19.90 19.25 30.00

Step 3: Formulation into LPP

Maximize Z = 10.40X1 + 11.5X2 + 23.50X3 + 19.90X4 + 19.25X5 + 30X6


Subject to
80X1 + 100X2 + 160X3 + 120X4 + 150X5 + 200X6 < 25,00,000 (amount available for investment)
200X6 < 2,50,000 (maximum in F)
80X1 + 100X2 < 5,00,000 (maximum investment in A and B)
X1 , X2 , X3 , X4 , X5 , X6 > 100
80X1 +100X2 1
> 10 (Minimum proportion in A and B)
80X1 +100X2 +160X3 +120X4 +150X5 +200X6
4X1 + 4.5X2 + 7.5X3 + 5.5X4 + 5.75X5 + 0X6 > 10,000 (Minimum dividend)
(0.05 x 80X1) +(0.03 x 100X2) +(0.01 x 160X3) +(0.20 x 120X4) +(0.06 x 150X5) +(0.08 x 200X6)
< 0.10 (Maximum risk)
80X1 +100X2 +160X3 +120X4 +150X5 +200X6

E M Reddy Page | 55
AMA-Notes

3. ASSIGNMENT PROBLEMS

3.1. Introduction

1) Features:
a) A type of linear programming problem
b) Assigning jobs to men & women
c) Can solve only minimization types
d) The assignments pre-condition is one to one relationship
2) Types of Problems
a) Minimization balanced assignment problems
b) Maximization balanced assignment problems
c) Minimization unbalanced assignment problems
d) Maximization unbalanced assignment problems
e) Degeneracy
f) Prohibited routes
g) Multiple optimal solution
h) Travelling sales men problem (Concept of looping)
3) Types of LPP
a) Linear Programing Problems
i. Normal Linear Programming Problems
ii. Transportation problems
1. Normal transportation problems
2. Assignment problems

3.2. Minimization balanced assignment problem – Hungarian Method

Question no 1: A machine tool company decides to make hour subassemblies through four
persons. Each person is to be received only one subassembly. The cost of each assembly is
determined by the bids by each person and shown in the table in hundreds of rupees. Assign the
different subassemblies to contactors so as to minimize the total cost.
Persons
Subassembly 1 2 3 4
1 15 13 14 17
2 11 12 15 13
3 13 12 10 11
4 15 17 14 16

Solution:

Step 1: Row operation

Take the minimum number in the row and subtract if from others.
Row Operation
15 13 14 17 2 0 1 4
11 12 15 13 0 1 4 2
13 12 10 11 3 2 0 1

E M Reddy Page | 56
AMA-Notes

15 17 14 16 1 3 0 2

Step 2: Column Operation

Take the minimum number in the column and subtract it from others.
Column Operation
2 0 1 4 2 0 1 3
0 1 4 2 0 1 4 1
3 2 0 1 3 2 0 0
1 3 0 2 1 3 0 1

Step 3: Covering zeros with minimum number of lines


2 0 1 3 2
0 1 4 1 3
3 2 0 0 1
1 3 0 1 4

Step 4: Is number of lines equal to order of matrix?

Number of lines = 4
Order of Matrix = 4 x 4
Yes, we can proceed to make assignment.

Step 4: Assignment
2 ⓪ 1 3 i
⓪ 1 4 1 ii
3 2 0 ⓪ iv
1 3 ⓪ 1 iii

Step 5: Final Solution


Sub assembly Person Cost
1 2 13
2 1 11
3 4 11
4 5 14
Total Cost 49

Notes:

1) We can formulate the above assignment problem into a LPP as follows:


Let X11 be first job given to 1st person, X12 first job given to 2nd person and so on...
Minimize Z = 15X11 + 13X12 + 14X13 + 17X14 + 11X21 + 12X22 + 15X23 + 13X24 + 13X31 + 12X33
+ 10X33 + 11X34 + 15X41 + 17X42 + 14X43 + 16X44
Subject to
X11 + X12 + X13 + X14 = 1 (one job should be given to 1 person)

E M Reddy Page | 57
AMA-Notes

X21 + X22 + X23 + X24 = 1 (one job should be given to 1 person)


X31 + X32 + X33 + X34 = 1 (one job should be given to 1 person)
X41 + X42 + X43 + X44 = 1 (one job should be given to 1 person)
X11 + X21 + X31 + X41 = 1 (one job person should not be given more than 1 assignment)
X12 + X22 + X32 + X42 = 1 (one job person should not be given more than 1 assignment)
X13 + X23 + X33 + X43 = 1 (one job person should not be given more than 1 assignment)
X14 + X24 + X34 + X44 = 1 (one job person should not be given more than 1 assignment)
Xij = (0,1)
2) The row operation is done to identify the efficient cell in each row. Efficient cells are those cells having
zero value. Those cells represent zero regret.
3) At the end of row operation, we ensure that each row has a zero but we cannot guarantee that each
column will have a zero because sometimes the same person may be efficient in more than one job and
another person not efficient in any job. For example, person 3 is good at job 3 and job 4 and person 4
not good at any job.
4) To have zeros in each column we perform column operation.
5) While allocating we always give preference to row or column having one zero because if we miss that
zero we do not have any more efficient point there.

3.3. Maximization balanced assignment problem – Hungarian Method

Question no 2: A manager has 4 subordinates and 4 tasks. The subordinates differ in efficiency His
estimate of the production each would do is given in the table. How the task should be allocated
one to one man, so that total production is maximized.
Subordinates
Task I II III IV
1 8 26 17 11
2 13 28 4 26
3 38 19 18 15
4 19 26 24 10

Solution:

Step 1: Conversion of maximization problem into Minimization (Regret matrix):


I II III IV
1 30 12 21 27
2 25 10 34 12
3 0 19 20 23
4 19 12 14 28

Notes:

1) Assignment steps are devised to solve minimization. Hence, a maximization problem should be
converted into minimization before applying assignment steps.
2) This can be done by taking the highest number in the matrix and reducing all the other numbers from it.
Due to this we convert the matrix into regret matrix and by minimizing regret we can maximize
production.

E M Reddy Page | 58
AMA-Notes

Step 2: Row Operation


I II III IV Row Operation
1 30 12 21 27 18 0 9 15
2 25 10 34 12 15 0 24 2
3 0 19 20 23 0 19 20 23
4 19 12 14 28 7 0 2 16

Step 3: Column Operation


I II III IV Column Operation
1 18 10 9 15 18 0 7 13
2 15 0 24 2 15 0 22 0
3 0 19 20 23 0 19 18 21
4 7 0 2 16 7 0 0 14

Step 4: Covering zeros with minimum number of lines


I II III IV
1 18 0 7 13
2 15 0 22 0
3 0 19 18 21
4 7 0 0 14

Step 5: Is number of lines equal to order of matrix?

Number of lines = 4
Order of Matrix = 4 x 4
Hence, we can proceed to make assignment.

Step 6: Assignment
I II III IV
1 18 ⓪ 7 13 i
2 15 ᴓ 22 ⓪ ii
3 ⓪ 19 18 21 iii
4 7 ᴓ ⓪ 14 iv

Step 7: Final Solution


Task Subordinates Production
1 II 26
2 IV 26
3 I 38
4 III 24

3.4. Minimization unbalanced assignment problem – Hungarian Method & Degeneracy

**Question no 3: A has one surplus truck in each city A, B, C, D & E and one deficit truck in each
of the cities 1, 2,3,4,5, & 6. The distance between the cities in kilometers is shown in the matrix
below.

E M Reddy Page | 59
AMA-Notes

Cities 1 2 3 4 5 6
A 12 10 15 22 18 8
B 10 18 25 15 16 12
C 11 10 3 8 5 9
D 6 14 10 13 13 12
E 8 12 11 7 3 10
Find the assignment of truck from the cities in surplus to cities in deficit so that the total distance
covered by vehicles in minimum?

Solution:

1) The above problem is unbalanced because we have 5 rows and 6 columns. We should first make the
matrix balanced by adding a dummy row. Always the value in the dummy row shall be ‘0’.
2) In this problem, dummy represents an imagined city and the distance between the imagined city and real
city is ‘0’ kilometers.

Step 1: Balanced matrix


1 2 3 4 5 6
A 12 10 15 22 18 8
B 10 18 25 15 16 12
C 11 10 3 8 5 9
D 6 14 10 13 13 12
E 8 12 11 7 3 10
F 0 0 0 0 0 0

Step 2: Row Operation


1 2 3 4 5 6
A 4 2 7 14 10 0
B 0 8 15 5 6 2
C 8 7 0 5 2 6
D 0 8 4 7 7 6
E 5 9 8 4 0 7
F 0 0 0 0 0 0

Note: Since every column has ‘0’ in it we need not do column operation because it results in the same
matrix.

Step 2: Covering zeros with minimum number of lines


1 2 3 4 5 6
A 4 2 7 14 10 0
B 0 8 15 5 6 2
C 8 7 0 5 2 6
D 0 8 4 7 7 6
E 5 9 8 4 0 7
F 0 0 0 0 0 0

E M Reddy Page | 60
AMA-Notes

Step 3: Is number of lines equal to order of matrix?

Number of lines = 5
Order of Matrix = 6 x 6
Since, it is not equal the situation is called degeneracy. This matrix needs improvement before proceeding
for assignment.

Step 4: Improvement 1

Identify least uncovered number (2 in this problem)


Elements

Covered Uncovred

Non-
Junctioned Less (-)
Juncitoned

Add (+) No Change

1 2 3 4 5 6
A 6 2 7 14 10 0
B 0 6 13 3 4 0
C 10 7 0 5 2 6
D 0 6 2 5 5 4
E 7 9 8 4 0 7
F 2 0 0 0 0 0

Step 5: Covering zeros with minimum number of lines


1 2 3 4 5 6
A 6 2 7 14 10 0
B 0 6 13 3 4 0
C 10 7 0 5 2 6
D 0 6 2 5 5 4
E 7 9 8 4 0 7
F 2 0 0 0 0 0

Step 6: Is number of lines equal to order of matrix?

Number of lines = 5
Order of Matrix = 6 x 6
Degeneracy continues. Hence, do further improvement.

Step 7: Improvement 2

E M Reddy Page | 61
AMA-Notes

1 2 3 4 5 6
A 6 0 5 12 8 0
B 0 4 11 1 2 0
C 12 7 0 5 2 8
D 0 4 0 3 3 4
E 9 9 8 4 0 9
F 4 0 0 0 0 2

Step 8: Covering zeros with minimum number of lines


1 2 3 4 5 6
A 6 0 5 12 8 0
B 0 4 11 1 2 0
C 12 7 0 5 2 8
D 0 4 0 3 3 4
E 9 9 8 4 0 9
F 4 0 0 0 0 2

Step 9: Is number of lines equal to order of matrix?

Number of lines = 6
Order of Matrix = 6 x 6
Hence, proceed make assignment.

Step 10: Assignment


1 2 3 4 5 6
A 6 ⓪ 5 12 8 ᴓ Iv
B ᴓ 4 11 1 2 ⓪ iii
C 12 7 ⓪ 5 2 8 i
D ⓪ 4 ᴓ 3 3 4 ii
E 9 9 8 4 ⓪ 9 v
F 4 ᴓ ᴓ ⓪ ᴓ 2 vi

Step 11: Final solution


Cities Cities Kilometers
A 2 10
B 6 12
C 3 3
D 1 6
E 5 3
F 4 0
Total Kilometers 34

Notes:

1) There are 5 cities having surplus trucks and 6 cities requiring trucks. Naturally one city should be denied
the truck. Which is that city?

E M Reddy Page | 62
AMA-Notes

a. It is city 4 because it is assigned dummy.


2) It should be understood that dummy represents rejection (both assignment and transportation
chapters).

3.5. Maximization unbalanced assignment problem – Hungarian Method

Question no 4: A management consulting firm has a backlog of 4 contracts. Work on these


contracts must be started immediately. 3 project leaders are available for assignment to the
contractors. Because of the varying work experience of the leaders, the profit to consulting firm will
vary based on the assignment as shown below. The unassigned contract can be completed by
subcontracting the work to an outside consultant. The profit on the subcontract is zero. Find out
optimal assignment.
Contract
Project Leader 1 2 3 4
A 13 10 9 11
B 15 17 13 20
C 6 8 11 7

Solution:

1) The problem is maximization unbalanced. Hence, conversion and balancing are involved.
2) What should be done first?
a. First Balance and then convert

Step 1: Balancing
13 10 9 11
15 17 13 20
6 8 11 7
0 0 0 0

Step 2: Conversion
7 10 11 9
5 3 7 0
14 12 9 13
20 20 20 20

Step 3: Row Operation


0 3 4 2
5 3 7 0
5 3 0 4
0 0 0 0

Step 4: Covering zeros with minimum number of lines


0 3 4 2
5 3 7 0

E M Reddy Page | 63
AMA-Notes

5 3 0 4
0 0 0 0

Step 5: Is number of lines equal to order of matrix?

Number of lines = 4
Order of Matrix = 4 x 4
Hence, proceed to make assignment.

Step 6: Assignment

⓪ 3 4 2 i
5 3 7 ⓪ ii
5 3 ⓪ 4 iii
ᴓ ⓪ ᴓ ᴓ iv

Step 7: Final Solution


Project Leader Contract Profit
A 1 13
B 4 20
C 3 11
D 2 0
Total Profit 44

There are 3 leaders and 4 contracts. Hence, one of the contract should be subcontracted and the assignment
says contract 2 to be subcontracted because it is assigned to dummy leader.

Question 5: WELLDONE company has taken the third floor of a multi-storyed building for rent
with a view to locate one of their zonal offices. There are five main rooms in this floor to be
assigned to five managers. Each room has its own advantages and disadvantages. Some have
windows, some are closer to the washrooms or to the canteen or secretarial pool. The rooms are of
all different sizes and shapes. Each of the five managers was asked to rank their room preferences
amongst the rooms 301, 302, 303, 304 and 305. Their preferences were recorded in a table as
indicated below:
Manager
𝐌𝟏 𝐌𝟐 𝐌𝟑 𝐌𝟒 𝐌𝟓
302 302 303 302 301
303 304 301 305 302
304 305 304 304 304
* 301 305 303 *
* * 302 * *
Most of the managers did not list all the five rooms since they were not satisfied with some of these
rooms and they have left off these from the list. Assuming that their preferences can be quantified
by numbers, find out as to which manager should be assigned to which room so that their total
preference ranking is a minimum.

Solution:

E M Reddy Page | 64
AMA-Notes

Step 1: Formulation

M1 M2 M3 M4 M5
301 - 4 2 - 1
302 1 1 5 1 2
303 2 - 1 4 -
304 3 2 3 3 3
305 - 3 4 2 -

The cells where no ranking is given like 301, M1 etc., are called prohibited cells. In those cells allocation
should not be made. Hence, we assign a very high cost ‘M’ to those sells and if it is maximization assign “-
M”.

M1 M2 M3 M4 M5
301 M 4 2 M 1
302 1 1 5 1 2
303 2 M 1 4 M
304 3 2 3 3 3
305 M 3 4 2 M

Step 2: Row Operation

M1 M2 M3 M4 M5
301 M 3 1 M 0
302 0 0 4 0 1
303 1 M 0 3 M
304 1 0 1 1 1
305 M 1 2 0 M

Step 3: Covering zeros with minimum number of lines

M1 M2 M3 M4 M5
301 M 3 1 M 0
302 0 0 4 0 1
303 1 M 0 3 M
304 1 0 1 1 1
305 M 1 2 0 M

Step 4: Is number of lines equal to order of matrix?

Number of lines = 5
Order of Matrix = 5 x 5
Hence, proceed to make assignment.

Step 5: Assignment

M1 M2 M3 M4 M5
301 M 3 1 M ⓪ i
302 ⓪ ᴓ 4 ᴓ 1 v

E M Reddy Page | 65
AMA-Notes

303 1 M ⓪ 3 M ii
304 1 ⓪ 1 1 1 iii
305 M 1 2 ⓪ M iv

Step 5: Final Solution


Room Manager
301 M5
302 M1
303 M3
304 M2
305 M4

Note: If in spite of assigning ‘M’ to the prohibited cell, if allocation is made then the problem is said to be
having infeasible solution.

Question no 6: An organization producing four different products A, B, C, D having four operators


P, Q, R, S who are capable of producing any of the four products work effectively 7 hours a day.
The time in minutes required for each operator for producing each of the product given in the cells
of the following matrix along with the profit in rupees per unit.
Operator Product
A B C D
P 6 10 14 12
O 7 5 3 4
R 6 7 10 10
S 20 10 15 15
Profit (Rs. /unit) 3 2 4 1
Find out the assignment of operators to products which will maximize the profit.

Solution:

Part 1: Formulation into profit matrix


A B C D
P 420
x3 = 210
420
x2 = 84
420
x4 = 120
420
x1 = 35
6 10 14 12
Q 420
x3 = 180
420
x2 = 168
420
x4 = 560
420
x1 = 105
7 5 3 4

R 420
x3 = 210
420
x2 = 120
420
x4 = 168
420
x1 = 42
6 7 10 10

S 420
x3 = 63
420
x2 = 84
420
x4 = 112
420
x1 = 28
20 10 15 15

A B C D
P 210 84 120 35
Q 180 168 560 105
R 210 120 168 42
S 63 84 112 28

E M Reddy Page | 66
AMA-Notes

Part 2: Conversion into Minimization


A B C D
P 350 476 440 525
Q 380 392 0 455
R 350 440 392 518
S 497 476 448 532

Part 3: Solution using assignment steps

Step 1: Row Operation


A B C D
P 0 126 90 175
Q 380 392 0 455
R 0 90 42 168
S 49 28 0 84

Step 2: Column Operation


A B C D
P 0 98 90 91
Q 380 364 0 371
R 0 62 42 84
S 49 0 0 0

Step 3: Covering zeros with minimum number of lines


A B C D
P 0 98 90 91
Q 380 364 0 371
R 0 62 42 84
S 49 0 0 0

Step 4: Is number of lines equal to order of matrix?

Number of lines = 3
Order of Matrix = 4 x 4
There is degeneracy. Hence, proceed for improvement.

Step 5: Improvement 1
A B C D
P 0 56 48 49
Q 422 364 0 371
R 0 20 0 42
S 91 0 0 0

Step 6: Covering zeros with minimum number of lines

E M Reddy Page | 67
AMA-Notes

A B C D
P 0 56 48 49
Q 422 364 0 371
R 0 20 0 42
S 91 0 0 0

Step 7: Is number of lines equal to order of matrix?

Number of lines = 3
Order of Matrix = 4 x 4
Degeneracy continuous. Do further improvement.

Step 8: Improvement 2
A B C D
P 0 36 48 29
Q 422 344 0 351
R 0 0 0 22
S 111 0 20 0

Step 9: Covering zeros with minimum number of lines


A B C D
P 0 36 48 29
Q 422 344 0 351
R 0 0 0 22
S 111 0 20 0

Step 10: Is number of lines equal to order of matrix?

Number of lines = 4
Order of Matrix = 4 x 4
Proceed to do assignment.

Step 11: Assignment


A B C D
P ⓪ 36 48 29 i
Q 422 344 ⓪ 351 ii
R ᴓ ⓪ ᴓ 22 iii
S 111 ᴓ 20 ⓪ iv

Step 12: Final Solution


Operator Product Profit (Rs.)
P A 210
Q C 560
R B 120
S D 28
Total Profit 918

E M Reddy Page | 68
AMA-Notes

3.6. Multiple Optimal Solution

**Question no 7: An airline operates seven days a week has time table as shown below. Crews must
have a minimum layover of 5 hours between flights. Obtain the pairing of flight that minimizes
layover time away from home. For any give pairing the crew will be based at the city that results in
smaller layover. For each pair also mention the town where the crew should be passed.
Delhi Jaipur Jaipur Delhi
Flight No. Departure Arrival Flight No. Departure Arrival
1 7.00 8.00 101 8.00 9.15
2 8.00 9.00 102 8.30 9.45
3 13.30 14.30 103 12.00 13.15
4 18.30 19.30 104 17.30 18.45

Solution:

Part 1: Layover times when the crew is based in Delhi


101 102 103 104
1 24 24.30 28 9.30
2 23 23.30 27 8.30
3 17.30 18 21 27
4 12.30 13 16.30 22

Part 2: Layover times when the crew is based in Jaipur


101 102 103 104
1 21.45 21.15 17.45 12.15
2 22.45 22.15 18.45 13.15
3 28.15 27.45 24.15 18.45
4 9.15 8.45 5.15 23.45

Part 3: Mixed Matrix (whichever is best i.e. least)


101 102 103 104
1 21.45-J 21.15-J 17.45-J 9.30-D
2 22.45-J 22.15-J 18.45-J 8.30-D
3 17.30-D 18-D 21-D 18.45-J
4 9.15-J 8.45-J 5.15-J 22-D

Part 4: Converting the matrix into whole numbers

Let 1 unit = ¼ hour. For example, 21.15 can be written as 85 [(21x4) +1]
101 102 103 104
1 87 85 71 38
2 91 89 75 34
3 70 72 86 75
4 37 35 21 88

E M Reddy Page | 69
AMA-Notes

Part 5: Solving the problem using the assignment steps

Step 1: Row Operation


101 102 103 104
1 49 47 33 0
2 57 55 41 0
3 0 2 16 5
4 16 14 0 67

Step 2: Column Operation


101 102 103 104
1 49 45 33 0
2 57 53 41 0
3 0 0 16 5
4 16 12 0 67

Step 3: Covering zeros with minimum number of lines


101 102 103 104
1 49 45 33 0
2 57 53 41 0
3 0 0 16 5
4 16 12 0 67

Step 4: Is number of lines equal to order of matrix?

Number of lines = 3
Order of Matrix = 4 x 4
Degeneracy exists. Hence, do improvement.

Step 5: Improvement 1
101 102 103 104
1 16 12 0 0
2 24 20 8 0
3 0 0 16 38
4 16 12 0 100

Step 6: Covering zeros with minimum number of lines


101 102 103 104
1 16 12 0 0
2 24 20 8 0
3 0 0 16 38
4 16 12 0 100

Step 7: Is number of lines equal to order of matrix?

E M Reddy Page | 70
AMA-Notes

Number of lines = 3
Order of Matrix = 4 x 4
Degeneracy continuous. Do further improvement.

Step 8: Improvement 2
101 102 103 104
1 4 0 0 0
2 12 8 8 0
3 0 0 28 50
4 4 0 0 100

Step 9: Covering zeros with minimum number of lines


101 102 103 104
1 4 0 0 0
2 12 8 8 0
3 0 0 28 50
4 4 0 0 100

Step 10: Is number of lines equal to order of matrix?

Number of lines = 4
Order of Matrix = 4 x 4
There is no Degeneracy. Proceed for assignment

Step 11: Assignment


101 102 103 104
1 4 ⓪ ᴓ ᴓ iii
2 12 8 8 ⓪ i
3 ⓪ ᴓ 28 50 ii
4 4 ᴓ ⓪ 100 iv

Step 12: Final Solution


Flight No. Paired Flight No. Place Layover Time
1 102 Jaipur 21.15
2 104 Delhi 8.30
3 101 Delhi 17.30
4 103 Jaipur 5.15
Total Layover time 52.30

Step 13: Alternate assignment


101 102 103 104
1 4 ᴓ ⓪ ᴓ iii
2 12 8 8 ⓪ i
3 ⓪ 0 28 50 ii

E M Reddy Page | 71
AMA-Notes

4 4 ⓪ ᴓ 100 iv

Step 14: Final Solution


Flight No. Paired Flight No. Place Layover Time
1 103 Jaipur 17.45
2 104 Delhi 8.30
3 101 Delhi 17.30
4 102 Jaipur 8.45
Total Layover time 52.30

Notes:

1) In step 6, we had two rows with two ‘0’s (Row 1 & Row 3) and two columns with two ‘0’s (Column 3 &
Column 4). As per the procedure since there is a tie we can draw either against the columns or against
the rows.
2) We decided to draw against the columns and ultimately covered all the zeros with 3 lines. Had we
chosen rows we would have covered using 4 lines. This is a small drawback in the procedure.
3) In the final assignment we had two ‘0’s to choose from i.e. cell (1,102) or cell (1,103). Either cell if we
choose we will get the same minimum layover time. Thus the problem is having multiple solution.

**Question no 8: A travelling salesman has to visit 5 cities. He wishes to start from a particular city,
visit each city once and return to his starting point. The travelling cost for each city from a
particular city is given below:
F To City
r A B C D E
o A X 4 7 3 4
m B 4 X 6 3 4
C C 7 6 X 7 5
it D 3 3 7 X 7
y E 4 4 5 7 X
What is the sequence of visit of the salesman, so that the cost is minimum?

Solution:

Part 1: Assigning ‘M’ to prohibited cells


A B C D E
A M 4 7 3 4
B 4 M 6 3 4
C 7 6 M 7 5
D 3 3 7 M 7
E 4 4 5 7 M

Note: There is no travel possible from ‘A’ to ‘A’, ‘B’ to ‘B’, ‘C’ to ‘C’, ‘D’ to ‘D’ and ‘E’ to ‘E’. Hence, these
cells are prohibited cells and a very high penalty imposed on it.

Part 2: Solving the problem using assignment steps

E M Reddy Page | 72
AMA-Notes

Step 1: Row Operation


A B C D E
A M 1 4 0 1
B 1 M 3 0 1
C 2 1 M 2 0
D 0 0 4 M 4
E 0 0 1 3 M

Step 2: Column Operation


A B C D E
A M 1 3 0 1
B 1 M 2 0 1
C 2 1 M 2 0
D 0 0 3 M 4
E 0 0 0 3 M

Step 3: Covering zeros with minimum number of lines


A B C D E
A M 1 3 0 1
B 1 M 2 0 1
C 2 1 M 2 0
D 0 0 3 M 4
E 0 0 0 3 M

Step 4: Is number of lines equal to order of matrix?

Number of lines = 4
Order of Matrix = 5 x 5
Degeneracy exists. Hence, do improvement.

Step 5: Improvement 1
A B C D E
A M 0 2 0 0
B 0 M 1 0 0
C 2 1 M 3 0
D 0 0 3 M 4
E 0 0 0 4 M

Step 6: Covering zeros with minimum number of lines


A B C D E
A M 0 2 0 0
B 0 M 1 0 0
C 2 1 M 3 0
D 0 0 3 M 4
E 0 0 0 4 M

E M Reddy Page | 73
AMA-Notes

Step 7: Is number of lines equal to order of matrix?

Number of lines = 5
Order of Matrix = 5 x 5
There is no Degeneracy. Hence, proceed for assignment.

Step 8: Assignment
A B C D E
A M ⓪ 2 ᴓ ᴓ iv
B ᴓ M 1 ⓪ ᴓ iii
C 2 1 M 3 ⓪ i
D ⓪ ᴓ 3 M 4 v
E ᴓ ᴓ ⓪ 4 M ii

Step 9: Final Solution as an assignment problem


City City Assigned Cost
A B 4
B D 3
C E 5
D A 3
E C 5
Total Cost 20

Step 10: Grouping Condition

Loops:

A→B→D→A and C→E→C

Note:

The assignment should form a loop. The loop would ensure the terminal point and the starting point as
same. In the initial basic solution two loops were formed for a total cost of Rs.20 whereas the problem
precondition is to form a single loop. Therefore, an improvement is to be sought.

Step 11: Improved Solution

Break the smaller loop of C→E→C i.e. do not assign against ‘0’ in row C (in 1st iteration). Instead second
lowest value of ‘1’ shall be assigned that comes against column ‘B’. The overall cost is likely to exceed
marginally. But the sequencing condition is complied.
A B C D E
A M ᴓ 2 ᴓ ⓪ iv
B ᴓ M 1 ⓪ ᴓ v
C 2 ① M 3 ᴓ i
D ⓪ ᴓ 3 M 4 iii

E M Reddy Page | 74
AMA-Notes

E ᴓ ᴓ ⓪ 4 M ii

Step 12: Final Solution


City City Assigned Cost
A E 3
B D 3
C B 6
D A 3
E C 5
Total Cost 21

Sequence: A→E→C→B→D→A

Question no 9: Suppose there exists a 4X4 matrix having ‘0’s in all the cells after row and column
operation. How many solutions are possible?

Solution:

No. of Possible solutions = 4! = 4 x 3 x 2 x 1 = 24

E M Reddy Page | 75
AMA-Notes

4. TRANSPORTATION

4.1. Introduction

1) Features:
a) A type of linear programming problem
b) It is all about matching demand and supply
c) No need to have one to one relationship i.e. a factory can supply to multiple warehouses or a
warehouse can take from multiple factories.
d) Like assignment transportation steps also can solve only minimization types
2) Stages in solving a transportation problem:
a) Obtaining Initial basic feasible solution (IBFS)
i. Northwest corner method
ii. Lease cost method
iii. Vogel’s approximation method
b) Testing for optimality – Moody’s optimality test

4.2. Minimization Balanced

Question no 1: Obtain the IBFS (Intima Basic Feasible Solution) for the following & also
determine whether they satisfy the optimality test.
W-1 W-2 W-3 Supply
Factory 1 6 8 4 14
Factory 2 4 9 8 12
Factory 3 1 2 6 5
Demand 6 10 15

Solution:

Stage 1: Obtaining Initial Basic Feasible Solution (IBFS)

4.2.1. Northwest Corner Method

W-1 W-2 W-3 Availability


Factory 1 6 8 14/8/0
6 8 4 6 = supplied
Factory 2 2 10 12/10/0 (Bold letters
4 9 8 are supplied
Factory 3 5 5/0 quantity)
1 2 6
Requirement 6/0 10/2/0 15/5/0 31

Cost associated with the above solution:


Allocated Cells Computation Cost (Rs.)
F1 – W1 6 Units X Rs.6 36
F1 – W2 8 Units X Rs.8 64
F2 – W2 2 Units X Rs.9 18

E M Reddy Page | 76
AMA-Notes

F2 – W3 10 Units X Rs.8 80
F3 – W3 5 Units X Rs.6 30
Total transportation cost 228

4.2.2. Least Cost Method

W-1 W-2 W-3 Availability


Factory 1 14 14/8/0
6 8 4
Factory 2 1 10 1 12/11/10/0
4 9 8
Factory 3 5 5/0
1 2 6
Requirement 6/1/0 10/0 15/1/0 31

Cost associated with the least cost method solution:


Allocated Cells Computation Cost (Rs.)
F1 – W3 14 Units X Rs.4 56
F2 – W1 1 Units X Rs.4 4
F2 – W2 10 Units X Rs.9 90
F2 – W3 1 Units X Rs.8 8
F3 – W1 5 Units X Rs.1 5
Total Transportation Cost 163

4.2.3. Vogel’s approximation method

W-1 W-2 W-3 Availability I II III


Factory 1 14 14/0 6–4=2 6–4=2 8–4=4
6 8 4
Factory 2 6 5 1 12/6/5/0 8–4=4 8–4=4 9–8=1
4 9 8
Factory 3 5 5/0 2 – 1= 1 - -
1 2 6
Requirement 6/0 10/5 15/1 31
I 4 – 1= 3 8–2=6 6–4=2
II 6–4=4 9–8=1 8 – 4= 4
III - 9–8=1 8 – 4= 4

Cost associated with the Vogel’s solution:


Allocated Cells Computation Cost (Rs.)
F1 – W3 14 Units X Rs.4 56
F2 – W1 6 Units X Rs.4 24
F2 – W2 5 Units X Rs.9 45
F2 – W3 1 Units X Rs.8 8
F3 – W2 5 Units X Rs.2 10
Total Transportation Cost 143

E M Reddy Page | 77
AMA-Notes

Notes:

1) North West corner method is the worst method because in selecting the cells ‘cost’ is not considered.
2) Between Least Cost and Vogel, the later is better because Least Cost Method allocates in absolute least
cost cell while Vogel allocates in relative least cost cell.
3) For example, Least Cost Method made its first allocation in the lowest cost cell in the entire matrix
which is F3 – W1 having Rs.1 cost. On the other hand, Vogel allocated in F3 – W12 having a cost of
Rs.2 because the regret of missing that cell and moving to next least cell in that column (F1 – W2) is
Rs.6 (Rs.8 – Rs.2) which is higher than the regret of Rs.3 for F3 – W1.

Stage 2: Checking for Degeneracy

Initial Basic Feasible Solution:


14 14
6 8 4
6 5 1 12
4 9 8
5 5
1 2 6
6 10 15 31

Number of allocated cells = 5


M + N – 1 = 3 + 3 – 1 = 5; where ‘M’ is number of rows and ‘N’ is number of columns.
Since both are equal we can proceed for optimality test.

Stage 3: Optimality test

Step 1: Allocated cells


Allocated Cells Cost
U1+V3= 4
U2+V1= 4
U2+V2= 9
U2+V3= 8
U3+V2= 2

Step 2: Values

Assume U1 value as ‘0’ and the remaining values will be as follows:


U1 = 0, U2 = 4, U3 = -3, V1 = 0, V2 = 5, V3 = 4

Step 3: Unallocated Cells


Unallocated Cells 𝐂𝐣 𝐙𝐣 𝐂𝐣 − 𝐙𝐣
U1+V1= 6 0 6
U1+V2= 8 5 3
U3+V1= 1 -3 4
U3+V3= 6 1 5

E M Reddy Page | 78
AMA-Notes

Since all the numbers in Cj − Zj are positive the current solution is optimal.

Notes:

1) The Number of variables in the optimality test will be ‘M+N’. In which we assume one variable to be
‘0’. Hence we need to find out the values for ‘M+N-1’ variables for which we required ‘M+N-1’
equations. Since allocated cells becomes equations in the optimality test, the number of allocated cells
should necessarily be ‘M+N-1’.
2) How to understand Cj − Zj ?
a. ‘Cj − Zj ’ value of the unallocated cell U1V1 is ‘6’. This means if this cell is made allocated for
every allocation made the cost increases by Rs.6.
b. The above can be understood as follows:
(+) 1 (-) 13 14
6 8 4
(-) 5 5 (+) 2 12
4 9 8
5 5
1 2 6
6 10 15 31

Additional Cost = 6 – 4 + 8 – 4 = 6 which is Cj − Zj ,


New allocation cost (Cj ) = 6
Change in existing allocation (Zj ) = – 4 + 8 – 4 = 0
c. If there exists any negative number improvement is made because there exists scope for
reduction of cost.

4.3. Minimization Balanced - Degeneracy

Question no 2: Find the optimal solution for the following problem.


W-1 W-2 W-3 Supply
Factory 1 50 30 220 1
Factory 2 30 45 170 3
Factory 3 250 200 50 4
Demand 4 2 2

Solution:

Stage 1: Obtaining Initial Basic Feasible Solution (IBFS) using Vogel’s Method
W-1 W-2 W-3 Availability I II
Factory 1 1 1/0 20 20
50 30 220
Factory 2 3 3/0 15 15
30 45 170
Factory 3 2 2 4/2/0 150 50
250 200 50
Requirement 4/1/0 2/0 2/0 8

E M Reddy Page | 79
AMA-Notes

I 20 15 120
II 20 15 -

Cost associated with the Vogel’s solution:


Allocated Cells Computation Cost (Rs.)
F1 – W1 1 Units X Rs.50 50
F2 – W1 3 Units X Rs.30 90
F3 – W2 2 Units X Rs.200 400
F3 – W3 2 Units X Rs.50 100
Total Transportation Cost 640

Stage 2: Checking for Degeneracy

Number of allocated cells = 4


M + N – 1 = 3 + 3 – 1 = 5; where ‘M’ is number of rows and ‘N’ is number of columns.
We cannot proceed for optimality test because number of allocated cells are not equal to ‘M+N-1’. This is
referred to as ‘Degeneracy’.
The number of allocated cells should be made as ‘5’. This should be done by allocating an infinitely small
quantity ‘e’ in the LEAST COST UNALLOCATED INDEPENDEDNT CELL.
W-1 W-2 W-3 Availability
Factory 1 1 e 1
50 30 220
Factory 2 3 3
30 45 170
Factory 3 2 2 4
250 200 50
Requirement 4 2 2 8

Stage 3: Optimality test

Step 1: Allocated cells


Allocated Cells Cost
U1+V1= 50
U1+V2= 30
U2+V1= 30
U3+V2= 200
U3+V3= 50

Step 2: Values

Assume U1 value as ‘0’ and the remaining values will be as follows:


U1 = 0, U2 = -20, U3 = 170, V1 = 50, V2 = 30, V3 = -120

Step 3: Unallocated Cells


Unallocated Cells 𝐂𝐣 𝐙𝐣 𝐂𝐣 − 𝐙𝐣

E M Reddy Page | 80
AMA-Notes

U1+V3= 220 -120 340


U2+V2= 45 10 35
U2+V3= 170 -140 310
U3+V1= 250 220 30

Since there is no negative number in Cj − Zj column, there is no scope for improvement and the current
solution is optimal.

4.4. Maximization Unbalanced and Improvement through Looping

**Question no 3: Consider the following transportation profit table and determine the optimal
solution.
W-1 W-2 W-3 W-4 Supply
Factory 1 40 25 22 33 100
Factory 2 44 35 30 30 30
Factory 3 38 38 28 30 70
Demand 40 20 60 30

Solution:

Step 1: Balancing
W-1 W-2 W-3 W-4 W-5 Supply
Factory 1 40 25 22 33 0 100
Factory 2 44 35 30 30 0 30
Factory 3 38 38 28 30 0 70
Demand 40 20 60 30 50 200

Step 2: Conversion into minimization and obtaining IBFS using Vogel


W-1 W-2 W-3 W-4 W-5 Availability I II III IV V
Factory 1 20 30 50 100/70 7 7 7 11 22
4 19 22 11 44
Factory 2 30 30/0 9 - - - -
0 9 14 14 44
Factory 3 10 20 40 70/50/40/0 0 0 8 2 28
6 6 16 14 44
Requirement 40/10/0 20/0 60/20 30/0 50 200
I 4 3 2 3 0
II 4 13 6 3 0
III 2 - 6 3 0
IV - - 6 3 0
V - - 6 - 0

Cost associated with the Vogel’s solution:


Allocated Cells Computation Cost (Rs.)
F1 – W3 20 Units X Rs.22 440
F1 – W4 30 Units X Rs.11 330

E M Reddy Page | 81
AMA-Notes

F1 – W5 50 Units X Rs.44 2200


F2 – W1 30 Units X Rs.0 0
F3 – W1 10 Units X Rs.6 60
F3 – W2 20 Units X Rs.6 120
F3 – W3 40 Units X Rs.16 640
Total Cost 3790

Step 2: Checking for degeneracy

Initial Basic Feasible Solution:


W-1 W-2 W-3 W-4 W-5 Availability
Factory 1 20 30 50 100
4 19 22 11 44
Factory 2 30 30
0 9 14 14 44
Factory 3 10 20 40 70
6 6 16 14 44
Requirement 40 20 60 30 50 200

Number of allocated cells = 7


M+N–1=3+5–1=7
Since there is no degeneracy we can proceed for optimality test.

Step 3: Moody’s optimality test

A: Allocated cells
Allocated Cells Cost
U1+V3= 22
U1+V4= 11
U1+V5= 44
U2+V1= 0
U3+V1= 6
U3+V2= 6
U3+V3= 16

B: Values

Assume U1 value as ‘0’ and the remaining values will be as follows:


U1 = 0, U2 = -12, U3 = -6, V1 = 12, V2 = 12, V3 = 22, V4 = 11, V5 = 44

C: Unallocated Cells
Unallocated Cells 𝐂𝐣 𝐙𝐣 𝐂𝐣 − 𝐙𝐣
U1+V1= 4 12 -8
U1+V2= 19 12 7
U2+V2= 9 0 9
U2+V3= 14 10 4

E M Reddy Page | 82
AMA-Notes

U2+V4= 14 -1 15
U2+V5= 44 32 8
U3+V4= 14 5 9
U3+V5= 44 38 6

The solution given by Vogel is not optimal and improvement is possible because one of the unallocated cell
U1V1 had -8 column in Cj − Zj i.e. if we make that cell as allocated, for every allocation made the cost
decreases by Rs.8.

Step 4: Improvement through looping

A: Looping Rules

1) Start from most negative cell


2) Use only Horizontal or Vertical Lines
3) Terminal point of the lines used should be in allocated cells
4) Start and complete by retiring into the same place of start. This is loop
5) Assign ‘+’ & ‘-ve’ from the start alternatively.
a. Choose from the quantities where ‘-ve’ were assigned.
6) Choose the lowest allocation for reshuffling

B: Improvement through looping

Quantity allocated in ‘-ve’ cells is 10 & 20. Least of them i.e. 10 Units should be made as new allocation.

Note: Suppose we allocate 11 units in the new cell U1V1, we will end up with a negative allocation (-1) in
the cell U3V1. Hence, the maximum allocation possible is 10 units. This is similar selecting least
Replacement Raito (RR) in simplex table.

Improved Solution:

E M Reddy Page | 83
AMA-Notes

Cost associated with improved solution:


Allocated Cells Computation Cost (Rs.)
F1 – W1 10 Units X Rs.4 40
F1 – W3 10 Units X Rs.22 220
F1 – W4 30 Units X Rs.11 330
F1 – W5 50 Units X Rs.44 2200
F2 – W1 30 Units X Rs.0 0
F3 – W2 20 Units X Rs.6 120
F3 – W3 50 Units X Rs.16 800
Total Cost 3710

The regret has reduced from 3790 in Vogel solution to 3710 in improved solution i.e. it has decreased by
Rs.80 because 10 units of new allocation results in cost reduction of 80 (10 Units x 8).

Step 5: Moody’s optimality test

A: Allocated cells
Allocated Cells Cost
U1+V1= 4
U1+V3= 22
U1+V4= 11
U1+V5= 44
U2+V1= 0
U3+V2= 6
U3+V3= 16

B: Values

Assume U1 value as ‘0’ and the remaining values will be as follows:


U1 = 0, U2 = -4, U3 = -6, V1 = 4, V2 = 12, V3 = 22, V4 = 11, V5 = 44

C: Unallocated Cells
Unallocated Cells 𝐂𝐣 𝐙𝐣 𝐂𝐣 − 𝐙𝐣
U1+V2= 19 12 7
U2+V2= 9 8 1
U2+V3= 14 18 -4
U2+V4= 14 7 7
U2+V5= 44 40 4
U3+V1= 6 -2 8
U3+V4= 14 5 9
U3+V5= 44 38 6

The above solution is not optimal. Hence, do further improvement.

Step 6: Improvement through looping

E M Reddy Page | 84
AMA-Notes

Quantity allocated in ‘-ve’ cells is 10 & 30. Least of them i.e. 10 Units should be made as new allocation.

Improved Solution:
W-1 W-2 W-3 W-4 W-5 Availability
Factory 1 20 30 50 100
4 19 22 11 44
Factory 2 20 10 30
0 9 14 14 44
Factory 3 20 50 70
6 6 16 14 44
Requirement 40 20 60 30 50 200

Cost associated with improved solution:


Allocated Cells Computation Cost (Rs.)
F1 – W1 20 Units X Rs.4 80
F1 – W4 30 Units X Rs.11 330
F1 – W5 50 Units X Rs.44 2200
F2 – W1 20 Units X Rs.0 0
F2 – W3 10 Units X Rs.14 140
F3 – W2 20 Units X Rs.6 120
F3 – W3 50 Units X Rs.16 800
Total Cost 3670

The regret has reduced from 3710 in improved solution to 3671 in further improved solution i.e. it has
decreased by Rs.40 because 10 units of new allocation results in cost reduction of 40 (10 Units x 4).

Step 7: Moody’s optimality test

A: Allocated cells
Allocated Cells Cost
U1+V1= 4
U1+V4= 11
U1+V5= 44
U2+V1= 0
U2+V3= 14
U3+V2= 6
U3+V3= 16

E M Reddy Page | 85
AMA-Notes

B: Values

Assume U1 value as ‘0’ and the remaining values will be as follows:


U1 = 0, U2 = -4, U3 = -2, V1 = 4, V2 = 8, V3 = 18, V4 = 11, V5 = 44

C: Unallocated Cells
Unallocated Cells 𝐂𝐣 𝐙𝐣 𝐂𝐣 − 𝐙𝐣
U1+V2= 19 8 11
U1+V3= 22 18 4
U2+V2= 9 4 5
U2+V4= 14 7 7
U2+V5= 44 40 4
U3+V1= 6 2 8
U3+V4= 14 9 5
U3+V5= 44 42 2

The above solution is optimal.

Step 8: Final Solution


Allocated Cells Computation Profit (Rs.)
F1 – W1 20 Units X Rs.40 800
F1 – W4 30 Units X Rs.33 990
F1 – W5 50 Units X Rs.0 0
F2 – W1 20 Units X Rs.44 880
F2 – W3 10 Units X Rs.30 300
F3 – W2 20 Units X Rs.38 760
F3 – W3 50 Units X Rs.28 1400
Total Profit 5130

Important Notes:

1) Suppose there is a tie in the negative cells quantities. For example, instead of 10 and 20 we have 10 and
10, then by making new allocation two existing old allocation will be replaced against degeneracy will
arise. Resolve the degeneracy before proceeding for the optimality test for the improved solution. Thus
degeneracy may occur in two stages:
i. During IBFS (Initial Basic Feasible Solution) – Vogel’s Approximation
ii. During Moody’s improvement
2) What happens if you are not able to form a loop during improvement?
Answer: Such situation will never arise because the procedures ensure that all unallocated cells during
optimality test are depended cells. That is reason why during degeneracy we make ‘e’ allocation in
LEAST COST INDEPENDEDN UNALLOCATED CELL. So that no independent unallocated cell
remains.
3) If one of the negative cell has ‘e’ quantity the reshuffling will change the allocations but will not change
the cost, should we do the reshuffling?
Answer: Yes, because the rearranged matrix will change UiVj values highlighting more possible
improvements.

E M Reddy Page | 86
AMA-Notes

4) Can we make ‘e’ allocation in dummy cell?


Answer: Yes
5) Will there be two ‘e’ allocations?
Answer: Yes, it is possible.

4.5. Multiple Optimal Solution

Question no 4: Solve the following transportation problem.


W-1 W-2 W-3 W-4 Supply
Factory 1 5 3 6 2 19
Factory 2 4 7 9 1 37
Factory 3 3 4 7 5 34
Demand 16 18 31 25

Solution:

Step 1: Obtaining IBFS using Vogel’s method


W-1 W-2 W-3 W-4 Availability I II III IV
Factory 1 18 1 19/1/0 1 2 2 1
5 3 6 2
Factory 2 12 25 37/12/0 3 3 - -
4 7 9 1
Factory 3 4 30 34/30/0 1 1 1 4
3 4 7 5
Requirement 16/4/0 18/0 31/30/0 25/0 200
I 1 1 1 1
II 1 1 1 -
III 2 1 1 -
IV 2 - 1 1

Cost associated with the Vogel’s solution:


Allocated Cells Computation Cost (Rs.)
F1 – W2 18 Units X Rs.3 54
F1 – W3 1 Units X Rs.6 6
F2 – W1 12 Units X Rs.4 48
F2 – W4 25 Units X Rs.1 25
F3 – W1 4 Units X Rs.3 12
F3 – W3 30 Units X Rs.7 210
Total Cost 355

Step 2: Checking for degeneracy

Initial Basic Feasible Solution:


W-1 W-2 W-3 W-4 Availability
Factory 1 18 1 19
5 3 6 2

E M Reddy Page | 87
AMA-Notes

Factory 2 12 25 37
4 7 9 1
Factory 3 4 30 34
3 4 7 5
Requirement 16 18 31 25 90

Number of allocated cells = 6


M+N–1=3+4–1=6
Since there is no degeneracy we can proceed for optimality test.

Step 3: Moody’s optimality test

A: Allocated cells
Allocated Cells Cost
U1+V2= 3
U1+V3= 6
U2+V1= 4
U2+V4= 1
U3+V1= 3
U3+V3= 7

B: Values

Assume U1 value as ‘0’ and the remaining values will be as follows:


U1 = 0, U2 = 2, U3 = 1, V1 = 2, V2 = 3, V3 = 6, V4 = -1

C: Unallocated Cells
Unallocated Cells 𝐂𝐣 𝐙𝐣 𝐂𝐣 − 𝐙𝐣
U1+V1= 5 2 3
U1+V4= 2 -1 3
U2+V2= 7 5 2
U2+V3= 9 8 1
U3+V2= 4 4 0
U3+V4= 5 0 5

The above solution is optimal because there are no negativity numbers to give improvement. But there
exists another optimal solution i.e. if we make U3V2 allocated cell, for every allocation made the cost
changes by Rs.0 i.e. it does not change. Hence the problem has multiple optimal solution.

Step 4: Alternative Solution – Checking the multiple solution


W-1 W-2 W-3 W-4 Availability
Factory 1 (-) 18 (+) 1 19
5 3 6 2
Factory 2 12 25 37
4 7 9 1

E M Reddy Page | 88
AMA-Notes

Factory 3 4 + (-) 30 34
3 4 7 5
Requirement 16 18 31 25 90

Quantity allocated in ‘-ve’ cells is 18 & 30. Least of them i.e. 18 Units should be made as new allocation.

Improved Matrix – Reshuffled Matrix:


W-1 W-2 W-3 W-4 Availability
Factory 1 19 19
5 3 6 2
Factory 2 12 25 37
4 7 9 1
Factory 3 4 18 12 34
3 4 7 5
Requirement 16 18 31 25 90

Cost associated with the alternative solution:


Allocated Cells Computation Cost (Rs.)
F1 – W3 19 Units X Rs.6 104
F2 – W1 12 Units X Rs.4 48
F2 – W4 25 Units X Rs.1 25
F3 – W1 4 Units X Rs.3 12
F3 – W2 18 Units X Rs.4 72
F3 – W3 12 Units X Rs.7 84
Total Cost 355

It is proved that there exists another solution with the same 355 cost.

Question no 5: The following table gives the unit transportation costs and the quantities demanded
/supplied at different locations for a minimization problem:
C1 C2 C3 C4 Total Units
R1 100 120 200 110 20000
R2 160 80 140 120 38000
R3 180 140 60 100 16000
Total Units 10000 18000 22000 24000
You are required to find out which cell gets the 3rd allocation in the initial basic feasible solution
under each of the following methods and to give the cell reference, cost per unit of that cell and the
quantity allocated to that cell:
(i) North west corner rule
(ii) Vogel’s Approximation Method
(iii) Least Cost Method

Solution:

North West corner method:


C1 C2 C3 C4 Supply

E M Reddy Page | 89
AMA-Notes

R1 10000 10000 20000/10000/0


100 120 200 110
R2 8000 38000/30000
160 80 140 120
R3 16000
180 140 60 100
Demand 10000/0 18000/8000/0 22000 24000 74000

In North West corner method is in the cell is R2C2 with a quantity of 8000 units at a cost Rs.80 per unit.

Vogel’s Method:
C1 C2 C3 C4 Supply I II III
R1 10000 20000/10000 10 10 10
100 120 200 110
R2 6000 38000 40 40 40
160 80 140 120
R3 16000 16000/0 40 - -
180 140 60 100
Demand 10000/0 18000 22000/6000/0 24000 74000
I 60 40 80 10
II 60 40 60 10
III - 40 60 10

In Vogel the 3rd allocation is R2C3 and quantity allocated is 6000 units with a cost of Rs.140 per unit.

Least Cost method:


C1 C2 C3 C4 Supply
R1 10000 20000/10000
100 120 200 110
R2 18000 38000/20000
160 80 140 120
R3 16000 16000/0
180 140 60 100
Demand 10000/0 18000/0 22000/6000 24000 74000

In this method the 3rd allocation is made in R1C1 and quantity allocated 10000 units and the cost is Rs.100
per unit.

Question no 6: The following matrix is a minimization problem for transportation cost. The unit
transportation costs are given the right hand corners of the cells and ∆ij values are encircled.
Destination Supply
Factory
D1 D2 D3 Units
F1 3 4 4 500
F2 ⑧ 9 300 6 ② 7 300
F3 ⓪ 4 ② 6 200 5 200
Demand 300 400 300 1000
Find the optimum solution (s) and the minimum cost?

E M Reddy Page | 90
AMA-Notes

Solution:
Destination Supply
Factory Bold = Allocated quantity
D1 D2 D3 Units
F1 300 3 100 4 100 4 500 Circled numbers = Zj - Zj for
F2 ⑧ 9 300 6 ② 7 300 that cell
F3 ⓪ 4 ② 6 200 5 200
Demand 300 400 300 1000

Cost associated with the above solution:


Allocated Cell Computation Cost (Rs.)
F1 – D1 300 Units X Rs.3 900
F1 – D2 100 Units X Rs.4 400
F1 – D3 100 Units X Rs.4 400
F2 – D2 300 Units X Rs.6 1800
F3 – D3 200 Units X Rs.5 1000
Total Cost 3700

Notes:

1) ∆ij indicates the change in the cost that occurs when an unallocated cell in ith row and jth column is made
allocated. It is nothing but Cj − Zj calculation.
2) Hence all the cells having the circled numbers are unallocated cells.
3) The solution is optimal because there exists no negative number to go for improvement.
4) The solution is multiple optimal even if cell U3V1 is made allocated the cost does not change because of
its ‘0’ ∆ij value.

Alternate solution or Improvement:

Make U3V1 as allocated cell.


Destination Supply
Factory
D1 D2 D3 Units
F1 300 (-) 3 100 4 100 (+) 4 500
F2 ⑧ 9 300 6 ② 7 300
F3 ⓪ (+) 4 ② 6 200 (-) 5 200
Demand 300 400 300 1000

Improved Solution:
Destination Supply
Factory
D1 D2 D3 Units
F1 100 3 100 4 400 4 500
F2 ⑧ 9 300 6 ② 7 300
F3 200 4 ② 6 ⓪ 5 200
Demand 300 400 300 1000

Cost associated with the above solution:

E M Reddy Page | 91
AMA-Notes

Allocated Cell Computation Cost (Rs.)


F1 – D1 100 Units X Rs.3 300
F1 – D2 100 Units X Rs.4 400
F1 – D3 100 Units X Rs.4 400
F2 – D2 300 Units X Rs.6 1800
F3 – D1 200 Units X Rs.4 800
Total Cost 3700

4.6. Formulation

Question no 7: Madhav ltd. has decided to launch an addition to its product range. The new
product may be distributed through any combination of the two company warehouse W1 and W2.
The available annual production capacities for the new product are:
100 Units at Plant P1
200 Units at Plant P2
300 Units at Plant P3
The three major concentrations of customer demand are at locations D1, D2 and D3 which are
estimated to require each year:
90 Units at D1
80 Units at D2
90 Units at D3
The unit production costs amount to 3, 4 and 1 at P1, P2 and P3 respectively. The unit handling
costs at the warehouse amount to 2 and 3 at W1 and W2 respectively.
The unit transportation costs from plant to warehouse and unit delivery costs from warehouse to
customer are as follows:
W1 W2 D1 D2 D3
P1 6 6 W1 3 5 8
P2 5 5 W2 5 3 9
P3 13 4
Required:
Determine an optimum production and distribution schedule.
Solution:
Step 1: Cost when routed through warehouse 1
Particulars Production Transport Handling Delivery Total
Cost (Rs.) Cost (Rs.) Cost (Rs.) Cost (Rs.) Cost (Rs.)
P1 – W1 – D1 3 6 2 3 14
P1 – W1 – D2 3 6 2 5 16
P1 – W1 – D3 3 6 2 8 19
P2 – W1 – D1 4 5 2 3 14
P2 – W1 – D2 4 5 2 5 16
P2 – W1 – D3 4 5 2 8 19
P3 – W1 – D1 1 13 2 3 19
P3 – W1 – D2 1 13 2 5 21
P3 – W1 – D3 1 13 2 8 24

E M Reddy Page | 92
AMA-Notes

D1 D2 D3 Supply
P1 14 16 19 100
P2 14 16 19 200
P3 19 21 24 100
Demand 90 80 90

Step 2: Cost when routed through warehouse 2


Particulars Production Transport Handling Delivery Total
Cost (Rs.) Cost (Rs.) Cost (Rs.) Cost (Rs.) Cost (Rs.)
P1 – W2 – D1 3 6 3 5 17
P1 – W2 – D2 3 6 3 3 15
P1 – W2 – D3 3 6 3 9 21
P2 – W2 – D1 4 5 3 5 17
P2 – W2 – D2 4 5 3 3 15
P2 – W2 – D3 4 5 3 9 21
P3 – W2 – D1 1 4 3 5 13
P3 – W2 – D2 1 4 3 3 11
P3 – W2 – D3 1 4 3 9 17
D1 D2 D3 Supply
P1 17 15 21 100
P2 17 15 21 200
P3 13 11 17 100
Demand 90 80 90

Step 3: Mixed Matrix (The Least should be selected)


D1 D2 D3 Supply
P1 14 – W1 16 – W2 19 – W1 100
P2 14 – W1 15 – W2 19 – W1 200
P3 13 – W2 11 – W2 17 – W1 100
Demand 90 80 90

The above problem is minimization unbalanced. Balance it by adding by a dummy column with 140 units
and assign ‘0’cost to the cells in the dummy column.

Step 1: Obtaining IBFS using Vogel’s method


D1 D2 D3 D4 Supply I II III IV
P1 80 1 1 100 14 16 2 1
14 16 19 0
P2 140 200/60 14 15 - -
14 15 19 0
P3 30 30 100 11 11 1 4
13 11 17 0
Demand 90 80 90 140/0 400
I 1 4 2 0
II - 1 1 0

E M Reddy Page | 93
AMA-Notes

III 2 1 1
IV 2 - 1

Cost associated with the Vogel’s solution:


Allocated Cells Computation Cost (Rs.)
F1 – W2 18 Units X Rs.3 54
F1 – W3 1 Units X Rs.6 6
F2 – W1 12 Units X Rs.4 48
F2 – W4 25 Units X Rs.1 25
F3 – W1 4 Units X Rs.3 12
F3 – W3 30 Units X Rs.7 210
Total Cost 355

Step 2: Checking for degeneracy

Initial Basic Feasible Solution:


W-1 W-2 W-3 W-4 Availability
Factory 1 18 1 19
5 3 6 2
Factory 2 12 25 37
4 7 9 1
Factory 3 4 30 34
3 4 7 5
Requirement 16 18 31 25 90

Number of allocated cells = 6


M+N–1=3+4–1=6
Since there is no degeneracy we can proceed for optimality test.

Step 3: Moody’s optimality test

A: Allocated cells
Allocated Cells Cost
U1+V2= 3
U1+V3= 6
U2+V1= 4
U2+V4= 1
U3+V1= 3
U3+V3= 7

B: Values

Assume U1 value as ‘0’ and the remaining values will be as follows:


U1 = 0, U2 = 2, U3 = 1, V1 = 2, V2 = 3, V3 = 6, V4 = -1

C: Unallocated Cells

E M Reddy Page | 94
AMA-Notes

Unallocated Cells 𝐂𝐣 𝐙𝐣 𝐂𝐣 − 𝐙𝐣
U1+V1= 5 2 3
U1+V4= 2 -1 3
U2+V2= 7 5 2
U2+V3= 9 8 1
U3+V2= 4 4 0
U3+V4= 5 0 5

The above solution is optimal because there are no negativity numbers to give improvement. But there
exists another optimal solution i.e. if we make U3V2 allocated cell, for every allocation made the cost
changes by Rs.0 i.e. it does not change. Hence the problem has multiple optimal solution.

**Question no 8: ABC manufacturing company wishes to develop a monthly production schedule


for the next months. Depending upon the sales commitments, the company can either keep the
production constant, allowing fluctuations in inventory or inventories can be maintained at a
constant level, with fluctuating production. Fluctuating production necessities in working
overtime, the cost of which is estimated to be double the normal production cost of Rs.12 per unit.
Fluctuating inventories result in inventory carrying cost of Rs.2 per unit. If the company fails to
fulfill its sales commitment, it incurs a shortage cost of Rs.4 per unit per month. The production
capacities for the next three months are shows below.
Production Capacity
Month Regular Overtime Sales
1 50 30 60
2 50 0 120
3 60 50 400
Determine the optimal production schedule.

Solution:

Matrix:
Month 1 2 3 Available
1 Normal 12 14 16 59
1 Overtime 24 26 28 30
2 Normal 16 12 14 50
3 Normal 20 16 12 60
3 Overtime 32 28 23 50
Requirement 60 120 40

Notes:

1) The objective of this sum is to match production and sales at the minimum inventory cost.
2) There are 3 possibilities:
i. Produce and sell in same month – Will incur only production cost of Rs.12 per unit.
ii. Produce in previous month and sell in subsequent months – Will incur production cost &
carrying cost Rs 2 per unit per month.

E M Reddy Page | 95
AMA-Notes

iii. Sell in previous month and produce in subsequent months – Will incur production cost &
shortage cost of Rs.4 per unit per month.
3) If production is done during overtime an additional time over time cost of Rs.12 per unit will be
incurred.
4) The above is a minimization unbalanced transportation problem. Balance it by adding dummy column
for 20 units with ‘0’ cost.

Question no 9: As a result of an expansion in production capacity, the management of Minerva


Manufacturing Ltd., has decided to take additional employees at each of its five plants in the
south-west of India. The numbers required at each plant are:
Plant 1 2 3 4 5
Employees required 45 74 50 82 63
All its employees currently come from three large towns in the area.
Upon contracting the main employment agency in each town, Minerva finds that the numbers of
suitable people available for employment are as follows:
Agency (Town) A B C
People available 120 100 154
Because of the rural situations of the five plant Minerva has agreed with the trade union concerned
that daily return traveling expenses from each town will be paid by the company to employees.
The rate is currently Rs.12 per mile, and the distance (in miles) between each plant and each town
are as follows:
Town 1 2 3 4 5
A 6 2 2 6 3
B 14 9 4 5 3
C 10 4 11 3 4
(a) How many people should Minerva aim to employ from each town in order to minimize the
additional travelling expenses incurred?
(b) What is the minimum value of these expenses in connection with the additional 314 employees?
(c) In order to appear not to be unfair the potential employees from any one of the three towns, it
has now been decided that the 60 people who are surplus to requirements should be spread equally
between the three towns i.e. 20 from each. How much more than in A would the company have to
pay out each day in travelling expenses in order to achieve the minimum cost?

Solution:

Step 1: Formulation
Agency/Plant 1 2 3 4 5 Dummy Supply
A 72 24 24 72 36 0 120
B 168 108 48 60 36 0 100
C 120 48 132 36 48 0 154
Demand 45 74 50 82 63 60 374

When the problem is absolved, obviously in the final solution allocations in the dummy will be there. For
example, in Agency A, dummy allocated and in Agency C, dummy 40 allocated. This means rejects 20
employees from town A and 40 employees from B that means dummy allocation indicates rejection.

E M Reddy Page | 96
AMA-Notes

Step 2: Problem reformulated when surplus to requirement should be spread evenly between three cities
Agency/Plant 1 2 3 4 5 Supply
A 72 24 24 72 36 100
B 168 108 48 60 36 80
C 120 48 132 36 48 134
Demand 45 74 50 82 63 314

Notes:

1) 374 persons applied and there are only 314 vacancies. Surely 60 applications need to be rejected which
can be done in two ways.
i. Rejections through dummy – The freedom to reject is given to Vogel and Moody and the only
consideration is cost minimization.
ii. Rejection by adjusting the supply – In the question does the reduction through a condition then
rejection should be done by adjusting the demand or supply.

Question 10: The brown chemical company produces a special oil-based material which is
currently in short supply. Four of Brown’s customers have already placed orders which in total
exceed the combined capacity of its two plans and the company needs to know how it should
allocate its production capacity to maximize profits.
The following distribution costs per unit have been determined.
Customer C1 (Rs.) C2 (Rs.) C3 (Rs.) C4 (Rs.)
Plant X 16 15 14 18
Plant Y 15 15 14 15
The variable unit production costs are Rs.10 per plant X and Rs.12 for plant Y. Since the four
customers are in different industries, the pricing structure allows different prices to be charged to
different customers. (The material undergoes slight variations for each customer at negligible
costs). These prices are Rs.46 for C1, Rs.42 for C2, Rs.40 for C3 and Rs.44 for C4.
The customer’s orders (in units) are:
C1 C2 C3 C4
2000 5000 3500 2500
And the plant capacities at X and Y in the period concerned are 6000 and 3000 units respectively.
Due to an industrial dispute the company can only supply customer C3 from plant Y.
Required:
(a) Use the transportation algorithm to determine the optimum solution.
(b) If the industrial disputes were to be resolved so that customer C3 would be supplied from plant
X, how would this affect your solution?
Solution:
Part 1: Profit Matrix (Selling Price – Variable Cost)
C1 (Rs.) C2 (Rs.) C3 (Rs.) C4 (Rs.)
X 46 – 26 = 20 42 – 25 = 17 40 – 24 = 16 44 – 28 = 16
Y 46 – 27 = 19 42 – 27 = 15 40 – 26 = 14 44 – 27 = 17

Part 2: Formulating into transportation matrix when there is industrial dispute

E M Reddy Page | 97
AMA-Notes

Customer 3 should be supplier only from Plan Y. Hence the cell X, C3 is a prohibited cell. To prevent
allocation in that cell, assign a high loss ‘-M’ too that cell.
C1 C2 C3 C4 Available
X 20 17 -M 16 6000
Y 19 15 14 17 3000
Requirement 2000 5000 3500 2500

The problem is maximization unbalanced. First balance it by adding a dummy row having profit of ‘0’ in
each cell and then the convert the problem into minimization then apply Vogel and moody for solution.

Initial Basic Solution for the above problem:

Optimality test for final improved solution:

A: Allocated cells
Allocated Cells Cost
U1+V1= 0
U1+V2= 3
U2+V1= 1
U2+V4= 3
U3+V2= 20
U3+V3= 20

B: Values

Assume U1 value as ‘0’ and the remaining values will be as follows:


U1 = 0, U2 = 1, U3 = 17, V1 = 0, V2 = 3, V3 = 3, V4 = 2

C: Unallocated Cells
Unallocated Cells 𝐂𝐣 𝐙𝐣 𝐂𝐣 − 𝐙𝐣
U1+V3= M 3 M
U1+V4= 4 2 2
U2+V2= 5 4 1
U2+V3= 6 4 2
U3+V1= 20 17 3
U3+V4= 20 19 1

Part 2: Formulation when industrial dispute resolved

E M Reddy Page | 98
AMA-Notes

C1 C2 C3 C4 Available
X 20 17 16 16 6000
Y 19 15 14 17 3000
Requirement 2000 5000 3500 2500

This part is similar to previous one except that in the cell X, C3 the regret is not M. But the regret is 4. We
need not do from the beginning the solution, we can test for optimality in the final solution obtained in the
previous part. For unallocated cell U1+V3, now the Cj − Zj = 4 – 3 = 1. Hence removing the dispute will
not alter the solution.

Question no 11: Management of Ranga ltd is very much worried about the continuing recession on
the country. The company has 7 divisions (A to G). They have decided to close four divisions
namely A, B, C and D transfer some of the employees to the remaining divisions. Personnel at the
units to be closed has signified a willingness to move to any of the three remaining units and the
company is willing to provide them with removal costs.
The technology of production is different to some degree at each unit and retraining expenses will
be incurred on transfer. Not all existing personnel can be absorbed by transfer and a number of
redundancies will arise. Cost of redundancy is given as a general figure at each unit is to be closed.
Number employed A-200, B-400, C-300, D-200.
Retraining Cost A B C D
Transfer to:
Unit E 0.5 0.4 0.6 1.3
Unit F 0.6 0.4 0.6 0.3
Unit G 0.5 0.3 0.7 0.3
Removal Cost A B C D
Transfer to:
Unit E 2.5 3.6 3.4 3.7
Unit F 2.4 4.6 3.4 1.7
Unit G 2.5 2.7 3.3 2.7
Redundancy Payment 6 5 6 7
Additional personnel required at units remaining open: E-350, F-450, G-200.
Use the transportation method to obtain an optimal solution to the problem of the cheapest mean
to transfer personnel from the units to be closed to those which will be expanded.
Solution:
Availability Demand
A 200 E 350
B 400 F 450
C 300 G 200
D 200
Total 1100 Total 1000
Redundant People 1100 – 1000 = 100

E F G Redundancy Supply
A 3 3 3 6 200
B 4 5 3 5 400

E M Reddy Page | 99
AMA-Notes

C 4 4 4 6 300
D 5 2 3 7 200
Requirement 350 450 200 100 1100

The above is minimization balanced problem. Solve using Vogel and Moody.

E M Reddy Page | 100


AMA-Notes

5. STANDARD COSTING or VARIANCE ANALYSIS:

5.1. Learning Objectives

1) Material Variance
i. Single Raw Material
ii. Mix of Raw Materials
2) Labour Variances
i. With Idle time
ii. Without Idle time
3) Variable Overhead Variances
4) Fixed Overhead Variances
i. Without Calendar
ii. With Calendar
iii. With WIP (Work in progress)
5) ABC (Activity Based Costing) and Overhead Variances
6) Sales Variances
i. Total Approach
ii. Margin Approach
iii. Reconciliation Problem
7) Reconciliation Problems
i. Budgeted profit to actual profit (Absorption Costing System)
ii. Budgeted profit to actual profit (Marginal Costing System)
iii. Standard profit to actual profit (Absorption Costing System)
iv. Standard profit to actual profit (Marginal Costing System)
8) Reconciliation with WIP
9) Reconciliation with opportunity cost
10) Reverse Working Problems
11) Planning and Operating Variance
12) Market size and market share variance
13) Miscellaneous Problems

5.2. Introduction

1) Standard Costing system provides information for control purpose.


2) Companies prepare budgets at the beginning of the period and had budgeted profit as the period’s
target. At the end of the period, when actual profits are earned they are compared with budget and
reasons analyzed. This is called variance analysis.

E M Reddy Page | 101


AMA-Notes

Profit
Variance

Sales Cost
Variance Variance

Overhead
Material Cost Labour Cost
Cost
Variance Variance
Variance

Variable Overhead Fixed Overhead


Variance (VOH) Variance (FOH)

3)
4) If the variances increase the actual profit, then it is favorable else it is adverse.
5) In the first segment we will learn calculation of variances. In the next segment we will learn preparation
of reconciliation statements (or) Operating Statements in various possible ways. The last segment we
will discuss the following concepts:
i. Partial Plan vs. Single Plan
ii. Reverse working problems
iii. Investigation of Variances

5.3. Understanding Standard Cost

Example:
Budgeted output = 10000 Units
Standard Cost per unit = Rs.10
Actual Output = 8000 Units
Actual Cost = Rs.96000
1) The company plan to spend Rs.1,00,000 but actually spent only Rs. 96,000 thereby saving a cost of Rs.
4,000. Is this right?
Answer: No, because we cannot compare a target cost for 10,000 units with the actual cost of 8,000
units.
2) We have to revise the target for actual output and then compare.
3) Three types of costs:

E M Reddy Page | 102


AMA-Notes

Budgeted Cost Standard Cost Actual Cost

BO x SC/U AO x SC/U AO x AC/U

Cost Planned Cost Allowed Cost Incurred

10,000 Units x Rs.10 8,000 Units x Rs.10 = 8,000 Units x Rs.12 =


= Rs.1,00,000 Rs.80,000 Rs.96,000
4) Budgeted cost is for “budgeted cost for budget output” and Standard cost is “Budgeted cost for actual
output”. Cost Variance is the difference between the Standard cost & actual cost and not budgeted cost
& actual cost.

5.4. Cost Variances

5.4.1. Material Cost Variances – Single Raw Material Input

Step 1: Computation Table


[1] [2] [3]
SQ x SP AQ x AP AQ x SP

Where SQ = Standard Quantity for actual output (or) Quantity allowed


Where SP = Standard Price
Where AQ = Actual Quantity
Where AP = Actual Price

Step 2: Variance Calculation

Material Cost
Variance (1-2)

Material Price Material Usage


Variance (3-2) Variance (1-3)

Question no 1: The following data is available for a product.


Standard material cost per unit of output 2kgs at Rs.20 per kg.
During a period, actual details are as under:
Actual Output = 10,000 Units

E M Reddy Page | 103


AMA-Notes

Material Used = 22,000 Kgs


Actual price is Rs.25 per kg.
Find out material variances?

Solution:

Step 1: Computation table


[1] [2] [3]
SQ x SP AQ x AP AQ x SP
20000 x 20 22000 x 25 22000 x 20
4,00,000 5,50,000 4,40,000

Step 2: Variance Calculation

Material Cost
Variance (1-2) =
4,00,000 – 5,50,000 =
1,50,000 (Adverse)
Material Price Material Usage
Variance (3-2) = Variance (1-3) =
4,40,000 – 5,50,000 = 4,00,000 – 4,40,000 =
1,10,00 (Adverse) 40,000 (Adverse)

Working Note: Calculation of Standard quantity


Input Output
2 Kgs 1 Unit
10000 x 2 Kgs = 20000 Kgs 10000 Units

Notes:

1) For the output of 10,000 units the factory is allowed to consume 20,000 Kgs at a standard price of Rs.20
i.e. the cost allowed is Rs.4,00,000.
2) However, it actually consumes 22,000 Kgs at Rs.25 per Kg i.e. Rs.5,50,000. Thus there is an
overspending of material cost of Rs.1,50,000.
3) The material cost may vary due to two reasons:
i. Variation in Purchase Price – Price Variance
ii. Variation in Quantity Consumed – Usage Variance
4) While calculating usage variance we multiply the excess usage of 2,000 Kgs with standard price of Rs.20
and not the actual price Rs.25?
Answer: Because by taking the actual price we are considering the efficiency/inefficiency of purchase
manager in evaluating the production manager performance.
5) However, while calculating price variance we multiply the price variance with actual quantity because the
duty of the purchase manager is to purchase the entire quantity ordered at efficient price.

E M Reddy Page | 104


AMA-Notes

6) Alternatively, the price variance can be calculated for the standard quantity and the balance reported as
“Joint Variance”.
Material Cost
Variance (1-2) =
4,00,000 – 5,50,000 =
1,50,000 (Adverse)

Material Price Material Usage Material Joint


Variance = (Rs.20 – Variance = (20,000 – Variance = Rs.5 x
Rs.25) x 20,000 = 22,000) x Rs.20 = 2000 = Rs.10,000
Rs.1,00,000 (Adverse) Rs.40,000 (Adverse) (Adverse)

5.4.2. Material Cost Variances – Mix of Raw Materials

Step 1: Computation Table


[1] [2] [3] [4]
SQ x SP AQ x AP AQ x SP RAQ x SP

Where SQ = Standard Quantity for actual output (or) Quantity allowed


Where SP = Standard Price
Where AQ = Actual Quantity
Where AP = Actual Price
Where RAQ = Revised Actual Quantity i.e. actual quantity in standard mix

Step 2: Variance Calculation

Material Cost
Variance (1-2)

Material Price Material Usage


Variance (3-2) Variance (1-3)

Material Mix Variance Material Yield


(4-3) Variance (1-4)

E M Reddy Page | 105


AMA-Notes

Question no 2: The standard set for a chemical mixture of a firm is as under:


Material Standard Mix% Standard Price per kg (Rs.)
A 40 20
B 60 30
The standard loss in production is 10%. During a period, the actual consumption and price paid for
a good output of 189 Kg are as under:
Material Quantity in Kg. Actual Price per kg (Rs.)
A 90 18
B 110 34
Calculate material variances.
Solution:
Step 1: Computation table
[1] [2] [3] [4]
SQ x SP AQ x AP AQ x SP RAQ x SP
A 84 x 20 90 x 18 90 x 20 80 x 20
B 126 x 30 110 x 34 110 x 30 120 x 30
Total 5460 5360 5100 5200

Step 2: Variance Calculations

Material Cost
Variance (1-2) = 5460
– 5360 = 100
(Favourable)

Material Price Material Usage


Variance (3-2) = 5100 Variance (1-3) = 5460
– 5360 = 260 – 5100 = 360
(Adverse) (Favourable)

Material Yield
Material Mix Variance
Variance (1-4) = 5460
(4-3) = 5200 – 5100 =
– 5200 = 260
100 (Favourable)
(Favourable)

Working Note 1: Computation of SQ


Input Output
100 90
189 x 100 / 90 = 210 189

Working Note 2: Computation of RAQ

Actual Quantity = 110 + 90 = 200 Kgs


A = 200 x 40% = 80 Kgs
B = 200 x 60% = 120 Kgs

E M Reddy Page | 106


AMA-Notes

Notes:

1) Material Mix Variance:


i. Material Mix Variance is “Column 4 – Column 3”
ii. The difference between column 3 and column 4 is not price (both are SPs), is not quantity (both
are AQs) but it is mix. Column 4 in standard mix and column 3 is actual mix. Hence, the
difference is mix variance.
iii. In this problem we increased Raw Material A proportion which is a cheaper Raw Material and
decreased Raw Material B proportion which is an expensive Raw Material there by having a
favorable mix variance of 100.
2) Material Yield Variance:
i. Material Yield Variance is “Column1 – Column 4”.
ii. Difference between column 1 and column 4 is not price (both are SPs), is not mix (both are
standard mix) but is the quantity.
iii.
Particulars Standard (Kgs) Actual (Kgs)
A. Input 210 200
B. Loss 21 11
C. Output (A – B) 189 189
D. % Loss (B/A x 100) 10% 5.5%
E. % Yield (100 – D) 90% 94.5%

This extra yield is called of 4.5% is called variance.


iv. A ‘favorable’ yield varicose indicates ‘abnormal gain’ while an ‘adverse’ variance yield variance
indicates ‘abnormal loss’.
3) Material Usage Variance:
i. Material Usage Variance is ‘Column 1 – Column 3’
ii. The difference between column 1 and column is not the price (both are SPs) but it is mixed
quantity. Column is standard quantity in standard mix and column is actual quantity in actual
mix.
iii. If we look at mix angle alone it is mix variance and quantity angle alone it is yield variance, both
it is usage variance.

Question no 3: SC Limited manufactures a special floor tile which measures ½ m x ¼ m x 0.01m.


The tiles are manufactured in a process, which requires the following standard mix:
Material Quantity (Kgs) Price (Rs.) Amount (Rs.)
A 40 1.5 60
B 30 1.2 36
C 10 1.4 14
D 20 0.5 10
120
Each mix should produce 100 square meters of floor tiles of 0.01m thickness. During April, the
actual output was 46,400 tiles from an input of:
Material Quantity (Kgs) Price (Rs.) Amount (Rs.)
A 2,200 1.6 3,520
B 2,000 1.1 2,200
C 500 1.5 750

E M Reddy Page | 107


AMA-Notes

D 1,400 0.5 700


7,170
Calculate material variances.
Solution:
Step 1: Computation table
[1] [2] [3] [4]
SQ x SP AQ x AP AQ x SP RAQ x SP
A 2,320 x 1.5 2,200 x 1.6 2,200 x 1.5 2,440 x 1.5
B 1,740 x 1.2 2,000 x 1.1 2,000 x 1.2 1,830 x 1.2
C 580 x 1.4 500 x 1.5 500 x 1.4 610 x 1.4
D 1,160 x 0.5 1,400 x 0.5 1,400 x 0.5 1,220 x 0.5
Total 6,960 7,170 7,100 7,320

Step 2: Variance Calculations

Material Cost
Variance (1-2) = 6960
– 7170 = 210
(Adverse)

Material Usage
Material Price
Variance (1-3) = 6960
Variance (3-2) = 7100
– 7100 = 140
– 7170 = 70 (Adverse)
(Adverse)

Material Yield
Material Mix Variance
Variance (1-4) = 6960
(4-3) = 7320 – 7100 =
– 7320 = 360
220 (Favourable)
(Adverse)

Working Note 1: Calculation of Standard Quantity

Area of one tile = Length x Breadth = 0.5m x 0.25m = 0.125 sq. mts.
So 100 sq. mts. = 100/0.125 = 800 tiles. 1 standard mix should produce 800 tiles.
Input Output
1 Standard Mix 800 tiles
58 Standard Mix 46,400 tiles
A 58 x 40 2,320
B 58 x 30 1,740
C 58 x 10 580
D 58 x 20 1,160

E M Reddy Page | 108


AMA-Notes

Working Note 2: Computation of RAQ


Actual Quantity = 2,200 + 2,000 + 510 + 1,400 = 6100
A = 6,100 x 40% = 2,440 Kgs
B = 6,100 x 30% = 1,830 Kgs
C = 6,100 x 10% = 610 Kgs
D = 6,100 x 20% = 1,220 Kgs

5.4.3. Labour Variances – without mix

Step 1: Computation Table


[1] [2] [3]
SH x SR AH x AR AH x SR

Where SH = Standard Hours for actual output (or) Hours allowed


Where SR = Standard Rate
Where AH = Actual Hours
Where AR = Actual Rate
Step 2: Variance Calculation

Labour Cost Variance


(1-2)

Labour Rate Variance Labour Efficiency


(3-2) Variance (1-3)

5.4.4. Labour Variances – with mix

Step 1: Computation Table


[1] [2] [3] [4]
SH x SR AH x AR AH x SR RAH x SR

Where SH = Standard Hours for actual output (or) Hours allowed


Where SR = Standard Rate
Where AH = Actual Hours
Where AR = Actual Rate
Where RAH = Revised Actual Hours i.e. actual hours in standard mix

E M Reddy Page | 109


AMA-Notes

Step 2: Variance Calculation

Labour Cost Variance


(1-2)

Labour Rate Variance Labour Efficiency


(3-2) Variance (1-3)

Labour Mix Variance/


Labour Productivity
Labour Gang
Variance (1-4)
Variance (4-3)

Question no 4: The data obtained from a manufacturing concern are:


Particulars Men Women
Number in standard gang 20 10
Standard rate per hour 9 8
Number in actual gang 16 14
Actual rate per hour (Rs.) 10 7
In a 48 hour-week, the gang as actually composed, produced 1200 standard hours. Compute labour
variances.
Solution:
Step 1: Computation table
[1] [2] [3] [4]
SH x SR AH x AR AH x SR RAH x SR
Men 800 x 9 768 x 10 768 x 9 960 x 9
Women 400 x 9 672 x 7 672 x 8 480 x 8
Total 10,400 12,384 12,288 12,480

Step 2: Variance Calculation

Labour Cost Variance


(1-2) = 10400 – 12384
= 1984 (Adverse)

Labour efficiency
Labour Rate Variance
Variance (1-3) =
(3-2) = 12288 – 12384
10400 – 12288 = 1888
= 96 (Adverse)
(Adverse)

Labour Mix/Gang Labour Productivity


Variance (4-3) = Variance (1-4) =
12480 – 12288 = 192 10400 – 12480 = 2080
(Favourable) (Adverse)

E M Reddy Page | 110


AMA-Notes

Working Note 1: Calculation of standard hours

Standard hours = 1200 hours


Men = 1200 x 20/30 = 800 hours
Women = 1200 x 10/30 = 400 hours

Working Note 2: Calculation of actual hours

Men = 16 people x 48 hours = 768 hours


Women = 14 people x 48 hours= 672 hours

Working Note 3: Calculation of Revised actual hours – Actual hours in standard mix

Actual hours = 768 + 672 = 1440 hours


Men = 1440 x 20/30 = 960 hours
Women = 1440 x 10/30 = 480 hours

5.4.5. Labour Variance – Idle time

Question no 5: In a certain factory


House paid for in a week 40
Standard rate per hour (Rs.) 8
Standard output of department per hour, taking into account the normal idle time (units) 20
Actual rate per hour (Rs.) 9
In a particular week, it was ascertained that 1000 units were produced despite 20% of the time paid
for was lost owing to power failure. Calculate labour variances.
Solution:
Step 1: Computation table
[1] [2] [3]
SH x SR AH x AR AH x SR
50 x 8 = 400 40 x 9 = 360 40 x 8 = 320

Working Note 1: Calculation of standard hours


Input Output
1 Hour 20 Units
50 Hours 1000 Units

Step 2: Revised Labour efficiency variance


[1] [2] [3]
SH x SR AH x AR AH (W) x SR
50 x 8 = 400 40 x 9 = 360 32 x 8 = 256

Labour revised efficiency ratio (1 – 3) = 400 – 256 = 144 (Favorable)

E M Reddy Page | 111


AMA-Notes

Step 3: Variance Calculation

Labour Cost Variance


(1-2) = 400 – 360 =
40 (Favourable)

Labour Rate Variance Labour Efficiency


(3-2) = 320 – 360 = Variance (1-3) = 400 –
40 (Adverse) 320 = 80 (Favourable)

Labour idle time


Variance = Idle time Labour revised
x Standard Rate = 8 efficiency Variance (1-
Hours x 8 = 64 3) = 144 (favourable)
(Adverse)

Notes:

1) 50 Hours job was completed in 40 hours and the workers have saved the company 10 Hours wages. Is
this right?
Answer: No, because 8 hours was idle time due to power cut and the workers have completed the job
in 32 hours. The real time saving is 18 hours. This revised efficiency variance.
2) There are two types of actual hours:
i. Actual hours paid – Considered for calculating rate variance
ii. Actual hours worked – Actual Hours paid – idle time – used to calculated revised efficiency
variance

5.4.6. Variable Overhead Variances

Step 1: Calculation of standard rate (SR)


SR per unit BVO/BO
SR per Hour BVO/BH

Step 2: Computation table


[1] [2] [3]
SH x SR or AO x SR AVO AH x SR or SO x SR

Where SH = Standard Hours for actual output (or) Hours allowed


Where SO = Standard Output for actual Hours
Where AH = Actual Hours
Where SR = Standard Rate
Where AVO = Actual Variable overhead

E M Reddy Page | 112


AMA-Notes

When we multiply by hours use SR per hour and when we multiply by output use SR per unit.

Step 3: Variance Calculation

Variable Overhead
Cost Variance (1-2)

Variable overhead Variable overhead


expenditure Variance Efficiency Variance
(3-2) (1-3)

Question no 6: XYZ company has established the following standards for variable factory overhead.
Standard hours per unit: 6
Variable overhead per hour: Rs.2/-
The actual data for the month are as follows:
Actual variable overheads incurred Rs.2,00,000
Actual output (units) 20,000
Actual hours worked 1,12,000
Required to calculate variable overhead variances:
a. Variable overhead cost variance.
b. Variable overhead expenditure variance
c. Variable overhead efficiency variance

Solution:

Step 1: Standard rates (SR)

SR per hour = Rs.2 (Given)


SR per unit = 6 Hours x Rs.2 = Rs.12 per hour

Step 2: Computation table


[1] [2] [3]
SH x SR or AO x SR AVO AH x SR or SO x SR
1,20,000 Hours x Rs.2 1,12,000 x Rs.1.79 1,12,000 Hours x Rs.2
(or) or
20,000 Units x Rs.12 18,667 Units x Rs.12
Rs.2,40,000 Rs.2,00,000 Rs.2,24,000

Working Note 1: Standard Hours for actual output


Input Output
6 Hours 1 Unit
1,20,000 Hours 20,000 Units

Working Note 2: Standard output for actual hours

E M Reddy Page | 113


AMA-Notes

Input Output
6 Hours 1 Unit
1,12,000 Hours 18677 Units

Step 3: Variance Calculation

Variable Overhead Cost


Variance (1-2) =
2,40,000 – 2,00,000 =
Variable overhead 40,000 (Favourable) Variable overhead
expenditure Variance Efficiency Variance
(3-2) = 2,24,000 – (1-3) = 2,40,000 –
2,00,000 = 24,000 2,24,000 = 16,000
(Favourable) (Favourable)

Notes:

1) In the entire chapter we assume the labour hours as the primary cost driver for overheads and the
output as secondary cost driver.
2) In Other words, more the output more the labour hours and more the labour hours more the variable
overheads.
3) For the 20,000 units output the cost allowed is Rs.2,40,000 but actually spent is only Rs.2,00,000. Thus
there is a savings in variable overhead cost of Rs. 40,000.
4) The variable overhead may vary due to two reasons:
i. Expenditure change – Plan to pay Rs.2 for hour but paid only Rs.1.79 per hour saving Rs.0.21
per 1,12,000 Hours which is equal to Rs. 24,000 (approximately)
ii. Change in efficiency
5) Efficiency can be seen in two ways:
i. In terms of hours – 1,20,00 Hours job has been done in 1,12,000 hours saving 8,000 Hours
variable overhead (8,000 Hours x Rs.2 = Rs. 16,000)
ii. In terms of output – In 1,12,000 Hours the company should have produced 18,677 units but
produced 20,000 units therefore the 1,333 units are produced without any extra labour hours
and variable overheads there by saving a cost of Rs. 16,000 (1,333 Units x Rs.12)

5.4.7. Fixed Overhead Variances – With Calendar

Step 1: Calculation of standard rate (SR)


SR per unit BFO/BO
SR per Hour BFO/BH

Step 2: Computation table


[1] [2] [3] [4] [5]
SH x SR AFO BFO [BO x SR AH x SR PFO
or or or [BFO x
AO x SR BH x SR] SO x SR AD/BD]

E M Reddy Page | 114


AMA-Notes

Where SH = Standard Hours for actual output (or) Hours allowed


Where SO = Standard Output for actual Hours
Where AH = Actual Hours
Where SR = Standard Rate
Where AFO = Actual Fixed overhead
PFO = Possible fixed overheads
Where AD = Actual Days
Where BD = Budgeted days

Step 3: Variance Calculation

Fixed Overhead Cost


Variance (1-2)

Fixed overhead expenditure Fixed overhead Volume


Variance (3-2) Variance (1-3)

Fixed overhead Capacity Fixed overhead Calendar Fixed overhead efficiency


Variance (4-5) Variance (5-3) Variance (1-4)

Question no 7: A manufacturing company operating a standard costing system has the following
data in respect of July, 2006: -
Actual number of working days 22
Actual man-hours worked during the month 8,600
Units produced 850
Actual fixed overhead incurred Rs. 3,600
The following information is obtained from the company’s budget and standard cost data: -
Budgeted number of working days per month 20
Budgeted man-hours per month 8,000
Standard man-hours per unit 10
Standard fixed overhead rate per man hour Rs.0.50
Calculate fixed overhead variances.

Solution:

Step 1: Calculation of Standard rates

Standard rate per hour = Rs.0.50 (given)


Standard rate per unit = 10 Hours x Rs.0.50 = Rs.5 per hour

Step 2: Computation table

E M Reddy Page | 115


AMA-Notes

[1] [2] [3] [4] [5]


SH x SR AFO BFO [BO x SR AH x SR PFO
or or or [BFO x
AO x SR BH x SR] SO x SR AD/BD]
850 x 5 3,600 8,000 x 0.5 8,600 x 0.5 4,000 x 22/20
4,250 3,600 4,000 4,300 4,400

Step 3: Variance Calculation

Fixed Overhead Cost


Variance (1-2) = 4250
– 3600 = 650
(Favourable)

Fixed overhead expenditure Fixed overhead Volume


Variance (3-2) = 4000 – 3600 Variance (1-3) = 4250 –
= 400 (Favourable) 4000 = 250 (Favourable)

Fixed overhead Capacity Fixed overhead Calendar Fixed overhead efficiency


Variance (4-5) = 4300 – Variance (5-3) = 4400 – Variance (1-4) = 4250 –
4400 = 100 (Adverse) 4000 = 400 (Favourable) 4300 = 50 (Adverse)

Notes:

1) Page no 168 (11 points)

Question no 8: From the following figures extracted from the books of a company, compute
appropriate variances:
Particulars Budget Actual
Output in units 12,000 13,000
Hours 6,000 6,600
Fixed Overhead Rs. 2,400 Rs. 2,500
Variable Overhead Rs. 12,000 Rs. 13,300
No. of days 50 54

Solution:

Part 1: Variable overhead variances

Step 1: Computation of Standard rates (SR)


BVO 12,000
Standard Rate/Unit = = 12,000 = Rs.1 per unit
BO

E M Reddy Page | 116


AMA-Notes

BVO 12,000
Standard Rate/Hour = = = Rs.2 per hour
BO 6,000

Step 2: Computation table


[1] [2] [3]
SH x SR or AO x SR AVO AH x SR or SO x SR
13,000 Units x Rs.1 Rs. 13,300 6,600 Hours x Rs.2
Rs. 13,000 Rs. 13,300 Rs. 13,200

Step 3: Variance Calculation

Variable Overhead Cost


Variance (1-2) = 13,300
– 13,000 = 300
Variable overhead (Adverse) Variable overhead
expenditure Variance Efficiency Variance
(3-2) = 13,200 – (1-3) = 13,000 –
13,300 = 100 13,200 = 200
(Adverse) (Adverse)

Part 2: Fixed overhead variances

Step 1: Computation of Standard rates (SR)


BFO 2,400
Standard Rate/Unit = = 12,000 = Rs.0.2 per unit
BO

BFO 2,400
Standard Rate/Hour = = 6,000 = Rs.0.4 per hour
BO

Step 2: Computation table


[1] [2] [3] [4] [5]
SH x SR AFO BFO [BO x SR AH x SR PFO
or or or [BFO x AD/BD]
AO x SR BH x SR] SO x SR
13,000 Units x Rs.0.2 Rs. 2,500 Rs. 2,400 6,600 Hours x Rs.0.4 Rs. 2,400 x 54/50
Rs. 2,600 Rs. 2,500 Rs. 2,400 Rs. 2,640 Rs. 2,592

Step 3: Variance Calculation

Fixed Overhead Cost Variance (1 – 2) = 2600 – 2500 = 100 (Favorable)


Fixed Overhead Expenditure Variance (3 – 2) = 2400 – 2500 = 100 (Adverse)
Fixed Overhead Volume Variance (1 – 3) = 2600 – 2400 = 200 (Favorable)
Fixed Overhead Capacity Variance (4 – 5) = 2640 – 2592 = 48 (Favorable)
Fixed Overhead Calendar Variance (5 – 3) = 2593 – 2400 = 192 (Favorable)
Fixed Overhead Efficiency Variance (1 – 4) = 2600 – 2640 = 40 (Adverse)

E M Reddy Page | 117


AMA-Notes

5.4.8. Fixed Overhead Variances – Without Calendar

Step 1: Calculation of standard rate (SR)


SR per unit BFO/BO
SR per Hour BFO/BH

Step 2: Computation table


[1] [2] [3] [4]
SH x SR AFO BFO [BO x SR AH x SR
or or or
AO x SR BH x SR] SO x SR

Where SH = Standard Hours for actual output (or) Hours allowed


Where SO = Standard Output for actual Hours
Where AH = Actual Hours
Where SR = Standard Rate
Where AFO = Actual Fixed overhead

Step 3: Variance Calculation

Fixed Overhead Cost


Variance (1-2)

Fixed overhead expenditure Fixed overhead Volume


Variance (3-2) Variance (1-3)

Fixed overhead Capacity Fixed overhead efficiency


Variance (4-3) Variance (1-4)

Question no 9: Calculate fixed production overhead variances in as much as details as possible, in


the following situation:
Particulars Budget Actual
Fixed Overhead (Rs.) 2,46,000 2,59,000
Direct labour (hours) 1,23,000 1,41,000
Output (units) 6,15,000 (see below)

The company operates a process costing system. At the beginning of the period 42,000 half
completed units were in stock. During the period 6,80,000 units were completed and 50,000 half
completed units remained in stock at the end of the period.

E M Reddy Page | 118


AMA-Notes

Solution:

Step 1: Commutation of actual output using Statement of Equivalent units


Output Equivalent Units
Item Units % Units
Opening WIP 42,000 50% 21,000
Introduced and completed 6,38,000 100% 6,38,000
Closing WIP 50,000 50% 25,000
Total 6,84,000

Step 2: Computation of Standard Rates (SR)


BFO 2,46,000
Standard Rate per unit = = 5,15,000 = Rs.0.40 per unit
BO

BFO 2,46,000
Standard Rate per hour = = 1,23,000 = Rs.2 per hour
BH

Step 3: Computation table


[1] [2] [3] [4]
SH x SR AFO BFO [BO x SR AH x SR
or or or
AO x SR BH x SR] SO x SR
6,84,000 Units x Rs.0.4 Rs. 2,59,000 Rs. 2,46,000 1,41,000 Hours x Rs.2
Rs. 2,73,600 Rs. 2,59,000 Rs. 2,46,000 Rs. 2,82,000

Step 4: Variance Calculation

Fixed Overhead Cost


Variance (1-2) =
2,73,600 – 2,59,000 =
14,600 (Favourable)

Fixed overhead Volume


Fixed overhead expenditure
Variance (1-3) = 2,73,600
Variance (3-2) = 2,46,000 –
– 2,46,000 = 27,600
2,59,000 = 13,000 (Adverse)
(Favourable)

Fixed overhead Capacity Fixed overhead efficiency


Variance (4-3) = 2,82,000 Variance (1-4) = 2,73,600
– 2,46,000 = 36,000 – 2,82,000 = 8,400
(Favourable) (Adverse)

E M Reddy Page | 119


AMA-Notes

5.4.9. Cost Variances Reconciliation

**Question 10: ABC ltd uses flexible budgets and standard costing for its single product PCM 30
produced at its factory at Solan. The following details relate to a particular month’s actual and also
provide brief details of ‘Standards’ established.
Standard quantity required for producing 1 unit of PCM 30 3 Kgs
Standard cost of Raw Materials Rs. 4.40 per kg
Cost of actual materials purchased and used in the relevant month Rs. 3,36,000
Actual price paid for the raw material in the relevant month Rs.4.20 per Kg
Standard labour time required to produce 1 unit of PCM 30 30 minutes
Standard wage rate Rs.5 per hour
Actual wage rate Rs.5.40 per hour
Sufficient direct labour time equivalent for producing 28,000 units was utilized but the actual
production in the relevant month was only 25,000 units.
The company has a normal operating capacity of 15,000 hours per month and flexible overhead
budgets are:
Hours of operation 12,500 14,000 15,000
Variable production overhead Rs. 1,50,000 Rs. 1,68,000 Rs. 1,80,000
Fixed production overhead Rs. 2,70,000 Rs. 2,70,000 Rs. 2,70,000
Actual fixed overheads incurred did not deviate from the budgeted amount. However, the variable
overheads incurred amounted to Rs. 1,60,000 in the concerned month.
1. You are required to calculate the appropriate variances material, labour and overhead.
2. Show the variances in a statement suitable for presentation to management, reconciling the
standard cost with the actual cost of production.
Solution:
Part 1: Analyzing volume (Output) and hours
Particulars Budgeted Standard Actual
Output (units) 30,000 (WN-1) 28,000 25,000
Hours 15,000 12,500 (WN-2) 14,000 (WN-3)

Working Note 1: Calculation of budgeted output


Input Output
30 Minutes 1 Unit
15,000 Hours 15,000 x 2 = 30,000 Units

Working Note 2: Calculation of Standard hours


Input Output
30 Minutes 1 Unit
25,000 x 30 = 12,500 Hours 25,000 Units

Working Note 3: Calculation of Actual Hours


Input Output
30 Minutes 1 Unit

E M Reddy Page | 120


AMA-Notes

28,000 x 30 = 14,000 Hours 28,000 Units

Part 1: Material Variances

Step 1: Computation table


[1] [2] [3]
SQ x SP AQ x AP AQ x SP
75,000 Kg x 80,000 Kg x 80,000 Kg x
Rs.4.4 Rs.4.2 Rs.4.4
Rs. 3,30,000 Rs. 3,36,000 Rs. 3,52,000

Step 2: Variance Calculation

Material Cost
Variance (1-2) =
3,30,000 – 3,36,000 =
6,000 (Adverse)
Material Price Material Usage
Variance (3-2) = Variance (1-3) =
3,52,000 – 3,36,000 = 3,30,000 – 3,52,000 =
16,000 (Favourable) 22,000 (Adverse)

Working Note: Calculation of Standard quantity for actual output (SQ)


Input Output
3 Kgs 1 Unit
25,000 x 3 Kgs = 75,000 Kgs 25,000 Units

Part 2: Labour Cost Variances


Step 1: Computation Table
[1] [2] [3]
SH x SR AH x AR AH x SR
12,500 x Rs.5 14,000 x Rs.5.4 14,000 x Rs.5
Rs. 62,500 Rs. 75,600 Rs. 70,000

Step 2: Variance Calculation

Labour Cost Variance


(1-2) = 62,500 –
75,600 = 13,100
(Adverse)
Labour Rate Variance Labour Efficiency
(3-2) = 70,000 - Variance (1-3) =
75,600 = 5,600 62,500 – 70,000 =
(Adverse) 7,500 (Adverse)

E M Reddy Page | 121


AMA-Notes

Part 3: Variable Overhead Variances

Step 1: Computation of Standard rates (SR)


BVO 1,80,000
Standard Rate/Unit = = = Rs.6 per unit
BO 30,000

BVO 1,80,000
Standard Rate/Hour = = = Rs.12 per hour
BO 15,000

Step 2: Computation table


[1] [2] [3]
SH x SR or AO x SR AVO AH x SR or SO x SR
12,500 Hours x Rs.12 Rs. 1,60,000 28,000 Hours x Rs.6
or or
25,000 Units x Rs.6 14,000 Units x Rs.12
Rs. 1,50,000 Rs. 1,60,000 Rs. 1,68,200

Step 3: Variance Calculation

Variable Overhead Cost


Variance (1-2) =
1,50,000 – 1,60,000 =
Variable overhead 10,000 (Adverse) Variable overhead
expenditure Variance Efficiency Variance
(3-2) = 1,68,000 – (1-3) = 1,50,000 –
1,60,000 = 8,000 1,68,000 = 18,000
(Favourable) (Adverse)

Part 4: Fixed Overhead Variances

Step 1: Computation of Standard Rates (SR)


BFO 2,70,000
Standard Rate per unit = = = Rs.9 per unit
BO 30,000

BFO 2,70,000
Standard Rate per hour = = = Rs.18 per hour
BH 15,000

Step 2: Computation table


[1] [2] [3] [4]
SH x SR AFO BFO [BO x SR AH x SR
or or or
AO x SR BH x SR] SO x SR
12,500 Hours x Rs.18 Rs. 2,70,000 Rs. 2,70,000 14,000 Hours x Rs.18
or or
25,000 Units x Rs.9 28,000 Units x Rs.9
Rs. 2,25,000 Rs. 2,70,000 Rs. 2,70,000 Rs. 2,52,000

E M Reddy Page | 122


AMA-Notes

Step 3: Variance Calculation

Fixed Overhead Cost


Variance (1-2) =
2,25,000 – 2,70,000 =
45,000 (Adverse)

Fixed overhead Volume


Fixed overhead expenditure
Variance (1-3) = 2,25,000
Variance (3-2) = 2,70,000 –
– 2,70,000 = 45,000
2,70,000 = 0
(Adverse)

Fixed overhead Capacity Fixed overhead efficiency


Variance (4-3) = 2,52,000 Variance (1-4) = 2,25,000
– 2,70,000 = 18,000 – 2,52,000 = 27,000
(Adverse) (Adverse)

Part 5: Computation of Standard Cost and Actual Cost

Step 1: Standard Cost per unit


Particulars Computation Amount per Unit (Rs.)
Direct Materials 3 Kg x Rs.4.4 13.20
Direct Labour ½ Hour x 5 2.50
Variable Overhead 6
Fixed Overhead 9
Total 30.70

Step 2: Computation of Standard Cost (Standard Cost for actual output)

Actual Output = 25,000 Units


Standard Cost per unit = Rs.30.70
Standard Cost = 25,000 Units x Rs.30.70 = Rs. 7,67,500
Alternatively, standard cost is the total of column 1 in all the variances table.
Particulars Amount (Rs.)
Direct Materials 3,30,000
Direct Labour 62,500
Variable Overhead 1,50,000
Fixed Overhead 2,25,000
Total 7,67,500

Step 3: Computation of actual cost

Actual cost is column 2 total in all variances tables.


Particulars Amount (Rs.)
Direct Materials 3,36,000
Direct Labour 75,600

E M Reddy Page | 123


AMA-Notes

Variable Overhead 1,60,000


Fixed Overhead 2,70,000
Total 8,41,600

Notes:

1) There are 3 types of costs:


i. Budgeted Cost – 30,000 Units x Rs.30.70 = Rs. 9,21,000
ii. Standard Cost – 25,000 Units x Rs.30.70 = Rs. 7,67,500
iii. Actual Cost – Rs.8,41,600
2) We should not compare the target cost for 30,000 units (Budgeted Cost) i.e. Rs.9,21,000 with actual cost
spent for 25,000 units i.e. Rs.8,41,600. The right way is to compare the revised target of Rs.7,67,500
(Standard Cost) with actual cost.

Part 6: Reconciliation Statement between standard cost and actual cost


Particulars Favorable (Rs.) Adverse (Rs.) Amount (Rs.)
Standard Cost 7,67,500
Material Price Variance 16,000 -
Material Usage Variance - 22,000
Labour Rate Variance - 5,600
Labour Efficiency Variance - 7,500
Variable overhead expenditure variance 8,000 -
Variable overhead efficiency variance - 18,000
Fixed overhead expenditure variance - -
Fixed overhead capacity variance - 18,000
Fixed overhead efficiency variance 24,000 27,000
Total 24,000 98,100 74,100 (Adverse)
Actual Cost 8,41,600

5.5. Marginal Costing vs. Absorption Costing

1) Profit = Sales – Expired Cost


2) Cost can be classified into two types:
i. Product Cost – Are those costs considered for stock valuation
i. Cost of goods sold - Expires
ii. Stock - Unexpired
ii. Period Cost – Are those cost not considered for stock valuation – Expires fully

E M Reddy Page | 124


AMA-Notes

Cost

Non-Manufacturing
Manuafacturing (Administration and
selling)

Material Labour Overheads Always period cost

Always Product
Variable Fixed
Cost

Product cost under Period cost under


absorption costing marginal costing
system system
3)
4) Thus Marginal Costing system values stock at Variable Manufacturing Cost (VMC) and Absorption
costing system values stock at Full Manufacturing Cost (FMC).
5) Hence, in absorption costing we should unitize fixed manufacturing overheads which is done at
standard rate (also called pre-determined rate or budgeted rate).
BFO
6) Standard rate = BO where the budgeted output is normal capacity and in the absence of information
100% capacity can be considered as normal.
7) Fixed Manufacturing Overheads are charged to the Cost of Production at standard rate for actual
output produced. This is called absorbed overheads.
8) At the end of year what is actually spent and what is absorbed may be different. This difference is called
under/over absorption which needs to be adjusted while calculating absorption costing profit.
9) If the question silent is regarding the actual fixed cost assumed to be same as budgeted fixed cost.
10) Income statement under Absorption Costing:
A Sales XXX
B Cost of Goods Sold:
Opening Stock of Finished Goods (FMC) XXX
Add: Cost of Production XXX
Less: Closing stock of Finished Goods (FMC) (XXX) (XXX)
C Gross Profit (A – B) XXX
D Administration and Selling Expenses (XXX)
E Under absorption (XXX)
F Over absorption XXX
G Profit XXX

E M Reddy Page | 125


AMA-Notes

11) Income statement under Marginal Costing:


A Sales XXX
B Variable Cost of Goods Sold:
Opening Stock of Finished Goods (VMC) XXX
Add: Variable Cost of Production XXX
Less: Closing stock of Finished Goods (VMC) (XXX) (XXX)
C Gross Contribution (A – B) XXX
D Variable Administration and Selling Expenses (XXX)
E Contribution (C – D) XXX
F Fixed Manufacturing/Administration/Selling Expenses XXX
G Profit (E – F) XXX

Question no 10: From the following data compute the profit under (a) Marginal Costing, and (b)
Absorption costing and reconcile the difference in profit.
Particulars Rs. Per unit
Selling Price 8
Variable Cost 4
Fixed Cost 2
Normal volume of production is 26,000 units per quarter.
Both opening and closing stocks consisting of both finished goods and equivalent units of work-in-
progress are as follows: -
Particulars Q1 Q2 Q3 Q4 Total
Opening Stock - - 6,000 2,000 -
Production 26,000 30,000 24,000 30,000 1,10,000
Sales 26,000 24,000 28,000 32,000 1,10,000
Closing Stock - 6,000 2,000 - -

Solution:

Part 1: Calculation of profit under marginal costing system


Particulars Q1 (Rs.) Q2 (Rs.) Q3 (Rs.) Q4 (Rs.)
A Sales 2,08,000 1,92,000 2,24,000 2,56,000
B Variable Cost of Goods Sold:
Opening Stock of Finished Goods (VMC) 0 0 24,000 8,000
Add: Variable Cost of Production 1,04,000 1,20,000 96,000 1,20,000
Less: Closing stock of Finished Goods (VMC) 0 (24,000) (8,000) 0
Variable cost of Goods Sold 1,04,000 96,000 1,12,000 1,28,000
C Gross Contribution (A – B) 1,04,000 96,000 1,12,000 1,28,000
D Variable Administration and Selling Expenses 0 0 0 0
E Contribution (C – D) 1,04,000 96,000 1,12,000 1,28,000
F Fixed Manufacturing/Administration/Selling 52,000 52,000 52,000 52,000
Expenses
G Profit (E – F) 52,000 44,000 60,000 76,000

Part 2: Calculation of Profit under absorption costing system


Particulars Q1 (Rs.) Q2 (Rs.) Q3 (Rs.) Q4 (Rs.)

E M Reddy Page | 126


AMA-Notes

A Sales 2,08,000 1,92,000 2,24,000 2,56,000


B Cost of Goods Sold:
Opening Stock of Finished Goods (FMC) 0 0 36,000 12,000
Add: Cost of Production 1,56,000 1,80,000 1,44,000 1,80,000
Less: Closing stock of Finished Goods (FMC) 0 (36,000) (12,000) 0
Cost of Goods Sold 1,56,000 1,44,000 1,68,000 1,92,000
C Gross Profit (A – B) 52,000 48,000 56,000 64,000
D Administration and Selling Expenses 0 0 0 0
E (Under)/Over Absorption 0 8000 (4000) 8000
F Profit 52,000 56,000 54,000 72,000

Working Note 1: Calculation of under/over absorption of fixed overheads


Particulars Q1 (Rs.) Q2 (Rs.) Q3 (Rs.) Q4 (Rs.)
A Actual Fixed Overhead 52,000 52,000 52,000 52,000
B Absorbed Overhead 52,000 60,000 48,000 60,000
(Actual Output X Standard Rate) (26,000 x 2) (30,000 x 2) (24,000 x 2) (30,000 x 2)
C (Under)/Over Absorption (B – A) 0 8000 (4000) 8000

Part 3: Reconciling Marginal Costing and Absorption Costing profit


Particulars Q1 Q2 Q3 Q4
A Opening Stock (Quantity) - - 6,000 2,000
B Closing Stock (Quantity) - 6,000 2,000 -
C Net Stock (B – A) - 6,000 (4,000) (2,000)
D Fixed Cost in net stock (C x Rs.2) - 12,000 (8,000) (4,000)
E Marginal Costing profit 52,000 44,000 60,000 76,000
F Absorption Costing Profit (D + E) 52,000 56,000 54,000 72,000

Notes:

1) Is the difference between the profits under the two systems due to under/over absorption?
Answer: No, because it is already adjusted while calculating absorption costing profit.
2) Then why the profit is difference?
Answer: It is due to the stock valuation. Marginal costing values stock at variable manufacturing cost
and absorption costing at full manufacturing cost. The difference is the fixed inside the net stock
(Closing Stock – Opening Stock).
i. Net Stock ‘0’ – No change in Stock Position – Both system shows same profit
ii. Net Closing Stock – Stock Increases – Absorption costing system shows more profit
iii. Net Opening Stock – Stock Decreases – Marginal Costing system shows more profit.

5.6. Sales Variances

Part 1: Total Approach

Step 1: Computation table


[1] [2] [3]
BQ x BP AQ x AP AQ x BP

E M Reddy Page | 127


AMA-Notes

Where BQ = Budgeted Quantity planned to be sold


Where AQ = Actual quantity sold
Where BP = Budgeted Selling Price
Where AP = Actual Selling Price

Step 2: Variance Computation

Total Sales Variance


(1-2)

Selling Price Variance Sales Volume


(3-2) Variance (1-3)

Part 2: Sales Variance Margin Approach

Step 1: Calculation of Margins

Absorption Costing Marginal Costing

BM = BP – SC BM = BP – SVC
AM = AP – SC AM = AP – SVC

Where SVC = Standard Variable Cost


Where SC = Standard Cost
Where BP = Budgeted Selling Price
Where AP = Actual Selling Price
Where BM = Budgeted Margin
Where AM = Actual Margin

Note: In absorption costing margin should be under stood as profit per unit and in marginal margin should
be understood as contribution per unit.

Step 2: Computation table


[1] [2] [3]
BQ x BM AQ x AM AQ x BM

Where BQ = Budgeted Quantity planned to be sold


Where AQ = Actual quantity sold

E M Reddy Page | 128


AMA-Notes

Where BM = Budgeted Margin


Where AM = Actual Margin

Step 3: Variance Computation

Total Sales Variance


(1-2)

Selling Price Variance Sales Volume Profit


(3-2) Variance (1-3)

Example:
Budgeted Output = 10,000 Units
Budged Selling price = Rs.15 per unit
Actual Output = 8,000 Units
Actual Selling Price = Rs.14 per unit
Standard Cost per unit = Material: Rs.4
Standard Cost per unit = Labour: Rs.3
Standard Cost per unit = Variable overhead: Rs.2
Standard Cost per unit = Fixed Overhead: Rs.2

Solution:

Part 1: Total Sales Approach

Step 1: Computation table


[1] [2] [3]
BQ x BP AQ x AP AQ x BP
10,000 x 15 8,000 x 14 8,000 x 15
1,50,000 1,12,000 1,20,000

Step 2: Computation of Variance

Total Sales Variance


(1-2) = 1,50,000 –
1,12,000 = 38,000
(Adverse)
Selling Price Variance Sales Volume
(3-2) = 1,20,000 – Variance (1-3) =
1,12,000 = 8,000 1,50,000 – 1,20,000 =
(Adverse) 30,000 (Adverse)

E M Reddy Page | 129


AMA-Notes

Notes:

1) The company targeted to have a sale of Rs.1,50,000 but actually sold only goods worth Rs.1,12,000. Due
to sales drop there is an adverse of variance of Rs. 38,000.
2) The sales changes due to two reasons:
i. Due to change in selling price – Plant to Sell at Rs.15 but sold only at Rs.14 resulting in adverse
price variance of Rs.1 per unit for 8,000 units.
ii. Due to change in sales volume – Targeted to sell 10,000 units but sold only 8,000 units. Hence
there is a volume drop of 2,000 units at Rs.15 resulting 30,000 adverse volume variance.

Part 2: Margin Approach – Absorption Costing System

Step 1: Calculation of Margins

BM = BP – SC = Rs.15 – (Rs.4 + Rs.3 + Rs.2 + Rs.2) = Rs.15 – Rs.10 = Rs.5


AM = AP – SC = Rs.14 – (Rs.4 + Rs.3 + Rs.2 + Rs.2) = Rs.14 – Rs.10 = Rs.4

Step 2: Computation table


[1] [2] [3]
BQ x BM AQ x AM AQ x BM
10,000 x 5 8,000 x 4 8,000 x 5
50,000 32,000 40,000

Step 3: Variance Computation

Total Sales Variance


(1-2) = 50,000 –
32,000 = 18,000
(Adverse)
Selling Price Variance Sales Volume Profit
(3-2) = 40,000 – Variance (1-3) =
32,000 = 8,000 50,000 – 40,000 =
(Adverse) 10,000 (Adverse)

Part 3: Margin Approach – Marginal Costing System

Step 1: Calculation of Margins

BM = BP – SVC = Rs.15 – (Rs.4 + Rs.3 + Rs.2) = Rs.15 – Rs.8 = Rs.7


AM = AP – SVC = Rs.14 – (Rs.4 + Rs.3 + Rs.2) = Rs.14 – Rs.8 = Rs.6

Step 2: Computation table


[1] [2] [3]
BQ x BM AQ x AM AQ x BM
10,000 x 7 8,000 x 6 8,000 x 7
70,000 48,000 56,000

E M Reddy Page | 130


AMA-Notes

Step 3: Variance Computation

Total Sales Variance


(1-2) = 70,000 –
48,000 = 22,000
(Adverse)
Selling Price Variance Sales Volume Profit
(3-2) = 56,000 – Variance (1-3) =
48,000 = 8,000 70,000 – 56,000 =
(Adverse) 14,000 (Adverse)

Notes:

1) In all the three approaches the selling price variance will be same because the difference in margins is
also the difference in price because in budgeted and actual margin the cost is standard.
2) Understanding sales volume variance in all the three approaches:

Total Approach Absorption Costing Marginal Costing

2000 Units decrease x (Rs.15 2000 Units decrease x (Rs.15


2000 Units decrease x Rs.15
– Rs.10) = Rs. 10,000 – Rs.8) = Rs. 14,000
= Rs. 30,000 (Adverse)
(Adverse) (Adverse)
a. When volume drops by 2,000 units the sales drop by Rs. 30,000. This is sales volume variance
under total sales approach.
b. We want to know the impact of volume drop on profit and not on sales.
c. When we don’t produce and sell 2,000 units we not only lose Rs.15 selling price but also save
Rs.10 cost. Thus the loss in profit per unit is Rs.5 and for 2,000 units is Rs. 10,000. This is sales
volume profit variance in absorption costing system.
d. When volume drops only variable cost could be saved and hence the cost saved per unit is not
Rs.10 but only Rs.8. Hence the profit drop due to volume drop is 2,000 Units x Rs.7 = Rs.
14,000 adverse marginal costing volume variance.
3) Correction made by absorption costing system:
a. Since marginal costing does not absorb fixed overheads it has only one variance which is fixed
overhead expenditure variance.
b. In absorption costing system we also have fixed overhead volume variance.
[1] [3]
AO x SR BFO (BO x SR)
8,000 Units x Rs.2 10,000 Units x Rs.2
Rs. 16,000 Rs. 20,000

Fixed Overhead Volume Variance = 1 – 3 = 16,000 – 20,000 = Rs. 4,000 (Adverse)


c. The volume has dropped by 2,000 Units. Had it been a variable cost we could have saved Rs.
4,000. Because of the cost being fixed this saving didn’t happen but while calculating sales

E M Reddy Page | 131


AMA-Notes

volume profit variance we wrongly considered this saving to happen which is now reversed
through fixed overhead volume varices Rs. 4,000 adverse.
4) Cost Volume Variance:
a. There are 3 types of costs:
Budgeted Cost Standard Cost Actual Cost
BO x SC per unit AO x SC per unit AO x AC per unit
10,000 Units x Rs.10 8,000 Units x Rs. 10 Assumed as Rs. 90,000
Rs. 1,00,000 Rs. 80,000 Rs. 90,000
Cost Volume Variance = Rs. 20,000
Cost Variance = Rs. 10,000

b. This cost volume variance is netted off against sales volume variance and reported as sales
volume profit variance in the reconciliation statement.
Sales Volume Variance (Total Approach) = Rs. 30,000 (Adverse)
Less: Cost Volume Variance = Rs. 20,000 (Favorable)
Sales Volume Profit Variance = Rs. 10,000 (Adverse)

5.7. Types of Reconciliation Problems

1) Standard Reconciliation
2) Reconciliation with WIP and Finished Goods
3) Opportunity Cost Method of Reconciliation
4) Reconciliation with Planning Variance and Operating Variance
5) Balance Score Card Method of Reconciliation

5.7.1. Standard Reconciliation Statement

Question no 11: The budgeted production of a company is 20,000 units per month. The standard
cost sheet is as under:
Direct Material 1.5 Kgs @ Rs.6 per Kg
Direct Labour 6 Hours @ Rs.5 per hour
Variable Overhead 6 Hours @ Rs.4 per hour
Fixed Overhead Rs.3 per unit
Selling Price Rs. 72 per unit
The following are the actual details for a month:
Actual Sales 18,750 Units
Actual Production 18,750 Units
Direct Material 29,860 Kgs @ Rs.5.25 per Kg
Direct Labour 1,18,125 Hours @ Rs.6 per hour
Fixed Overhead Rs. 40,000
Variable Overhead Rs. 5,25,000
Required:
(i) Calculate all variances
(ii) Prepare reconciliation statement from budgeted profit as well as from standard profit.
Solution:
Part 1: Material Cost Variances

E M Reddy Page | 132


AMA-Notes

Step 1: Computation table


[1] [2] [3]
SQ x SP AQ x AP AQ x SP
28,125 Kg x Rs.6 29,860 Kg x Rs.5.25 29,860 Kg x Rs.6
Rs. 1,68,750 Rs. 1,56,765 Rs. 1,79,160

Step 2: Variance Calculation

Material Cost
Variance (1-2) =
1,68,750 – 1,56,765 =
11,985 (Favourable)
Material Price Material Usage
Variance (3-2) = Variance (1-3) =
1,79,160 – 1,56,765 = 1,68,750 – 1,79,160 =
22,395 (Favourable) 10,410 (Adverse)

Working Note: Calculation of Standard quantity for actual output (SQ)


Input Output
1.5 Kgs 1 Unit
18,750 x 1.5 Kgs = 28,125 Kgs 18,750 Units

Part 2: Labour Cost Variances


Step 1: Computation Table
[1] [2] [3]
SH x SR AH x AR AH x SR
1,12,500 x Rs.5 1,18,125 x Rs.6 1,18,125 x Rs.5
Rs. 5,62,500 Rs. 7,08,750 Rs. 5,90,625

Step 2: Variance Calculation

Labour Cost Variance


(1-2) = 5,62,500 –
7,08,750 = 1,46,250
(Adverse)
Labour Rate Variance Labour Efficiency
(3-2) = 5,90,625 – Variance (1-3) =
7,08,750 = 1,18,125 5,62,500 – 5,90,625 =
(Adverse) 28,125 (Adverse)

Working Note: Calculation of Standard Hours for actual output (SQ)


Input Output
6 Hours 1 Unit

E M Reddy Page | 133


AMA-Notes

18,750 x 6 Hours = 1,12,500 Hours 18,750 Units

Part 3: Variable Overhead Variances

Step 1: Computation of Standard rates (SR)

Standard Rate/Hour = Rs.4 per Hour (Given)

Standard Rate/Unit = Rs.4 x 6 Hours = Rs.24 per Unit

Step 2: Computation table


[1] [2] [3]
AO x SR AVO AH x SR
18,750 Units x Rs.24 Rs. 5,25,000 1,18,125 Hours x Rs.4
Rs. 4,50,000 Rs. 5,25,000 Rs. 4,72,500

Step 3: Variance Calculation

Variable Overhead Cost


Variance (1-2) =
4,50,000 – 5,25,000 =
Variable overhead 75,000 (Adverse) Variable overhead
expenditure Variance Efficiency Variance
(3-2) = 4,72,500 – (1-3) = 4,50,000 –
5,25,000 = 52,500 4,72,500 = 22,500
(Adverse) (Adverse)

Part 4: Fixed Overhead Variances

Step 1: Computation of Standard Rates (SR)

Standard Rate/Unit = Rs.3 per Unit (Given)

Standard Rate/Hour = Rs.3/6 Hours = Rs.0.5 per hour

Step 2: Computation table


[1] [2] [3] [4]
AO x SR AFO BFO [BO x SR] AH x SR
18,750 Units x Rs.3 Rs. 40,000 20,000 Units x 1,18,125 Hours x
Rs.3 Rs.0.5
Rs. 56,250 Rs. 40,000 Rs. 60,000 Rs. 59,062.5

Step 3: Variance Calculation

E M Reddy Page | 134


AMA-Notes

Fixed Overhead Cost


Variance (1-2) =
56,250 – 40,000 =
16,250 (Favourable)

Fixed overhead expenditure Fixed overhead Volume


Variance (3-2) = 60,000 – Variance (1-3) = 56,250 –
40,000 = 20,000 (Favourable) 60,000 = 3,750 (Adverse)

Fixed overhead Capacity Fixed overhead efficiency


Variance (4-3) = 59,062.5 Variance (1-4) = 56,250 –
– 60,000 = 937.5 59,062.5 = 2,812.5
(Adverse) (Adverse)

Part 5: Sales Variances

Step A: Total Approach

Step 1: Computation table


[1] [2] [3]
BQ x BP AQ x AP AQ x BP
20,000 x 72 18,750 x 72 18,750 x 72
14,40,000 13,50,000 13,50,000

Step 2: Computation of Variance

Total Sales Variance


(1-2) = 14,40,000 –
13,50,000 = 9,000
(Adverse)
Sales Volume
Selling Price Variance
Variance (1-3) =
(3-2) = 13,50,000 –
14,40,000 – 1,12,000
1,12,000 = 0
= 90,000 (Adverse)

Step B: Margin approach – Absorption Costing System

Step 1: Calculation of Margins

Standard Cost per unit:

Material (1.5 Kg x Rs.6) = Rs.9


Labour (6 Hours x Rs.5) = Rs.30
Variable Overhead (6 Hours x Rs.4) = Rs.24
Fixed Overhead = Rs.3
Standard Cost per unit = Rs.66

BM = BP – SC = Rs.72 – Rs.66 = Rs.6


AM = AP – SC = Rs.72 – Rs.66 = Rs.6

E M Reddy Page | 135


AMA-Notes

Step 2: Computation table


[1] [2] [3]
BQ x BM AQ x AM AQ x BM
20,000 x 6 18,750 x 6 18,750 x 6
1,20,000 1,12,500 1,12,500

Step 3: Variance Computation

Total Sales Variance (1 – 2) = 1,20,000 – 1,12,500 = 7,500 (Adverse)


Sales Price Variance (3 – 2) = 1,12,500 – 1,12,500 = 0
Sales Volume Profit Variance (1 – 3) = 1,20,000 – 1,12,500 = 7,500 (Adverse)

Step C: Margin approach – Margin Costing System

Step 1: Calculation of Margins

Standard Variable Cost per unit:

Material (1.5 Kg x Rs.6) = Rs.9


Labour (6 Hours x Rs.5) = Rs.30
Variable Overhead (6 Hours x Rs.4) = Rs.24
Standard Cost per unit = Rs.63

BM = BP – SVC = Rs.72 – 63 = Rs.9


AM = AP – SVC = Rs.72 – 63 = Rs.9

Step 2: Computation table


[1] [2] [3]
BQ x BM AQ x AM AQ x BM
20,000 x 9 18,750 x 9 18,750 x 9
1,80,000 1,68,750 1,68,750

Step 3: Variance Computation

Total Sales Variance


(1-2) = 1,80,000 –
1,68,750 = 11,250
(Adverse)
Sales Volume Profit
Selling Price Variance
Variance (1-3) =
(3-2) = 1,68,750 –
1,80,000 – 1,68,750
1,68,750 = 0
= 11,250 (Adverse)

Part 6: Budgeted profit and actual profit

Step 1: Computation of Budgeted profit

E M Reddy Page | 136


AMA-Notes

Budgeted Units = 20,000 Units


Budgeted Profit per unit = Rs.6
Budgeted Profit = Rs. 1,20,000

Step 2: Computation of Actual profit


Particulars Computation Amount (Rs.)
Sales 18,750 Units x Rs.72 13,50,000
Less: Materials 29,860 Kgs x Rs.5.25 1,56,765
Less: Labour 1,18,125 Hours x Rs.6 7,08,750
Less: Variable overhead 5,25,000
Less: Fixed overhead 40,000
Actual loss 80,515

Part 7: Reconciliation between budgeted profit and actual profit – Absorption Costing System
Particulars Favorable (Rs.) Adverse (Rs.) Amount (Rs.)
Budgeted profit 1,20,000
Material Price Variance 22,395 -
Material Usage Variance - 10,410
Labour Rate Variance - 1,18,125
Labour Efficiency Variance - 28,125
Variable overhead expenditure variance - 52,500
Variable overhead efficiency variance - 22,500
Fixed overhead expenditure variance 20,000 -
Fixed overhead capacity variance - 937.5
Fixed overhead efficiency variance - 2812.5
Sales volume profit variance - 7,500
Total 42,395 2,42,910 2,00,515 (Adverse)
Actual Loss 80,515

Part 8: Reconciliation between budgeted profit and actual profit – Marginal Costing System
Particulars Favorable (Rs.) Adverse (Rs.) Amount (Rs.)
Budgeted profit 1,20,000
Material Price Variance 22,395 -
Material Usage Variance - 10,410
Labour Rate Variance - 1,18,125
Labour Efficiency Variance - 28,125
Variable overhead expenditure variance - 52,500
Variable overhead efficiency variance - 22,500
Fixed overhead expenditure variance 20,000 -
Sales volume profit variance - 11,250
Total 42,395 2,42,910 2,00,515 (Adverse)
Actual Loss 80,515

Notes:

E M Reddy Page | 137


AMA-Notes

1) Sales Volume profit variance Marginal Costing System = Sales Volume profit variance Absorption
Costing System + Fixed Overhead Volume Variance = Rs.7500 (Adverse) + Rs. 3,750 (Adverse) = Rs.
11,250 (Adverse).
Rs.6
2) Standard Net Profit Ratio = Rs.72 = 8.33%
Rs.9
Standard P/V Ratio = Rs.72 = 12.5%
Sales Volume Profit Variance – Absorption Costing System = Sales Volume Variance X Net Profit
Ratio = Rs. 90,000 (Adverse) x 8.33% = Rs. 7,500 (Adverse)
Sales Volume Profit Variance – Margin Costing System = Sales Volume Variance X P/V Ratio = Rs.
90,000 (Adverse) x 12.5% = Rs. 11,250 (Adverse)

5.8. Concept of Standard Profit

1) There are 3 types of Profits:


a) Budgeted Profit – Budgeted profit for Budgeted output
b) Standard Profit – Budgeted profit for Actual output (Everything is budgeted except for volume)
(Flexible Budgeted profit)
c) Actual Profit – Actually earned
2) All these 3 profits under Absorption Costing System:

Budgeted Profit Standard Profit Actual Profit

20,000 Units x Rs.6 = 18,750 Units x Rs.6 =


Rs.80,515 (Loss)
Rs.1,20,000 (Profit) Rs.1,12,500 (Profit)

Sales Volume Profit


Variance = Rs.7,500
(Adverse)
3) The three profits in Marginal Costing System:

E M Reddy Page | 138


AMA-Notes

Budgeted Profit Standard Profit Actual Profit

Budgeted Contribution – Standard Contribution –


Rs.80,515 (Loss)
Budgeted Fixed overheads Budgeted Fixed overheads

20,000 Units x Rs.9 - 18,750 Units x Rs.9 -


Rs.60,000 = Rs.1,20,00 Rs.60,000 = Rs.1,08,750
(Profit) (Profit)

Sales Volume Profit


Variance = Rs.11,250
(Adverse)

4) Reconciliation between budgeted profit and actual profit – Absorption Costing System:
Particulars Favorable (Rs.) Adverse (Rs.) Amount (Rs.)
Budgeted Profit 1,20,000
Sales volume profit variance - 7,500 7,500 (Adverse)
Standard Profit 1,12,500
Material Price Variance 22,395 -
Material Usage Variance - 10,410
Labour Rate Variance - 1,18,125
Labour Efficiency Variance - 28,125
Variable overhead expenditure variance - 52,500
Variable overhead efficiency variance - 22,500
Fixed overhead expenditure variance 20,000 -
Fixed overhead capacity variance - 937.5
Fixed overhead efficiency variance - 2812.5
Total 42,395 2,35,410 1,93,015 (Adverse)
Actual Loss 80,515

5) Reconciliation between budgeted profit and actual profit – Marginal Costing System
Particulars Favorable (Rs.) Adverse (Rs.) Amount (Rs.)
Budgeted profit 1,20,000
Sales volume profit variance - 11,250 11,250 (Adverse)
Standard Profit 1,08,750
Material Price Variance 22,395 -
Material Usage Variance - 10,410
Labour Rate Variance - 1,18,125

E M Reddy Page | 139


AMA-Notes

Labour Efficiency Variance - 28,125


Variable overhead expenditure variance - 52,500
Variable overhead efficiency variance - 22,500
Fixed overhead expenditure variance 20,000 -
Total 42,395 2,31,660 1,89,265 (Adverse)
Actual Loss 80,515

6) An overview of the various numbers:


Items Budget (20,000 Standard (18,750 Actual (18,750
Units) (Rs.) Units) (Rs.) Units) (Rs.)
Materials 1,80,000 1,68,750 1,56,765
Labour 6,00,000 5,62,500 7,08,750
Variable Overhead 4,80,000 4,50,000 5,25,000
Fixed Overhead 60,000 56,250 40,000
Total Cost 13,20,000 12,37,500 14,30,515
Sales 14,40,000 13,50,000 13,50,000
Profit 1,20,000 1,12,500 (80,515)

7) Cost Volume Variance = Rs. 13,20,000 – Rs. 12,37,500 = Rs. 82,500 (Favorable)
Sales Volume Variance (Total Approach) = Rs.14,40,000 – Rs. 13,50,000 = Rs. 90,000 (Adverse)
Sales Volume Profit Variance = Rs. 90,000 (Adverse) – Rs. 82,500 (Favorable) = Rs. 7,500 (Adverse)
8) By adjusting sales volume profit variance to budgeted profit we arrive at standard profit. The standard
profit should be adjusted for cost variances and selling price variance to arrive at actual profit or loss.

5.9. Reconciliation with Work in Progress

Question no 12: The following particulars being a standard for a product set as under:
Particulars Qty. or Hrs. per unit Rate in Rs. Amount per Unit
Direct Material
A 2 Kgs 3 6
B 1 Kg 4 4
Direct Wages 5 Hours 4 20
Variable Overheads 5 Hours 1 5
Fixed Overheads 5 Hours 2 10
Total 45
Standard Profit 5
Standard Selling Price 50
Budgeted output is 8,000 units per month. June 2008, the company produced and sold 6,000 Units.
Other actual data are as follows:
Particulars Rs.
Sales Value 3,05,000
Material A 14,850 Kgs 43,065
Material A 7,260 Kgs 29,750
Direct Wages 32,000 Hours 1,27,500
Variable Overhead 30,000
Fixed Overhead 80,600

E M Reddy Page | 140


AMA-Notes

Closing working in progress was 600 units in respect of which material A and B were fully issued
and labour and overhead were 50% complete. The direct labour hours worked was 31,800.
Analyze the variances and present reconciliation statement in all possible ways.
Solution:
Analyzing question:
1) Cost Variances are related to production and sales variances are related to units sold.
2) When we mean units produced, it included both completed production and work in progress.
3) But units completed and WIP cannot be added. WIP has to be converted into equivalent units produced
and then added to other completed units.
Part 1: Calculation of equivalent completed units
Particulars Material Labour Overhead
Completed Units 6,000 6,000 6,000
Closing Work in progress 600 300 300
Total 6,600 6,300 6,300

For calculating sales variance, the actual output is 6,000 Units, for Material Variance the actual output is
6,600 units and for labour and overhead variances the actual output is 6,300 Units.

Part 2: Material Variances


Step 1: Computation table
[1] [2] [3] [4]
SQ x SP AQ x AP AQ x SP RAQ x SP
A 13,200 x 3 43,065 14,850 x 3 14,740 x 3
B 6,600 x 4 29,750 7,260 x 4 7,370 x 4
Total 66,000 72,815 73,590 73,700

Step 2: Variance Calculations

Material Cost
Variance (1-2) =
66,000 – 72,815 =
6,815 (Adverse)

Material Price Material Usage


Variance (3-2) = Variance (1-3) =
73,590 – 72,815 = 775 66,000 – 73,590 =
(Favourable) 7,590 (Adverse)

Material Mix Variance Material Yield


(4-3) = 73,700 – Variance (1-4) =
73,590 = 110 66,000 – 73,700 =
(Favourable) 7,700 (Adverse)

E M Reddy Page | 141


AMA-Notes

Working Note 1: Computation of SQ (Standard Quantity of Raw Material for actual Output)
Input Output
3 Kgs 1 Unit
6,600 Units x 3 Kgs = 19,800 Kgs 6,600 Units

Material A = 19,800 Kgs x 2/3 = 13,200 Kgs


Material B = 19,800 Kgs x 1/3 = 6,600 Kgs

Working Note 2: Computation of RAQ

Actual Quantity = 14,850 Kgs + 7,260 Kgs = 22,110 Kgs


Material A = 22,110 Kgs x 2/3 = 14,740 Kgs
Material B = 22,110 Kgs x 1/3 = 7,370 Kgs

Part 3: Labour Variances


Step 1: Computation table
[1] [2] [3]
SH x SR AH x AR AH x SR
31,500 x 4 1,27,500 32,000 x 4
1,26,000 1,27,500 1,28,000

Working Note 1: Calculation of SH (Standard Hours for actual Output)


Input Output
5 Hours 1 Unit
6,300 Units x 5 Hours = 31,500 Hours 6,300 Units

Step 2: Variance Calculation

Labour Cost Variance


(1-2) = 1,26,000 –
1,27,500 = 1,500
(Adverse)

Labour Rate Variance Labour Efficiency


(3-2) = 1,28,000 – Variance (1-3) =
1,27,500 = 500 1,28,000 – 1,26,000 =
(Favourable) 2,000 (Favourable)

Labour idle time


Variance = Idle time Labour revised
x Standard Rate = 200 efficiency Variance =
Hours x 4 = 800 1,200 (Adverse) (WN)
(Adverse)

E M Reddy Page | 142


AMA-Notes

Working Note: Revised Labour efficiency variance


[1] [2] [3]
SH x SR AH x AR AH (W) x SR
31,500 x 4 1,27,500 31,800 x 4
1,26,000 1,27,500 1,27,200

Labour revised efficiency ratio (1 – 3) = 1,26,000 – 1,27,200 = 1,200 (Adverse)

Part 4: Variable Overhead Variances

Step 1: Computation of Standard rates (SR)

Standard Rate/Hour = Rs.1 per hour


Standard Rate/Unit = 5 Hours x Rs.1 = Rs.5 per unit

Step 2: Computation table


[1] [2] [3]
AO x SR AVO AH (W) x SR
6,300 Units x Rs.5 Rs. 30,000 32,800 Hours x Rs.1
Rs. 31,500 Rs. 30,000 Rs. 31,800

Step 3: Variance Calculation

Variable Overhead Cost


Variance (1-2) = 31,500
– 30,000 = 1,500
Variable overhead (Favourable) Variable overhead
expenditure Variance Efficiency Variance
(3-2) = 31,800 – (1-3) = 31,500 –
30,000 = 1,800 31,800 = 300
(Favourable) (Adverse)

Part 5: Fixed Overhead Variances

Step 1: Computation of Standard Rates (SR)

Standard Rate/Hour = Rs.2 per hour


Standard Rate/Unit = 5 Hours x Rs.2 = Rs.10 per unit

Step 2: Computation table


[1] [2] [3] [4]
AO x SR AFO BFO [BO x SR] AH x SR
6,300 Units x Rs.10 Rs. 80,600 8,000 Units x Rs.10 31,800 Hours x Rs.2
Rs. 63,000 Rs. 80,600 Rs. 80,000 Rs. 63,600

Step 3: Variance Calculation

E M Reddy Page | 143


AMA-Notes

Fixed Overhead Cost


Variance (1-2) =
63,000 – 80,600 =
17,600 (Adverse)

Fixed overhead Volume


Fixed overhead expenditure
Variance (1-3) = 63,000 –
Variance (3-2) = 80,000 –
80,000 = 17,000
80,600 = 600 (Adverse)
(Adverse)

Fixed overhead Capacity


Fixed overhead efficiency
Variance (4-3) = 63,600 –
Variance (1-4) = 63,000 –
80,000 = 16,400
63,600 = 600 (Adverse)
(Adverse)

Part 6: Sales Variances

Step 1: Standard net profit ratio and P/V ratio


Budgeted Profit per unit 5
Standard Net Profit Ratio = x 100 = 50 x 100 = 10%
Budgeted Selling Price

Budgeted Contibution per unit 5+10


Standard PV Ratio = x 100 = x 100 = 30%
Budgeted Selling Price 50

Step 2: Computation table – Total Approach


[1] [2] [3]
BQ x BP AQ x AP AQ x BP
8,000 x 50 3,05,000 6,000 x 50
4,00,000 3,05,000 3,00,000

Step 2: Computation of Variance

Total Sales Variance


(1-2) = 4,00,000 –
3,05,000 = 95,000
(Adverse)
Selling Price Variance Sales Volume
(3-2) = 3,00,000 – Variance (1-3) =
3,05,000 = 5,000 4,00,000 – 3,00,000 =
(Favourable) 1,00,000 (Adverse)

Sales Volume profit variance – Absorption Costing = Sales Volume Variance X Standard Net Profit Ratio
= Rs. 1,00,000 x 10% = Rs. 10,000 (Adverse)
Sales Volume profit variance – Marginal Costing = Sales Volume Variance X Standard PV Ratio
= Rs. 1,00,000 x 30% = Rs. 30,000 (Adverse)

E M Reddy Page | 144


AMA-Notes

Part 7: Closing Work in Progress Valuation


Particulars Computation Absorption Marginal
Costing (Rs.) Costing (Rs.)
Material A 600 Equivalent Units x 2 Kgs x Rs.3 3,600 3,600
Material A 600 Equivalent Units x 1 Kg x Rs.4 2,400 2,400
Labour 300 Equivalent Units x 5 Hours x Rs.4 6,000 6,000
Variable 300 Equivalent Units x 5 Hours x Rs.1 1,500 1,500
Overhead
Fixed Overhead 300 Equivalent Units x 5 Hours x Rs.2 3,000 -
Total Cost 16,500 13,500

Part 8: Computation of Actual Profit


Particulars Absorption Costing (Rs.) Marginal Costing (Rs.)
Sales 3,05,000 3,05,000
Less: Materials 72,815 72,815
Less: Labour 1,27,500 1,27,500
Less: Variable overhead 30,000 30,000
Less: Fixed overhead 80,600 80,600
Total Cost 3,10,915 3,10,915
Less: Closing WIP 16,500 13,500
Cost of Goods Sold 2,94,415 2,97,415
Profit 10,585 7,585

Part 9: Reconciliation Statement – Absorption Costing System


Particulars Favorable (Rs.) Adverse (Rs.) Amount (Rs.)
Budgeted profit (8,000 Units x Rs.5) 40,000
Material Price Variance 775 -
Material Mix Variance 110 -
Material Yield Variance - 7,700
Labour Rate Variance 500 -
Labour Idle Time Variance - 800
Labour Revised Efficiency Variance - 1,200
Variable overhead expenditure variance 1,800 -
Variable overhead efficiency variance - 300
Fixed overhead expenditure variance - 600
Fixed overhead capacity variance - 16,400
Fixed overhead efficiency variance - 600
Selling Price variance 5,000 -
Selling Volume Profit Variance - 10,000
Total 8,185 37,600 29,415 (Adverse)
Actual Profit 10,585

Part 10: Reconciliation between budgeted profit and actual profit – Marginal Costing System
Particulars Favorable (Rs.) Adverse (Rs.) Amount (Rs.)
Budgeted profit (8,000 Units x Rs.5) 40,000

E M Reddy Page | 145


AMA-Notes

Material Price Variance 775 -


Material Mix Variance 110 -
Material Yield Variance - 7,700
Labour Rate Variance 500 -
Labour Idle Time Variance - 800
Labour Revised Efficiency Variance - 1,200
Variable overhead expenditure variance 1,800 -
Variable overhead efficiency variance - 300
Fixed overhead expenditure variance - 600
Selling Price variance 5,000 -
Selling Volume Profit Variance - 30,000
Total 8,185 40,600 32,415 (Adverse)
Actual Profit 7,585

Notes:

1) When we have idle time for overhead variances AH means Actual Hours Worked because in working
hours only production takes place and when production takes place only resources will be consumed.
2) Stock should be valued at Normal Cost (or) Standard Cost. Valuing the stocks at actual cost results in
postponement of variance recognition to the next accounting period which is not desirable.
3) In previous problems, we have seen that Sales Volume Profit Variance – Marginal Costing = Sales
Volume Profit Variance – Absorption Costing + Fixed Overhead Volume Variance. In this sum the
equality is absent. Rs. 30,000 (Adverse) is not equal to ‘Rs. 10,000 (Adverse) + Rs. 17,000 (Adverse)’.
4) The difference is nothing but the difference between the profits under two systems which in turn is due
to fixed cost inside net stock.
5) We targeted to produce and sell 8,000 units but produced and sold only 6,000 units. Due to 2,000 Units
volume drop the profit lost is 2,000 Units x (Rs.50 – Rs.45) = Rs. 10,000 (Adverse) (Absorption Costing
System). However, the real profit loss is 2,000 Units x (Rs.50 – Rs.35) = Rs. 30,000 (Adverse) (Marginal
Costing System) because due to 2,000 Units volume drop Rs. 20,000 fixed cost (2,000 Units x Rs.10)
could not be saved.
6) This mistake Absorption Costing System rectifies through Fixed Overhead Volume Variance. However,
in this case the reversal didn’t happen for 2,000 Units drop but happened only for 1,700 Units (8,000
Units – 6,300 Units) drop.
7) The fixed cost for those 300 Units which is Rs. 3,000 (300 Units x Rs.10) is carried forward to next year.
Hence, is not the cost of the current year and will not form part of variance.

5.10. Opportunity Cost Method of Reconciliation

Question no 13: Blue Ltd manufactures a single product, the standards of which are as follows:
Standard per unit (Rs.) (Rs.)
Standard Selling Price 268
Less: Standard Cost
Material (16 Kgs at Rs.4) 64
Labour (4 Hours at Rs.3) 12
*Overheads (4 Hours at Rs.24) 96 172
Standard Profit 96

E M Reddy Page | 146


AMA-Notes

Total overhead costs are allocated on the basis of budgeted direct labour hours.
The following information relates to last month’s activities:
Budgeted Actual
Production and sales 600 Units 500 Units
Direct Labour 2,400 Hours at Rs.3 2,300 Hours at Rs.3
Fixed Overheads Rs. 19,200 Rs. 20,000
Variable Overheads Rs. 38,400 Rs. 40,400
Materials 9,600 Kgs at Rs.4 per Kg 9,600 Kgs at Rs.4 per Kg
The actual selling price was identical to the budgeted selling price and there was no opening or
closing stocks during the period.
You are required to calculate the variances and reconcile the budgeted and actual profit for each of
the following methods:
a) Traditional Method (Absorption Costing System)
b) The opportunity cost method assuming materials are the limiting factor and materials are
restricted to 9,600 Kgs for the period.
c) The opportunity cost method assuming labour hours are the limiting factor and labour hours are
restricted to 2,400 hours for the period.
d) The opportunity cost method assuming there no scarce inputs.
Solution:
A. Traditional Method Reconciliation
Part 1: Material Cost Variances

Step 1: Computation table


[1] [2] [3]
SQ x SP AQ x AP AQ x SP
8,000 Kgs x Rs.4 9,600 Kgs x Rs.4 9,600 Kgs x Rs.4
Rs. 32,000 Rs. 38,400 Rs. 38,400

Step 2: Variance Calculation

Material Cost
Variance (1-2) =
32,000 – 38,400 =
6,400 (Adverse)
Material Usage
Material Price
Variance (1-3) =
Variance (3-2) =
32,000 – 38,400 =
38,400 – 38,400 = 0
6,400 (Adverse)

Working Note: Calculation of Standard quantity for actual output (SQ)


Input Output
16 Kgs 1 Unit
500 Units x 16 Kgs = 8,000 Kgs 500 Units

E M Reddy Page | 147


AMA-Notes

Part 2: Labour Cost Variances


Step 1: Computation Table
[1] [2] [3]
SH x SR AH x AR AH x SR
2,000 x Rs.3 2,300 Hours x Rs.3 2,300 Hours x Rs.3
Rs. 6,000 Rs. 6,900 Rs. 6,900

Step 2: Variance Calculation

Labour Cost Variance


(1-2) = 6,000 – 6,900
= 900 (Adverse)
Labour Efficiency
Labour Rate Variance
Variance (1-3 = 6,000
(3-2) = 6,900 – 6,900
– 6,900 = 900
=0
(Adverse)

Working Note: Calculation of Standard Hours for actual output (SQ)


Input Output
4 Hours 1 Unit
500 Units x 4 Hours = 2,000 Hours 500 Units

Part 3: Variable Overhead Variances

Step 1: Computation of Standard rates (SR)


BVO 38,400
Standard Rate/Unit = = = Rs.64 per unit.
BO 600

BVO 38,400
Standard Rate/Hours = = = Rs.16 per hour
BH 2,400

Step 2: Computation table


[1] [2] [3]
AO x SR AVO AH x SR
500 Units x Rs.64 Rs. 40,400 2,300 Hours x Rs.16
Rs. 32,000 Rs. 40,400 Rs. 36,800

Step 3: Variance Calculation

E M Reddy Page | 148


AMA-Notes

Variable Overhead Cost


Variance (1-2) = 32,000
– 40,400 = 8,400
Variable overhead (Adverse) Variable overhead
expenditure Variance Efficiency Variance
(3-2) = 36,800 – (1-3) = 32,000 –
40,400 = 3,600 36,800 = 4,800
(Adverse) (Adverse)

Part 4: Fixed Overhead Variances

Step 1: Computation of Standard Rates (SR)


BFO 19,200
Standard Rate/Unit = = = Rs.32 per unit.
BO 600

BFO 19,200
Standard Rate/Hours = = = Rs.8 per hour
BH 2,400

Step 2: Computation table


[1] [2] [3] [4]
AO x SR AFO BFO [BO x SR] AH x SR
500 Units x Rs.32 Rs. 20,000 Rs. 19,200 2,300 Hours x Rs.8
Rs. 16,000 Rs. 20,000 Rs. 19,200 Rs. 18,400

Step 3: Variance Calculation

Fixed Overhead Cost


Variance (1-2) =
16,000 – 20,000 =
4,000 (Adverse)

Fixed overhead expenditure Fixed overhead Volume


Variance (3-2) = 19,200 – Variance (1-3) = 16,000 –
20,000 = 800 (Adverse) 19,200 = 3,200 (Adverse)

Fixed overhead Capacity Fixed overhead efficiency


Variance (4-3) = 18,400 – Variance (1-4) = 16,000 –
19,200 = 800 (Adverse) 18,400 = 2,400 (Adverse)

Part 5: Sales Variances – Total Approach

Step 1: Computation table


[1] [2] [3]
BQ x BP AQ x AP AQ x BP
600 Units x Rs. 268 500 Units x Rs. 268 500 Units x Rs. 268
Rs. 1,60,800 Rs. 1,34,000 Rs. 1,34,000

E M Reddy Page | 149


AMA-Notes

Step 2: Computation of Variance

Total Sales Variance


(1-2) = 1,60,800 –
1,34,000 = 26,800
(Adverse)
Sales Volume
Selling Price Variance
Variance (1-3) =
(3-2) = 1,34,000 –
1,60,800 – 1,34,000 =
1,34,000 = 0
26,800 (Adverse)

Step 3: Calculation of Standard PV Ratio and Net Profit Ratio


Standard Profit per unit 96
Standard Net Profit Ratio = x 100 = 268 x 100 = 35.83%
Budgeted Selling Price

Standard Contibution per unit (268−64−12−64) or (96+32) 128


Standard PV Ratio = x 100 = x 100 =268x100=47.76%
Budgeted Selling Price 268

Step 4: Sales Volume Profit Variance

Sales Volume profit variance – Absorption Costing = Sales Volume Variance X Standard Net Profit Ratio
= Rs. 26,800 x 35.83% = Rs. 9,600 (Adverse)
Sales Volume profit variance – Marginal Costing = Sales Volume Variance X Standard PV Ratio
= Rs. 26,800 x 47.76% = Rs. 12,800 (Adverse)

Part 6: Reconciliation Statement

Step 1: Budgeted profit

Budgeted Output = 600 Units


Budgeted Profit per unit = Rs.96
Budgeted Profit = 600 Units x Rs.96 = Rs. 57,600

Step 2: Computation Actual profit


Particulars Amount (Rs.)
Sales 1,34,000
Less: Materials 38,400
Less: Labour 6,900
Less: Variable overhead 40,400
Less: Fixed overhead 20,000
Actual Profit 28,300

Step 3: Reconciliation between budgeted profit and actual profit – Traditional Method (Absorption Costing)
Particulars Favorable (Rs.) Adverse (Rs.) Amount (Rs.)
Budgeted profit 57,600
Material Price Variance - -

E M Reddy Page | 150


AMA-Notes

Material Usage Variance - 6,400


Labour Rate Variance - -
Labour Efficiency Variance - 900
Variable overhead expenditure variance - 3,600
Variable overhead efficiency variance - 4,800
Fixed overhead expenditure variance - 800
Fixed overhead capacity variance - 800
Fixed overhead efficiency variance - 2,400
Sales Price Variance - -
Sales volume profit variance - 9,600
Total - 29,300 29,300 (Adverse)
Actual Profit 28,300

B. Reconciliation Using Opportunity Cost Method – When Raw Material is Limiting Factor

Step 1: Contribution per Kg of Raw Material

Contribution per unit = Rs.128


Kgs of Raw Material Required =16 Kgs
Rs.128
Contribution per unit of Raw Material = 16 Kgs = Rs.8 per Kg
Cost per Kg of Raw Material Wasted = Purchase Cost + Opportunity Cost = Rs.4 + Rs. 8 = Rs.12

Step 2: Material Usage Variance

Material Usage Variance = [SQ x SP] – [AQ x SP]


= [500 Units x 16 Kgs x Rs.12] – [9,600 Kgs x Rs.12]
= Rs. 96,000 – Rs.1,15,200
= Rs. 19,200 (Adverse)

Step 3: Reconciliation Statement


Particulars Favorable (Rs.) Adverse (Rs.) Amount (Rs.)
Budgeted profit 57,600
Material Price Variance - -
Material Usage Variance - 19,200
Labour Rate Variance - -
Labour Efficiency Variance - 900
Variable overhead expenditure variance - 3,600
Variable overhead efficiency variance - 4,800
Fixed overhead expenditure variance - 800
Total - 29,300 29,300 (Adverse)
Actual Profit 28,300

Notes:

1) When Raw Material is a limiting factor a Kg of Raw Material wastes not only results in loss of Rs.4
purchase price, it also results in loss of profit at Rs.8 per kg wasted.
2) Why Raw Material Usage affects the profit?

E M Reddy Page | 151


AMA-Notes

Answer: Since it is a limiting factor, when we waste a Kg we cannot purchase one more Kg and do the
production. A Kg wasted results in production drop, sales drop and consequently profit drop.
3) The company targeted to produce and sell 600 Units but produced and sold only 500 Units. Prima facie
we may think that the sales volume drop is due to inability of sales department to create demand thus
loosing Rs. 12,800 (100 Units x Rs.8)
4) However, in this case the volume drop is not due to inability to sell but due to inability to produce and
account of adverse usage of scarce Raw Material.
5) Company should have used 8,000 Kgs for the output but used 9,600 Kgs leading to 1,600 Kgs over
usage. This has two impacts:
i. Wastage of Purchase Price – 1,600 Kgs x Rs.4 = Rs. 6,400 (Adverse)
1,600 Kgs
ii. Loss of Volume – = 100 Units x Rs. 128 = Rs. 12,800 (Adverse)
16 Kgs
Total = Rs. 6,400 (Adverse) + Rs. 12,800 (Adverse) = Rs. 19,200 (Adverse)
This means, the production manager not only should take responsibility for the usage of Rs.
6,400 but also should take responsibility for the sales volume profit variance of Rs. 12,800

C. Reconciliation Using Opportunity Cost Method – When Labour Hours is Limiting Factor

Step 1: Contribution per hour

Contribution per unit = Rs.128


Number of Hours per unit = 4 Hours
Rs.128
Contribution per hour = 4 Hours = Rs.32
Cost of wasted labour hour = Wage Rate + Opportunity Cost = Rs.3 + Rs.32 = Rs.35

Step 2: Labour Efficiency Variance

Labour Efficiency Variance = [SH x SR] – [AH x SR]


= [2,000 Hours x Rs.35] – [2,300 Hours x Rs.35]
= Rs. 70,000 – Rs. 80,500
= Rs. 10,500 (Adverse)

Step 3: Labour Idle Capacity Variance

The factory plan to work 2,400 Hours but actuary worked for 2,300 hours thus underutilizing a capacity of
100 Hours. For these 100 Hours there is no wage cost but there exists opportunity cost which is Rs. 3,200
100 Hours
(100 Hours x Rs.32). Alternatively, 100 Hours loss is equal to 25 Units ( ) loss. When 25 units are
4 Hours
lost the profit lost is Rs. 3,200 (25 Units x Rs.128).

An overview:

E M Reddy Page | 152


AMA-Notes

Labour Variance =
13,700 (Adverse)

Labour Efficiency Sales Volume Profit Variance


Variance = 900 = 12,800 (Adverse) (100
(Adverse) Units x Rs.128)

Due to Under
Due to Inefficiency
Utilisation

300 Hours x Rs.75 = 25 units x Rs.128 =


9,600 (Adverse) 3,200 (Adverse)

Conclusion:

The efficiency variance of 10,500 (Adverse) is the responsibility of the worker and supervisor and the
underutilization of capacity variance of 3,200 (Adverse) is the responsibility of the person who is
responsible for the underutilization. For example, if the 100 Hours are lost due to a major machine break
down, then maintenance department is responsible. If it is due to strike, HR and Management is responsible
and so on….

Step 4: Reconciliation Statement


Particulars Favorable (Rs.) Adverse (Rs.) Amount (Rs.)
Budgeted profit 57,600
Material Price Variance - -
Material Usage Variance - 19,200
Labour Rate Variance - -
Labour Capacity Variance - 3200
Labour Efficiency Variance - 10,500
Variable overhead expenditure variance - 3,600
Variable overhead efficiency variance - 4,800
Fixed overhead expenditure variance - 800
Total - 29,300 29,300 (Adverse)
Actual Profit 28,300

D. Reconciliation using Opportunity Cost Method when there is no scarce input

In this case prepare reconciliation statement under Marginal Costing System.

E M Reddy Page | 153


AMA-Notes

5.11. Planning Variance vs. Operating Variance

Example:
Budgeted Output 10,000 Units
Actual Output 8,000 Units
Original Budget
Standard Quantity per unit 10 Kgs
Standard Price Rs.4 per Kg
Revised Budget
Standard quantity per unit (due to change in produce design) 12 Kg
Standard price (due to general price level changes) Rs.3.5 per Kg
Actual
Actual Quantity 85,000 Kgs
Actual price paid Rs.3.75 per Kg

Solution:
Part 1: Material Planning Variances

Step 1: Computation table


[1] [2] [3]
SQ x SP AQ x AP AQ x SP
10,000 Units x 10 Kgs x Rs.4 10,000 Units x 12 Kgs x Rs.3.5 10,000 Units x 12 Kgs x Rs.4
Rs. 4,00,000 Rs. 4,20,000 Rs. 4,80,000

Step 2: Variance Calculation

Material Cost
Variance (1-2) =
4,00,000 – 4,20,000 =
20,000 (Adverse)
Material Price Material Usage
Variance (3-2) = Variance (1-3) =
4,80,000 – 4,20,000 = 400,000 – 4,80,000 =
60,000 (Favourable) 80,000 (Adverse)

Part 2: Material Operating Variances

Step 1: Computation table


[1] [2] [3]
SQ x SP AQ x AP AQ x SP
8,000 Units x 12 Kgs x Rs.3.5 85,000 Kgs x Rs.3.75 85,000 Kgs x Rs.3.5
Rs. 3,36,000 Rs. 3,18,750 Rs. 2,97,500

Step 2: Variance Calculation

E M Reddy Page | 154


AMA-Notes

Material Cost
Variance (1-2) =
3,36,000 – 3,18,750 =
17,250 (Adverse)
Material Price Material Usage
Variance (3-2) = Variance (1-3) =
2,97,500 – 3,18,750 = 3,36,000 – 2,97,500 =
21,250 (Adverse) 38,500 (Favourable)

Notes:

1) When the budget is implemented sometimes there may be a change in environment and the budget
needs to be revised for proper performance evaluation.
2) We have three sets of data:
a. Budget Planning Variance
b. Revised Budget
c. Actuals Operating Variance
3) Planning variance are generally being uncontrolled and no person can be made responsible for that
variance. Operating Variance are controllable and reflects the efficiency or inefficiency in performance.
People should be made accountable only for operating variances.
4) In the above example, if you don’t analyses variance into planning and operating it may lead to wrong
performance evaluations.
5) For example, the original budget asked Purchase Manger to purchase at Rs.4 per Kg and he actually
purchase at Rs.3.75 per Kg. It seems he is efficient but in reality at the time of purchase there was a
general price level decrease where everybody was purchasing at Rds.3.5 per Kg. In this background the
Purchase Manager is really inefficient. This problem can be overcome by analyzing the variance into
planning and operating variance.
6) Planning Variance compares two budgets Original and Revised. The word standard should be
understood as Original Budget and actual as Revised Budget.
7) While calculating operating variance understand standard as revised budget and actual as actuals.

Question no 14: Tungach Ltd makes and sells a single product. Demand for the product exceeds
the expected production capacity of Tungach ltd. The holding of stocks of the finished product is
avoided if possible because the physical nature of the produce is such that it deteriorates quickly
and stocks may become unsaleable.
A standard marginal cost system is in operation. Feedback reporting takes planning and
operational variances into consideration.
The Mgt accountant has given the following operating statement for period 9:
Tungach Ltd.
Operating Statement – Period 9
(Rs.) (Rs.)
Original budgeted contribution 36,000
Revision Variances:
Material usage 9,600 (Adverse)
Material Price 3,600 (Favorable)

E M Reddy Page | 155


AMA-Notes

Wage rate 1,600 (Favorable) 4,400 (Adverse)


Revised budgeted contribution 31,600
Sales volume variance:
Causal factor
Extra Capacity 4,740 (Favorable)
Productivity drop 987.5 (Adverse)
Idle time 592.5 (Adverse)
Stock increase 2,370 (Adverse) 790 (Favorable)
Revised Standard contribution for sales achieved 32, 390
Other Variances
Material usage 900 (Favorable)
Material Price 3,120 (Adverse)
Labour Efficiency 1,075 (Adverse)
Labour Idle time variance 645 (Adverse)
Labour rate variance 2,760 (Adverse) 6,700 (Adverse)
Actual Contribution 25,690
Other data are available are as follows:
(i) The original standard contribution per product unit as determined at period 1 was:
Particulars Rs. Rs.
Selling Price 30
Less: Direct Material 1.5 Kilos at Rs.8 12
Direct Labour 2 hours at Rs.4.50 9 21
Contribution 9
(a) A permanent change in the product specification was implemented from period 7
onwards. It was estimated that this change would require 20% additional material per
product unit. The current efficient price of the material has settled at Rs.7.50 per kilo.
(b) Actual direct material used during period 9 was 7,800 Kilos of Rs.7.90 per Kilo. Any
residual value is due to operational problems.
(c)The original standard wage rate overestimated the degree of trade union pressure during
negotiations and was 20 Paisa higher than the rate subsequently agreed. Tungach Ltd
made a short-term operational decision to pay the workforce at Rs.4.60 per hour during
period 7 to 9 in an attempt to minimize the drop in efficiency likely because of the
produce specification change. The management succeeded in extending the production
capacity during the period 9 and the total labour hours paid was 9,200 Hours which is
included 150 Hours of Idle time.
(ii)
Budgeted production and sales quantity (period 9) 4,000 Units
Actual Sales quantity (period 9) 4,100 Units
Actual production quantity (period 9) 4,400 Units
(iii) Stock of finished goods are valued at the current efficient standard cost.
Required:
(a) Prepare detailed figures showing how the material and labour variances in the operating
statement have been calculated.
(b) Prepare detailed figures showing how the sales volume variance has been calculated for each
casual factor shown in the operating statement.

Solution:

E M Reddy Page | 156


AMA-Notes

A.
Part 1: Material Planning Variances

Step 1: Computation table


[1] [2] [3]
SQ x SP AQ x AP AQ x SP
4,000 Units x 2 Hours x Rs.4.5 4,000 Units x 2 Hours x Rs.4.6 4,000 Units x 2 Hours x Rs.4.5
Rs. 48,000 Rs. 54,000 Rs. 57,600

Step 2: Variance Calculation

Total Material
Planning Variance (1-
2) = 48,000 – 54,000
= 6,000 (Adverse)
Planning Material Planning Material
Price Variance (3-2) = Usage Variance (1-3)
57,600 – 54,000 = = 48,000 – 57,600 =
3,600 (Favourable) 9,600 (Adverse)

Part 2: Material Operating Variances

Step 1: Computation table


[1] [2] [3]
SQ x SP AQ x AP AQ x SP
7,920 Kgs x Rs.7.5 7,800 Kgs x Rs.7.9 7,800 Kgs x Rs.7.5
Rs. 59,400 Rs. 61,620 Rs. 58,500

Step 2: Variance Calculation

Material Cost
Variance (1-2) =
59,400 – 61,620 =
2,220 (Adverse)
Material Price Material Usage
Variance (3-2) = Variance (1-3) =
58,500 – 61,620 = 59,400 – 58,500 = 900
3,120 (Adverse) (Favourable)

Working Note: Calculation of Standard quantity for actual output (SQ)


Input Output
1.8 Kgs 1 Unit
4,400 Units x 1.8 Kgs = 7,920 Kgs 4,400 Units

E M Reddy Page | 157


AMA-Notes

Part 3: Labour Planning Variances

Step 1: Computation table


[1] [2] [3]
SH x SR AH x AR AH x SR
4,000 Units x 2 Hours x Rs.4.5 4,000 Units x 2 Hours x Rs.4.3 4,000 Units x 2 Hours x Rs.4.5
Rs. 36,000 Rs. 34,400 Rs. 36,000

Step 2: Variance Calculation

Labour Planning
Variance (1-2) =
36,000 – 34,400 =
1,600 (Favourable)
Labour Planning Rate Labour Planning
Variance (3-2) = Efficiency Variance
36,000 – 34,400 = (1-3) = 36,000 –
1,600 (Favourable) 36,000 = 0
Part 4: Labour Operating Variances

Step 1: Computation table


[1] [2] [3]
SH x SR AH x AR AH x SR
8,800 Hours x Rs.4.3 9,200 Hours x Rs.4.6 9,200 Hours x Rs.4.3
Rs. 37,840 Rs. 42,320 Rs. 39,560

Step 2: Variance Calculation

Labour Cost Variance


(1-2) = 37,840 –
42,320 = 4,480
(Adverse)
Labour Rate Variance Labour Efficiency
(3-2) = 39,560 – Variance (1-3) =
42,320 = 2,760 37,480 – 39,560 =
(Adverse) 1,720 (Adverse)

Labour Idle time variance = Idle time x SR = 150 Hours x Rs.4.3 = 645 (Adverse)
Labour Revised Efficiency Variance:
[1] [3]
SH x SR AH (W) x SR
8,800 Hours x Rs.4.3 9,050 Hours x Rs.4.3
Rs. 37,840 Rs. 38,915

E M Reddy Page | 158


AMA-Notes

Labour Revised efficiency variance (1-3) = 37,840 – 38,915 = 1,075 (Adverse)

Working Note: Calculation of Standard Hours for actual output (SH)


Input Output
2 Hours 1 Unit
4,400 Units x 2 Hours = 8,800 Hours 4,400 Units

B.

Part 1: Sales Volume Profit Variance – Margin Approach

Step 1: Standard Cost per unit


Particulars Computation Amount (Rs.)
Materials 1.8 Kgs x Rs.7.5 13.50
Labour 2 Hours x Rs.4.3 8.6
Standard Cost 22.1

Step 2: Calculation of Margins

BM = BP – SC = Rs. 30 – Rs.22.1 = Rs.7.9


AM = AP – SC = Rs. 30 – Rs.22.1 = Rs.7.9

AP = BP, because there is no selling price variance in the sum.

Step 3: Computation table


[1] [2] [3]
BQ x BM AQ x AM AQ x BM
4,000 Units x Rs.7.9 4,100 Units x Rs.7.9 4,100 Units x Rs.7.9
Rs. 31,600 Rs. 32,390 Rs. 32,390

Step 4: Variance Calculation

Total Sales Variance


(1-2) = 31,600 –
32,390 = 790
(Favourable)
Sales Volume
Sales Price Variance
Variance (1-3) =
(3-2) = 32,390 –
31,600 – 32,390 = 790
32,390 = 0
(Favourable)

Part 2: Causal factors of sales volume variance

Step 1: Extra Capacity


Budgeted Hours 4,000 Units x 2 Hours = 8,000 Hours

E M Reddy Page | 159


AMA-Notes

Actual Hours 9,200 Hours


Extra Capacity 9,200 Hours – 8,000 Hours = 1,200 Hours
Extra Output 1,200 Hours/2 Hours = 600 Units
Capacity Variance 600 Units x Rs.7.9 = Rs. 4,740 (Favorable)

Step 2: Productivity drop


Standard Hours 4,400 Units x 2 Hours = 8,800 Hours
Actual Hours Worked 9,200 Hours – 150 Hours = 9,050 Hours
Extra hours due to inefficiency 9,050 Hours – 8,800 Hours = 250 Hours
Volume Drop 250 Hours/2 = 125 Units
Productivity Variance 125 Units x Rs.7.9 = Rs.987.5 (Adverse)

Step 3: Idle time variance


Hours List 150 Hours
Production Lost 150 Hours/2 Hours = 75 Units
Idle time variance 75 Units x Rs.7.9 = 595.5 (Adverse)

Step 4: Stock Increase


Production 4,400 Units
Sales 4,100 Units
Stock Variance 300 Units x Rs.7.9 = Rs. 2,370 (Adverse)

Notes:

1) There is a volume increase of 100 Units and since a unit can give Rs.7.9 profit the sales volume variance
is Rs.790 (Favourable) (100 Units x Rs.7.9)
2) Understanding the reason for 4,000 Units sales increasing to 4,100 Units:
Particulars Units
Budgeted Output 4,000
Add: Extra Capacity 600
Less: Idle time (75)
Less: Productivity Drop (125)
Actual Production 4,400
Less: Increase in stock 300
Actual Sales 4,100

3) When volume drops by 1 Unit, we do not produce and sell 1 unit. Due to not selling 1 unit we lose
Rs.30 selling price and by not producing 1 Unit we saved production cost of Rs.22.1, thereby loosing
net Rs.7.9.
4) However, in case of stock increase of 300 Units we did not sell those units and hence lost selling price
of Rs.30 but we cannot save Rs.22.1 because the units are produced. Then how we consider the Rs.22.1
to be saved in our calculation?
Answer: The cost is incurred but deferred to the next year through closing stock. Hence it is considered
as savings.
5) The sales department has managed to create an extra demand of 100 units, hence should be rewarded
for their performance.

E M Reddy Page | 160


AMA-Notes

6) Production department has produced 400 units extra out of which 300 units does not have demand. It
reflects poor co-ordination between sales and production department. These inventories would involve
extra holding cost.
7) Moreover, the production increase was achieved by creating extra capacity for which additional wage
cost are involved and it is not due to efficiency, the volume has increased. Hence, no credit should be
given to the production manager for the volume increase.

Question no 15: Country Preserves produce Jams, marmalade and preserves. All products are
produced in a similar fashion; the fruits are low temperature cooks in a vacuum process and then
blended with glucose syrup with added citric acid and protein to help setting.
Margins are tight and the firm operates a system of standard costing for each batch of jam.
The standard cost data for a batch of raspberry jam are:
Fruit extract 400 Kg At Rs.0.16 Per Kg
Glucose Syrup 700 Kg At Rs.0.10 Per Kg
Pectin 99 Kg At Rs.0.332 Per Kg
Citric Acid 1 Kg At Rs.2.00 Per Kg
Labour 18 Hours At Rs.36.25 Per Hour
Standard processing loss 3%.
The summer of 2002 proved disastrous for the raspberry crop with a late frost and cool, cloudy
conditions at the ripening period, resulting in a low national yield. As a consequence, normal
prices in the trade were Rs.0.19 per kg for fruit extract although good buying could achieve some
savings. The impact of exchange rates on imports of sugar has caused the price of syrup to
increase by 20%.
The actual results for the batch were:
Fruit extract 428 Kg At Rs.0.18 Per Kg
Glucose Syrup 742 Kg At Rs.0.12 Per Kg
Pectin 125 Kg At Rs.0.328 Per Kg
Citric Acid 1 Kg At Rs.0.95 Per Kg
Labour 20 Hours At Rs.30 Per Hour
Actual output was 1,164 Kgs of raspberry jam.
You are required to:
(a) Calculate the ingredients planning variances that are deemed uncontrollable;
(b) Calculate the ingredients operating variances that are deemed controllable;
(c) Comment on the advantages and disadvantages of variance analysis using planning and
operating variance.
(d) Calculate the mixture and yield variances.
(e) Calculate the total variances for the batch.
Solution:
Part 1: Material Planning Variances

Step 1: Computation table


Particulars [1] [2] [3]
SQ x SP AQ x AP AQ x SP
Fruit extract 400 Kgs x Rs.0.16 400 Kgs x Rs.0.19 400 Kgs x Rs.0.16
Glucose Syrup 700 Kgs x Rs.0.10 700 Kgs x Rs.0.12 700 Kgs x Rs.0.10

E M Reddy Page | 161


AMA-Notes

Pectin 99 Kgs x Rs.0.332 99 Kgs x Rs.0.332 99 Kgs x Rs.0.332


Citric Acid 1 Kg x Rs.2 1 Kg x Rs.2 1 Kg x Rs.2
Total Rs. 168.87 Rs. 194.87 Rs. 168.87

Step 2: Variance Calculation

Material Planning
Variance (1-2) =
168.87 – 194.87 = 26
(Adverse)
Planning Material
Planning Material
Price Variance (3-2) =
Usage Variance (1-3)
168.87 – 194.87 = 26
= 168.87 – 168.87 = 0
(Adverse)

Planning Material Price for Fruit extract = 400 Kgs x Rs.0.03 = Rs.12 (Adverse)
Planning Material Price for Glucose Syrup = 700 Kgs x Rs.0.02 = Rs. 14 (Adverse)
Part 2: Material Operating Variances

Step 1: Computation table


Particulars [1] [2] [3] [3]
SQ x SP AQ x AP AQ x SP RAQ x SP
Fruit extract 400 Kgs x Rs.0.19 428 Kgs x Rs.0.18 428 Kgs x Rs.0.19 432 Kgs x Rs.0.19
Glucose Syrup 700 Kgs x Rs.0.12 742 Kgs x Rs.0.12 742 Kgs x Rs.0.12 756 Kgs x Rs.0.12
Pectin 99 Kgs x Rs.0.332 128 Kgs x Rs.0.328 128 Kgs x Rs.0.332 106.92 Kgs x
Rs.0.332
Citric Acid 1 Kg x Rs.2 1 Kg x Rs.0.95 1 Kg x Rs.2 1.08 Kg x Rs.2
Total Rs. 194.87 Rs. 208.03 Rs. 213.86 Rs. 210.46

Step 2: Variance Calculation

Material Cost
Variance (1-2) =
194.87 – 208.03 =
13.16 (Adverse)

Material Price Material Usage


Variance (3-2) = Variance (1-3) =
213.86 – 208.03 = 194.87 – 213.86 =
5.83 (Favourable) 18.99 (Adverse)

Material Mix Variance Material Yield


(4-3) = 210.46 – Variance (1-4) =
213.86 = 3.40 194.87 – 210.46 =
(Adverse) 15.59 (Adverse)

E M Reddy Page | 162


AMA-Notes

Working Note 1: Calculation of SQ (Standard quantity for actual output)


Input Output
100 Kgs 97 Kgs
1,164/97 x 100 = 1,200 Kgs 1,164 Kgs

Fruit extract = 400 Kgs, Glucose Syrup = 700 Kgs, Pectin = 99 Kgs, Citric Acid = 1 Kg
Working Note 2: RAQ (Actual Quantity in Standard Mix)
Actual Quantity = 428 + 742 + 125 +1 = 1,296 Kgs (in 400:700:99:1)
1296
Fruit Extract = 1,200 x 400 = 432 Kgs
1296
Glucose Syrup = 1,200 x 700 = 756 Kgs
1296
Pectin = x 99 = 106.92 Kgs
1,200
1296
Citric Acid = 1,200 x 1 = 1.08 Kgs

Part 3: Labour Variances


Step 1: Computation table

Step 1: Computation table


[1] [2] [3]
SH x SR AH x AR AH x SR
18 Hours x Rs.36.25 20 Hours x Rs.30 20 Hours x Rs.36.25
Rs. 652.5 Rs. 600 Rs. 725

Step 2: Variance Calculation

Labour Cost Variance


(1-2) = 652.5 – 600 =
52.5 (Favourable)
Labour Efficiency
Labour Rate Variance
Variance (1-3) = 652.5
(3-2) = 725 – 600 =
– 725 = 1,720
125 (Favourable)
(Adverse)

5.12. Planning vs. Operating Variance – Market Size and Market Share Variance

**Question no 16: Super computers manufacture and sell three related PC models. The budgeted
and actual data for 2008 is as follows:
Budgeted for 2008
Selling Price per Variable cost per Contribution Sales Volume in
unit Rs. unit Rs. margin per unit units Rs.
Rs.

E M Reddy Page | 163


AMA-Notes

PC 24,000 14,000 10,000 7,000


Portable PC 16,000 10,000 6,000 1,000
Super PC 1,00,000 60,000 40,000 2,000
Actual for 2008
Selling Price per Variable cost perContribution Sales Volume in
unit Rs. unit Rs. margin per unit units Rs.
Rs.
PC 22,000 10,000 12,000 8,250
Portable PC 13,000 8,000 5,000 1,650
Super PC 70,000 50,000 20,000 1,100
Super Computers derived its total unit sales budget for 2008 from the internal management
estimate of a 20% market share and an industry sales forecast by computer manufacturers
association of 50,000 units. At the end of the year the association reported actual industry sales of
68,750 Units.
Required to compute:
1. Market Share Variance
2. Market Size Variance
3. Sales Quantity Variance
Solution:
Part 1: Calculation of Sales Quantity Variance
Particulars [1] [4]
BQ x BM RAQ x BM
PC 7,000 Units x Rs. 10,000 7,700 Units x Rs. 10,000
Portable PC 1,000 Units x Rs. 6,000 1,100 Units x Rs. 6,000
Super PC 2,000 Units x Rs. 40,000 2,200 Units x Rs. 40,000
Total Rs. 15,60,00,000 Rs. 17,16,00,000

Sales Quantity Variance (1 – 4) = Rs. 15,60,00,000 – Rs. 17,16,00,000 = Rs. 1,56,00,000 (Favorable)

Working Note: Calculation of RAQ (Actual Quantity in standard mix)

Actual Quantity = 8,250 + 1,650 + 1,100 = 11,000 Units (7:1:2)


PC = 7,700 Units
Portable PC = 1,100 Units
Super PC = 2,200 Units

Alternatively, the sales quantity variance can be calculated as follows:

Weighted Contribution (or) Average Contribution:


Particulars Computation Budgeted Margin (Rs.)
PC Rs. 10,000 x 70% 7,000
Portable PC Rs. 6,000 x 10% 600
Super PC Rs. 40,000 x 20% 8,000
Average Weighted Margin 15,600

E M Reddy Page | 164


AMA-Notes

Budgeted Quantity = 10,000 Units


Actual Quantity = 11,000 Units
Increase in Quantity = 1,000 Units
Sales Quantity Variance = 1,000 Units x Rs. 15,600 = Rs. 1,56,00,000 (Favorable)

Part 2: Market Size Variance


Budgeted Quantity 50,000 Units x 20% = 10,000 Units
Revised Budgeted Quantity 68,750 Units x 20% = 13,750 Units
Increase in Quantity 13,750 Units – 10,000 Units = 3,750 Units
Market Size Variance 3,750 Units x Rs. 15,600 = Rs. 5,85,00,000 (Favourable)

Alternatively, Market Size variance can be calculated as follows:


Market Size Variance = (Budgeted Market Size – Actual Market Size) x Budgeted Market Share X Average
Contribution per unit
= (50,000 Units – 68,750 Units) x 20% x Rs. 15,600 = Rs, 5,85,00,000 (Favourable)

Part 3: Market Share Variance


Revised Budgeted Quantity 13,750 Units
Actual Quantity 11,000 Units
Decrease in Quantity 13,750 Units – 11,000 Units = 2,750 Units
Market Share Variance 2,750 Units x Rs. 15,600 = Rs. 4,29,00,000 (Adverse)

Alternatively, Market Share variance can be calculated as follows:


Market Share Variance = (Budgeted Market Share – Actual Market Share) x Actual Market Size X Average
Contribution per unit
= (20% – 16%) x 68,750 Units x Rs. 15,600 = Rs, 4,29,00,000 (Favourable)

Working Note: Actual Market Share


11,000 Units
Actual Market Share = 68,750 Units x 100 = 16%

Summary:

Sales Quantity Variance


= Rs. 1,56,00,000
(Favourable)

Change in Market Size Change in Market Size

Market size Variance = Market Share Variance


Rs. 5,85,00,000 = Rs. 4,29,00,000
(Favourable) (Adverse)

E M Reddy Page | 165


AMA-Notes

Notes:

1) The company targeted to sell 10,000 Units but actually sold 11,000 Units. Can we conclude that the sales
department has done an efficient job?
Answer: No, the sales quantity increase may happen due to reasons
i. The overall market base has increased hence the quantity increased – It is Market Size Variance
and Uncontrollable – Should not be considered for performance evaluation i.e. it is a planning
variance
ii. The company penetrated more into the market increasing its market share thereby increasing its
sales quantity – Increasing Market Share is indicative of Sales department operational
performance. It is operating Variance and hence controllable.
2) In this problem, due to increase in market size the sales should have increased by 3,750 Units had they
retained their targeted market share of 20%
3) However, since they could achieve only 16% penetration it could increase the quantity only by 1,000
Units thereby unable to sell possible 2,750 Units.

5.13. Balance Score Card Method of Reconciliation

Question no 17: ABC ltd manufactures three types of products namely P1, P2 and P3. The
production process requires a single input raw material, a single type of direct labour and a single
energy input. Overheads are shared by all the three products. Budgeted details of the three
products are shown below.
Particulars P1 P2 P3
Labour Hours 0.20 0.25 0.40
Material Kg per unit 1.0 1.1 1.3
Kilo watt Hours 0.5 0.6 0.8
Budgeted sales in units 10,000 6,000 2,000
Forecasted price 15 20 40
The committed fixed overheads are expected to cost Rs. 80,000 per period and the unit cost for the
input resources are as follows:
Labour Rs.20 per hour
Material Rs.4 per Kg
Energy Rs.6 per kilo watt hour
The actual financial results for ABC ltd for the concerned budgeted period are show below:
Sales Rs. 3,85,000
Labour Rs. 1,09,452
Material Rs. 96,448
Energy Rs. 61,671
Total Cost Rs. 2,67,571
Contribution Rs. 1,17,429
Committed fixed cost Rs. 84,000
Profit Rs. 33,429
Additional information regarding inputs and outputs during the concerned period are provided to
you below:
Outputs Inputs
Product Quantity Price Cost Quantity Price
P1 12,000 16 Labour 5,212 Hours 21

E M Reddy Page | 166


AMA-Notes

P2 5,500 22 Materials 21,920 Kg 4.4


P3 1,800 40 Energy 10,633 KWH 5.8
With the help of the above information you are required to calculate the standard margin
(contribution) and subsequently compute the following variances in order to reconcile budgeted
profits with the actual profits.
A. Sales-Activity Variance
b. Price-Recovery Variance
c. Productivity Variance

Solution:

Part 1: Calculation of Material variances


[1] [2] [3]
SQ x SP AQ x AP AQ x SP
20,390 Kg x Rs.4 21,920 Kg x Rs.4.4 21,920 Kg x Rs.4
Rs. 81,560 Rs. 96,448 Rs. 87,680

Working note: SQ for AO


P1 1 Kg x 12,000 Units 12,000 Kgs
P2 1.1 Kg x 5,500 Units 6,050 Kgs
P3 1.3 Kg x 1,800 Units 2,340 Kgs
Total 20,390 Kgs

Material Variances:

Material Price Variance (3 – 2) = Rs. 8,768 (Adverse) – Price Recovery


Material Usage Variance (1 – 3) = Rs. 6,120 (Adverse) - Productivity

Part 2: Calculation of Labour variances


[1] [2] [3]
SH x SR AH x AR AH x SR
4,495 Hours x Rs20 5,212 Hours x Rs.21 5,212 Hours x Rs.20
Rs. 89,900 Rs. 1,09,452 Rs. 1,02,240

Working note: SH for AO


P1 0.2 Hours x 12,000 Units 2,400 Hours
P2 0.25 Hours x 5,500 Units 1,375 Hours
P3 0.4 Hours x 1,800 Units 720 Hours
Total 4,495 Hours

Labour Variances:

Labour rate variance (3 – 2) = 5,212 (Adverse) – Price recovery


Labour efficiency variance (1 – 3) = 14,340 (Adverse) – Productivity

E M Reddy Page | 167


AMA-Notes

Part 3: Energy cost variance (Variable Overhead Variance)


[1] [2] [3]
SKWH x SR AKWH x AR AKWH x SR
10,740 KWH x Rs.6 10,633 KWH x Rs.5.8 10,633 KWH x Rs.6
Rs. 64,440 Rs. 61,671 Rs. 63,798

Working note: SKWH for AO


P1 0.5 KWH x 12,000 Units 6,000 KWH
P2 0.6 KWH x 5,500 Units 3,300 KWH
P3 0.8 KWH x 1,800 Units 1,440 KWH
Total 10,740 KWH

Energy Variances:

Energy expenditure variance (3 – 2) = 2,127 (Favourable) – Price Recovery


Energy efficiency variance (1 – 3) = 642 (Favourable) – Productivity

Part 4: Fixed overhead variance – Committed cost

Expenditure variance = BFO – AFO = 80,000 – 84,000 = 4,000 (Adverse) – Price Recovery

Part 5: Sales Variances under Total Approach


Products [1] [2] [3] [4]
BQ x BP AQ x AP AQ x BP RAQ x BP
P1 10,000 x 15 12,000 x 16 12,000 x 15 10,723 x 15
P2 6,000 x 20 5,500 x 22 5,500 x 20 6,433 x 20
P3 2,000 x 40 1,800 x 40 1,800 x 40 2,144 x 40
Total 3,50,000 3,85,000 3,62,000 3,75,265

Sales Variances:

Selling Price Variance (3 – 2) = 23,000 (Favourable) – Price recovery


Sales volume Variance (1 – 3) = 12,000 (Favourable) – Growth component
Sales Mix Variance (4 – 3) = 13,265 (Adverse) – Growth Component
Sales Quantity Variance (1 – 4) = 25,265 (Favourable) – Growth Component

Working Note: Computation of RAQ (Actual Quantity in Standard Mix)

Actual Quantity = 12,000 + 5,500 + 1,800 = 19,300 Units


P1 = 19,300 Units x 10/18 = 10,723 Units
P2 = 19,300 Units x 6/18 = 6,433 Units
P3 = 19,300 Units x 2/18 = 2,144 Units

Part 6: Material Cost volume variance


[1] [2]
BQ x SP SQ x SP

E M Reddy Page | 168


AMA-Notes

19,200 Kgs x Rs.4 20,390 Kgs x Rs.4


Rs. 76,800 Rs. 81,560

Working note: SQ for BO


P1 1 Kg x 10,000 Units 10,000 Kg
P2 1.1 Kg x 6,000 Units 6,600 Kg
P3 1.3 Kg x 2,000 Units 2,600 Kg
Total 19,200 Kg

Material cost variance due to growth in volume = 4,760 (Adverse) – Growth Component

Part 7: Labour cost volume variance


[1] [2]
BH x SR SH x SR
4,300 Hours x 20 4,495 Hours x 20
86,000 89,900

Working note: SH for BO


P1 0.2 Hours x 10,000 Units 2,000 Hours
P2 0.25 Hours x 6,000 Units 1,500 Hours
P3 0.4 Hours x 2,000 Units 800 Hours
Total 4,300 Hours

Labour cost variance due to growth in volume = 3,900 (Adverse) – Growth Component

Part 8: Energy cost volume variance (Variable Overhead Variances)


[1] [2]
SKWH x SR SKWH x SR
10,200 x 6 10,740 x 6
61,200 64,440

Working note: SKWH for AO


P1 0.5 KWH x 10,000 Units 5,000 KWH
P2 0.6 KWH x 6,000 Units 3,600 KWH
P3 0.8 KWH x 2,000 Units 1,600 KWH
Total 10,200 KWH

Energy cost variance due to growth in volume = 3,240 (Adverse) – Growth Component

Part 9: Computation of Budgeted Profit


Particulars Amount (Rs.)
Budgeted Sales 3,50,000
Less:
Budgeted Material Cost 76,800

E M Reddy Page | 169


AMA-Notes

Budgeted Labour Cost 86,000


Budgeted Energy Cost 61,200
Budgeted Committed Cost 80,000
Budgeted Profit 46,000

Part 10: Reconciliation Statement


Items Budget Growth Price Recovery Productivity Actual
Sales 3,50,000 12,000 (F) 23,000 (F) - 3,85,000
Materials 76,800 4,760 (A) 8,768 (A) 6,120 (A) 96,448
Labour 86,000 3,900 (A) 5,212 (A) 14,340 (A) 1,09,452
Energy 61,200 3,240 (A) 2,127 (F) 642 (F) 61,671
Committed 80,000 - 4,000 (A) - 84,000
Profit 46,000 100 (F) 7,147 (F) 19,818 (A) 33,429

Notes - Balance Score card approach of presenting Reconciliation Statement:

1) A company’s profit can increase or decrease due to following 3 factors:


i. Increase in volume results in increase in profits. – This is called “Growth Component”.
ii. Increase in Volume alone cannot assure increase in profits. The output prices should be
properly recovering the input prices. – This is called “Price Recovery Component”.
iii. Increase in volume and efficient recovery of prices need not assure increased profit. It also
depends on how the resources are utilized with minimum wastage. – This is called “Productivity
Component”.
2) In our Reconciliation Statement we classified all the variances into one of the three categories.
i. Growth Component
a) Sales Volume Variance
b) Material Cost Volume Variance
c) Labour Cost Volume Variance
d) Variable Overhead Volume variance
ii. Price Recovery Component
a) Selling Price Variance
b) Material Price Variance
c) Labour Rate Variance
d) Variable Overhead Expenditure Variance
e) Fixed Overhead Expenditure Variance
iii. Productivity Component
a) Material Usage Variance
b) Labour Efficiency Variance
c) Variable Overhead Efficiency Variance
3)
Items Growth Price Recovery Productivity
Sales Yes Yes No
Material Yes Yes Yes
Labour Yes Yes Yes
Variable Overhead Yes Yes Yes
Fixed Overhead - Yes -

E M Reddy Page | 170


AMA-Notes

5.14. Miscellaneous Concepts – Standard Costing Ratios

1) There are 3 Standard Costing Ratios:

Volume Ratio (1/3)

Capacity Ratio (4/3) Efficiency Ratio (1/4)

Note: Columns has to be taken from the Fixed Overhead Computation table
2) It is further expanded as follows:
SH x SR SH
i. Volume Ratio = BH x SR = BH
AH x SR AH
ii. Capacity Ratio = BH x SR = BH
SH x SR SH
iii. Efficiency Ratio = =
AH x SR AH

Question no 18: The budgeted production for July in the finishing department of a pottery
manufacturer is 4,500 cups, 4,000 saucers and 6,250 plates. In one standard hour a direct operative
is expected to be able to finish either, 30 cups, or 40 saucers, or 25 plates. During period July, 400
direct labour hours were worked and actual production was 4,260 cups, 6,400 Saucers and 3,950
plates.
Required:
Using the above information calculate for July:
(i) Productivity/Efficiency Ratio
(ii) Production Volume/Activity Ratio
(iii) Capacity utilization Ratio

Solution:

Step 1: Calculation of Budgeted Hours


Particulars Computation Hours
Cups 4,500 Cups / 30 Cups 150 Hours
Saucers 4,000 Saucers / 40 Saucers 100 Hours
Plates 6,250 Plates / 25 Plates 250 Hours
Total Hours 500 Hours

Step 2: Calculation of Standard Hours


Particulars Computation Hours
Cups 4,260 Cups / 30 Cups 142 Hours
Saucers 6,400 Saucers / 40 Saucers 160 Hours
Plates 3,950 Plates / 25 Plates 158 Hours
Total Hours 460 Hours

Step 3: Actual Hour (Given) = 400 Hours

E M Reddy Page | 171


AMA-Notes

Step 4: Calculation of Production Ratios/Standard Costing Ratios

Volume Ratio (SH/


BH) = 460/500 = 92%

Efficiency Ratio (SH/


Capacity Ratio (AH/
AH) = 460/400 =
BH) = 400/500 = 80%
115%

Notes:

1) The Company Plan to work 500 Hours but actually worked only for 400 Hours producing 460 Hours of
Output.
2) In other words, it utilized only 80% of its capacity at 115% efficiency to achieve 92% if the targeted
volume.
3) Volume Ratio = Capacity Ration X Efficiency Ratio
4) Volume Ratio also called “Activity Ratio”, Efficiency Ratio also called “Productivity Ratio” and
Capacity Raito also called “Capacity Utilization Ratio”.

**Question no 19: Following a strategy of production differentiation, West Wood Corporation


makes a high-end kitchen range good, KE8. Westwood presents the following data for the years
2008 and 2009.
2008 2009
Units of KE8 produced and sold 40,000 42,000
Selling price Rs. 1,000 Rs. 1,100
Direct Materials (Sq. feet) 1,20,000 1,23,000
Direct materials costs per square feet Rs.100 Rs.110
Manufacturing capacity of KE8 50,000 units 50,000 units
Total conversation costs Rs. 100,00,000 Rs. 110,00,000
Conversion costs per unit of capacity Rs.200 Rs.220
Selling and customer-service capacity 300 Customers 290 Customers
Total selling and customer-service costs Rs.72,00,000 Rs.72,50,000
Cost per customer of selling and customer – Service Capacity Rs. 24,000 Rs. 25,000
Westwood produces no defective units, but it reduces direct materials usage per unit of KE8 in
2009. Conversion costs in each year depend upon production capacity defined in terms of KE8
units that can be produced. Selling and customer – service costs depend upon the number of
customers that the selling and service functions are designed to support. Westwood has 230
customers in 2008 and 250 customers in 2009.
Required:
1. Describe briefly key elements that you would include in Westwood's balanced score card.
2. Calculate the growth, price-recovery, and productivity components that explain the change in
operating Income from 2008 to 2009.

E M Reddy Page | 172


AMA-Notes

Solution:

Part 1: Calculation of Budgeted Profit (2008) and Actual Profit (2009)


Items Budget (Rs. in ‘000) Actual (Rs. in ‘000)
Sales 40,000 46,200
Less:
Material Cost 12,000 13,530
Conversion Cost (Labour + Overheads) 10,000 11,000
Selling and Customer Service Cost 7,200 7,250
Profit 10,800 14,420

Increase in Profit = Rs. 36,20,000

Part 2: Change in operating income due to growth component

Step 1: Sales Volume Variance


[1] [3]
BQ x BP AQ x BP
40,000 Units x Rs. 1,000 42,000 Units x Rs. 1,000
Rs. 4,00,00,000 Rs. 4,20,00,000

Sales Volume Variance (1 – 3) = 20,00,000 (Favourable)

Step 2: Material Cost Volume Variance


[1] [2]
BQ x BP SQ x BP
40,000 Units x 3 Sq. feet x Rs.100 42,000 Units x 3 Sq. feet x Rs.100
Rs. 1,20,00,000 Rs. 1,26,00,000

Material Cost change due to growth in volume = 6,00,000 (Adverse)

Note: The change in operating income due to change in volume or growth in business is Rs. 20,00,000
(Favourable) – Rs. 6,00,000 (Adverse) = Rs. 14,00,000 (Favourable). This is nothing but Sales Volume
Variance under margin approach (Marginal Costing System)

Part 3: Change in operating income due to price recovery component

Step 1: Selling Price Variance


[2] [3]
AQ x AP AQ x BP
42,000 Units Rs. 1,100 42,000 Units Rs. 1,000
Rs. 4,62,00,000 Rs. 4,20,00,000

Selling Price Variance (3 – 2) = 42,00,000 (Favourable)

Step 2: Material Price Variance

E M Reddy Page | 173


AMA-Notes

[2] [3]
AQ x AP AQ x SP
1,23,000 Sq. feet x Rs.110 1,23,000 Sq. feet x Rs.100
Rs. 1,35,30,000 Rs. 1,23,00,000

Material Price Variance (3 – 2) = Rs. 12,30,000 (Adverse)

Step 3: Conversion cost expenditure variance


[2] [3]
AFOH BFOH
Rs. 1,10,00,000 Rs. 1,00,00,000

Fixed Overhead Expenditure Variance (3 – 2) = 10,00,000 (Adverse)

Step 4: Selling and customer service capacity variance


[2] [3]
Actual Capacity x AP Actual Capacity x SP
290 Customer Capacity x Rs. 25,000 290 Customer Capacity x Rs. 24,000
Rs. 72,50,000 Rs. 69,60,000

Customer service Capacity variance (3 – 2) = 2,90,000 (Adverse)

Part 4: Change in operating income due to productivity

Step 1: Material Usage Variance


[1] [3]
SQ x SP AQ x SP
42,000 Units x 3 Sq. feet x Rs.100 1,23,000 Kgs x Rs.100
Rs. 1,26,00,000 Rs. 1,23,00,000

Material Usage Variance (1 – 3) = Rs. 3,00,000 (Favourable)

Step 2: Selling and customer service capacity variance


[1] [3]
Budgeted Capacity x SP Actual Capacity x SP
300 Customer Capacity x Rs. 24,000 290 Customer Capacity x Rs. 24,000
Rs. 72,00,000 Rs. 69,60,000

Customer service Capacity variance (1 – 3) = 2,40,000 (Favourable)

Part 5: Reconciliation Statement


Rs. in ‘000
Items Budget Growth Price Recovery Productivity Actual
Sales 40,000 2,000 (F) 4,200 (F) - 46,200
Materials (12,000) 600 (A) 1,230 (A) 300 (F) (13530)

E M Reddy Page | 174


AMA-Notes

Conversion Cost (10,000) - 1,000 (A) - (11,000)


Customer Support Cost (7,200) - 290 (A) 240 (F) (7250)
Profit 10,800 1,400 (F) 1,680 (F) 540 (F) 14,420

Notes:

1) Variable costs are Volume driven costs and fixed costs are Capacity driven costs.
2) We can compare two years fixed cost only if they are designed to support the same capacity.
3) Conversion cost incurred in 2008 and 2009, both are to support the same 50,000 Units capacity. Thus
the Rs. 10,00,000 extra fixed cost is purely due to expenditure and classified under price recovery
component.
4) The 2008 customer service cost Rs. 72,00,000 is incurred to support 300 customer capacity. However,
the 2009 cost can support only 290 customer capacity. Hence we cannot compare 2008 and 2009 cost
and say expenditure variance is Rs. 50,000 (Adverse) because they are supporting different capacities.
5) The increase in fixed cost of Rs. 50,000 should be analyzed as follows:
i. Change due to Capacity – (300 – 290) x Rs. 24,000 = Rs. 2,40,000 (Favourable) – By downsizing
excess capacity the management is saving a fixed cost of Rs. 2,40,000 which is due to they are
efficient decision making. Hence it is productivity component.
ii. Change due to Expenditure – (Rs. 69,60,000 – Rs.72,50,000) = Rs. 2,90,00 (Adverse) – Had they
negotiated the expenditure at the same price level of 2008 they should have incurred only Rs.
6,60,000 but they incurred actually Rs. 72,50,000. This extra cost is price recovery component.

5.15. Partial Plan vs. Single Plan

Partial Plan Single Plan


“WIP & Finished Goods” Stock is valued at All the “three stocks” are valued at “Standard Cost”
“Standard Cost” and “Raw Material” Stock at
“Actual Cost”
WIP account is debited with Actual Cost WIP account is debited with Standard Cost
All Variances arises from WIP account Variances arises from their respective accounts
Both Material Price & Usage Variance are calculated Material price variance calculated at the time of
at the time of consumption purchase and Usage variance at the time of
consumption
For Price & Usage variance AQ means Actual For Price Variance AQ means Actual Quantity
Quantity Consumed purchased and for usage AQ means Actual Quantity
consumed
Variances are calculated at the end of the period Variance are calculated on real time basis as and
when they arise

Question no 20:
Material purchased 10,000 pieces at Rs.1.10 Rs. 11,000
Material consumed 9,500 pieces at Rs.1.10 Rs. 10,450
Actual wages paid 2,475 hours at Rs.3.50 Rs. 8,662.50
Actual factory expenses incurred Rs. 17,000
Budgeted factory overheads Rs. 16,500
Units produced and sold 900 units @ Rs.60 per unit.
The standard rates and prices are as under:

E M Reddy Page | 175


AMA-Notes

Direct Materials Rs.1.00 per piece


Standard input 10 pieces per unit
Direct Labour rate Rs.3.00 per hour
Standard labour requirement 2.5 hours per unit
Overheads Rs.6.00 per labour hour
Prepare ledger accounts under partial and single plan.

Solution:

Part 1: Calculation of Cost Variances

Step 1: Material Cost Variance


[1] [2] [3]
SQ x SP AQ x AP AQ x SP
9,000 Pcs x Rs.1 9,500 Pcs x Rs.1.1 9,500 Pcs x Rs.1
9,000 10,450 9,500

Material Variances:

Material Price Variance (3 – 2) = 950 (Adverse)


Material usage variance (1 – 3) = 500 (Adverse)

Step 2: Labour cost variances


[1] [2] [3]
SH x SR AH x AR AH x SR
900 x 2.5 Hours x 3 2,475 Hours x 3.5 2,475 Hours x 3
6,750 8,662.5 7,425

Labour Variances:

Labour rate variance (3 – 2) = 1,237.5 (Adverse)


Labour efficiency variance (1 – 3) = 675 (Adverse)

Step 3: Fixed Overhead Variance


[1] [2] [3] [4]
SH x SR or AO x SR AFOH BFOH AH x SRs or SO x SR
900 Units x 15 17,000 16,500 2,475 x 6
13,500 17,000 16,500 14,850

Fixed Overhead Variances:

Fixed Overhead expenditure variance (3 – 2) = 500 (Adverse)


Fixed Overhead volume variance (1 – 3) = 3,000 (Adverse)
Fixed Overhead capacity variance (4 – 3) = 1,650 (Adverse)
Fixed Overhead efficiency variance (1 – 4) = 1,350 (Adverse)

E M Reddy Page | 176


AMA-Notes

Part 2: Ledger accounts under partial plan


Raw Material Control Account
To Cash 11,000 By WIP 10,450
By Bal C/d 550
Total 11,000 Total 11,000

WIP Control Account


To Raw Material Control a/c 10,450 By MPV a/c 950
To Wages Control a/c 8,662.5 By MUV a/c 500
To Production OH Control a/c 17,000 By LRV a/c 1,237.5
By LEV a/c 675
By FOEXV a/c 500
By FOVVOLV a/c 3,000
BY FG a/c 29,250
Total 36,112.5 Total 36,112.5

Finished Goods Control Account


To WIP 29,250 By Cost of Sales 29,250
Total 29,250 Total 29,250

Wages Control Account


To Cash 8,662.5 By WIP 8,662.5
Total 8,662.5 Total 8,662.5

POH Control Account


To Cash 17,000 By WIP 17,000
Total 17,000 Total 17,000

Cost of Sales Account


To FG 29,250 By Costing P&L 29,250
Total 29,250 Total 29,250

Costing P&L Account


To Cost of sales a/c 29,250 By Sales 54,000
To MPV a/c 950
To MUV a/c 500
To LRV a/c 1,237.5
To LEV a/c 675
To FOEXV a/c 500
To FOVOLV a/c 3,000
To Profit 17,887.5
Total 54,000 Total 54,000

E M Reddy Page | 177


AMA-Notes

Notes:

1) In Partial Plan the material price variance is calculated only for consumed units. The units in stock will
be valued at actual cost and the price variance inside it will be recognized in the next period when the
stock is consumed.
Raw Material – 10,000 Pcs purchased at Rs.1.1
i. 9,500 Pcs Consumed – Rs.0.10 price variance for the 9,500 Pcs i.e. Rs.950 (Adverse) is
recognized in this year.
ii. 500 Pcs in Stock – Valued at Rs.1.10 i.e. actual cost. 500 Pcs x Rs.1.10 = Rs.550. The Price
variance of 50 (Adverse) taken to next year.
2) The WIP has been debited with actual cost but WIP & Finished Goods should be valued at standard
cost. Hence, the actual cost WIP account should be brought to standard cost by debiting and crediting
variances.
i. Favourable Variances – Credit the Variance Account and Debit the WIP Account
ii. Adverse Variances – Debit the Variance Account and Credit the WIP Account
3) Since the WIP account is brought to standard cost, what goes out of that account to finished goods will
also be standard cost and what remains as closing WIP will also be at standard cost.
4) Standard Cost per unit:
Material (10 Pcs x Rs.1) = Rs.10
Labour (2.5 Hours x Rs.3) = Rs.7.5
Fixed Overhead (2.5 Hours x Rs.6) = Rs.15
Total Cost = Rs.325
Standard Cost of FG produced = 900 Units x Rs.32.5 = Rs. 29,250
5) In the costing P&L account, we credit actual sales but debit standard cost of sales. Hence, we need to
bring the standard cost of sales to actual cost of sales. This can be done by glossing all the variance
accounts and transferring into Costing P&L account.

Part 3: Preparation of Ledger accounts under single plan

A. Calculation of Material Price Variances:


[2] [3]
AQ (Purchased) x AP AQ (Purchased) x SP
10,000 Pcs x Rs.1.1 10,000 Pcs x Rs.1
11,000 10,000

Material Price Variance (3 – 2) = 1,000 (Adverse)

B. Ledger Accounts

Raw Material Control Account


To Cash 11,000 By MPV 1,000
By MUV 500
By WIP 9,000
By Bal c/d 500
Total 11,000 Total 11,000

E M Reddy Page | 178


AMA-Notes

WIP Control Account


To Raw Material Control a/c 9,000 BY FG a/c 29,250
To Wages control a/c 6,750
To Production OH control a/c 13,500
Total 29,250 Total 29,250

Finished Goods Control Account


To WIP 29,250 By Cost of Sales 29,250
Total 29,250 Total 29,250

Wages Control Account


To Cash 8,662.5 By WIP 6,750
By LRV 1,237.5
By LEV 675
Total 8,662.5 Total 8,662.5

POH Control Account


To Cash 17,000 By WIP 13,500
By FOEXP 500
By FOVOLV 3,000
Total 17,000 Total 17,000

Cost of Sales Account


To FG 29,250 By Costing P&L 29,250
Total 29,250 Total 29,250

Costing P&L Account


To Cost of sales a/c 29,250 By Sales 54,000
To MPV a/c 1,000
To MUV a/c 500
To LRV a/c 1,237.5
To LEV a/c 675
To FOEXV a/c 500
To FOVOLV a/c 3,000
To Profit 17,837.5
Total 54,000 Total 54,000

Notes:

1) In Single Plan Material Price Variance is recognized as soon as the purchase is over. Hence, the Rs.0.10
excess payment for entire pieces is booked as variance in this year itself.
2) The Raw Material Stock of 500 Pieces is valued at Standard Price of Rs.1 i.e. Rs.500 (500 Pieces x Rs.1).
3) The Variances are booked on Real time basis. For example, for a job order of 900 Units the bill of
materials allows 9,000 Pcs (900 Units x 10 Pcs). In the first instance the stores department issues only
9,000 Pcs. Any additional requisition will be issued by booking the cost to usage variance and not the
WIP account.

E M Reddy Page | 179


AMA-Notes

4) Profit under Single Plan is Rs. 17,837.5 and in Partial Plan it is Rs. 17,887.5. The difference in profit is
nothing but the price variance inside the Raw material stock.

Question no 21: ST company manufactures ceramic vases. It uses its standard costing system when
developing its flexible budget amounts. In April 2007, 4,000 finished units were produced. The
following information is related to its two direct manufacturing cost categories: Direct Materials
and direct manufacturing labour.
Direct Materials used were 8,000 Kilos. The standard direct material input allowed for one output
unit is 2 Kilos at Rs.15 per kilo. ST purchased 10,000 Kg of materials at Rs.16.50 per kg at a total
cost of Rs. 16,500.
Actual direct manufacturing labour hours were 6,500 hours at a total cost of Rs. 1,32,600. Standard
manufacturing labour time allowed is 1.5 hours per output unit and the standard direct
manufacturing labour cost is Rs.20 per hour.
Required:
1. Calculate the direct materials price variance and efficiency variance and the direct
manufacturing labour price variance and efficiency variance. Base the direct materials price
variance on a flexible budget for actual quantity purchased but base the direct materials
efficiency variance on a flexible budget for actual quantity used
2. Prepare journal entries for a standard costing system that isolated variances at the earliest time
possible.

Solution:

Part 1: Calculation of Material Variances

Step 1: Material Price Variances

Material Price Variance = [AQ Purchased x SP] – [AQ Purchased x AP]


= [10,000 Kgs x Rs.15] – [10,000 Kgs x Rs.16.5]
= Rs. 1,50,000 – Rs. 1,65,000
= Rs. 15,000 (Adverse)

Step 2: Material Usage Variance

Material Usage Variance = [SQ x SP] – [AQ Consumed x SP]


= [8,000 Kgs x Rs.15] – [8,000 Kgs x Rs.15]
= Rs. 1,20,000 – Rs. 1,20,000
=0

Working Note: Calculation of Standard quantity for actual output (SQ)


Input Output
2 Kgs 1 Unit
4,000 Units x 2 Kgs = 8,000 Kgs 4,000 Units

Part 2: Labour Variances

Step 1: Computation table

E M Reddy Page | 180


AMA-Notes

[1] [2] [3]


SH x SR AH x AR AH x SR
6,000 Hours x Rs.20 Rs. 1,32,600 6,500 Hours x Rs.20
Rs. 1,20,000 Rs. 1,32,600 Rs. 1,30,000

Step 2: Variance Calculation

Labour Rate Variance (3 – 2) = Rs.1,30,000 – Rs.1,32,600 = Rs. 2,600 (Adverse)


Labour Efficiency Variance (1 – 3) = Rs. 1,20,000 – Rs. 1,30,000 = Rs. 10,000 (Adverse)

Working Note: Calculation of Standard Hours for actual output (SH)


Input Output
1.5 Hours 1 Unit
4,000 Units x 1.5 Hours = 6,000 Hours 4,000 Units

Part 3: Ledger Accounts


Raw Material Control Account
To Cash 1,65,000 By MPV 15,000
By WIP 1,20,000
By Bal c/d 30,000
Total 1,65,000 Total 1,65,000
Closing Raw Material = [10,000 Kgs – 8,000 Kgs] x Rs.15 = Rs. 30,000
Wages Control Account
To Cash 1,32,6000 By WIP 1,20,000
By LRV 2,600
By LEV 10,000
Total 1,32,600 Total 1,32,600

WIP Control Account


To Raw Material Control a/c 1,20,000 BY FG a/c 2,40,000
To Wages control a/c 1,20,000
Total 2,40,000 Total 2,40,000

Amount transferred to FG = 4,000 Units x Rs.60 = Rs.2,40,000

Working Note: Standard Cost per unit


Items Computation Amount (Rs.)
Material 2 Kgs x Rs.15 30
Labour 1.5 Hours x Rs.20 30
Total 60

Question no 22: X ltd. produces and sells a single product. Standard cost card per unit of the
product is as follows:

E M Reddy Page | 181


AMA-Notes

Particulars Rs.
Direct Material: A 10 Kgs @ Rs.5 per Kg 50.00
: B 5 Kgs @ Rs.6 per Kg 30.00
Direct wages 5 Hours @ Rs.5 per hour 25.00
Variable production overheads 5 hours @ 12 per hour 60.00
Fixed production overheads 25.00
Total Standard Cost 190.00
Standard gross profit 35.00
Standard selling price 225.00
The fixed production overhead has been absorbed on the expected annual output of 25,200 units
produced evenly throughout the year. During the month of December, 2009, the following were for
the actual production of 2,000 unit:
Particulars Rs.
Sales 2,000 Units @ Rs.225 4,50,000
Direct Material: A 18,900 Kg 99,225
: B 10,750 Kg 61,275
Direct wages 10,500 hours (actually worked 10,300 hours) 50,400
Variable production overheads 1,15,000
Fixed production overheads 56,600
Total 3,82,500
Gross profit 67,500
The material price variance is extracted at the time of receipt materials. Material purchases were A:
20,000 Kgs @ Rs. 5.25 per kg; B 11,500 Kgs @ Rs. 5.70 per kg.
Required:
(i) Calculate all variances.
(ii) Prepare an operating statement showing Standard gross profit, Variances Actual gross profit.
(iii) Explain the reason for the difference in actual gross profit in the question and calculated in (ii)
above.
Solution:
Notes:
1) Material usage variance is related to consumption. So for its calculation AQ means Actual Quantity
consumed always.
2) Regarding price variance, the variance can be recognized as soon as the materials are purchased (Single
plan) or only at the time of consumption (Partial plan).
3) In case of single plan, AQ for price variance calculation means AQ purchased. The raw material stock is
valued at standard material cost.
4) In case of partial plan, for price variance computation AQ means AQ consumed and raw material stock
is valued at actual cost.
A.
Part 1: Material Variances
Step 1: Material Usage Variances
Raw Materials [1] [3] [4]

E M Reddy Page | 182


AMA-Notes

SQ x SP AQ Consumed x SP RAQ x SP
A 20,000 Kgs x Rs.5 18,900 Kgs x Rs.5 19,766.66 Kgs x Rs.5
B 10,000 Kgs x Rs.6 10,750 Kgs x Rs.6 9883.33 Kgs x Rs.6
Total Rs. 1,60,000 Rs. 1,59,000 Rs. 1,58,133

Working note 1: Calculation of SQ (Standard quantity for actual output)


Input Output
15 Kgs 1 Unit
2,000 Units x 15 Kgs = 30,000 Kgs 2,000 Units

A = 30,000 Kgs x 10/15 = 20,000 Kgs


B = 30,000 Kgs x 5/15 =10,000 Kgs

Working note no 2: Calculation of RAQ (Actual Quantity in standard mix)

Actual Quantity = 18,900 Kgs + 10,750 Kgs = 29,650 Kgs


A = 29,650 Kgs x 10/15 = 19,766.66 Kgs
B = 29,650 Kgs x 5/15 =9,883.33 Kgs

Material Variances:

Material Usage Variance (1


– 3) = Rs.1,60,000 –
Rs.1,59,000 = Rs.1,000
(Favourable)

Material Yield Variance Material Mix Variance


(1 – 4) = Rs.1,60,000 – (4 – 3) = Rs.1,58,133 –
Rs.1,58,133 = Rs.1867 Rs.1,59,000 = Rs.867
(Favourable) (Adverse)

Step 2: Material Price Variance


Raw Materials [2] [3]
AQ Purchased x AP AQ Purchased x SP
A 20,000 Kgs x Rs.5.25 20,000 Kgs x Rs.5
B 11,500 Kgs x Rs.5.7 11,500 Kgs x Rs.6
Total Rs. 1,70,550 Rs. 1,69,000

Material Price Variance (3 – 2) = Rs. 1,69,000 – Rs. 1,70,550 = Rs. 1,550 (Adverse)

Part 2: Labour Variances


Step 1: Computation table
[1] [2] [3]

E M Reddy Page | 183


AMA-Notes

SH x SR AH x AR AH x SR
10,000 Hours x Rs.5 10,500 Hours x Rs.4.8 10,500 Hours x Rs.5
Rs. 50,000 Rs. 50,400 Rs. 52,500

Working Note: Calculation of SH (Standard Hours for Actual Output)


Input Output
5 Hours 1 Unit
2,000 Units x 5 Hours = 10,000 Hours 2,000 Units

Step 2: Variance Calculation

Labour Cost Variance (1 –


2) = Rs. 50,000 – Rs.
50,400 = Rs.400 (Adverse)

Labour Rate Variance Labour Efficiency


(3 – 2) = Rs. 52,500 – Variance (1 – 3) = Rs.
Rs. 50,400 = Rs.2,100 50,000 – Rs. 52,500 =
(Favourable) Rs 2,500 (Adverse)

Step 3: Idle time Variance and Revised Labour efficiency variance

Idle time Variance = Idle time x Standard Rate = 200 Hours x Rs.5 = Rs.1000 (Adverse)
Revised efficiency variance = Rs. 2,500 (Adverse) – Rs. 1,000 (Adverse) = Rs. 1,500 (Adverse)

Part 3: Variable Overhead variances

Step 1: Standard Rates

Standard Rate/Hour = Rs.12 (Given)


Standard Rate/Unit = 5 Hours x Rs.12 = Rs.60

Step 2: Computation table


[1] [2] [3]
AO x SR AVO AH x SR
2,000 Units x Rs.60 Rs. 1,15,000 10,300 Hours x Rs.12
Rs. 1,20,000 Rs. 1,15,000 Rs. 1,23,600

E M Reddy Page | 184


AMA-Notes

Step 3: Variance Calculation

Variable Overhead Cost


Variance (1-2) =
Rs.1,20,000 – Rs.1,15,000
= Rs.5,000 (Favourable)

Variable overhead Variable overhead


expenditure Variance Efficiency Variance (1-
(3-2) = Rs.1,23,600 – 3) = Rs.1,20,000 –
Rs.1,15,000 = Rs.8,600 Rs.1,23,600 = Rs.3,600
(Favourable) (Adverse)

Part 3: Variable Overhead variances

Step 1: Standard Rates

Standard Rate/Unit = Rs.25 (Given)


Rs.25
Standard Rate/Hour = 5 Hours = Rs.5

Step 2: Computation table


[1] [2] [3] [4]
AO x SR AFO BFO AH x SR
2,000 Units x Rs.25 Rs. 56,600 2,100 Units x Rs.25 10,300 Hours x Rs.5
Rs. 50,000 Rs. 56,600 Rs. 52,500 Rs. 51,500

Note: Budgeted output per annum = 25,000 Units


25,000 Units
Budgeted output per month = = 2,100 Units
12 Months

Step 3: Variance Calculation

E M Reddy Page | 185


AMA-Notes

Fixed Overhead Cost


Variance (1-2) = Rs.50,000
– Rs.56,600 = Rs.6,600
(Adverse)

Fixed overhead Fixed overhead


expenditure Variance Volume Variance (1-3)
(3-2) = Rs.52,500 – = Rs.50,000 –
Rs.56,600 = Rs.4,100 Rs.52,500 = Rs.2,500
(Adverse) (Adverse)

Fixed overhead Fixed overhead


Capacity Variance (4-3) efficiency Variance (1-
= Rs.51,500 – 4) = Rs.51,500 –
Rs.52,500 = Rs.1,000 Rs.50,000 = Rs.1,500
(Adverse) (Adverse)

Part 5: Sales Variances – Margin Approach

Step 1: Calculation of Margins

BM = BP – SC = Rs.225 – Rs.190 = Rs.35


AM = AP – SC = Rs.225 – Rs.190 = Rs.35

Step 2: Computation table


[1] [2] [3]
BQ x BM AQ x AM AQ x BM
2,100 Units x Rs.35 2,000 Units x Rs.35 2,000 Units x Rs.35
Rs. 73,500 Rs. 70,000 Rs. 70,000

Step 3: Variance Computation

Total Sales Variance


(1-2) = Rs.73,500 –
Rs.70,000 = Rs.3,500
(Adverse)
Sales Volume Profit
Selling Price Variance
Variance (1-3) =
(3-2) = Rs.70,000 –
Rs.73,500 – Rs.70,000
Rs.70,000 = 0
= Rs.3,500 (Adverse)

B. Reconciliation Statement – Absorption Costing System

E M Reddy Page | 186


AMA-Notes

Particulars Favorable (Rs.) Adverse (Rs.) Amount (Rs.)


Budgeted profit 73,500
Sales volume profit variance - 3,500 (3,500)
Standard Profit - - 70,000
Material Price Variance - 1,550
Material Usage Variance 1,000 -
Labour Rate Variance 2,100 -
Labour Idle Time Variance - 1,000
Labour Revised Efficiency Variance - 1,500
Variable Overhead Expenditure Variance 8,600 -
Variable Overhead Efficiency Variance - 3,600
Fixed Overhead Expenditure Variance - 4,100
Fixed Overhead Capacity Variance - 1,000
Fixed Overhead Efficiency Variance - 1,500
Total 11,700 14,250 2,550 (Adverse)
Actual Profit 67,450

C.

Step 1: Actual profit Absorption costing system (Single Plan followed)


Sales Rs. 4,50,000
Material Rs. 1,70,550
Labour Rs. 50,400
Variable Overhead Rs. 1,15,000
Fixed Overhead Rs. 56,600
Total Rs. 3,92,550
Less: Value of Closing Raw Material Rs. 10,000 Rs. 3,82,550
Profit 67,450

Valuation of Raw Material Stock using Single Plan:

Material A: [20,000 Kgs – 18,900 Kgs] x Rs.5 = 1,100 Kgs x Rs.5 = Rs. 5,500
Material B: [11,000 Kgs – 10,250 Kgs] x Rs.6 = 750 Kgs x Rs.6 = Rs. 4,500
Total Value of Raw Material Stock = Rs. 5,500 + Rs. 4,500 = Rs. 10,000

Step 2: Understanding the difference in profit

1) In the question, the actual profit was calculated using Partial Plan. This can be proved as follows:
Sales Rs. 4,50,000
Material Rs. 1,70,550
Labour Rs. 50,400
Variable Overhead Rs. 1,15,000
Fixed Overhead Rs. 56,600
Total Rs. 3,92,550
Less: Value of Closing Raw Material Rs. 10,050 Rs. 3,82,500
Profit 67,500

E M Reddy Page | 187


AMA-Notes

Valuation of Raw Material Stock using Single Plan:

Material A: [20,000 Kgs – 18,900 Kgs] x Rs.5.25 = 1,100 Kgs x Rs.5.25 = Rs. 5,775
Material B: [11,000 Kgs – 10,250 Kgs] x Rs.5.70 = 750 Kgs x Rs.5.70 = Rs. 4,275
Total Value of Raw Material Stock = Rs. 5,775 + Rs. 4,275 = Rs. 10,050
2) We value under Single Plan the stock at Rs. 10,000 but the question has valued under Partial Plan at Rs.
10,050. This difference is the reason for the difference between two profits.
3) To be more elaborate the Rs.50 difference is nothing but the variance inside Raw Material Stock which
can be proved as follows:
Material A: 1,100 Kgs x [Rs.5 – Rs.5.25] = 1,100 Kgs x Rs.0.25 = Rs.275 (Adverse)
Material B: 750 Kgs x [Rs.6 – Rs.5.70] = 750 Kgs x Rs.0.30 = Rs.225 (Favourable)
Net Amount = Rs. 275 (Adverse) + Rs.225 (Favourable) = Rs.50 (Adverse)

**Question no 23: From the following intonation show how profit had gone up in detail:
Particulars 2007 2008
Materials 1,00,000 1,32,000
Labour 60,000 66,000
Variable Overhead 12,000 14,000
Fixed Overhead 20,000 24,000
Total Cost 1,92,000 2,36,000
Profit 8,000 17,000
Sales 2,00,000 2,53,000
During the year 2008, selling price and material price have each gone up by 10% and labour by
10%, when compared to 2007.

Solution:

Part 1: Sales Variance – Total Approach

Step 1: Computation table


[1] [2] [3]
BQ x BP AQ x AP AQ x BP
Rs. 2,00,000 Rs. 2,53,000 Rs. 2,30,000

Working Note: Calculation of “AQ x BP”


AP
AP = BP x 110% → BP = 110%

AP AQ x AP Rs.2,53,000
AQ x BP = AQ x 110% = = = Rs. 2,30,000
110% 110%

Step 2: Variance Calculation

Total Sales Variance (1 – 2) = Rs. 2,00,000 – Rs. 2,53,000 = Rs. 53,000 (Favourable)
Selling Price Variance (3 – 2) = Rs. 2,30,000 – Rs. 2,53,000 = Rs. 23,000 (Favourable)
Sales Volume Variance (1 – 3) = Rs. 2,00,000 – Rs. 2,30,000 = Rs. 30,000 (Favourable)

E M Reddy Page | 188


AMA-Notes

Notes:

1) Actual Price = Budgeted Price x 110% (Given in the question)


Budgeted Price = Actual Price / 110%. Therefore, AQ x BP = [AQ x AP]/110%.
2) Budgeted Sales [BQ x BP] is Rs. 2,00,000 and Standard Sales [Budgeted Sales for Actual Output – AQ x
Rs.30,000
BP] is Rs. 2,30,000. Hence, we can conclude that the volume has increased by Rs.2,00,000 x 100 = 15%

Step 3: Sales Volume Profit Variance


Budgeted Profit Rs.8,000
Standard Net Profit Ratio = = Rs.2,00,000 x 100 = 4%
Budgeted Sales

Budgeted Contribution Rs.28,000


Standard P/V Ratio = = Rs.2,00,000 x 100 = 14%
Budgeted Sales

Sales Volume Profit Variance – Absorption Costing System = Sales Volume Variance X Net Profit Ratio
= Rs. 30,000 (F) x 4% = Rs. 1,200 (F)
Sales Volume Profit Variance – Marginal Costing System = Sales Volume Variance X PV Ratio
= Rs. 30,000 (F) x 14% = Rs. 4,200 (F)

Part 2: Material Cost Variances

Step 1: Computation table


[1] [2] [3]
SQ x SP AQ x AP AQ x SP
Rs. 1,00,000 x 115% Rs. 1,32,000 Rs. 1,20,000
Rs. 1,15,000 Rs. 1,32,000 Rs. 1,20,000

Working Note: Calculation of “AQ x SP”


AP
AP = SP x 110% → SP = 110%

AP AQ x AP Rs.1,32,000
AQ x SP = AQ x 110% = = = Rs. 1,20,000
110% 110%

Step 2: Variance Calculation

Material Cost Variance (1 – 2) = Rs. 1,15,000 – Rs. 1,32,000 = Rs. 17,000 (Adverse)
Material Price Variance (3 – 2) = Rs. 1,20,000 – Rs. 1,32,000 = Rs. 12,000 (Adverse)
Material Usage Variance (1 – 3) = Rs. 1,15,000 – Rs. 1,20,000 = Rs. 5,000 (Adverse)

Part 3: Labour Variances

Step 1: Computation table


[1] [2] [3]
SH x SR AH x AR AH x SR
Rs. 60,000 x 115% Rs. 66,000 Rs. 1,20,000

E M Reddy Page | 189


AMA-Notes

Rs. 69,000 Rs. 66,000 Rs. 60,000

Working Note: Calculation of “AH x SR”


AR
AR = SR x 110% → SR = 110%

AR AH x AR Rs.66,000
AH x SR = AH x 110% = = = Rs. 60,000
110% 110%

Step 2: Variance Calculation

Labour Cost Variance (1 – 2) = Rs. 69,000 – Rs. 66,000 = Rs. 3,000 (Favourable)
Labour Rate Variance (3 – 2) = Rs. 60,000 – Rs. 66,000 = Rs. 6,000 (Adverse)
Labour Efficiency Variance (1 – 3) = Rs. 69,000 – Rs. 60,000 = Rs. 9,000 (Favourable)

Part 4: Variable overhead Variances


[1] [2]
SH x SR or AO x SR AVO
Rs. 12,000 x 115% Rs. 14,000
Rs. 13,800 Rs. 14,000

Variable Overhead Cost Variance (1 – 2) = Rs. 13,800 – Rs. 14,000 = Rs. 200 (Adverse)

Part 5: Fixed Overhead Variance

Step 1: Computation table


[1] [2] [3]
SH x SR or AO x SR AFO BFO
Rs. 20,000 x 115% Rs. 24,000 Rs. 20,000
Rs. 23,000 Rs. 24,000 Rs. 20,000

Step 2: Variance Calculation

Fixed Overhead Cost Variance (1 – 2) = Rs. 23,000 – Rs. 24,000 = Rs. 1,000 (Adverse)
Fixed Overhead Expenditure Variance (3 – 2) = Rs. 20,000 – Rs. 24,000 = Rs. 4,000 (Adverse)
Fixed Overhead Volume Variance (1 – 3) = Rs. 23,000 – Rs. 20,000 = Rs. 3,000 (Favourable)

Part 6: Reconciliation Statement – Absorption Costing System


Particulars Favorable (Rs.) Adverse (Rs.) Amount (Rs.)
Budgeted profit 8,000
Material Price Variance - 1,200
Material Usage Variance - 5,000
Labour Rate Variance - 6,000
Labour Efficiency Variance 9,000 -
Variable Overhead Cost Variance - 200
Fixed Overhead Expenditure Variance - 4,000

E M Reddy Page | 190


AMA-Notes

Fixed Overhead Volume Variance 3,000 -


Selling Price Variance 23,000 -
Sales Volume Profit Variance 1,200 -
Total 36,200 27,200 9,000 (Favourable)
Actual Profit 17,000

Part 7: Reconciliation Statement – Marginal Costing System


Particulars Favorable (Rs.) Adverse (Rs.) Amount (Rs.)
Budgeted profit 8,000
Material Price Variance - 1,200
Material Usage Variance - 5,000
Labour Rate Variance - 6,000
Labour Efficiency Variance 9,000 -
Variable Overhead Cost Variance - 200
Fixed Overhead Expenditure Variance - 4,000
Fixed Overhead Volume Variance - -
Selling Price Variance 23,000 -
Sales Volume Profit Variance 4,200 -
Total 36,200 27,200 9,000 (Favourable)
Actual Profit 17,000

5.16. Reverse Working Problems

Question no 24: A company manufactures a food product, data for which for one week have been
analyzed as follows:
Standard Cost Data (Rs.)
Direct Materials: 10 Kgs at Rs.1.50 15
Direct Wages: 5 Hours at Rs.4.00 20
Production Overhead: 5Hours at Rs.5.00 25
Total 60
Profit Margin is 20% of sales price.
Budgeted sales are Rs. 30,000 per week.
Actual Data (Rs.)
Sales 29,880
Direct Materials 6,435
Direct Wages 8,162
Analysis of variances:
Adverse (Rs.) Favourable (Rs.)
Direct Materials Price 585
Direct Materials Usage 375
Direct Labour Rate 318
Direct Labour Efficiency 180
Production Expenditure 200
Overhead Volume 375
It can be assumed that the production and sales achieved resulted in no changes of stock. You are
required, from the data given, to calculate:
a. The actual output;

E M Reddy Page | 191


AMA-Notes

b. The actual profit;


c. The actual price per unit of material;
d. The actual rate per labour hour;
e. The amount of production overhead incurred;
f. The amount of production overhead absorbed;
g. The production overhead efficiency variance;
h. The selling price variance;
i. The sales volume profit variance

Solution:

Part 1: Material Variances


[1] [2] [3]
SQ x SP AQ x AP AQ x SP
4,150 Kgs x Rs.1.5 3,900 Kgs x Rs.1.65 3,900 Kgs x Rs.1.5
Rs. 6,225 Rs. 6,435 Rs. 5,850

Working Note 1: Actual Quantity of Raw Materials

Material Price Variance (3 – 2) = (AQ x Rs.1.5) – Rs. 6,435 = – Rs.585


AQ x Rs.1.5 = Rs. 6,435 – Rs.585
AQ x Rs.1.5 = Rs. 5,850
AQ = Rs. 5,850/Rs.1.5 = 3,900 Kgs

Working Note 2: Calculation of Standard Quantity

Material Usage Variance (1 – 3) = (SQ x Rs.1.5) – Rs. 5,850 = Rs.375


AQ x Rs.1.5 = Rs. 5,850 + Rs.375
AQ x Rs.1.5 = Rs. 6,225
AQ = Rs. 6,225/Rs.1.5 = 4,150 Kgs

Working note 3: Actual Output

SQ means Standard Quantity for actual output.


Input Output
10 Kgs 1 Unit
415 Units x 10 Kgs = 4,150 Kgs 415 Units

Part 2: Labour Variances


[1] [2] [3]
SH x SR AH x AR AH x SR
2,075 Hours x Rs.4 2,120 Hours x Rs.3.85 2,120 Hours x Rs.4
Rs. 8,300 Rs. 8,162 Rs. 60,000

Working Note 1: Calculation of AH

E M Reddy Page | 192


AMA-Notes

Labour Rate Variance (3 – 2) = (AH x Rs.4) – Rs. 8,162 = Rs.318


AQ x Rs.4 = Rs. 8,162 + Rs.318
AQ x Rs.4 = Rs. 8,480
AQ = Rs. 8,480/Rs.4 = 2,120 Hours

Working note 2: Calculation of SH (Standard Hours for Actual Output)


Input Output
5 Hours 1 Unit
415 Units x 5 Hours = 2,075 Hours 415 Units

Part 3: Fixed Overheads


[1] [2] [3] [4]
AO x SR AFO BFO (BO x SR) AH x SR
415 Units x 5 Hours x Rs.5 Rs. 9,800 400 Units x 5 Hours x Rs.5 2,120 Hours x Rs.5
Rs. 10,375 Rs. 9,800 Rs. 10,000 Rs. 10,600

Working Note 1: Calculation of BFO

Fixed Overhead Volume Variance (1 – 3) = Rs. 10,375 – BFO= Rs. 375


BFO = Rs. 10,375 – Rs.375
BFO = Rs. 10,000

Working Note 2: Calculation of AFO

Fixed Overhead Expenditure Variance (3 – 2) = Rs. 10,000 – AFO= Rs. 200


AFO = Rs. 1,000 – Rs.200
AFO = Rs. 9,800

Working Note 3: Break Up of Volume Variance

Fixed Overhead Volume Variance (1 – 3) = Rs. 10,375 – Rs. 10,000 = Rs. 375 (Favourable)
Fixed Overhead Capacity Variance (4 – 3) = Rs. 10,600 – Rs. 10,000 = Rs. 600 (Favourable)
Fixed Overhead Capacity Variance (1 – 4) = Rs. 10,375 – Rs. 10,600 = Rs. 225 (Adverse)

Part 4: Sales Variances

Step 1: Calculation of Margins


Budgeted Sales Rs.30,000
Budgeted Price = Budgeted Output = 400 Units = Rs.75

Actual Sales Rs.29,880


Actual Price = Actual Output = 450 Units = Rs.72

BM = BP – SC = Rs.75 – Rs.60 = Rs.15


AM = AP – SC = Rs.72 – Rs.60 = Rs.12

E M Reddy Page | 193


AMA-Notes

Step 2: Computation table


[1] [2] [3]
BQ x BM AQ x AM AQ x BM
400 Units x Rs.15 450 Units x Rs.12 450 Units x Rs.15
Rs. 6,000 Rs. 4,980 Rs. 6,225

Step 3: Variance Computation

Total Sales Variance (1 – 2) = Rs. 6,000 – Rs. 4,980 = Rs. 1,020 (Adverse)
Selling Price Variance (3 – 2) = Rs. 6,225 – Rs. 4,980 = Rs. 1,245 (Adverse)
Sales Volume Variance (1 – 3) = Rs. 6,000 – Rs. 6,225 = Rs.225 (Favourable)

Part 5: Actual Profit


Items Amount (Rs.)
Sales 29,880
Less:
Material (6435)
Labour (8162)
Production Overhead (9800)
Actual Profit 5,483

Part 6: Final Solution


Particulars Answer
Actual Output 415 Units
Actual Profit Rs. 5,483
Actual Price per Unit of Material 1.65 per Kg
Actual Rate per Labour Hour 3.85 per Hour
Amount of Production Overhead Incurred Rs. 9,800
Amount of Production Overhead Absorbed Rs. 10,375
Production Overhead Efficiency Variance Rs. 225 (Adverse)
Selling Price Variance Rs. 1,245 (Adverse)
Sales Volume Profit Variance Rs. 225 (Favourable)

Question no 25: A company produces a product, which has a standard variable production cost of
Rs.8 per unit made up as follows:
Direct Materials Rs.4.6 (2 Kg x Rs.2.3)
Direct Labour Rs.2.1 (0.7 Hours x Rs.3 per Hour)
Variable Overheads Rs.1.3
Fixed manufacturing costs are treated as period cost.
The following information is available for the period just ended:
Particulars Rs.
Variable manufacturing cost of sales (at standard cost) 2,63,520
Opening stock of finished goods (at standard cost) 1,20,800
Closing stock of finished goods (at standard cost) 1,46,080

E M Reddy Page | 194


AMA-Notes

Direct Material price variance 2,571 (Adverse)


Raw Material used in manufacture (at actual cost) 1,70,310
Direct labour rate variance 4,760 (Adverse)
Direct labour efficiency variance 3,240 (Favourable)
Required for the period ended:
1. Number of units produced
2. Raw Material usage variance
3. Total actual direct labour
4. Actual cost per Kg of raw material

Solution:

Step 1: Calculation of Number of Units Produced


Particulars Computation Units
Sales 2,63,520/8 32,940
Add: Closing Stock 1,46,080/8 18,260
Less: Opening Stock 1,20,800/8 15,100
Units Produced 36,100

Step 2: Material Variances


[1] [2] [3]
SQ x SP AQ x AP AQ x SP
72,200 Kgs x Rs.2.3 72,930 Kgs x Rs.2.34 72,930 Kgs x Rs.2.3
Rs. 1,66,060 Rs. 1,70,310 Rs. 1,67,739

Material Usage Variance (1 – 3) = Rs. 1,66,060 – Rs. 1,67,739 = Rs. 1,679 Adverse)

Working Note 1: Actual Quantity of Raw Materials

Material Price Variance (3 – 2) = (AQ x Rs.2.3) – Rs. 1,70,310= – Rs. 2,571


AQ x Rs.2.3 = Rs. 1,70,310 – Rs. 2,571
AQ x Rs.2.3 = Rs. 1,67,739
AQ = Rs. 1,67,739/Rs.2.3 = 72,930 Kgs

Working Note 2: Calculation of SQ (Standard Quantity for Actual Output)


Input Output
2 Kgs 1 Unit
36,100 Units x 2 Kgs = 72,200 Kgs 36,100 Units

Step 3: Labour Variances


[1] [2] [3]
SH x SR AH x AR AH x SR
25,2705 Hours x Rs.3 24,190 Hours x Rs.3.20 24,190 Hours x Rs.3
Rs. 75,810 Rs. 77,330 Rs. 72,570

E M Reddy Page | 195


AMA-Notes

Working note 1: Calculation of SH (Standard Hours for Actual Output)


Input Output
0.7 Hours 1 Unit
36,100 Units x 0.7 Hours = 25,270 Hours 36,100 Units

Working Note 2: Calculation of AH

Labour Efficiency Variance (1 – 3) = Rs. 75,810 – (AH x Rs.3) = Rs. 3,240


AQ x Rs.3 = Rs. 75,810 – Rs. 3,240
AQ x Rs.3 = Rs. 72,570
AQ = Rs. 72,570/Rs.3 = 24,190 Hours

Working Note 3: Calculation of Actual Labour Cost

Labour Rate Variance = (AH x SR) – (AH x AR)


- Rs. 4,760 = Rs. 72,570 – (AH x AR)
AH x AR = Rs. 72,570 + Rs. 4,760
AH x AR = Rs. 77,330

Step 4: Final Solution

Number of units produced = 36,100 Units


Raw Material usage variance = Rs. 1,670 (Adverse)
Total actual direct labour = Rs. 77,330
Actual cost per Kg of raw material = Rs.2.34

5.17. Investigation of Variances

Question no 26: A company using a detailed system of standard costing finds that the cost of
investigation of variances is Rs. 20,000. If after investigation an out of control situation is
discovered, the cost of correction is Rs. 30,000. If no Investigation is made, the present value of
extra cost involved is Rs. 1,50,000. The probability of the process being in control involved is 0.82
and the probability of the process being out of control is 0.18. You are required to advise:
i. Whether investigation of the variances should be undertaking or not; and
ii. The probability at which it is desirable not to institute investigation into variances.

Solution:

E M Reddy Page | 196


AMA-Notes

Conclusion: The expected cost when investigation is ordered is Rs. 25,400 but the expected without the
investigation is Rs. 27,000. Hence, it is desirable to investigate.

Calculation of Indifference probability:

Let probability of system in control be ‘P’ and system out of control be ‘1 – P’. The indifference probability
is the one where both the decisions has the same expected cost.

[20,000 x P] + [50,000 x (1 – P)] = 1,50,000 (1 – P)


20,000P + 50,0000 – 50,000P = 1,50,000 – 1,50,000 P
20,000P + 1,50,0000P – 50,000P = 1,50,000 – 50,000
1,20,000P = 1,00,000
P = 1,00,000/1,20,000
P = 0.83
1 – P = 1 – 0.83 = 0.17

Conclusion: If the company believes that there is 17% chance of the system being out of control then it is
indifferent between Investigation or No investigation. If the probability exceeds 17%, it will order for
investigation else it will not.

Notes:

1) When variances are reported the company has to make a decision whether or not to order investigation.
2) The advantage of investigation is that it may discover frauds and help the company to prevent the
variances from recurring but there is a cost involved in investigation.
3) 3 factors affect the investigation decisions:

E M Reddy Page | 197


AMA-Notes

i. The Materiality of the Variances


ii. Cost involved in investigation and correction
iii. Probability of system being out of control
4) Decision should be made based on expected cost.

E M Reddy Page | 198


AMA-Notes

6. RELEVANT COSTING

6.1. Learning Objectives

1) Relevant Cost of Materials


2) Relevant Cost of Labour
3) Relevant Cost of Overheads
4) Comprehensive Problems
5) Limiting Factor and Relevant Costing
6) Relevant Costing under uncertainty

6.2. Relevant Cost of Materials

1) In Stock
i. Regularly Used – Relevant Cost (RC) = Purchase Price (or) Replacement Cost
ii. Not Regularly Used
a) Has Disposal Value – Relevant Cost (RC) = Realizable Value
b) Has Alternative Use – Relevant Cost (RC) = The Cost of Material Substituted
c) Has Both – Relevant Cost (RC) = Higher of the Two
2) Not in Stock – Relevant Cost (RC) = Purchase Price
Question no 1: X ltd. has been approached by a customer who would like a special job to be done
for him and is willing to pay Rs.22,000 for it. The job would require the following materials.
Material Total Units Units already Book value of units Realizable Replacement
Required in stock in stock Rs. /Unit value Rs./Unit Cost Rs./Unit
A 1,000 0 - - 6
B 1,000 600 2 2.5 5
C 1,000 700 3 2.5 4
D 200 200 4 6 9
a) Material B is used regularly by X ltd. And if stocks were required for this job they would need to
be replaced to meet other production demand.
b) Material C and D are in stock as the result of previous excess purchase and they have a
restricted use.
No other use could be found for material C but material D could be used in another job as
substitute for 300 units of material E, which currently cost Rs.5 per unit (of which the company has
no units in stock at the moment).
What are the relevant costs of material, in deciding whether or not to accept the contract? Assume
all other expenses on this contract to be specially incurred beside the relevant cost of material are
Rs.550.
Solution:
Part 1: Relevant Material Cost for this Job
Step 1: Material A
Material A is not in stock and hence need to be specifically purchased for this job. Therefore, relevant cost
is its “Purchase Price”.
Relevant Cost = 1,000 Units x Rs.6 = Rs.6,000

E M Reddy Page | 199


AMA-Notes

Step 2: Material B
Material B is partly in stock and partly needs to be purchased. For the units to be purchased, the relevant
cost is “Purchase Price” and since the units in stock are regularly used, they need to be replaced if used for
this job, the relevant cost is “Replace Cost (or) Purchase Price”.
1,000 Units
In Stock = 600 Units – Relevant Cost = 600 Units x Rs.5 = Rs.3,000
Not in Stock = 400 Units – Relevant Cost = 400 Units x Rs.5 = Rs.3,000
Total Relevant Cost = Rs.5,000
Step 3: Material C
Material C also is partly in stock and partly to be purchased. For the units to be purchased, the relevant cost
is “Purchase Price”. The units in stock could be sold for Rs.2.5 if it is not used hence the relevant cost of
using it is “Realizable Value lost”.
1,000 Units
In Stock = 700 Units – Relevant Cost = 700 Units x Rs.2.5 = Rs.1,750
Not in Stock = 300 Units – Relevant Cost = 300 Units x Rs.4 = Rs.1,200
Total Relevant Cost = Rs.2,950
Step 4: Material D
Material D is in stock and not regularly used. If it is not used for this contract the company has two options.
i. Can be disposed off for a realizable value of Rs. 1,200 (200 Units x Rs.6)
ii. Cab be used in place of 300 Units of E. Thereby saving a cost of Rs.1,500 (300 Units x Rs.5)
Obviously, the company will select best of the two. Hence, relevant cost is higher of the two Rs.1,500.
Part 2: Statement of Benefit and Cost
Particulars Amount (Rs.)
A. Benefit (Job Price) 22,000
B. Cost
Material A 6,000
Material B 5,000
Material C 2,950
Material D 1,500
Other Cost 550
C. Net Benefit (A – B) 6,000
Since, net benefit is positive the job should be accepted.
Notes: The book value of units in stock is irrelevant because it is Historical or Sunk Cost.

E M Reddy Page | 200


AMA-Notes

6.3. Relevant Cost of Labour

1) Skilled Labour – Paid on Time guaranteed basis


i. Currently Idle – Relevant Cost (RC) = Nil
ii. Busy
a) Cannot be substituted – Relevant Cost = Contribution lost from abandoned job
b) Can be substituted – Relevant Cost = Wages paid to the substitute worker
2) Unskilled Labour – Appointed on Hire & Fire Basis, hence Relevant Cost is Wages paid
Question no 2: Ram Ltd is evaluating the feasibility of a contract requiring supply of 1,000 units of
component ZED. The labour specification for this contract is as follows:
Type of Labour Hours/Unit Rate/Hour Remarks
Skilled labour 4 5 # Difficult to recruit.
# Paid on time-guarantee basis.
Unskilled labour 6 3 # To be specifically hired for this contract

Ascertain the relevant cost of labour for this contract.


Solution:
1) Skilled Labour:
a) Skilled worker are paid on time-guarantee basis. Hence, the wages paid to them is irrelevant because
it is committed cost.
b) Since, no other detail is given we assume that at present they are idle and the Relevant Cost is “Nil”.
2) Unskilled Labour:
a) They are specifically hired for this contact. Hence, their entire wages is relevant.
b) Relevant Cost (RC) = 1,000 Units x 6 Hours x Rs.3 = Rs.18,000.
Question no 3: A ltd is at present carrying out a research project, which requires spending of
Rs.40,000 towards skilled labour. They are paid on time guaranteed basis. Had they not been
employed in this project, they could have been in some other productive job fetching revenue of
Rs.1,50,000 to the company. For this job, the company has to incur a prime cost of Rs.1,00,000.
Ascertain the relevant labour cost for this research project.
Solution:
1) The time guaranteed wages of Rs.40,000 is irrelevant because it is committed.
2) If the research project is done, then A ltd. could have used this employee for other productive job. By
doing this project, we lose contribution from other job which is opportunity cost.
Revenue = Rs.1,50,000
Less: Cost = Rs.60,000
Contribution Loss = Rs.90,000
3) The Relevant wage cost is contribution loss which is Rs.90,000
Note:
1) While calculating contribution loss (or) profit loss, we ignored the wages of Rs.40,000 because whether
or not the other job is done it will still be incurred.
2) In study material answer is presented as follows:

E M Reddy Page | 201


AMA-Notes

Labour Cost = Wages + Opportunity Cost


= Rs.40,000 + Rs.50,000
= Rs.90,000
Question no 4: XYZ ltd received an order to produce 10,000 units of a component named super-x. It
requires 5 hours of skilled labour. The company already has in its roll an employee possessing the
necessary skills, who is currently paid Rs.5 per hour on time guaranteed basis. At present he is
busy with an urgent job, which would be affected on undertaking this order. To get this job
continued the company has to hire a temporary employee who will paid at Rs.4 per hour.
Ascertain the relevant labour cost for producing the component of super-X.
Solution:
1) The Rs.5 per hour, time guaranteed wages is irrelevant because it is committed.
2) The skilled worker is busy with another job. If he is used for this job, the company has to hire a
substitute worker to continue the other job. The Relevant cost is “Wages paid to substitute worker”.
3) Relevant Cost = 10,000 Units x 5 Hours x Rs.4 = Rs.2,00,000.
Question no 5: A ltd is in construction business, which also carries out painting and maintenance
work during severe winter not being conducive for construction activities. At present the company
is evaluating the viability of a proposal to build a housing complex, for which it has to employ
contract basis a team of highly skilled craftsmen. The compensation for this team works out of
Rs.3,00,000. Though a period of 9 months is sufficient for completion of the contract, due to spells
of bad weather it is estimated to be over in over in a year’s time. During winter, this team could be
used for painting and maintenance work already undertaken by the company, which otherwise
would require to be subcontracted to outsiders. For this, the company has received quotations from
two jobbing builders, one for Rs.50,000 and another for Rs.40,000. This painting and maintenance
work, which if done by the company requires spending on material Rs.10,000. Ascertain the
relevant labour cost for building the housing complex.
Solution:
1) Since the craftsmen or specifically hired for this contract, their wages of Rs.3,00,000 is relevant.
2) Due to the craftsmen, the company can save a net cost of Rs.30,000 (Rs.40,000 – Rs.10,000) on its
maintenance work i.e. Subcontracting charges saved – Rs.40,000 (Lowest Quotation) and material cost
incurred – Rs.10,000.
3) Relevant Labour Cost = Rs.3,00,000 – Rs.30,000 = Rs.2,70,000

6.4. Relevant Cost of Overheads

1) Variable Overheads – Always Relevant


2) Fixed Overheads
i. General Fixed Cost - Irrelevant
ii. Specific Fixed Cost – Relevant
Question no 6: ABC ltd receives an offer for producing 1,000 units of components used in
manufacture of aircraft. For manufacturing each and every unit 4 machine hours are required. The
company absorbs overheads on the basis of machine hours. Currently, the machine hour rate is
Rs.20 per hour, of which Rs.7 us variable and Rs.13 is fixed. If the contract is accepted, the

E M Reddy Page | 202


AMA-Notes

company will additionally incur a fixed overhead of Rs.3,200. Ascertain the relevant overhead cost
for this contract.
Solution:
1) Variable overheads are relevant because they are incremental cost. Relevant Variable overheads = 1,000
Units x 4 Hours x Rs.7 = Rs.28,000
2) The absorbed fixed overheads will be incurred whether or not the units are produced. It is non-
incremental, hence irrelevant.
3) Additional fixed cost of Rs.3,200 is specific for this contract. Hence, it is relevant.
4) The relevant overhead cost for this contract is Rs.31,200 (Rs.28,000 + Rs.3,200)

6.5. Comprehensive Problems

Question no 7: A research project, which to date has cost the company Rs.1,50,000 is under review.
It is anticipated that should the project be allowed to proceed, it will be completed in
approximately one year when the results would be sold to a government agency for Rs.3,00,000.
Shown below are the additional expenses, which the managing director estimated will be necessary
to complete the work.
Material – Rs.60,000.
This material, which has just been received, is extremely toxic and if not used on the project would
have to be disposed of by special means, at a cost of Rs.5,000.
Labour – Rs.40,000.
The men are highly skilled and very difficult to recruit. They were transferred to the project from a
production department and, at a recent board meeting, the works director claimed that if the men
were returned to him he could earn the company each year Rs.1,50,000 extra sales. The accountant
calculated that the prime cost of those sales would be Rs.1,00,000 and the overhead absorbed (all
fixed) would amount to Rs.20,000.
Research staff – Rs.60,000.
A decision has already been taken that this will be the last major piece of research undertaken, and
consequently when work on the project ceases the staff involved will be made redundant.
Redundancy and severance pay have been estimated at Rs.25,000.
Share of general building service – Rs.35,000.
The managing director is not very sure what is included in this expenses. He knows, however, that
the accounts staff charges similar amount every year to each department.
Required:
Assuming the estimates are accurate, advice the managing director whether the project should be
allowed to proceed. You must carefully and clearly explain the reasons for your treatment of each
expense item.
Solution:
1) The cost spent to date on this project of Rs.1,50,000 is irrelevant because it is Sunk cost.
2) The government agency’s fee of Rs.3,00,000 is the relevant benefit of continuing the project.
3) The material price of Rs.60,000 is irrelevant because it is historical (the material is just received and
hence is in stock).
4) If the material is not used for this project, the company should dispose it off by spending Rs.5,000.
The usage avoids this spending. Hence, Rs.5,000 is relevant benefit.

E M Reddy Page | 203


AMA-Notes

5) The relevant labour cost is Rs.90,000 (Refer Question no 3)


6) The Research staff wages of Rs.60,000 is a relevant cost because it will be paid if the project is
continued and avoided if the project is abandoned.
7) The redundancy cost of Rs.25,000 is irrelevant because if the project is abandoned it will be paid today
and if continued will be paid a year later.
8) Share of general building service of Rs.35,000 is irrelevant because it is an apportion expense.
9) Fixed overheads of Rs.20,000 is irrelevant because it is not incremental.
Statement of Cost-Benefit analysis:
Particulars Amount (Rs.) Amount (Rs.)
A. Benefits
Agency fees 3,00,000
Savings in material disposal cost 5,000 3,05,000
B. Cost
Labour 90,000
Research Staff Wages 60,000 1,50,000
C. Net Benefit (A – B) 1,55,00
Recommended to continue the research project since the net benefit is positive.
Question no 8: A company has been making a machine to order for a customer but the customer
has since gone into liquidation and there is no prospect that any money will be obtained from the
winding up of the company.
Costs incurred to date in manufacturing the machine are Rs.50,000 and the progress payments of
Rs.15,000 had been received from the customer prior to the liquidation.
The sale department has found another company willing to buy the machine for Rs.34,000 once it
has been completed.
To complete the work, the following costs would be incurred.
1. Materials: These have been bought at a cost of Rs.6,000. They have no other use and if the
machine is not finished they would be sold for scrap of Rs.2,000.
2. Further labour cost would be Rs.8,000. Labour is in short supply and if the machine is not
finished, the work force would be switched to another job which would earn Rs.30,000 in revenue
and incur direct cost of Rs.12,000.
3. The absorbed fixed overheads is Rs.8,000.
4. Consultancy fees of Rs.4,000. If the work is not completed the consultants contract would be
cancelled at a cost of Rs.1,500.
5. General overheads of Rs.8,000 would be added to the cost of additional work.
Required: Asses whether new customer order should be accepted.
Solution:
Cost-benefit analysis of accepting the new order:
Particulars Amount (Rs.) Amount (Rs.)
A. Benefits
Sales value of new order 34,000 34,000
B. Cost
Material 2,000

E M Reddy Page | 204


AMA-Notes

Labour (30,000 – 4,000) 26,000


Consultancy fee (4,000 – 1,500) 2,500 30,500
C. Net Benefit (A – B) 3,500
1)

Continue the work by accepting new order since the net benefit is positive.
Note: Reasons should be written as above.
**Question no 9: Vishwakarma is a builder. His business will have spare capacity over the coming
six months and he has been investigating two projects.
Project A:
Vishwakarma is tendering for a Scholl extension contract. Normally he prices a contract by adding
100% to direct costs, to cover overheads and profit. He calculates direct costs as the actual cost of
materials valued on first-in-firs-out basis, plus the estimated wages of direct labour. But for this
contract he has prepared more detailed information.
Four types of material will be needed:
Material Quantity (units): Price per unit: (in Rs.)
Needed for Already Purchase price of Current Current
contract in stock units in stock purchase price resale price
Z 1,100 100 7.00 10.00 8.00
Y 150 200 40.00 44.00 38.00
X 600 300 35.00 33.00 25.00
W 200 400 20.00 21.00 10.00
Z and Y are in regular use. Neither X nor W is currently used; X has no foreseeable use in the
business, but W could be used on other jobs in place of material currently costing Rs.16 per unit.
The contract will last for six months and requires two craftsmen, whose basic annual wage cost is
Rs.16,000 each. To complete the contract in time it will also be necessary to pay them a bound of
Rs.700 each. Without the contract they would be retained at their normal pay rate, doing work,
which will otherwise be done by temporary workers, engaged for the contract period at a total cost
of Rs.11,800. Three casual labourers would also be employed specifically for the contract at a cost
of Rs.4,000 each.
The contract will require two types of equipment: general – purpose equipment already owned by
Vishwakarma, which will be retained at the end of the contract, and specialized equipment to be
purchased second-hand, which will be sold at the end of the contract.
The general-purpose equipment cost Rs.21,000 two years ago is being depreciated on a straight line
basis over a seven year life (with assumed zero scrap value). Equivalent new equipment can be
purchased currently for Rs.49,000. Second-hand prices for comparable general-purpose equipment,
and those for the relevant specialized equipment, are shown below:
General – Purpose equipment Specialized equipment
Purchase Price Resale Price Purchase Price Resale Price
(Rs.) (Rs.) (Rs.) (Rs.)
Current 20,000 17,200 9,000 7,400
After 6 months:
If used for 6 months: 15,000 12,600 7,000 5,800
If not used 19,000 16,400 8,000 6,500
The contract will require the use of a yard on which Vishwakarma has a four-year lease at a fixed
rental of Rs.2,000 per year. If Vishwakarma does not get the contract the yard will probably remain
empty. The contact will also incur administrative expenses estimated at Rs.5,000.

E M Reddy Page | 205


AMA-Notes

Project B:
If Vishwakarma does not get the contract he will buy a building plot for Rs.20,000 and build a
house. Building cost will depend on weather conditions:
Weather Condition A B C
Probability 0.4 0.4 0.2
Building costs (excluding land) (000s) Rs.60 Rs.80 Rs.95
Similarly the price obtained for the house will depend on market conditions:
Weather Condition D E
Probability 0.7 0.3
Sale Price (net of selling expenses) Rs.1,00,000 Rs.1,20,000
Vishwakarma does not have the resources to undertake both projects. The costs of his supervision
time can be ignored.
Requirements:
(a) Ignoring the possibility of undertaking Project B, calculate:
i. The price at which Vishwakarma would tender for the school extension contract if the used
his normal pricing method, and
ii. The tender price at which you consider Vishwakarma would neither gain nor lose by taking
the contract.
(b) Explain, with supporting calculations, how the availability of Project B should affect
Vishwakarma’s tender for the school extension contract.
Solution:
a.
Part 1: Tender price when Vishwakarma uses normal pricing method
Tender Price = Direct Cost + 100% Mark Up
Particulars Computation Amount (Rs.)
A. Materials
Z [100 Units x Rs.7] + [1,000 Units x Rs.10] 10,700
Y 150 Units x Rs.40 6,000
X [300 Units x Rs.35] + [300 Units x Rs.33] 20,400
W 200 Units x Rs.20 4,000
Total Material Cost 41,100
B. Wages
Crafts men wages 2 men x Rs.16,000 x 6/12 16,000
Crafts men bonus 2 men x Rs.700 1,400
Casual labour wages 3 men x Rs.4,000 12,000
Wage Total 29,400
C. Direct Cost A+B 70,500
D. Margin C x 100% = Rs.70,500 x 100% 70,500
E. Tender Price C+D 1,41,000

E M Reddy Page | 206


AMA-Notes

Part 2: Calculation of Relevant Cost for doing this school extension contract
Step 1: Relevant cost of Materials
Particulars Computation Amount (Rs.)
Z [100 Units x Rs.10] + [1,000 Units x Rs.10] 11,000
Y 150 Units x Rs.44 6,600
X [300 Units x Rs.25] + [300 Units x Rs.33] 17,400
W 200 Units x Rs.16 3,200
Total Relevant Cost 38,200
Step 2: Relevant cost of wages
Particulars Computation Amount (Rs.)
Crafts Men Wages Temporary workers wages 11,800
Crafts Men Bonus 2 Men x Rs.700 1,400
Casual labour wages 3 Men x Rs.4,000 12,000
Total Relevant Cost 25,200
Step 3: Relevant cost of Special Purpose equipment
A Specialized equipment are specifically purchased for the contract and will be disposed off after the
contract gets completed. Hence, the relevant cost is current purchase price – Resale price after 6 months =
Rs.9,000 – Rs.5,800 = Rs.3,200.
Step 4: General Purpose Equipment
1) General purpose equipment is already with the company and the company will continue to have it even
after the contract completion.
2) The cost at which it was purchased 2 years ago i.e. Rs.21,000 is irrelevant because it is historical or sunk.
3) If the general purpose equipment is sold today we can realize Rs.17,200 but if it is used for 6 months
and then sold it will realize only Rs.12,600. Thus there is a loss in value of Rs.17,200 – Rs.12,600 =
Rs.4,600.
i. Loss in value due to efflux of time – Rs.17,200 – Rs.16,400 = Rs.800 – Irrelevant because it will
happen whether or not the contract is accepted.
ii. Loss in value due to usage – Rs.16,400 – Rs.12,600 = Rs.3,800 – Relevant because it happens
due to usage in this contract.
Step 5: Other relevant costs
1) The fixed rental for the yard of Rs.2,000 is irrelevant because it is a committed cost
2) If the yard is not used for the school contract it will remain empty and idle resource has got no relevant
cost.
3) Administration expense of Rs.5,000 is relevant because it is incremental.
Step 6: Minimum tender price where Vishwakarma will neither gain nor lose
Particulars Amount (Rs.)
Materials 38,200
Wages 25,200
Special Purpose Equipment 3,200

E M Reddy Page | 207


AMA-Notes

General Purpose Equipment 3,800


Administrative Expenses 5,000
Total Relevant cost 75,400
If tender price is fixed as Rs.75,400 then Vishwakarma could recover all the relevant costs and make no gain
or no loss from the contract.
b. Tender Price for the school extension contract if House building contract is available
Step 1: Expected profit from house building contract
Particulars Computation Amount (Rs.)
Expected Sales [0.7 x Rs.1,00,000] + [0.3 x Rs.1,20,000] 1,06,000
Less:
Cost of land Given (20,000)
Expected building cost [0.4 x Rs.60,000] + [0.4 x Rs.80,000] + [0.2 x Rs.95,000] (75,000)
Expected profit from building contract 11,000

Step 2: Revised tender price for the school extension contract


Particulars Amount (Rs.)
Relevant cost in part 2 75,400
Add: Profit lost from building contract (Opportunity cost) 11,000
Revised tender price 86,400

6.6. Limiting Factor and Relevant Costing

***Question no 10: Following a fire at the factor of Elgar ltd, the management team met to review
the proposed operations for the next quarter. The fire has destroyed all the finished goods stock,
some of the raw materials and about half of the machined in the forming shop.
At the meeting of the management team the following additional information was provided.

i. Only 27,000 machine hours of forming capacity will be available in the forthcoming quarter.
Although previously it was thought that sales demand would be the only binding limitation on
production it has now become apparent that for the forthcoming quarter the forming capacity is
also limiting factor.
ii. It will take about three months to reinstate the forming shop to its previous operational
capacity. Hence the restriction on forming capacity is for the next quarter only.
iii. Some details of the product range manufactured by Elgar are provided in the following table:
Product A B C D E
Sales price (Rs.) 50 60 40 50 80
Units of special material required for production:
W or X 2 2 2 1 3
Y - - - - 6
Z 1 2 1 1 -
Other direct materials cost (Rs.) 6 12 6 5 13
Other variable production costs (Rs.) 8 4 8 4 4
Fixed production costs (based on standard costs) (Rs.) 6 3 6 3 3
Forming hours required 5 6 2 10 6

E M Reddy Page | 208


AMA-Notes

iv. The forecasts of demand, in units, for the forthcoming quarter are:
Product Product Product Product Product
A B C D E
Units demanded 2,000 2,000 4,000 3,000 4,000

v. Due to purchasing error there is an excess of material ‘W’ in stock. This has a book value of
Rs.6 per unit, which is also its current replacement cost. This could be sold to realize Rs.4 per
unit after sales and transportation costs. Material ‘X’ could be used instead of material ‘W’;
material ‘X; is not in stock and has a current replacement cost of Rs.5 per unit.
vi. Material ‘Y; was in stock at a book value of Rs.2 per unit, which is its normal cost if ordered 3
months in advance, but the stocks of this material were entirely destroyed by the fire. In order
to obtain the material quickly a price of Rs.3 per unit will have to be paid for the first 3,000 units
obtained in the quarter and any additional units required will cost Rs.6 per unit. These special
prices will apply only to this quarter’s purchases.
vii. The fire destroyed some of the stock of material ‘Z’. The remaining stock of 2,000 units have a
book value of Rs.7 per unit. The replacement price for ‘Z’ is currently Rs.8 per unit.
viii. As a result of the fire it is estimated that the fixed production costs will be Rs.42,000 for the next
quarter and the administration and office overheads will amount to Rs.11,500.
ix. The demand figures shown in note (iv) include a regular order from a single customer for 3,000
units of C, and 3,000 units of E. The order is usually placed quarterly and the customer always
specifies that the order be fulfilled in total or not at all.
Required:
(a) Ignoring the information contained in note (ix) for the section of the question, determine the
optimum production plan for the forthcoming quarter and the resulting profit.
(b) Prepare the statement, which clearly shows the management of the company the financial
consequences of both acceptance and rejection of the order mentioned in note (ix).

Solution:

Part 1:

Step 1: Calculation of contribution per unit and contribution per hour


Items A B C D E E1
Selling Price (Rs.) 50 60 40 50 80 80
Less: Variable Cost
Material ‘W’ (Rs.) 8 8 8 4 12 12
Material ‘Y’ (Rs.) - - - - 18 36
Material ‘Z’ (Rs.) 8 16 8 8 - -
Other direct material cost 6 12 6 5 13 13
Other variable cost 8 4 8 4 4 4
Contribution per unit 20 20 10 29 33 15
Hours per unit 5 6 2 10 6 6
Contribution per hour 4 3.33 5 2.9 5.5 2,5
Rank 3 4 2 5 1 6

E M Reddy Page | 209


AMA-Notes

Notes:

1) Material ‘W’ is in stock and irregularly used. Its book value is irrelevant because it is historical and
current replacement cost is also irrelevant because it will not be replaced.
2) If Material ‘W’ is used for production we lose a realizable value of Rs.4 per unit which is its relevant
cost.
3) Instead of ‘W’ if we use ‘X’ which is not in stock we should spend Rs.5 per unit to purchase it. Hence,
its relevant cost is Rs.5.
4) Between ‘W’ and ‘X’, ‘W’ is cheaper and should be used.
5) Units of ‘Y’ required to produce 4,000 units of E = 4,000 x 6 Units = 24,000 Units
i. First 3,000 units of ‘Y’ purchased at Rs.3 per unit – This can produce 500 units of E.
ii. Balance 21,000 units ‘Y’ of purchased at Rs.6 per unit – This can produce 3,500 units of E
which we called as E1.
6) ‘Z’ is partly in stock and partly to be purchased. The book value of units in stock is irrelevant because it
is historical. Since the units in stock are regularly used, the relevant cost is its replacement cost of Rs.8.
For the units to be purchased also the relevant cost is Rs.8.

Step 2: Allocation of limiting factor


Product Units Hours/Unit Hours ∑ 𝐇𝐨𝐮𝐫𝐬 Contribution/hour (Rs.) Contribution (Rs.)
E 500 6 3,000 3,000 5.5 16,500
C 4,000 2 8,000 11,000 5 40,000
A 2,000 5 10,000 21,000 4 40,000
B 1,000 6 6,000 27,000 3.3 20,000
Total Contribution 1,16,500

Part 2:

Step 1: Allocation of limiting factor – Specific order rejected


Product Units Hours/Unit Hours ∑ 𝐇𝐨𝐮𝐫𝐬 Contribution/hour (Rs.) Contribution (Rs.)
E 500 6 3,000 3,000 5.5 16,500
C 1,000 2 2,000 5,000 5 10,000
A 2,000 5 10,000 15,000 4 40,000
B 2,000 6 12,000 27,000 3.3 40,000
Total Contribution 1,06,500

Step 2: Allocation of limiting factor – Specific order accepted


Product Units Hours/Unit Hours ∑ 𝐇𝐨𝐮𝐫𝐬 Contribution/hour (Rs.) Contribution (Rs.)
E 500 6 3,000 3,000 5.5 16,500
E1 2,500 6 15,000 18,000 2.5 37,500
C 4,000 2 8,000 26,000 5 10,000
A 200 5 1,000 27,000 4 4,000
Total Contribution 98,000

It is recommended to reject the specific order because the contribution is highest on rejection.

E M Reddy Page | 210


AMA-Notes

6.7. Joint product and Relevant Costing

1) When from a process more than one product emerges, the process is called joint process and the
products are called joint products.
2) The stage at which the product separate is called “Separation Point” or “Split off Point”.
3) All costs spent before split off point are called “Joint Costs” and cost spent after it are called “Further
processing cost”.
4) Joint cost should be apportioned to the joint products for the purpose of stock valuation because value
of stock is share of joint cost plus further processing cost.
5) However, for decision making purpose the point costs are irrelevant which can be seen in the following
two types of decisions:
i. Further Process or not:
The company may sell the joint product at split off point or after further processing it. It
depends on cost and benefit.
a) Benefit – Incremental Sales from improved product
b) Cost – Further processing cost
In this decision joint costs are irrelevant because they will anyhow be incurred whether are not
we further process i.e. they are non-incremental.
ii. Continuance or Discontinuance of a product:
For example, the product ‘B’ has a share of joint cost of Rs.8 per unit and further processing
cost of Rs.15 per unit and selling price of Rs.20 per unit. It gives a loss of Rs.3 (Rs.20 – Rs.23)
per unit. Can we discontinue it?
Answer: No, A independent joint product cannot be discontinued because if we produce ‘Á’,
‘B; will automatically get produced and joint cost cannot be avoided just because we don’t sell

E M Reddy Page | 211


AMA-Notes

‘B’ i.e. Rs.8 is irrelevant. By selling ‘B’ we get Rs.20 and incur Rs.15 further processing cost
giving a Rs.5 contribution towards the joint cost recovery. Hence, once again joint cost
irrelevant for decision making.

Question no 11: A company incurs joint production cost of Rs.3,00,000 for production of two
products A & B. This joint cost comprises of Rs.2,40,000 as fixed cost and Rs.5 per unit as variable
cost. Other details are as follows:
Products Units Produced Units Sold FPC per unit Selling Price
A 10,000 10,000 8 40
B 2,000 2,000 10 35
A new customer approaches the company with an offer to purchase 600 units of product ‘B’ at
Rs.25 per unit. This sale will not affect the market price to the other customers. Should the specific
offer be accepted? What should be done to make the order acceptable?

Solution:

Step 1: Profit or loss from the specific offer

The joint produce ratio is A: B = 5:1


To produce 600 units of B, we should also produce 3,000 units of A. Thus the input processed in joint
process is 3,600 units.
Particulars Computation Amount (Rs.)
Sales 600 Units x Rs.25 15,000
Joint Cost 3,600 Units x Rs.5 (18,000)
Further processing cost 600 Units x Rs.10 (6000)
Profit/(Loss) (9000)

Step 2: Minimum selling price for 3,000 units of A


Particulars Computation Amount (Rs.)
Further processing cost of A 3,000 Units x Rs.8 24,000
Loss from specific offer 9,000
Required sales 33,000
Rs.33,000
Minimum selling price = 3,000 Units = Rs.11 per Unit

Conclusion: The specific order of 600 units of B can be accepted only when we can sell 3,000 units of A at
least at a selling price of Rs.11 per unit.

Check:
Particulars Computation Amount (Rs.)
Sales of ‘A’ 3,000 Units x Rs.11 33,000
Sales of ‘B’ 600 Units x Rs.25 15,000
Joint cost 3,600 Units x Rs.5 (18,000)
Further processing cost of ‘A’ 3,000 Units x Rs.8 (24,000)
Further processing cost of ‘B’ 600 Units x Rs.10 (6,000)

E M Reddy Page | 212


AMA-Notes

Profit/(Loss) 0

Independently ‘B’ alone cannot be accepted.

Question no 12: HTM ltd using 12,00,000 units of material M produces jointly 2,00,000 units of H
and 4,00,000 units of T. The cost and sales details are as under:
Direct material M @ Rs.5 per unit Rs.60,00,000
Other variable cost Rs.42,00,000
Total fixed cost Rs.18,00,000
Selling price of H per unit Rs.25
Selling price of T per unit Rs.20
The company receives an additional order for 40,000 units of T at the rate of Rs.15 per unit. If this
order is accepted, the existing price of T will not be affected. However the present price of H
should be reduced evenly on the entire sale of H to market the additional units to be produced.
Find the minimum average selling price to be charged on H to sustain the increased sales.

Solution:

Step 1: Profit or loss from accepting the specific offer

The joint produce ratio is T: H = 2:1


To produce 40,000 units of T, we should also produce 20,000 units of H by processing 1,20,000 units of
‘M’.
Joint Variable cost = Rs.60,00,000 + Rs.42,00,000 = Rs.1,02,00,000
Input processed = 12,00,000 Units
Rs.1,02,00,000
Joint Variable cost per unit = = Rs.8.5 per unit
12,00,000 Units

Particulars Computation Amount (Rs.)


Sales 40,000 Units x Rs.15 6,00,000
Joint cost of processing 1,20,000 Units x Rs.8.5 (10,20,000)
Profit/(Loss) (4,20,000)

This specific order independently cannot be accepted.

Step 2: Minimum selling price of H


Particulars Computation Amount (Rs.)
Existing Sales 2,00,000 Units x Rs.25 50,00,000
Loss to be recovered 4,20,000
Required sales 54,20,000
Units sold 2,00,000 Units + 20,000 Units 2,20,000 Units
Rs.54,20,000
Minimum selling price = 2,20,000 Units = Rs.24.64 per Unit

Conclusion: For the new order to be accepted we should be able to sell 2,20,000 units of ‘H’ at a minimum
selling price of Rs.24.64.

E M Reddy Page | 213


AMA-Notes

Question no 13: A company manufactures three joint products A, B and C. C has no NRV unless it
undergoes further processing. The cost details of C are as follows:
Particulars Rs. per unit
Up to point of separation
Marginal Cost 30
Fixed Cost 20
After point of separation
Marginal Cost 15
Fixed Cost 5
C can be sold at Rs.37 per unit and not more than that.
 Would you recommend production of C?
 Would your recommendation be different if A, B and C are not joint products?
Solution:
Step 1: Viability of ‘C’ if it is a joint product
The joint variable cost of Rs.30 is irrelevant because even if we decide to discontinue ‘C’ the production of
‘A’ and ‘B’ automatically results in production of ‘C’ and this Rs.30 cannot be avoided. By further
processing and selling ‘C’ we get a selling price of Rs.37 and spend variable cost of Rs.15 which gives Rs.22
contribution towards joint cost. Hence, should be continued.
Step 2: Viability of ‘C’ it is not a joint product
Selling price = Rs.37
Variable Cost = Rs.45
Conurbation = Rs.8
Since contribution is negative ‘Ç’ should be discontinued.

6.8. Relevant Costing under Uncertainty

Question no 14: W ltd is to produce new products in short-term venture which will utilize some
obsolete materials and expected spare capacity. The new product will be advertised in Quarter I
with production and sales taking place in Quarter II. No further production or sales are
anticipated.
Sales volume are uncertain but will, to some extent, be a function of sales price. The possible sales
volumes and the advertising costs associated with each potential sales price are follows:
Sales price Rs.20 per unit Sales price Rs.25 per unit Sales price Rs.40 per unit
Sales volume Probability Sales volume Probability Sales volume Probability
(units 000’s) (units 000’s) (units 000’s)
4 0.1 2 0.1 0 0.2
6 0.4 5 0.2 3 0.5
8 0.5 6 0.2 10 0.2
- - 8 0.5 15 0.1
Advertising Cost Rs.20,000 Advertising Cost Rs.50,000 Advertising Cost Rs.1,00,000
The resources used in the production of each unit of the product is:
Production Grade I – 2 Hours
Grade II – 1 Hours
Materials X – 1 Units

E M Reddy Page | 214


AMA-Notes

Y – 2 Units
The normal cost per hour of labour is:
Grade I – Rs.2
Grade II – Rs.3
However, before considering the effect of the current venture there is expected to be 4,000 hours of
idle time for each grade of labour in quarter II. Idle time is paid at the normal rates.
Material X is in stock at a book value of Rs.8 per unit is widely used within the firm and any usage
for the purpose of this venture will require replacing. Replacement cost in Rs.9 per unit.
Material Y is obsolete stock. There are 16,000 units in stock at a book value of Rs.3.50 per unit any
stock not used will have to be disposed of a cost, to W ltd. of Rs.2 per unit. Further quantities of Y
can be purchased for Rs.4 per unit.
Overhead recovery rates are:
Variable overhead Rs.2 per direct labour hour worked. Fixed overhead Rs.3 per direct labour hour
worked. Total fixed overheads not alter as a result of the current venture.
Feedback from advertising will enable the exact demand to be determined at the end of quarter I
and production in quarter II will be set to equal that demand. However it is necessary to decide
now on the sales price in order that it can be incorporated into the advertising campaign.
Required:
(a) Calculate the expected money value of the venture at each sales price and on the basis of this
advice W ltd of its best course of action.
(b) Briefly explain why the management of W ltd. might rationally reject the sales price leading to
the highest expected money value and prefer one of the other sales prices.
Solution:
Part A:
Step 1: Calculation of total cost at all output levels
Units Quant Rs.(X) Quant Rs.(Y) Grade Grade Grade Grade Variab Total
ity of ity of I I (Rs.) II II le OH
X Y Hours Hours (Rs.) (Rs.)
2,000 2,000 18,000 4,000 (8,000) 4,000 - 2,000 - 12,000 22,000
3,000 3,000 27,000 6,000 (12,000) 6,000 4,000 3,000 - 18,000 37,000
4,000 4,000 36,000 8,000 (16,000) 8,000 8,000 4,000 - 24,000 52,000
5,000 5,000 45,000 10,000 (20,000) 10,000 12,000 5,000 3,000 30,000 70,000
6,000 6,000 54,000 12,000 (24,000) 12,000 16,000 6,000 6,000 36,000 88,000
8,000 8,000 72,000 16,000 (32,000) 16,000 24,000 8,000 12,000 48,000 1,24,000
10,000 10,000 90,000 20,000 (16,000) 20,000 32,000 10,000 18,000 60,000 1,84,000
15,000 15,000 1,35,000 30,000 24,000 30,000 52,000 15,000 33,000 90,000 3,34,000

Notes:

1) Material ‘X’ is in stock and regularly used. Hence the relevant cost is its replacement cost of Rs.9 per
unit. The book value of per unit is irrelevant because it is historical cost.
2) 16,000 units of Material ‘Y’ is in stock due to excess purchase. If not used will be disposed off by
spending Rs.2 per unit. Hence, usage saves disposal cost which is a relevant benefit.

E M Reddy Page | 215


AMA-Notes

3) Any usage in excess of 16,000 units need to be purchased by spending Rs.4 per unit which is the
relevant cost for the excess consumption. For example:
30,000 Units
a. In stock – 16,000 Units – Relevant benefit = 16,000 Units x Rs.2 = Rs.32,000
b. To be purchased – 14,000 Units – Relevant Cost = 14,000 Units x Rs. 4 = Rs.56,000
Net Relevant cost = Rs,24,000
4) 4,000 hours of idle time of G – I and G – II available and relevant cost for using idle time is ‘Nil’. Any
excess hours above 4,000 will have a relevant cost of Rs.2 and Rs.3 for G – I and G – II respectively.
5) Variable cost per unit is Rs.6 (3 Hours x Rs.2).
6) Fixed costs are irrelevant because they do not alter due to the current venture.

Step 2: Profit when selling price is Rs.20


Units Probability Computation Profit Expected Profit
(Sales – Variable Cost – Advertising Cost) (Rs.) (Rs.)
4,000 0.1 80,000 – 52,000 – 20,000 8,000 800
6,000 0.4 1,20,000 – 88,000 – 20,000 12,000 4,800
8,000 0.5 1,60,000 – 1,24,000 – 20,000 16,000 8,000
Total Profit/(Total Loss) 13,600

Step 3: Profit when selling price is Rs.25


Units Probability Computation Profit Expected Profit
(Sales – Variable Cost – Advertising Cost) (Rs.) (Rs.)
2,000 0.1 50,000 – 22,000 – 50,000 (22,000) (2,200)
5,000 0.2 1,25,000 – 70,000 – 50,000 5,000 1,000
6,000 0.2 1,50,000 – 88,000 – 50,000 12,000 2,400
8,000 0.5 2,00,000 – 1,24,000 – 50,000 26,000 13,000
Total Profit/(Total Loss) 14,200

Step 4: Profit when selling price is Rs.40


Units Probability Computation Profit Expected Profit
(Sales – Variable Cost – Advertising Cost) (Rs.) (Rs.)
0 0.2 0 – 0 – 1,00,000 (1,00,000) (20,000)
3,000 0.5 1,20,000 – 37,000 – 1,00,000 (17,000) (8,500)
10,000 0.2 4,00,000 – 1,84,000 – 1,00,000 1,16,000 23,200
15,000 0.1 6,00,000 – 3,34,000 – 1,00,000 1,66,000 16,600
Total Profit/(Total Loss) 11,300

The highest expected money value comes in Option – 2. Hence it should be selected i.e. price the new
product at Rs.25.

Part B: Why management may reject Rs.25 selling price?

1) A selling price of Rs.20 guarantees no loss. Even when the demand is lowest it gives a profit of
Rs.8,000.
2) Selling price of Rs.40 promises a profit as high as Rs.1,66,000 and hence may be preferred by
management.

E M Reddy Page | 216


AMA-Notes

3) However selling price of Rs.25 neither guarantees no loss nor promises high profit. Hence, can be
rationally rejected by management.

Conclusion: Is expected money value decision wrong i.e. is the mean unreliable for decision making?
Answer: Mean (Average) is superior to all tactical methods. The selling price of Rs.25 has only 10% chance
of having loss. Due to this on should not go for selling price of Rs.20. If it is made, then the business men is
unwilling to take risk. Selling price of Rs.40 has 70% chance of loss. In that option it is undue risk. Business
is all about taking calculated risk which happens by making decisions based on expected values.

Question no 15: Ram ltd has spare capacity in two of its manufacturing departments – Department
4 and Department 5. A five-day week of 40 hours is worked but there is only enough internal work
for three days per week so that two days per week (16 hours) could be available in each department.
In recent months Ram ltd has sold this time to another manufacturer but there is some concern
about the profitability of this work.
The accountant has prepared a table giving the hourly operating costs in each department. The
summarized figures are as follows:
Particulars Department 4 (Rs.) Department 5 (Rs.)
Power Costs 40 60
Labour Costs 40 20
Overhead Costs 40 40
Total 120 120
The labour force is paid on time basis and there is no change in the weekly wage bill whether or
not the plant is working at full capacity. The overhead figures are taken from the firm’s current
capacity. The overheads figures are taken the firm’s current overhead absorption rates. These rates
are designed to absorb all budgeted overhead (fixed and variable) when the departments are
operating at 90% full capacity (assume a 50 week year). The budgeted fixed overhead attributed to
department 4 is Rs.36,000 p.a. and that for department 5 is Rs.50,400 p.a.
As a short term expedient the company has been selling processing time to another manufacturer
who has been paying Rs.70 per hour for time in either department. This customer is very willing to
continue this arrangement and to purchase any spare time available but Ram ltd is considering the
introduction of a new product on a minor scale to absorb the spare capacity.
Each unit of the new product would require 45 minutes in department 4 and 20 minutes in
department 5. The variable cost of the required input material is Rs.10 per unit. It is considered
that:
 With a selling price of Rs.100 the demand would be 1,500 units p.a.;
 With a selling price of Rs.110 the demand would be 1,000 units p.a.; and
 With a selling price of Rs.120 the demand would be 500 units p.a.;
You are required to calculate the best weekly programme for the slack time in the two
manufacturing departments and to determine the best price to charge for the new product.

Solution:

Step 1: Calculation of Variable Overheads per Hour


Particulars Department 4 Department 5
A. Full Capacity (100%) (40 Hours x 50 Weeks) 2,000 Hours 2,000 Hours

E M Reddy Page | 217


AMA-Notes

B. Normal Capacity (90% x 2,000 Hours) 1,800 Hours 1,800 Hours


C. Budgeted Fixed Overheads Rs.36,000 Rs.54,000
D. Fixed overheads per hour (C/B) Rs.20 Rs.28
E. Overheads per hour Rs.40 Rs.40
F. Variable overheads per hour Rs.20 Rs.12

Step 2: Relevant cost per hour for operating each department


Particulars Department 4 Department 5
Power Cost per Hour Rs.40 Rs.60
Variable overheads per hour Rs.20 Rs.12
Total Rs.60 Rs.72

Notes:

1) Labour cost per hour is irrelevant because it is paid on time guaranteed basis and hence committed.
2) Selling idle department 4 hour for Rs.70 is viable because we can earn Rs.10 contribution per hour by
selling the idle time.
3) However to run department 5 for one hour cost Rs.72 but the realization is only Rs.70 per hour. Hence
it is better to keep it idle rather than selling it.

Step 3: Variable cost per unit of new product


Particulars Computation Amount (Rs.)
Direct Materials cost per unit Given 10
Department 4 relevant cost per unit Rs.60 x 45 Minutes/60 Minutes 45
Department 4 opportunity cost per unit Rs.10 x 45 Minutes/60 Minutes 7.5
Department 5 relevant cost per unit Rs.72 x 20 Minutes/60 Minutes 24
Total Cost per unit 86.5

Step 4: Contribution per unit of new product


Particulars Option – 1 (Rs.) Option – 2 (Rs.) Option – 3 (Rs.)
Selling Price per unit 100 110 120
Variable cost per unit (86.5) (86.5) (86.5)
Contribution per unit 13.5 23.5 33.5

Step 5: Determination of Volume


16 Hours x 50 Weeks
Possible Production using department 4 idle time = 45 Minutes (or) 0.75 Hours = 1,067 Units

16 Hours x 50 Weeks
Possible Production using department 5 idle time = 20 Minutes (or) 0.33 Hours = 2,400 Units

To conclude, we can maximum produce only 1,067 units of the new product.
Particulars Option – 1 Option – 2 Option – 3
Selling Price Rs.100 Rs.110 Rs.120
Demand 1,500 Units 1,000 Units 500 Units

E M Reddy Page | 218


AMA-Notes

Production 1,067 Units 1,067 Units 1,067 Units


Possible Volume 1,067 Units 1,000 Units 500 Units

Step 6: Contribution of each option


Particulars Option – 1 Option – 2 Option – 3
Possible Volume 1,067 Units 1,000 Units 500 Units
Contribution per unit Rs.13.5 Rs.23.5 Rs.33.5
Total Contribution Rs.14,405 Rs.23,500 Rs,16,750

The company should fix a selling price of Rs.110 and sell 1,000 Units since this option gives highest
incremental contribution.

Step 7: Plan to use spare capacity and the resulting incremental profit

Department 4:- 800 Hours


- New Product – 1,000 Units x 0.75 Hours = 750 Hours
- Sell – 50 Hours
Department 5:- 800 Hours
- New Product – 1,000 Units x 0.333 Hours = 333 Hours
- Keep idle – 467 Hours

Due to the above decision the incremental profit to the company is Rs.23,500 due to using the spare
capacity for new product and Rs.934 (467 Hours x Rs.2) by avoiding Rs.2 loss on account of sale of the idle
capacity. Therefore, incremental profit is Rs.24,434 (Rs.23,500 + Rs.934).

E M Reddy Page | 219


AMA-Notes

7. MARGINAL COSTING

7.1. Learning Objectives

1) Basics in Marginal Costing


a) Income statement
b) PV Raito
c) Break-even point (Value and Units)
d) Margin of Safety (Value and Units)
e) Profit
2) Issues in the concept of Break-even point
a) Multiple Break-even points (Step fixed cost)
b) Break-even point and semi variable cost
c) Marginal costing break-even point vs. Absorption costing break-even point
d) Break-even point with specific fixed cost
3) Indifference point
a) Basic problem
b) Indifference point as a state of demand
c) Indifference point and Break-even point
d) Indifference point with specific fixed cost
e) Indifference point expressed as a limiting factor
4) Differential Costing (marginal cost, Marginal Revenue analysis)
5) Limiting factor problems
a) Basic limiting factor allocation
b) Limiting factor in a make or buy situation
c) Limiting factor allocation with specific fixed cost
d) Multiple limiting factor allocation having consistent ranks
6) Concept of shutdown point
7) Miscellaneous problems

7.2. Basics in Marginal Costing

Example:
Selling price per unit = Rs.10
Variable cost per unit = Rs.6
Fixed Cost = Rs.10,000
Sales = 6,000 Units
Prepare or Calculate:
1) Income Statement
2) PV Ratio
3) Break-even point or Break-even sales (Units/Value)
4) Margin of Safety (Units & Value)
5) Profit
Solution:

E M Reddy Page | 220


AMA-Notes

7.2.1. Income Statement

Step 1: Income Statement


Particulars Computation Amount (Rs.)
Sales 6,000 Units x Rs.10 60,000
Less: Variable Cost 6,000 Units x Rs.6 (30,000)
Contribution 24,000
Less: Fixed Cost Given (10,000)
Profit 14,000

Notes:

1) The above income statement is very helpful in assessing the impact of volume on cost and profit.
2) Hence, the chapter is also referred as Cost Volume Profit analysis (CVP analysis).

7.2.2. PV Ratio

Step 2: PV Ratio
Contribution per unit Rs.4
PV Ratio = = Rs.10 = 40%
Selling Price

Contribution Rs.24,000
PV Ratio = = Rs.60,000 = 40%
Sales

Notes:

1) A PV ratio of 40% means, when the sales is Rs.100 contribution is Rs.40.


2) It also means when the sales changes by Rs.100, the profit and contribution changes by Rs.40.
3) Change in Profit = Change in Contribution because fixed cost does not change.

7.2.3. Breakeven Point

Step 3: Break-even Point

1) Break-even point is the sales level at which the profit is ‘0’.


2) At that sales level sales = Total cost & contribution = fixed cost.
3) This break-even point or sales can be expressed in two ways:
Fixed Cost Rs.10,000
a. In units = Contribution per unit = = 2,500 Units
Rs.4
b. In Value
i. Break-even point in units x Selling price per unit – 2,500 Units x Rs.10 = Rs.25,000
Fixed Cost Rs.10,000
ii. = = Rs.25,000
PV Ratio 40%

7.2.4. Margin of Safety

Step 4: Margin of Safety

1) Margin of Safety is the sales above break-even sales.

E M Reddy Page | 221


AMA-Notes

2) It is the sales that can drop before the company starts incurring losses.
3) It is profit generating sales.
4) This Margin of Safety also can be expressed in units or value:
a. In Units
i. Actual Sales – Break-even Point = 6,000 Units – 2,500 Units = 3,500 Units
Profit Rs.14,000
ii. = = 3,500 Units
Contribution per Unit Rs.4
b. In Value
i. Actual Sales – Break-even Sales = Rs.60,000 – Rs.25,000 = Rs.35,000
ii. Margin of Safety x Selling Price = 3,500 Units x Rs.10 = Rs.35,000
Profit Rs.14,000
iii. = = Rs.35,000
PV Ratio 40%

Notes:
1) Contribution is linked to sales, fixed cost to break-even sales and profit to Margin of Safety.
2) When we produce and sell 1 unit, we incur variable cost and also receive selling price. What stays in
hand is Contribution.
3) Contribution contributes:
a. Towards Fixed Cost Recovery – Up to Breakeven Sales
b. Towards Profit – For Margin of Safety Sales

7.2.5. Profit

Step 5: Profit
1) Profit = Sales – Cost = Rs.60,000 – Rs.46,000 = Rs.14,000
2) Profit = Contribution – Fixed Cost = Rs.24,000 – Rs.10,000 = Rs.24,000
3) Profit = Margin of Safety in Units x Contribution per unit = 3,500 Units x Rs.4 = Rs.14,000
4) Profit = Margin of Safety in value x PV Ratio = Rs.35,000 x 40% = Rs.14,000
Conclusion: For the all above formulas to be true the volume should not affect selling price, variable cost
per unit or the total fixed cost. In other words we assume in linearity in relationship. If the linearity
assumption does not hold good what happens is the subject matter of discussion.

7.3. Issues in the concept of Break-even point

7.3.1. Multiple break-even points (Step fixed cost)

1) Whenever the fixed cost and variable cost/unit are varying at different levels, then the usage of formula
will not be rewarding.
2) Use the spirit of basic knowledge of marginal costing in solving the Break-even Point.
3) At Break-even Point, Sales = Cost (or) Profit = 0
Question no 1: A firm sells its produce at Rs.25 per unit. Its cost behavior for various production
ranges is:
Units of production Cumulative fixed cost Variable cost per unit
0 – 16,000 2,50,000 16.00
16,001 – 60,000 3,50,000 17.00
60,001 and above 5,00,000 20.00

E M Reddy Page | 222


AMA-Notes

Identify the break-even point(s) in units.


Solution:

Step 1: Calculation of Break-even in range I


Items Particulars
Selling Price Rs.25
Less: Variable Cost (Rs.16)
Contribution Rs.9
Fixed Cost Rs.2,50,000
Break-even Point (Rs.2,50,000/Rs.9) 27,778 Units

We cannot call 27,778 units as Break-even point because Range I exist only up to 16,000 Units, beyond
which the cost pattern itself changes. We can conclude that there is no break-even in Range I i.e. Range I
generates only loss.

Step 2: Unrecovered fixed cost in Range I

Contribution (16,000 Units x Rs.9) = Rs.1,44,00


Less: Fixed Cost = Rs.2,50,000
Unrecovered fixed cost = Rs.1,06,000

Step 3: Calculation of Break-even in range II


Items Particulars
Selling Price Rs.25
Less: Variable Cost (Rs.17)
Contribution Rs.8
Incremental fixed cost for Range II (Rs.3,50,000 – Rs.2,50,000) Rs.1,00,000
Unrecovered fixed cost of Range I Rs.1,06,000
Fixed cost recoverable in Range II Rs.2,06,000
Units required to recover fixed cost (Rs.2,06,000/Rs.8) 25,750 Units
Break-even point (25,750 Units + 16,000 Units) 41,750 Units

Check:
Particulars Computation Amount (Rs.)
Contribution [16,000 Units x Rs.9] + [25,750 Units x Rs.8] 3,50,000
Less: Fixed Cost Given 3,50,000
Profit 0

Company breaths air of profit only on reaching 41,750 units in Range II. Range II extends up to 60,000
units. From 41,750 units to 60,000 units the contribution generated results in profit.

Step 4: Profit generated by Range II

Profit generating units (60,000 Units – 41,750 Units) = 18,250 Units


Contribution per unit = Rs.8
Profit (18,250 Units x Rs.8) = Rs.1,46,000

E M Reddy Page | 223


AMA-Notes

Step 5: Calculation of Break-even in Range III


Items Particulars
Selling Price Rs.25
Less: Variable Cost (Rs.20)
Contribution Rs.5
Incremental fixed cost for Range III (Rs.5,00,000 – Rs.3,50,000) Rs.1,50,000
Recovery through profit from Range II Rs.1,46,000
Fixed cost recoverable in Range III Rs.4,000
Units required to recover fixed cost (Rs.4,000/Rs.5) 800 Units
Break-even point (60,000 Units + 800 Units) 60,800 Units

Check:
Particulars Computation Amount (Rs.)
Contribution [16,000 Units x Rs.9] + [44,000 Units x Rs.8] + [800 Units x Rs.5] 5,00,000
Less: Fixed Cost Given 5,00,000
Profit 0

Step 6: Analysis of profitability and sales range


Sales Range Profitability
0 – 41,749 Units Loss
41,750 Units Zero (BEP1)
41,751 – 60,000 Units Profit
60,001 – 60,799 Units Loss
60,800 Units Zero (BEP2)
Above 60,800 units Profit

Notes:

1) When we have step fixed cost i.e. fixed cost increases at output different output ranges, we may have
multiple breakeven points.
2) In such situation break-even should be analyzed range by range.
3) Generally higher volumes results in higher profit only when linearity is satisfied. In case of step fixed
cost this linearity is absent. Hence, more volume need not give higher profits. For example, at 60,000
units the profit is Rs.1,46,000 but at 60,801 units the profit is only Rs.5.

Question no 2: SCV is a leading cable TV service provide with its operations spread over different
cities. It has recently been approached by the city of Chennai to operate its cable television
operations. Chennai city officials have become tired to reporting on the cable television company
they have operated for the past five years.
SCV makes the following assumptions in its planning after negotiations with key parties.
A basic set of 10 cable television stations will be offered at Rs.20 per month per subscriber. These
10 stations include a sports channel, a news channel and other general audience channels.
Chennai would retain ownership of the physical facilities and would maintain them in working
condition. Under a leasing agreement, SVC will pay Chennai the following charges:

E M Reddy Page | 224


AMA-Notes

 Fixed commitment charges: Rs.50,000 per month if number of subscribers is 10,000 or less and
Rs.75,000 per month, if the number of subscribers is more than 10,000.
 Variable revenue share: 10% of the monthly revenues from the first 10,000 subscribers and 5%
from additional subscribers.
SCV will receive the ten channels in its basic service from interlink cable. Interlink acts as a
intermediary between cable television stations and companies such as SCV, which sell to
individual subscribers. Interlink charges a monthly-fixed fees of Rs.20,000 plus monthly charge of
Rs.8 per subscriber for the first 20,000 subscribers and Rs.6 per subsequent subscriber.
SCV estimates its own operating costs to include both a fixed and variable component. The fixed
component if Rs.55,000 per month up to 20,000 subscribers. It is expected to increase by Rs.15,000
per month, if number of subscribers exceeds 20,000 subscribers. Variable cost per subscriber is
Rs.2 per month.
Required:
a) How does the contribution margin per subscriber behave over the 0 to 30,000 – subscriber
range?
b) Calculate the break-even number of subscribers per month for SCV.
c) What is the operating income per month to SCV with (a) 10,000 (b) 20,000 (c) 30,000
subscribers? Comment on the results.

Solution:

Step 1: Analysis of facts

1) SCV collects a revenue of Rs.20 from each subscriber.


2) It incurs the following costs:
i. Payment to Chennai Corporation
ii. Payment to Interlink
iii. Own Operating Cost
3) Payment to Chennai Corporation:
i. Variable Cost
a) Up to 10,000 Subscribers – 10% x Rs.20 = Rs.2
b) Beyond 10,000 Subscribers – 5% x Rs.20 = Rs.1
ii. Fixed Cost
a) Up to 10,000 Subscribers – Rs.50,000
b) Beyond 10,000 Subscribers – Rs.75,000
4) Payment to Interlink
i. Variable Cost
a) Up to 20,000 Subscribers – Rs.8 per Subscriber
b) Beyond 20,000 Subscribers – Rs.6 per Subscriber
ii. Fixed Cost – Rs.20,000
5) Own operating Cost
i. Variable Cost – Rs.2 per subscriber
ii. Fixed Cost
a) Up to 20,000 subscribers – Rs.55,000
b) Beyond 20,000 subscribers – Rs.70,000

E M Reddy Page | 225


AMA-Notes

Step 2: Behavior of costs at different Ranges


Particulars 0 – 10,000 (Rs.) 10,001 – 20,000 (Rs.) 20,001 – 30,000 (Rs.)
Selling Price 20 20 20
Less: Variable Cost
Own Variable Cost (2) (2) (2)
Paid to Interlink (8) (8) (6)
Chennai Corporation (2) (1) (1)
Contribution 8 9 11
Less: Fixed Cost
Own Variable Cost 55,000 55,000 70,000
Paid to Interlink 20,000 20,000 20,000
Chennai Corporation 50,000 50,000 75,000
Fixed Cost 1,25,000 1,50,000 1,65,000

Step 3: Summary of the above analysis


Range Fixed Cost (Rs.) Contribution/Subscriber (Rs.)
0 -10,000 1,25,000 8
10,001 – 20,000 1,50,000 9
20,001 – 30,000 1,65,000 11

Step 4: Break-even point in Range I

Fixed Cost = Rs.1,25,000


Contribution/Subscriber = Rs.8
Rs.1,25,000
Break-even point = = 15,625 Subscribers
Rs.8

We cannot call 15,625 subscribers as break-even point because Range I exists only up to 10,000 subcribers
beyond which the cost pattern changes. We can conclude that there is no break-even point in Range I and
Range I ends up with unrecovered fixed cost.

Step 5: Unrecovered fixed cost in Range I

Contribution (10,000 Subscribers x Rs.8) = Rs.80,000


Less: Fixed Cost = Rs.1,25,000
Unrecovered fixed cost = Rs.45,000

Step 6: Break-even point in Range II

Contribution per subscriber = Rs.9


Incremental fixed cost (Rs.1,50,000 – Rs.1,25,000) = Rs.25,000
Unrecovered fixed cost brought forward from Range I = Rs.45,000
Recoverable fixed cost in Range II (Rs.25,000 + Rs.45,000) = Rs.70,000
Subscribers required to recover (Rs.70,000/Rs.9) = 7,778 Subscribers
Break-even point (10,000 + 7,778) = 17,778 Subscribers

E M Reddy Page | 226


AMA-Notes

Range II extends up to 20,000 subscribers. The Margin of Safety in Range II is 2,222 subscribers (20,000 –
17,778).
Profit at the end of Range II = 2,222 Subscribers x Rs.9 = Rs.19,998 or Rs.20,000. Thus Range II ends with
profit of Rs.20,000.

Step 7: Break-even point in Range III

Contribution per subscriber = Rs.11


Incremental fixed cost in Range III (Rs.1,65,000 – Rs.1,50,000) = Rs.15,000
Profit brought forward from Range II = Rs.20,000

Range III need not break-even because its entire incremental fixed cost is recovered from Range II profit.
The Range III starts with a profit of Rs.5,000.

Step 8: Calculation of profit to SCV at the end of each Range


Subscribers Profit/(Loss) (Rs.)
10,000 (45,000)
20,000 20,000
30,000 5,000 + [10,000 x 2] = 1,50,000

7.3.2. Break-even point with semi-variable cost

**Question no 3: Kalyan University conducts a special course on computer application during


summer. For this purpose, it invites applications from graduates. An entrance test is given to the
candidates and based on the same, a final selection of a hundred candidates is made. The entrance
test consists of four objective type of examination and is spread over four days, one examination
per day. Each candidate is charged a fee of Rs.50 for raking up the entrance test. The following
data was gathering for the past two years.
Statement of net Revenue from the Entrance test for the course on “Computer Application”
Particulars Year 1 (Rs.) Year 2 (Rs.)
Gross Revenue (Fees collected) 1,00,000 1,50,000
Costs
Valuation 40,000 60,000
Question booklets 20,000 30,000
Hall rent at Rs.2,000 per day 8,000 8,000
Honorarium to chief 6,000 6,000
Administrator
Supervision charges 4,000 6,000
1 supervisor for every 100 candidates at Rs.50 per day
General Administration expenses 6,000 6,000
Total Cost 84,000 1,16,000
Net Revenue 16,000 34,000
Required to compute:
a) The budgeted net revenue, if 4,000 candidates take up the entrance test Year 3.
b) The break-even number of candidates.
c) The number of candidates to be enrolled if the net income desired is Rs.20,000

E M Reddy Page | 227


AMA-Notes

Solution:

Step 1: Analysis of the cost behavior pattern


Particulars Year 1 Year 2 Incremental Analysis
(Rs.)
No.of students 2,000 3,000 1,000
Revenue (Rs.) 1,00,000 1,50,000 50,000 Variable @ Rs.50
Cost:
Valuation 40,000 60,000 20,000 Variable @ Rs.20
Questions Booklet 20,000 30,000 10,000 Variable @ Rs.10
Supervision 4,000 6,000 2,000 Variable for every 100
students @ Rs.2 per student
Hall Rent 8,000 8,000 - Fixed
Honorarium 6,000 6,000 - Fixed
Administration expenses 6,000 6,000 - Fixed

Step 2: Calculation of contribution per student


Particulars Within 100 students (Rs.) For every 100 students (Rs.)
Fees collected 50 50
Less: Valuation (20) (20)
Less: Questions Booklet (10) (10)
Less: Supervision cost - (2)
Total Cost 30 32
Contribution 20 18

Step 3: Profit calculation when 4,000 students write the exam


Particulars Computation Amount (Rs.)
Contribution 4,000 Students x Rs.18 72,000
Less: Fixed Cost (20,000)
Profit 52,000

Alternatively,
Particulars Computation Amount (Rs.)
Gross Revenue 4,000 Students x Rs.50 2,00,000
Less: Costs
Valuation 4,000 Students x Rs.20 (80,000)
Question Booklet 4,000 Students x Rs.10 (40,000)
Supervision 40 Supervisors x Rs.200 (8,000)
Hall Rent (8,000)
Honorarium (6,000)
General Administration charges (6,00)
Net Revenue 52,000

E M Reddy Page | 228


AMA-Notes

Step 4: Calculation of Break-even point

Contribution per student for every 100 students = Rs.18


Fixed Cost = Rs.20,000
Rs.20,000
Break-even point = Rs.18 = 1,111 Students
Additional fixed to recover [200 – (11 x 2)] = Rs.178
Contribution per student within every 100 students = Rs.20
Rs.178
Additional students to recover additional fixed cost = Rs.20 = 9 Students
Break-even students (1,111 Students + 9 Students) = 1,120 Students

Check:
Particulars Computation Amount (Rs.)
Fees 1,120 Students x Rs.50 56,000
Less: Variable Cost 1,120 Students x Rs.20 33,600
Less: Supervision cost 12 Supervisors x Rs.12 2,400
Less: Fixed Cost 20,000
Profit/(Loss) 0

Step 5: Number of students to make a profit of Rs.20,000

Contribution per student for every 100 students = Rs.18


Fixed Cost = Rs.20,000
Profit = Rs.20,000
Contribution required (Rs.20,000 + Rs.20,000) = Rs.40,000
Rs.40,000
Number of students = Rs.18 = 2,222 Students
Additional fixed to recover [200 – (22 x 2)] = Rs.156
Contribution per student within every 100 students = Rs.20
Rs.156
Additional students to recover additional fixed cost = Rs.20 = 8 Students
Break-even students (2,222 Students + 8 Students) = 2,230 Students

7.3.3. Marginal Costing (vs.) Absorption Costing Break-even point

Question no 4: A company uses absorption costing system based on standard costs. The total
variable manufacturing cost is Rs.6 per unit. The standard production rate is 10 units per machine
hour. Total budgeted and actual fixed production overhead costs are Rs.8,40,000. Fixed production
overhead is allocated at Rs.14 per machine hour. Assume this same standard for the last year and
current year.
Selling price is Rs.10 per unit.
Variable selling overheads is Rs.2 per unit and fixed selling costs are Rs.2,40,000. Beginning
inventory was 30,000 units and ending inventory was 40,000 units.
1. Compute BEP under absorption costing system.
2. Compute BEP under marginal costing system.
3. Calculate profit under absorption costing system for the BEP sale under marginal costing system
with the stock level as given below.

E M Reddy Page | 229


AMA-Notes

Solution:

Facts:
Selling Price Rs.10
Variable manufacturing cost Rs.6
Budgeted and Actual fixed manufacturing cost Rs.8,40,000
Variable selling cost Rs.2
Fixed selling cost Rs.2,40,000
Opening stock 30,000 Units
Closing Stock 40,000 Units
Standard rate per hour Rs.14
Standard Rate per unit Rs.14
= Rs.1.4
10 Units

Step 1: Break-even point under absorption costing system:


Total Fixed Cost Rs.10,80,000
Fixed manufacturing cost in net stock c/f to next year 10,000 Units x Rs.1.4 = Rs.14,000
Net fixed cost recoverable this year through unit sold Rs.10,66,000
Contribution per unit Rs.10 – Rs.6 – Rs.2 = Rs.2
Break-even point Rs.10,66,000/Rs.2 = 5,33,000 Units

Step 2: Profit when sales is 5,33,000 Units


Particulars Computation Amount (Rs.)
Sales 5,33,000 Units x Rs.10 53,30,000
Cost of Goods Sold 5,33,000 Units x Rs.7.4 (39,44,200)
Gross Profit 13,85,800
Less: Variable Selling Expenses 5,33,000 Units x Rs.2 (10,66,000)
Less: Fixed Selling Expenses Given (2,40,000)
Less: Under Absorption 5,43,000 Units x Rs.1.4 – Rs.8,40,000 (79,800)
Profit/(Loss) 0

Working Notes:

1) Calculation of cost of goods sold:


Full Manufacturing cost per units (Rs.6 + Rs.1.4) = Rs.7.4
Value of opening Stock of finished goods (30,000 Units x Rs.7.4) = Rs.2,22,000
Cost of production (5,43,000 Units x Rs.7.4) = Rs.40,18,200
Value of closing Stock of finished goods (40,000 Units x Rs.7.4) = (Rs.2,96,000)
Cost of goods sold (5,33,000 Units x Rs.7.4) = Rs.39,44,200
Rs.7.4 is common, when we take it out the cost of goods sold is Rs.7.4 (30,000 + 5,43,000 – 40,000).
2) Units produced = Units Sold + Closing Stock – Opening Stock
= 5,33,000 Units + 40,000 Units – 30,000 Units
= 5,43,000 Units

E M Reddy Page | 230


AMA-Notes

Step 3: Profit under marginal costing system when 5,33,000 units are sold
Particulars Computation Amount (Rs.)
Sales 5,33,000 Units x Rs.10 53,30,000
Variable Cost of Goods Sold 5,33,000 Units x Rs.6 (31,98,000)
Gross Contribution 21,32,000
Less: Variable Selling Expenses 5,33,000 Units x Rs.2 (10,66,000)
Contribution 10,66,000
Less: Fixed Cost Rs.8,40,000 + Rs.2,40,000 (10,80,000)
Profit/(Loss) (14,000)

Step 4: BEP under marginal costing system


Total Fixed Cost Rs.10,80,000
Contribution per unit Rs.10 – Rs.6 – Rs.2 = Rs.2
Break-even point Rs.10,80,000/Rs.2 = 5,40,000 Units

Step 5: Profit under absorption costing system when sales is 5,40,000 units
Particulars Computation Amount (Rs.)
Sales 5,40,000 Units x Rs.10 54,00,000
Cost of Goods Sold 5,40,000 Units x Rs.7.4 (39,96,000)
Gross Profit 14,04,000
Less: Variable Selling Expenses 5,40,000 Units x Rs.2 (10,80,000)
Less: Fixed Selling Expenses Given (2,40,000)
Less: Under Absorption 5,50,000 Units x Rs.1.4 – Rs.8,40,000 (70,000)
Profit/(Loss) 14,000

Notes:

1) It is known that at a given sales level, marginal costing income statement and absorption costing
statement reports different profits when there exists stock.
2) This means when there is stock, the sales level at which the marginal costing reports ‘0’ profit will not
be the sales level at which the absorption costing has ‘0’ profit. Thus break-even point is different for
marginal and absorption costing systems.
3) In absorption costing system the sales need not recover the entire current year’s fixed cost when there is
net closing stock because that portion of fixed cost inside net closing stock escapes to next year. In this
problem, out of Rs.10,80,000 fixed cost Rs.14,000 goes to next year. What needs to be recovered only is
Rs.10,66,000.
4) Similarly, when there is net opening stock the sales should recover not only the current year fixed stock
but also a portion if fixed cost from previous year.
5) Thus when stock are given absorption costing break-even point is calculated as follows:
Total Fixed Cost – Fixed Cost in net Closing Stock
a. Net Closing Stock – Break-even point = Contribution per unit
Total Fixed Cost+ Fixed Cost in net Opening Stock
b. Net Opening Stock – Break-even point = Contribution per unit
6) We can observe that at absorption costing break-even point i.e. 5,33,000 units, the absorption costing
profit is ‘0’ and the marginal costing loss is Rs.14,000. Similarly, at marginal costing break-even point of
5,40,000 units, the marginal costing profit is ‘0’ and absorption costing profit is Rs.14,000. To conclude,

E M Reddy Page | 231


AMA-Notes

absorption costing system profit is Rs.14,000 is always higher than marginal costing system profit
because of fixed inside net stock (10,000 Units x Rs.1.4).

**Question no 5: The following is the production and sales given for six periods:
In ‘000 P1 P2 P3 P4 P5 P6
Sales 150 120 180 150 140 160
Production 150 150 150 150 170 140
Other details are:
Selling Price Rs.10
Variable manufacturing cost Rs.6
Fixed manufacturing cost (Budget and Actual) Rs.3,00,000
Non-manufacturing overhead Rs.1,00,000
Budgeted activity level for the period 1,50,000 Units
Calculate BEP under Absorption costing and Marginal costing system for all the periods.

Solution:

Facts:

Selling Price = Rs.10


Variable Manufacturing Cost = Rs.6
Fixed Manufacturing Overheads = Rs.3,00,000
Fixed non-manufacturing overheads = Rs.1,00,000
Units produced – Years 1 – 4 = 1,50,000 Units
Year 5 = 1,70,000 Units
Year 6 = 1,40,000 Units
Denominator level unit/Normal Capacity = 1,50,000 Units

Break-even Point under absorption costing system:

FMOH
TFC−[ UD xUP] TFC−Absorbed FMOH
Break-even Point = FMOH or SP−VC−Absorption Rate
SP−VC− UD

Period Computation Break-even Point


P1 – P4 [4,00,000−3,00,000x1,50,000] 50,000 Units
1,50,000
10−6−2
P5 [4,00,000−
3,00,000
x1,70,000] 30,000 Units
1,50,000
10−6−2
P6 [4,00,000−
3,00,000
x1,40,000] 60,000 Units
1,50,000
10−6−2

E M Reddy Page | 232


AMA-Notes

Break-even point formula:

FMOH
TFC−[ UD xUP] TFC−Absorbed FMOH
Break-even Point = FMOH or SP−VC−Absorption Rate; where
SP−VC− UD

TFC = Total Fixed Cost (FMOH + FNMOH)


FMOH = Fixed Manufacturing Capacity
UD = Normal Capacity/Denominator Level Units
VC = Variable cost per unit (VMC + VNMC)

Notes:

1) Derivation of the Formula:


a. Calculation of Total Cost:
FMOH
Manufacturing cost of goods sold – [VMC + ] x US
UD
Add: Variable non-manufacturing cost – [US x VNMC per unit]
Add: Fixed Non-manufacturing cost
FMOH
Add: FMOH – [UP x UD xUP] (Actual Overheads – Absorbed Overheads)
b. The total cost formula can be further simplified as follows:
FMOH FMOH
Total Cost = Units Sold x [VC + UD ] + FNMC + FMOH - xUP]
UD
c. It can be further simplified as follows:
FMOH FMOH
Total Cost = US x [VC + UD ] + TFC - UD xUP
d. The units sold can be called as break-even point only when sales = Total Cost
FMOH FMOH
US x SP = US x [VC + ] + TFC - xUP
UD UD
FMOH FMOH
US x SP – US x [VC + ] = TFC - xUP
UD UD
FMOH FMOH
US [SP – VC – ] = TFC - x UP
UD UD
FMOH
TFC−[ xUP]
UD
US = FMOH
SP−VC− UD
“US” is called Break-even point.
2) For decision making purpose always use marginal costing break-even point.
3) When manager’s financial rewards are linked to division’s profits calculated using absorption costing
system, they would like to know the sales demand to break-even and work towards achieving that
demand.
4) However, if the company is lenient on stock policy, the absorption costing may be counter-productive.
For example, if the manager feels the demand will be only 50,000 units compared to the budgeted
1,50,000 units, he may still break-even by producing more (1,50,000 Units). What happens is by
increasing production the stock is increased and by increasing stock fixed cost recondition is postponed
and low sales can even break-even which is for good for the company.
5) To conclude, both for decision making and performance evaluation it is better to use marginal costing
system.

E M Reddy Page | 233


AMA-Notes

7.3.4. Break-even point with two products

Question no 6: Ezee ltd. makes two products E and Z. All units produced are sold. There is no
inventory buildup. Production facilities may be used interchangeably for both the products. Sales
units are the limiting factor. The following information is given:
Price Level Proposed Increase
E Z Total Total
Contribution p.u. 25 20
Fixed Cost 46,000 47,500
Sales Units 3,000 2,000 5,000 4,000
For increase in quantities above 4,000 units for each product, there will be an increase in variable
selling costs (for the increased portion only) thereby reducing the contribution per unit to the
following figures:
Units Contribution per unit of E Contribution per unit of Z
4,001 – 5,000 20 15
5,001 – 6,000 15 10
Above 6,000 No Sales Possible
i. For the present level, find the break-even point with the present product.
ii. What is the minimum number of incremental units to be sold to recover the additional fixed
cost of Rs.47,500 to be incurred? (Present product mix need not be maintained)
iii. If you are allowed to choose the best product mix for the incremental level (while taking the
present mix given in the first table above for the present level) what would be the individual
product quantities and the corresponding total contributions, the total average contribution per
unit and the total profits for the complete production?
Solution:
Part 1: Break-even point for present situation
When we have multiple products with a given sales mix,
Fixed Cost
BEP = Weighted Contribution per unit
3 2
Weighted Contribution per unit = [5 x25] + [ 5 x20] = Rs.23
Rs.46,000
BEP = = 2,000 Units
Rs.23

Product ‘E’ = 2,000 Units x 3/5 = 1,200 Units


Product ‘Z’ = 2,000 Units x 3/5 = 800 Units
Check:
Contribution from product ‘E’ (1,200 Units x Rs.25) = Rs.30,000
Contribution from product ‘Z’ (800 Units x Rs.20) = Rs.16,000
Total Contribution = Rs.46,000
Less: Fixed Cost = (Rs.46,000)
Profit/(Loss) =0

E M Reddy Page | 234


AMA-Notes

Part 2: Calculation of units to be sold to recover incremental fixed cost


Increment Fixed Cost to be recovered = Rs.47,500
Contribution through additional units of product ‘E’ (1,000 Units x Rs.25) = (Rs.25,000)
Fixed cost to be recovered through product ‘Z’ = Rs.22,500
Contribution per unit of Product ‘Z’ = Rs.20
Rs.22,500
Additional units to be sold of product ‘Z’ = 1,125 Units
Rs.20

Alternatively,
Increment Fixed Cost to be recovered = Rs.47,500
Contribution through additional units of product ‘E’ (1,000 Units x Rs.25) = (Rs.25,000)
Contribution through incremental units of product ‘E’ (1,000 Units x Rs.20) = (Rs.20,000)
Fixed cost to be recovered through product ‘Z’ = Rs.2,500
Contribution per unit of Product ‘Z’ = Rs.20
Rs.2,500
Additional units to be sold of product ‘Z’ = 1,125 Units
Rs.20

In both the cases, we recover Rs.47,500 fixed cost.


Part 3: Production plan for the new plant
4,000 Units
- Product ‘E’ (3,000 Units – 4,000 Units) = 1,000 Units x Rs.25 = Rs.25,000
- Product ‘E’ (4,000 Units – 5,000 Units) = 1,000 Units x Rs.20 = Rs.20,000
- Product ‘Z’ (2,000 Units – 4,000 Units) = 2,000 Units x Rs.20 = Rs.40,000
Contribution = Rs.25,000 + Rs.20,000 + Rs.40,000 = Rs.85,000
Fixed Cost = Rs.47,500
Profit = Contribution – Fixed Cost = Rs.85,000 – Rs.47,500 = Rs.37,500
Rs.85,000
Average contribution per unit = 4,000 Units = Rs.21.25 per unit

7.4. Indifference Point

7.4.1. Introduction to Indifference point

1) Concept of indifference point will arise when we have two options:


a. Option 1 – Low fixed cost and high variable cost per unit
b. Option 2 – Low variable cost per unit and high fixed cost
2) When the volume is high it is better to go for low variable cost option and at low volume low fixed cost
option.
3) At some volume, both options will give same profit or have same cost which is called indifference point.
4) This indifference point volume can be expressed in two ways:
a. In Units
Difference in Fixed Cost
i. Difference in Variable Cost per Unit
Difference in Fixed Cost
ii. Difference in Contribution per Unit
b. In Volume
Difference in Fixed Cost
i. Difference in Variable Cost Ratio

E M Reddy Page | 235


AMA-Notes

Difference in Fixed Cost


ii. Difference in PV Ratio

Question no 7:
Company Variable cost per unit Fixed Cost
P 9 60,000
Q 5 50,000
At what sale range is P more profitable than Q and vice versa? Assume that both the products have
the same selling price.
Solution:
Step 1: Indifference point
Difference in Fixed Cost 90,000−60,000 30,000
Indifference point = Difference in Variable Cost per Unit = = = 7,500 Units
9−5 4

Check:
The cost of both companies at 7,500 units volume is as follows:
Particulars P Q
Variable Cost 7,500 Units x Rs.9 = Rs.67,500 7,500 Units x Rs.5 = Rs.37,500
Fixed Cost Rs.60,000 Rs.90,000
Total Cost Rs.127,500 Rs.127,500
At 7,500 units total cost is same for P & Q and since selling price is also same, they should have the same
profit.
Step 2: Conclusion
Range Company Reason
Less than 7,500 Units P Low fixed cost
At 7,500 Units P or Q Indifference Point
More than 7,500 Units Q Low Variable Cost

Question no 8: Two business AB ltd. and CD ltd. sells the same type of product in the same type of
market. Their budget profit and loss accounts for the year ending 2008 are as follows:
AB ltd. CD ltd.
Rs. Rs. Rs. Rs.
Sales 1,50,000 1,50,000
Less: Variable Costs 1,20,000 1,00,000
Less: Fixed Costs 15,000 35,000
1,35,000 1,35,000
Net profit budgeted 15,000 15,000
You are required to:
a) Calculate the break-even point of each business
b) Calculate the sales volume at which each of the business will earn Rs.5,000 profit; and
c) State which business is likely to earn greater profits in conditions of:
i. Heavy demand for the product;
ii. Low demand for the product.

E M Reddy Page | 236


AMA-Notes

Solution:

Step 1: Calculation of Break-even point


Particulars AB ltd. CD ltd.
Sales Rs.1,50,000 Rs.1,50,000
Less: Variable Cost (Rs.1,20,000) (Rs.1,00,000)
Contribution Rs.30,000 Rs.50,000
PV Ratio (Contribution/Sales) 20% 33.33%
Fixed Cost Rs.15,000 Rs.35,000
Break-even sales (Fixed Cost/PV Ratio) Rs.75,000 Rs.1,05,000

Step 2: Sales to earn profit of Rs.5,000


Particulars AB ltd. CD ltd.
Profit Rs.5,000 Rs.5,000
Add: Fixed Cost Rs.15,000 Rs.35,000
Contribution Rs.20,000 Rs.40,000
PV Ratio 20% 33.33%
Sales (Contribution/PV Ratio) Rs.1,00,000 Rs.1,20,00

Step 3: Calculation of indifference point


Difference in Fixed Cost 35,000−15,000 20,000
Indifference point = = = 13.33% = Rs.1,50,000
Difference in PV Ratio 33.33%−20%

Analysis of the company over the different sales range:


Sales Range Company Reason
Less than Rs.1,50,000 AB ltd. Low fixed cost (or) Low Break-even Sales
At Rs.1,50,000 AB ltd. or CD ltd. Indifference Point
Greater than Rs.1,50,000 CD ltd. Low Variable Cost Ratio (or) High PV Ratio

7.4.2. Indifference point as a state of demand

Question no 9: The current average weekly trading results of the Hotel Saravana Bhavan are shown
below.
(Rs.) (Rs.)
Turnover 2,800
Operating Costs:
Materials 1,540
Power 280
Staff 340
Building Occupancy costs 460 2,620
Profit 180
The average selling price of each meals is Rs.4; materials and power may be regarded as a variable
cost varying with the number of meals provided. Staff costs are semi-variable with a fixed cost of
Rs.200 per week; the building occupancy costs are all fixed.
Required:
Calculate the number of meals required to be sold in order to earn a profit of Rs.300 per week.

E M Reddy Page | 237


AMA-Notes

The owners of the restaurant are considering expanding their business and using under-utilized
space diversifying into either (1) take-away foods, or (2) high quality meals.
The sales estimates for both proposals are rather uncertain and it is recognized that actual sales
volume could be up to 20% either higher or lower than that estimated.
The estimated sales and costs of each proposal are:
Take-Away Foods High Quality Meals
Sales Volume, per week 720 200
Meals (Rs.) Meals (Rs.)
Average selling price, per meal 1.60 6.00
Variable costs, per meal 0.85 4.66
Incremental fixed costs, per week 610.00 282.00
If either of the above proposals were implemented it has been estimated that the existing
restaurant’s operations would be affected as follows:
i. As a result of bulk purchasing, material costs incurred would be reduced by 10 paisa per meal.
This saving would apply to all meals produced in the existing restaurant.
ii. Because more people would be aware of the existence of the restaurant it is estimated that
turnover would increase. If the take-away apply to all meals produced in the existing
restaurant’s sales would increase by one meal, alternatively if the high quality meals section
were open then for every five such meals sold the existing restaurant’s sales would increase by
one meal.
iii. A specific effect of implementing the take away food proposal would be a change in the terms
of the staff in the existing restaurant, the result of which would be that the staff wage of Rs.340
per week would have to be regarded as fixed cost.
Required:
Calculate, for each of the proposed methods of diversification:
i. The additional profit, which would be earned by the owners of the restaurant if the,
estimated sales were achieved.
ii. The sales volume at which the owners of the restaurant would earn no additional profits
from the proposed diversification.

Solution:

Part 1: Number of meals to be sold to earn a profit of Rs.300


Particulars Per Unit (Rs.) Existing (Rs.) Proposed (Rs.)
Sales (Rs.280/Rs.4 = 700 Units) 4 2,800
Less: Variable Cost
Material (Rs.1,540/700 Meals) 2.2 1,540
Power (Rs.280/700 Meals) 0.4 280
Staff (Rs.140/700 Meals) 0.2 140
Total Variable Cost 2.8 1,960
Contribution 1.2 840 960
Less: Fixed Cost
Building 460
Fixed Staff Cost 200
Total Fixed Cost 660 660
Profit 180 300

E M Reddy Page | 238


AMA-Notes

Target Contribution = Rs.960


Contribution per meals= Rs.1.2
Target meals = Rs.960/Rs1.2
= 800 Meals

Alternatively,

Additional profit = Rs.300 – Rs.180 = Rs.120


Incremental Meals = Rs.120/Rs.1.2 = 100 Meals
Target Units = 700 Meals + 100 Meals = 800 Meals

Part 2: Diversification

Step 1: Additional profit the hotel can earn when diversification is made
After Diversification
Before Diversification Take Away High Quality
Existing Take Away Existing High Quality
Selling Price 4 4 1.6 4 6
Material 2.2 2.1 0.85 Given 2.1 4.66 Given
Power 0.4 0.4 0.4
Staff 0.2 - - 0.2 -
Total Variable Cost 2.8 2.5 0.85 2.7 4.66
Contribution 1.2 1.5 0.75 1.3 1.34
Numbers (Meals) 700 772 720 740 200
Contribution 840 1158 540 962 268
1698 1230
Fixed Cost:
Building 460 460 610 Given 460 282 Given
Staff 200 200 + 140 200
Total Fixed Cost 660 800 610 660 282
1410 942
Profit 180 288 288

If the estimated diversification sales is achieved then both the options gives us Rs.108 additional profit
(Rs.288 – Rs.180).

Step 2: Number of take away meals to be sold to earn no additional profit from diversification
Particulars Quantity Contribution per unit Total Contribution Fixed Cost Profit
Existing 700 + 1/10 of X 1.5 1.5 (700 + 1/10X) 800
Take Away X 0.75 0.75X 610
1,410 180

1.5 (700 + 1/10X) + 0.75X – 1,410 = 180


1,050 + 0.15X + 0.75X – 1,410 – 180 = 0
0.9X = 540 → X = 600

Step 3: Number of High quality meals to be sold to earn no additional profit from diversification

E M Reddy Page | 239


AMA-Notes

Particulars Quantity Contribution per unit Total Contribution Fixed Cost Profit
Existing 700 + 1/5 of Y 1.30 1.3 (700 + 1/5Y) 660
High Quality Y 1.34 1.34Y 282
942 180

1.3 (700 + 1/5Y) + 1.34Y – 942 = 180


910 + 0.26Y + 1.34Y = 180 + 942
1.6Y = 180 + 942 – 910
Y = 133 Units

Conclusion: If the hotel is able to sell 600 take away meals or 133 high quality meals, it earns no additional
profit through diversification i.e. it just breaks-even.

Notes:

1) Interpretation of the two options viability:


Particulars Estimate Maximum Minimum Break-even
Take Away (Meals) 720 864 576 600
High Quality (Meals) 200 240 160 133

2) At normal state of demand both options are equally good as they give the same additional profit of
Rs.108.
3) If we fear a fall in demand below normal it is better to select high quality meals because even if
maximum 20% drop occurs still 160 meals could be sold which is above the break-even point of 133
meals i.e. this option surely gives additional profit even at lowest demand.
4) Since at normal demand both are good, at low demand high quality is good, at high demand take away
should be better.
5) Here indifference point is not expressed in units but expressed as a state of demand.

7.4.3. Limiting Factor and Indifference Point

Question no 10: Modern packaging corporation specializes in the manufacture of plastic bottles
through moulding operations. The firm has four moulding machines, each capable of producing of
100 bottles per hour. The firm estimates that the variable cost of producing a plastic bottler is 20
paisa. The bottles are sold for 50 paisa each.
A local toy company that would like the firm to produce a moulded plastic toy for them has
approached management. The toy company is willing to pay Rs.3 per unit for the toy. The variable
cost to manufacture the toy will be Rs.2.40. In addition, modern packaging corporation would have
to incur a cost of Rs.20,000 to construct the needed mould exclusively for this order. Because the
toy uses more plastic and is of more intricate shape than bottle, a moulding machine can produce
only 40 units per hour. The customer want 1,00,000 units. Assume that modern packaging
corporation has the total capacity of 10,000 machine hours available during the period in which the
toy company wants the delivery of toys. The firm’s fixed costs, excluding the costs to construct the
toy mould, during the same period will be Rs.2,00,000.
Required:

E M Reddy Page | 240


AMA-Notes

a) If the management predicts that the demand for its bottles will require the use of 7,500
machine hours (or) less during the period, should the special order be accepted? Give
reasons.
b) If the management predicts that the demand for its bottles will be higher than its ability to
produce bottles, should the order be accepted? Why?
c) The management has located a form that has just entered the moulded plastic business.
This firm has considerable excess capacity and more efficient moulding machines and is
willing to sub-contract the toy job (or) any portion of it, for Rs.2.80 per unit. It will
construct its own toy mould. Determine modern pacing corporation’s minimum expected
excess machine hour capacity needed to justify production any portion of the order itself
rather that subcontracting it entirely.
d) The management predicted that it would have 1600 hours of excess machine capacity
available during the period. Consequently, it accepted the toy order and subcontracted
36,000 units to the other plastic company. In fact, demand for bottles turned out to be
9,00,000 units for the period. The firm was able to produce only 8,40,000 units because it
has produced the toys. What was the cost of prediction error of failure to predict demand
correctly?

Solution:

Step 1: Calculation of contribution per machine hour and ranking


Particulars Bottle (Rs.) Toy (Rs.)
Selling Price 0.50 3.00
Variable Cost (0.20) (2.40)
Contribution 0.30 0.60
Units/Hour 100 40
Contribution/Hour 30 24
Rank 1 2
Fixed Cost 2,00,000 20,000

Step 2: Can toy order be accepted if the bottle uses maximum 7,500 hours

In this case we have unused machine capacity of 2,500 hours which is sufficient to produce the 1,00,000
toys (2,500 Hours x 40 Toys = 1,00,000 Toys). The toy order can be accepted if it is viable financially.

Contribution from Toy order (2,500 Hours x 24 = Rs.60,000 or (1,00,000 Toys x 0.6)
Less: Incremental fixed cost = Rs.20,000
Incremental Profit = Rs.40,000

Instead of keeping 2,500 hours idle it is better to use it for Toy order and earn an additional profit of
Rs.40,000.

Step 3: Can toy order be accepted if the bottle demand requires more than 10,000 machine hours

No, because we should not transfer the limiting factor from first rank product to second rank i.e. from
Bottle to toys hours should not be accepted. Toy can be manufactured only when hours of left after
manufacturing for bottle demand.

E M Reddy Page | 241


AMA-Notes

Step 4: Manufacture (vs.) Subcontract


Particulars Manufacture (Rs.) Sub-Contract (Rs.)
Selling Price 3.00 3.00
Less: Variable Cost (2.40) (2.80)
Contribution/unit 0.60 0.20
Fixed Cost 20,000 -

Advantage of manufacturing is low variable cost and advantage of sub-contracting is no fixed cost. At high
volumes manufacturing is better and at low volumes sub-contracting is better.
Difference in fixed csot 20,000−0
Indifference point = Difference in varibale cost per unit = = 50,000 Toys
2.8−24

50,000 Toys
Indifference point in hours = = 1,250 Hours
40 Toys/Hour

1) It is worth manufacturing toys only when we plan to manufacture more than 50,000 toys.
2) To manufacture more than 50,000 toys we require at least 1,250 hours after meeting bottle demand.
3) Thus the minimum excess capacity required to justify the toy manufacturing is 1,250 Hours.
4) The implication of the above number can be understood as follows:
a. Situation 1 – Bottle demand uses 9,000 Hours – Only 1,000 Hours left for toy manufacture.
Since it is less than 1,250 hours do not manufacture toys and sub-contract fully leaving 1,000
hours idle.
b. Situation 2 – Bottle demand uses 8,000 Hours – Since available hours 2,000 greater than the
minimum 1,250 hours, manufacture 80,000 (2,000 Hours x 40 Toys) toys in this 2,000 hours and
sub-contract balance 20,000 Toys.

Step 5: Cost of prediction error

Step A: Based on the actual data as implemented by the management


Particulars Bottles Toy Manufacture Toy Sub-Contract
Units (Actual) (Toys) 8,40,000 64,000 (1,600 Hours x 40 Toys) 36,000
Contribution/Unit (Rs.) 0.30 0.60 0.20
Total Contribution (Rs.) 2,52,000 38,400 7,200
Fixed Cost (Rs.) 2,00,000 20,000 -
Profit (Rs.) 52,000 18,400 7,200

Total Profit (Rs.) = 52,000 + 18,400 + 7,200 = 77,600

Step B: Should have been if the bottle demand is predicted correctly


Particulars Bottles Toy Manufacture Toy Sub-Contract
Units (Actual) (Toys) 9,00,000 - (Note 1) 1,20,000
Contribution/Unit (Rs.) 0.30 - 0.20
Total Contribution (Rs.) 2,70,000 - 20,000
Fixed Cost (Rs.) 2,00,000 - -
Profit (Rs.) 70,000 - 20,0000

E M Reddy Page | 242


AMA-Notes

Total Profit (Rs.) = 70,000 + 20,000 = 90,000

Note 1:

Surplus Hours = 10,000 Hours – [9,00,000 Bottles/100 Bottles per Hour)


= 1,000 Hours which is less than 1,250 Hours. Therefore, do not manufacture.

Cost of prediction error = Rs.90,000 – Rs.77,600 = Rs.12,400

Additional Notes:

1) It is generally believed that keeping capacity idle is bad but sometimes it may be good to keep the
capacity idle rather than use it.
Example: After bottle manufacture if hours available is less than 1,250 hours it is better to leave the
hours unused and subcontract the toy order.
2) Let us consider two situations:
i. Situation 1: Bottle requires 7,600 Hours
ii. Situation 2: Bottle requires 9,000 Hours
There is no sub-contracting option. How to deal with these two situations i.e. should toy order
be accepted or rejected.
3) In situation 1, the toy order can be accepted only by transferring 100 hours from bottle to toy.
i. Benefit → Profit from Toy order = Rs.40,000
ii. Cost → Contribution lost from Bottles = Rs.3,000 (100 Hours x Rs.30)
In this situation we break the myth that all the hours should be given to first rank product i.e. we
compromise on the limiting factor ranking to use full capacity.
4) In situation 2,the toy order can be accepted only by transferring 1,500 hours from bottle to toy.
i. Benefit → Profit from Toy Order = Rs.40,000
ii. Cost → Contribution lost from Bottles = Rs.45,000 (1,500 Hours x Rs.30)
Here, we should reject the toy order i.e. it is better to keep the capacity idle rather than
compromising on limiting factor ranking.
5) There should be an indifference point between limiting factor compromise and idle capacity. It is as
follows:
Particulars Fixed Profit Variable profit
Limiting Factor Rs.40,000 -
Idle Capacity - Rs.30
Change in profit Rs.40,000 Rs.30
Rs.40,000
Indifference point = = 1,333 Hours
Rs.30
Indifference point = 7,500 Hours + 1,333 Hours = 8,833 Hours
Range Selection
7,500 Hours – 8,833 Hours Do bottle and toy by compromising on limiting factor
8,833 Hours Compromise on limiting factor or keep the capacity is
idle i.e accept or reject toy order
Greater than 8,833 Hours Reject Toy order and keep the capacity is idle

E M Reddy Page | 243


AMA-Notes

7.4.4. Understanding how to analyze a semi-variable cost

Example 1:
Units Cost (Rs.)
1,000 15,000
2,000 20,000
3,000 25,000

1) The above cost is not fixed because it changes with volume. It is neither variable because it is not
constant per unit. This is semi-variable cost.
2) Semi-variable cost should be analyzed into variable and fixed portion which is done as follows:
Change in cost 20,000−15,000
Variable cost per unit = Change in units = = Rs.5 per unit
2,000−1,000
Fixed cost = Total Cost – Variable Cost = Rs.15,000 – Rs.5,000 (1,000 Units x Rs.5) = Rs.10,000

Example 2:
Units Cost (Rs.)
1,000 15,000
2,000 19,000
3,000 25,000

1) The cost is semi-variable but the relationship between volume and cost is not perfectly correlated
because the points do not fit into but are scattered.

E M Reddy Page | 244


AMA-Notes

2) We have to analyze the semi-variable cost using regression method by finding out line of best fit which
is done as follows:
Y = mX + C
∑ Y = nC + m∑ X → Equation no.1
∑ XY = C∑ X + m∑ X 2 → Equation no.2
When we solve the equations we will get m and C i.e. variable cost and fixed cost.
X Y XY 𝐗𝟐
1,000 Units 15,000 1,50,00,000 10,00,000
2,000 Units 19,000 3,80,00,000 40,00,000
3,000 Units 25,000 7,50,00,000 90,00,000
16,000 Units 59,000 12,80,00,000 1,40,00,000

59,000 = 3C + 6,000m
12,80,00,000 = 6,000C + 1,40,00,000m
When we solve the two equations we obtain m and C, m is variable cost and C is fixed cost.

Question no 11: Super press ltd is considering launching a new monthly magazine at a selling price
of Rs.1 per copy. Sales of the magazine are expected to be 5,00,000 copies for month, but it is
possible that the actual sales could differ quite significantly from this estimate.
Two different methods of producing the magazine are being considered and neither would involve
any additional capital expenditure. The estimated production costs for each of the two methods of
manufacture, together with the additional marketing and distribution costs of selling the new
magazine, are summarized below:
Method A Method B
Variable Costs 55 paisa per copy 50 paisa per copy
Specific Fixed Costs Rs.80,000 per month Rs.1,20,000 per month
For semi-variable cost the following estimate have been obtained:
3,50,000 copies Rs.55,000 pm Rs.47,500 pm
4,50,000 copies Rs.65,000 pm Rs.52,500 pm
6,50,000 copies Rs.85,000 pm Rs.62,500 pm
It may be assumed that the fixed cost content of the semi-variable cost will be remaining constant
throughout the range of activity shown.
The company currently sells a magazine covering related topics to those that will be included in
the new publication and consequently it is anticipated that sales of this existing magazine will be
adversely affected. It is estimated that for every ten copies sold of the new publication, sales of the
existing will be reduced by one copy.
Sales and cost data of the existing magazine are shown below:
Sales 2,20,000 copies per month
Selling Price 85 paisa per copy
Variable Costs 35 paisa per copy
Specific Fixed Costs Rs.80,000 per month
Required:
(a) Calculate, for each production method, the net increase in company profits which will result
from the introduction of the new magazine, at each of the following levels of activity.
 5,00,000 copies per month
 4,00,00o copies per month

E M Reddy Page | 245


AMA-Notes

 6,00,000 copies per month


(b) Calculate, for each production method, the amount by which sales volume of the new magazine
could decline from the anticipated 5,00,000 copies per month, before the company makes no
additional profit from the introduction of the new publication.
(c) Briefly identify and conclusions which may be drawn from your calculations.
(d) Calculate additional profit if the demand for the magazine is 7,00,000 copies.

Solution:

Step 1: Segregating semi-variable cost into variable and fixed portion


Change in Cost
Variable cost per unit = Change in units

65,000−55,000
Method A = 4,50,000−3,50,000 = Rs.0.10 per copy
52,500−47,500
Method B = 4,50,000−3,50,000 = Rs.0.05 per copy

Fixed Cost = Total Cost – Variable cost

Method A = Rs.55,000 – (3,50,000 Copies x Rs.0.10) = Rs.20,000


Method B = Rs.47,500 – (3,50,000 Copies x Rs.0.05) = Rs.30,000

Step 2: Contribution lost

Selling price (old magazine) = Rs.0.85


Variable cost (old magazine) = Rs.0.35
For every 10 new magazines sold, 1 old magazine sales lost i.e. for 10 new magazines contribution lost =
Rs.0.50. Therefore, contribution loss per new magazine sold Rs0.05 (Rs.0.5/10 Copies)

Step 3: Analyzing data for method ‘A’ and method ‘B’


Particulars Method A (Rs.) Method B (Rs.)
Selling Price 1.00 1.00
Less: Variable
Direct 0.55 0.50
Semi Variable 0.1 0.05
Opportunity Cost 0.05 0.05
Total Variable Cost 0.70 0.60
Contribution 0.30 0.40
Less: Fixed Cost 80,000 1,20,000
Fixed Cost 80,000 1,20,000
Semi-Variable 20,000 30,0000
Total Fixed Cost 1,00,000 1,50,000

Step 4: Calculation of profit at different sales level under both methods

Method – A:

E M Reddy Page | 246


AMA-Notes

Levels 5,00,000 Units 4,00,000 Units 6,00,000 Units


Contribution @ 30 paisa (Rs.) 1,50,000 1,20,000 1,80,000
Less: Fixed Cost 1,00,000 1,00,000 1,00,000
Profit 50,000 20,000 80,000

Method – B:
Levels 5,00,000 Units 4,00,000 Units 6,00,000 Units
Contribution @ 20 paisa (Rs.) 2,00,000 1,60,000 2,40,000
Less: Fixed Cost 1,50,000 1,50,000 1,50,000
Profit 50,000 10,000 90,000

Step 5: Indifference point


Difference in fixed cost 1,50,000−1,00,000 50,000
Indifference point = Differnce in contribution per unit = = = 5,00,000 Copies
0.4−0.3 0.1

Sales Range Method Selected Reason


Less than 5,00,000 copies Method A Low fixed cost
5,00,000 copies Method A or Method B Indifference point
More than 5,00,000 copies Method B Low Variable cost

Step 6: Calculation of break-even sales and margin of safety for the new magazine
Particulars Method A Method B
Fixed Cost 1,00,000 1,50,0000
Contribution per unit 0.30 0.40
BEP (Copies) 3,33,333 3,75,000
Total Sales 5,00,000 5,00,000
Margin of Safety 1,66,667 1,25,000

If method A is adopted, from the anticipated 5,00,000 copies we can tolerate 1,66,667 copies drop. If
method B is adopted, we can tolerate a drop of 1,25,000 copies drop. If the drop is beyond this level the
incremental profit is ‘0’ and existing profit is eroded.

Step 7: Analysis of the different sales range

We have to make two decisions based on marketing team’s inputs.


1) Old magazine or new magazines
2) Method A or Method B

First decision is based on break-even point and second decision based on indifference point.
Sales Range Decision
Less than 3,33,333 copies Only old Magazine
3,33,333 copies – 5,00,000 copies Old & New Magazine – Method A
5,00,000 Copies Produce new magazine using Method A or Method B along with old
magazine
More than 5,00,000 Copies Produce new magazine using Method B along with old magazine

E M Reddy Page | 247


AMA-Notes

Step 8: Incremental profit when new magazine demand is 7,00,000 copies

Contribution (7,00,000 Copies x Rs.0.4) = Rs.2,80,000


Less: Fixed Cost = (Rs.1,50,000) This calculation is wrong.
Incremental Profit = Rs.1,30,000

Alternatively,

Contribution from new magazine (7,00,000 Copies x Rs.0.45) = Rs.3,15,000


Less: Contribution lost from old magazine (70,000 x Rs.0.35) = (Rs.35,000) This calculation is wrong.
Less: Incremental fixed cost = (Rs.1,50,000)
Incremental profit = Rs.1,30,000

Specific fixed cost of old magazine = Rs.80,000.


Contribution per copy of old magazine = Rs.0.50
Rs.80,000
BEP (Old Magazine) = = 1,60,000 Units
Rs.0.50
If old magazine demand is less than 1,60,000 copies it is better to close the old magazine because the
contribution does not recover the specific fixed cost.

Current sales of old magazine = 2,20,000 Copies


Less: Break-even sales = 1,60,000 Copies
Drop in sales allowed = 60,000 Copies

Any drop beyond 60,000 copies the old magazine will be closed. The maximum new magazine that could be
sold for old magazine to continue is 6,00,000 copies, beyond that old magazine would be discontinued. At
7,00,000 copies only new magazine using method B.

Contribution from new magazine (7,00,000 copies x Rs.0.40) = Rs.3,15,000


Less: Contribution lost from old magazine (2,20,000 x Rs.0.5) = (Rs.1,10,000)
Add: Savings in specific fixed cost = Rs.80,000
Less: Incremental fixed cost of new magazine = (Rs.1,50,000)
Incremental Profit = Rs.1,35,000

Question no 12: XY ltd. makes two products X and Y, whose respective fixed costs are F1 and F2.
You are given that the unit contribution of Y is one fifth less than the unit contribution of X, that
the total of F1 and F2 is Rs.1,50,000, that the BEP of X is 1,800 units (for BEP of X F2 is not
considered) and that 3,000 units is the indifference point between X and Y (i.e. X and Y make equal
profits at 3,000 unit volume, considering their respective fixed costs). There is no inventory build
up as whatever is produced is sold. You are required to find out the values F1 and F2 and units
contribution of X and Y.

Solution:

Let Contribution per unit of ‘X’ be ‘C’. Therefore contribution per unit of ‘Y’ is ‘C – 1/5C = 4/5C’.
Fixed Cost
Breakeven Point = Contribution per unit

E M Reddy Page | 248


AMA-Notes

F1
Breakeven Point of ‘X’ = = 1,8000 → F1 = 1800C
C

Difference in Fixed Cost


Indifference point = Difference in contributino per unit = 3,000

F1 −F2
1 = 3,000
c
5
600C = F1 − F2
600C = 1,800 C - F2
F2 = 1,200C
F1 + F2 = 1,50,000
1,800 C + 1,200 C= 1,50,000
3,000C = 1,50,000
1,50,000
C= = Rs.50
3,000
4 4
C = x 50 = Rs.40
5 5
F1 = 1,800C = 1,800 x Rs.50 = Rs.90,000
F2 = 1,200C = 1,200 x Rs.50 = Rs.60,000

Conclusion:

Contribution per unit of ‘X’ = Rs.50


Contribution per unit of ‘Y’ = Rs.40
Fixed Cost of ‘X’ = Rs.90,000
Fixed Cost of ‘Y’ = Rs.60,000

7.5. Limiting Factor Problems

7.5.1. Basic Limiting factor allocation problems

Question no 12: As a part of its rural upliftment programme, the government has put under
cultivation a farm of 96 hectares to grow tomatoes of four varieties: Royal Red, Golden Yellow,
Juicy Crimson and Sunny Scarlet. Of the total, 68 hectares are suitable for all four varieties but the
remaining 28 hectares are suitable for growing only Golden Yellow and Juicy Crimson. Labour is
available for all kinds of farm and is no constraint. The market requirement is that all four varieties
of tomatoes must be produced with a minimum of 1,000 boxes of any one variety.
The farmers engaged have decided that the area devoted to any crop should be in terms of
complete hectares and not in fractions of a hectare. The other limitation is that not more than
20,000 boxes of any one variety should be produced. The following data re relevant.
Royal Golden Juicy Sunny
Red Yellow Crimson Scarlet
Annual Yield:
Boxes per hectare 350 100 70 180
Costs: Rs. Rs. Rs. Rs.
Direct Materials 476 216 196 312
Labour:

E M Reddy Page | 249


AMA-Notes

Growing per hectare Harvesting and 896 608 371 528


packaging per box
Transport per box 5.20 5.20 4.00 9.60
Market price per box 15.38 15.87 18.38 22.27
Fixed Overheads per annum Rs.
Growing 11,200
Harvesting 7,400
Transport 7,200
General Administration 10,200
Required:
(a) Within the given constraints, the area to be cultivated with each variety of tomatoes if the
largest total profit has to be earned.
(b) The amount of such profit in rupees.

Solution:

Step 1: Calculation for contribution per hectare of each variety of each variety
Particulars Royal Red Golden Yellow Juicy Crimson Sunny Scarlet
Selling Price 15.38 15.87 18.38 22.27
Less: Variable Cost
Direct Material 1.36 2.16 2.80 1.73
Labour 2.56 6.08 5.30 2.93
Harvesting 3.60 3.28 4.40 5.20
Transport 5.20 5.20 4.00 9.60
Total Variable cost (Rs.) 12.72 16.72 16.50 19.46
Contribution per box (Rs.) 2.66 (-0.85) 1.88 2.81
Yield/Hectare (Boxes) 350 100 70 180
Contribution per Hectare (Rs.) 931 -85 131.6 505.80
Rank – Versatile I IV III II
Rank – Specialized - II I -

Step 2: Calculation of Minimum and Maximum hectares

We should produce minimum 1,000 boxes of each variety and maximum 20,000 boxes of each variety.
Particulars Royal Red Golden Yellow Juicy Crimson Sunny Scarlet
A) Yield/Hectare (Boxes) 350 100 70 180
B) Minimum boxes to be produced 1,000 1,000 1,000 1,000
C) Maximum boxes can be produced 20,000 20,000 20,000 20,000
D) Allocation Category Versatile Specialized Specialized Versatile
E) No.of hectares for minimum 3 10 15 6
requirement (B/A)
F) No.of hectares for maximum 57 NA NA NA
requirement (C/A)

Step 3: Allocation of 96 hectares

E M Reddy Page | 250


AMA-Notes

Production Minimum Balance Total


Versatile Specialized Versatile Specialized
Royal Red 3 - 54 - 57
Golden Yellow - 10 - - 10
Juicy Crimson - 15 - 3 18
Sunny Scarlet 6 - 5 - 11
Total 9 25 59 3 96

Step 4: Calculation of the profit from the above mix


Production Hectare Contribution/Hectare (Rs.) Contribution (Rs.)
Royal Red 57 931 53,067
Golden Yellow 10 (85) (860)
Juicy Crimson 18 131.6 2,368.80
Sunny Scarlet 11 505.80 5,563.80
Total Contribution 60,149.60
Total Contribution (Approximately) 60,150
Less: Fixed Cost (36,000)
Profit 24,150

Notes:

1) We cannot allocate fraction of hectares to any crop. Hence we round off and allocate.
2) Minimum Hectares is always rounded off to the higher number. For example, for juicy crimson
1,000/70 =14.28 hectares but rounded off to 15 hectares because rounding off to 14 hectares will not
ensure minimum 1,000 boxes.
3) In case of maximum the rounding off should be understood as follows:
Royal Red = 20,000/350 = 57.14 hectares. The company has two options
i. Allocate 58 hectares to royal red and cultivate 57.14 hectares living balance 0.86 hectares idle
→ Advantage is maximum 1st rank is produced and disadvantage is idle capacity.
ii. Allocate 57 hectares to royal red and cultivate one full hectare sunny scarlet → Advantage is
no idle capacity and disadvantage is 0.14 hectares transferred from 1st rank to 2nd rank
product.
4) By rounding off to 57 hectares:
i. Benefit → Contribution from Sunny Scarlet = Rs.505.80
ii. Cost → Contribution lost from Royal Red – Rs.931 x 0.14 hectares = Rs.130.34
Hence, rounded off to 57 hectares
501.80
5) Indifference point = = 0.54 Hectares → 57 Hectares + 0.54 Hectares = 57.54 Hectares
931
If the fraction is above this round off to 58 hectares else round off to 57 hectares.

7.5.2. Limiting factor in Make or Buy Situation

1) Generally, it is believed that making is cheaper than buying because making involved only variable cost
and buying includes share of fixed cost, supplier’s profit etc., in purchase price.
2) Make (or) buy decision can be classified into types:

E M Reddy Page | 251


AMA-Notes

i. A Long term investment decision → For making we require machine. Hence capital outlay
occurs in year 0. Due to making operationally we saved cost each year which results inflow. If
NPV is positive make by investing in machine else buy.
ii. A Short term limiting factor allocation → In this case we have facilities to make but limiting
factor exists that prevents making the entire requirement. Here what should be made using
limited source and what should be bought from outside is the decision to be made.

Question no 13: A company is preparing its production budget for the year ahead. Two of its
processes are concerned with the manufacture of three components, which are used in several of
the company’s products. Capacity (machine hours) in each of these two processes is limited to
2,000 hours.
Production costs are as follows:
Component X Component Y Component Z
(Rs. Per unit) (Rs. Per unit) (Rs. Per unit)
Direct Materials 15.00 18.50 4.50
Direct Labour 12.00 12.50 8.00
Variable Overhead 6.00 6.25 4.00
Fixed Overhead:
Process M 6.00 6.00 4.50
Process N 10.50 10.50 3.50
Total Cost 49.50 53.75 24.50
Requirements for components X, Y and Z (in units) for the following year:
X 300
Y 300
Z 450
Fixed overhead is absorbed on the basis of machine hours at the following rates:
Process M Rs.3.00 per hour
Process N Rs.3.50 per hour
Components X and Z could be obtained from an outside supplier at following prices per unit.
X Rs.44.00
Z Rs.23.00
Required:
(a) Demonstrate the insufficient capacity is available to produce the requirements for components
X, Y and Z in the year ahead, and calculate the extent of the shortfall.
(b) Determine the requirements for bought in components in order to satisfy the demand for
components at minimum cost.

Solution:

Step 1: Identifying limiting factor


Particulars Requirement of Hours per unit Units Hours for Process.
M N M N
X 6/3 = 2 Hours 10.5/3.5 = 3 Hours 300 300 x 2 = 600 300 x 3 =900
Y 6/3 = 2 Hours 10.5/3.5 = 3 Hours 300 300 x 2 = 600 300 x 3 =900
Z 4.5/3 = 1.5 Hours 3.5/3.5 = 1 Hour 450 450 x 1.5 = 675 300 x 1 =300
Total Hours Required 1,875 2,250

E M Reddy Page | 252


AMA-Notes

Available Hours 2,000 2,000


Surplus/ (Short fall) 125 250

The entire requirement cannot be produced. From the above because we have insufficient process N hours.
Hence, we should buy some requirements from outside.

Product ‘Y’ should be necessarily manufactured because it does not have buying option. But ‘X’’ or ‘Z’
should be decided based on based on the relative extra cost.

Step 2: Make (or) Buy Table


Product Buy Variable Contri- Prod. ‘N’ Contribution/ Rank
Cost Cost bution hours/unit Hour (N)
X 44 33 11 3 3.67 2 #
Y 23 16.5 6.5 1 6.5 1 Manufacture

# Manufacture to the extent possible and the remaining shall be bought out.

Step 3: Allocation of limiting factor


Components No.of Manufacturing Manufacturing Cumulative Hours
Units hours/unit hours (Max 200)
Y 300 3 900 900
Z 450 1 450 450
X 216 3 650 (Balance) 2000
X 84 Buy (300 – 216)

Question no 14: X ltd manufactures and sells a range of sports equipments. The marketing director
would like to increase X ltd.’s share of the market, and is considering an advertising campaign in
order to stimulate demand for the products.
Two alternative sales budgets have been put forwarded for the year ahead.
Product (‘000 Units)
A B C D
Budget 1 – without advertising 180 280 260 150
Budget 1 – with advertising 200 310 285 165
The advertising campaign would cost Rs.2,90,000.
Selling prices and variable production costs are budgets as follows: [Rs. per unit]
Products A B C D
Selling Prices 9.95 11.95 22.95 19.95
Variable production costs:
Direct Materials 4.20 5.50 12.70 10.40
Direct Labour 1.70 1.70 2.80 2.65
Variable Overhead 0.60 0.60 1.00 0.90
The variable overheads are absorbed on a machine hour basis at a rate of Rs.1.00 per hour. Fixed
overheads total Rs.25,70,000. Production capacity is limited to 7,15,000 machine hours in the year
ahead. Products A and C could be bought in ad X ltd would be prepared to do this to make up any
shortfall of production requirements if necessary and justify. Products A and C could be bought-in
for Rs.8.90 per unit and Rs.20.00 per unit respectively.

E M Reddy Page | 253


AMA-Notes

If the advertising campaign was shown to be successful, increased production requirements would
then be met in the long run by investment in additional facilities. In the meantime, the company
would like to assesse the potential of the advertising campaign in the year ahead, and if justified,
determine the best way to obtain the required quantities of Product A and C.
Required:
On the basis of expectations for the year ahead, determine whether investment in the advertising
campaign would be worthwhile and how production facilities would be best utilized.

Solution:

Step 1: Identification of limiting factor


Product Machine Sales ‘000 (Units) Hours ‘000
Hours/Unit Without With Without With
Advertising Advertising Advertising Advertising
A 0.60 180 200 108 (180 x 0.6) 120 (200 x 0.6)
B 0.60 280 310 168 (280 x 0.6) 186 (310 x 0.6)
C 1.00 260 285 260 (260 x 1.0) 285 (285 x 1.0)
D 0.90 150 165 135 (150 x 0.9) 148.5 (165 x 0.9)
Total Hours Required 671 739.5
Available Hours 715 715

The machine hours per unit is calculated with the help of variable overhead absorption rate of Rs.1 per
hour. For example ‘A’ is charge 0.60 variable overhead per unit which means ‘A’ consumes 0.60 ( Rs.0.60 x
1 Hour)

Step 2: Make (or) Buy Decision

If the company decides to advertise to meet the increased demand it requires 7,35,000 machine hours but
only 7,15,000 hours are available. Thus it is not possible to produce and sell the entire demand. A part of
the demand has to be bought and sold.

Product ‘B’ and ‘D’ does not have buying option. They should be compulsorily manufactured. Hence will
not participate in make (or) buy ranking.
Product Buying Variable Contri- Manufacturing Contribution/ Rank
Cost Cost bution Hours Hour (N)
A 8.9 6.5 2.4 0.6 4 1
C 20 16.5 3.5 1 3.5 2

Manufacture ‘A’, Manufacture ‘C’ to the extent possible and balance requirement of ‘C’ should be bought
from outside.

Step 3: Allocation of Limiting factor (Machine Hours)


Product Units Hours/Unit Hours ∑ 𝐇𝐨𝐮𝐫𝐬
B 310 0.6 186 186
D 165 0.9 148.5 334.5
A 200 0.6 120 454.5

E M Reddy Page | 254


AMA-Notes

C 260.5 1 260.5 715


C Buy 24.5 (385 – 260.5)

Step 4: Advertise or not


Product Selling Variable Contribution per Before ‘000s After ‘000s
Price Cost Unit Units Contribution Units Contribution
A 9.95 6.50 3.45 180 621 200 690
B 11.95 7.80 4.15 280 1,162 310 1,286.50
C 22.95 16.5 6.45 260 1,677 260.5 1,680.22
C (Buy) 22.95 20.00 2.95 - - 24.5 72.28
D 19.95 13.95 6 150 900 165 990
Total Contribution 4,360 4,719
Less: Fixed Cost 2,570 2,860
Total Profit 1,790 1,859

Alternatively,

Incremental Contribution due to advertisement (4,719 – 4,360) = 359


Less: Specific Advertisement Cost = (290)
Incremental profit due to advertising = 69

Conclusion: Recommended to go for advertisement campaign and manage the shortfall in production
requirement by purchasing product ‘C’ to the extent of 24,500 units.

Notes:

1) Short term decisions should be purely short term, should not be extended to long periods. For example,
a company’s plant capacity is 1,00,000 units and current demand is 80,0000 units with a selling price of
Rs.50, variable cost of Rs.30. An offer comes for purchase of 20,000 units at a selling price of Rs.31.
Should this offer be accepted?
a) If it is a onetime offer?
b) If it is an offer for next four years?
Answer:
a) Instead of having the capacity idle it is better to accept the offer and have contribution of
Rs.20,000 (20,000 Units x Rs.1) towards the factory fixed cost but this decision should be purely
short term (or) one time.
b) If it is for four years, it become long term decision for which short term decision making
concepts should not be applied. If we feel that for next four years, the demand is going to be
only 80,000 units, the company should think off downsizing the plant and save fixed cost rather
than use the idle capacity for a nominal Rs.1 contribution.
2) In the above problem, the decision to buy a portion of product ‘C’s requirement is purely short term. In
long term the company should try to remove the limiting factor by investing in additional facilities.

**Question no 15: A processing company EF is extremely busy. It has increased its output and
sales from 12,900 kg in quarter 1 to 17,300 kg in quarter 2 but, though demand is still rising, it
cannot increase its outputs more than another 5% from the existing labour force which is now at its
maximum. Data in quarter 2 for its four products were:

E M Reddy Page | 255


AMA-Notes

P Q R S
Output (kg) 4,560 6,960 3,480 2,300
Selling price (Rs. per kg) 16.20 11.64 9.92 13.68
Costs (Rs. Per kg)
Direct Labour (at Rs6 per hour) 1.96 1.30 0.99 1.70
Direct Materials 6.52 4.90 4.10 5.42
Direct packaging 0.84 0.74 0.56 0.70
Fixed overhead (absorbed on basis of direct labour cost) 3.92 2.60 1.98 3.40
Total 13.24 9.54 7.63 11.22
The XY company has offered to supply 2,000 kg of any of the product at a delivered price of 90% of
EF’s selling price. The company will then be able to produce extra another product in its place up
to the plant’s total capacity.
Required to state, with supporting calculations:
Which product should be purchased and which other product should be produced in it place up to
the plant’s total capacity so that the company reports the maximum profit? Assume XY’s quality
and delivery are acceptable.
Solution:

Step 1: Identification of Limiting Factor


Product No. of Kgs Hours per Kg Hours
P 4,560 1.96/6 = 0.3267 1,489
Q 6,960 1.30/6 = 0.2167 1,508
R 3,480 0.99/6 = 0.1650 574
S 2,300 1.706 = 0.2833 651
Total Hours 4,223
Add: Overtime 5% (4,233 x 5%) 212
Maximum Hours 4,435

Step 2: Contribution per limiting factor (or) Contribution per hour


Product Selling Variable Contribution Hours Contribution per Rank Significance
Price Cost per Unit hour
P 16.2 9.32 6.88 0.3267 21.05(6.88/0.3267) III
Q 11.64 6.94 4.7 0.2167 21.69(4.7/0.2167) II Manufacture
R 9.92 5.65 4.27 0.1650 25.87(4.27/0.1650) I during the extra
S 13.68 7.82 5.86 0.2833 20.68(5.86/0.2833) IV 5% capacity

Step 3: Calculation of capacity release if 2,000 Kgs are bought


Product Hours Released Remark
P 2,000 x 0.3267 = 653.4 Hours Company will not buy
Q 2,000 x 0.2167 = 433.4 Hours all only one of them
R 2,000 x 0.1650 = 330 Hours will be purchased from
S 2,000 x 0.2833 = 566.6 Hours outside

Step 4: Make (or) Buy table

E M Reddy Page | 256


AMA-Notes

Product Supply price (90% of Variable Extra cost of Extra cost per Rank
SP) Cost Buy hour
(a) (b) (c) = (a) – (b) (d)
P 14.58 9.32 5.26 5.26/0.3267 = 16.10 2
Q 10.47 6.94 3.53 3.53/0.2167 = 16.28 3
R 8.93 5.65 3.28 3.28/0.165 = 19.87 4
S 12.31 7.82 4.49 4.49/0.2833 = 15.84 1

Step 5: Selection of the product to be purchased from outside


Product Hours Contribution per Contribution Incremental Incremental Rank
hour from R per hour lost Contribution Contribution
(a) (b) (c) (d) = (b) – (c) (e) = (a) x (d)
P 653.4 25.87 16.10 9.77 6,384 1
Q 433.4 25.87 16.28 9.59 4,156 3
S 566.6 25.87 15.84 10.03 5,683 2

Step 6: Quantity allocation for Buy & Manufacturing


Product Total Allocation
Quantity Manufacture Buy
P 4,560 2,560 2,000
Q 6,960 6,960 -
Old 574 Hours
5% 212 Hours 1,439 Hours
R (0.165) Released 653 Hours
8,721 8,721 -
S 2,300 2,300 -

Step 7: Calculation of profit as per the new allocation and identifying increase in profit
Product Present New
Units SP VC Cn. Total Cn. Units SP VC Cn. Total Cn.
P 4,560 16.2 9.32 6.88 31,373 2,560 (mfg.) 16.2 9.32 6.88 17,613
2,000 (buy) 16.2 14.58 1.62 3,240
Q 6,960 11.64 6.94 4.70 32,712 6,960 11.64 6.94 4.70 32,712
R 3,480 9.92 5.65 4.27 14,860 8,721 9.92 5.65 4.27 37,238
S 2,300 13.68 7.82 5.86 13,478 2,300 13.68 7.82 5.86 13,478
Total Contribution 92,423 Total Contribution 1,04,281
Less: Fixed Cost (WN – 1) 50,681 Less: Fixed Cost (WN – 1) 50,681
Profit 41,742 Profit 53,600

Due to our recommendation the profit has increased by Rs.11,858 (Rs.53,600 – Rs.41,742).

WN – 1:

Fixed Cost = (3.92 x 4,560) + (2.6 x 6,960) + (1.98 x 3,480) + (3.4 x 2,300) = 50,681

Notes:

E M Reddy Page | 257


AMA-Notes

1) From Step 4 it could be observed that it is cheaper to release a hour by purchasing product ‘S’ from
outside because the extra buying cost is only Rs.15.84 per hour.
2) In step 5 also it could be seen that product ‘S’ gives highest contribution per hour of Rs.10.03 but still
we decided to buy 2,000 Kgs of product ‘P’ because it may have few paisa extra cost per hour but it
releases more hours enabling us to produce more product ‘R’.

7.5.3. Multiple Limiting factors in Make or Buy Situation

Question no 16: A construction company has accepted a contract to lay underground pipe work.
The contract requires than 2500m of 10 inch pipe and 2000m 28 inch pipe be laid each week.
The limiting factor is the availability of specialized equipment. The company owns 15 excavating
machine (type A) and 13 lifting and joining machines (type B). The normal operating time is 40
hours a week but up to 50% overtime is acceptable to the employees.
The time taken to handle each meter of pipe is:
Size of Pipe Minutes per meter
Machine A Machine B
10 Inches 6 12
18 Inches 18 12
The cost of operating the machines are:
Machine A (Rs.) Machine B (Rs.)
Fixed Costs, per week, each 450 160
Labour, per crew, per hour:
Up to 40 hours per week 10 12
Over 40 hours per week 15 18
The cost of materials are supplies per meter are:
10 Inches Rs.10
18 Inches Rs.5
A subcontractor has offered to lay any quantity of the 10 inches’ pipe at Rs.18 per meter and of the
18 inches’ pipe atRs.12 per meter.
You are required to calculate:
(a) Calculate the most economical way of undertaking the contract;
(b) State they weekly cost involved in your solution to (a) above.

Solution:

Step 1: Identification of limiting factor


Pipe Quantity Minutes Hours
Size required per A B A B
week
10 2,500 15,000 (2,500 x 6) 30,000 (2,500 x 12) 250 500
Inches
18 2,000 36,000 (2,000 x 18) 24,000 (2,000 x 12) 600 1,000
Inches
Total Hours Required 850 1,500
Hours Available:
No. of machines 15 13

E M Reddy Page | 258


AMA-Notes

No. of hours per week 40 40


No. of machine hours available per week 600 520
It is a limiting It is a limiting
factor factor
Note: Overtime is not considered, will be considered later.

It is not possible to lay on our own the entire requirement because limiting factor exists. Some portion has
to be necessarily sub-contracted. The ranking will be given based on the make (or) buy table as below:

Step 2: Own Work (vs.) Sub-contract


Pipe Size Per unit VC SC Cn. Per Hour Rank
Material Labour Cn. Per limiting factor
A B Total A B A B
10 Inches 10 1 2.4 3.4 13.4 18 4.6 46 (4.6/6 x 60) 23 (4.6/12 x 60) 1 1
18 Inches 5 3 2.4 5.4 10.4 12 1.6 5.34 (1.6/18 x 60) 8 (1.6/12 x 60) 2 2

Both Machine ‘A’ and Machine ‘B ranks the 10 inches’ pipe as the first rank product. Hence, there is a
consistency in ranking. If there is a conflict in ranking between liming factors, then solve using linear
programming technique.

Step 3: Allocation of Normal Machine ‘A’ and Machine ‘B’ hours


Product Units Hours’ time Allocation of Hours Units Mfg.
Required requires
A B A (600) B (520)
10 inches’ 2,500 250 500 250 500 2,500 Units
pipe
18 inches’ 2,000 600 400 100 Units x 18/60 20 Hours 20 Hours x 60/12 =
pipe = 30 Hours 100 Units
520(Max)

Balance hours of A: 600 – 250 – 30 = 320 Hours

Step 4: Viability of normal Machine ‘A’ and overtime Machine ‘B’ against sub-contracting

Subcontract charges = Rs.12


Own work:
Material = Rs.5
Direct Labour – A (Rs.10 x 18/60) = Rs.3
Direct Labour – B (Rs.18 x 12/60) = Rs.3.60
Own work with OT B = Rs.11.60

It is better to lay on our own with overtime ‘B’ because it is Rs.0.40 cheaper than sub-contracting option.

Step 5: Allocation of normal ‘A’ and OT ‘B’ hours – Units possible


Machine Available Hours Machine Hours per unit Units Possible
A 320 18/60 320 x 60/18 = 1,067

E M Reddy Page | 259


AMA-Notes

B 260 (Overtime 50% of 520) 12/60 260 x 60/12 = 1,300


Only 1,067 can be produced

At the end of this stage the entire normal machine ‘A’ hours are used. The next step is to check the viability
of OT ‘A’ and OT ‘B’ against sub-contracting.

Step 6: Viability of OT ‘A’ and OT ‘B’

Subcontract charges = Rs.12


Own work:
Material = Rs.5.00
Direct Labour – A (Rs.15 x 18/60) = Rs.4.50
Direct Labour – B (Rs.18 x 12/60) = Rs.3.60
Own work with OT ‘A’ & ‘B’ = Rs.13.10

In this case it is better to contract rather than do on our own using OT ‘A’ and OT ‘B’.

Alternatively,

Advantage of own work (Normal ‘A’ & OT ‘B’) = Rs.0.4


(-) Over time ‘A’ premium [(Rs.15 – Rs.10) x 18/60] = (Rs.1.5)
Extra Advantage = (Rs.1.10)

Step 7: Allocation of products


Product Required Own Sub-contracting
Normal of A & B OT of B & Normal of A
10 Inches pipe 2,500 2,500 - -
18 Inches pipe 2,000 100 1,067 833 (Balance)

Step 8: Weekly Cost statement


Product Units Variable Cost (Rs.) Total Cost (Rs.)
10 Inches Pipe – Normal 2,500 Units 13.4 33,500
18 Inches Pipe – Normal 100 Units 10.4 1,040
18 Inches Pipe – OT ‘B’ 1,067 Units 11.6 12,377
18 Inches Pipe – Sub-contracting 833 Units 12.0 9,996
Total Variable Cost 56,913

Fixed Cost:
Machine Nos. Fixed Cost(Rs.) Total Cost (Rs.)
A 15 450 6,750
B 13 160 2,080
Total Fixed Cost 8,830

Total Weekly Cost = Variable Cost + Fixed Cost = Rs.56,913 + Rs.8,830 = Rs.65,743

E M Reddy Page | 260


AMA-Notes

Question no 17: A car manufacturing company needs four components W, X , Y, Z. The


manufacturing components may be procured from outside. The cost, purchase price for the
component and other information are given below:
W (Rs.) X (Rs.) Y (Rs.) Z (Rs.)
Direct Material 60 70 75 60
Direct Wages 30 40 60 40
Direct Expenses @ Rs.20 per MH 40 30 40 40
Fixed Cost 20 20 15 25
150 160 190 165
Purchase price from market 250 160 200 135
Units required for the year 3,000 3,500 2,000 3,000
(i) There are constrains in machine time is manufacturing all components. Total machine hours
available are only 12,000.
(ii) Other alternative is to use machine time in a second shift which will attract 20% extra wages
and other fixed overheads @ Rs.3,000 for 1,000 hours or part thereof.
Give your suggestion about to course of action for maximization of profit.

Solution:

Step 1: Identification of Limiting factor


Product Hours/Unit Units Hours
W 2 3,000 6,000
X 1.5 3,500 5,250
Y 2 2,000 4,000
Z 2 3,000 6,000
Total Requirement 21,250
Hours Available 12,000

If we compare the manufacturing cost & buying cost for product ‘Z’ is alone, buying is cheaper. Hence ‘Z’
should be compulsorily purchased and not manufactured.

6,000 Hours of ‘Z’ will not be counted for limiting factor. The hour requirement is only 15,250 Hours
(21,250 – 6,000). Still it is a limiting factor.

Step 2: Make (or) Buy Table


Product Buying Variable Contribution Machine Contribution per Rank
Cost (Rs.) Cost (Rs.) (Rs.) Hours hour (Rs.)
W 150 130 20 2 10 3
X 160 140 20 1.5 13.33 1
Y 200 175 25 2 12.5 2

Manufacture ‘X’ and ‘Y’. Manufacture ‘W’ to the extent possible and meet the remaining demand second
shift (or) outside purchase.

Step 3: Allocation of Machine Hours


Product Units Hours/Unit Hours ∑ 𝐇𝐨𝐮𝐫𝐬

E M Reddy Page | 261


AMA-Notes

X 3,500 1.5 5,250 5,250


Y 2,000 2 4,000 9,250
W 1,375 2 2,750 12,000

W (Second Shift /Purchase) = 1,625 units (3,000 Units – 1,375 Units)

Step 4: Second shift (vs.) Buy


Particulars Computation Amount (Rs.)
Variable cost/Unit – Second shift Rs.60 + Rs.30 x 120% + Rs.40 136
Variable cost 1,625 Units x Rs.136 2,21,000
Additional fixed cost for 3,250 Hours (1,625 x 2) 4 thousand’s Hours x Rs.3,000 12,000
Total Manufacturing cost in second shift 2,23,000
Buying cost 1,625 Units x Rs.150 2,43,750

Recommended to go for second shift rather than buying.

Additional improvement to the solution:

We can manufacture 1,500 units in second shift and buy the balance 125 units from outside.

Variable Manufacturing cost (1,500 Units x Rs.136) = Rs.2,04,000


Additional Fixed Cost (3 thousand’s hours x Rs.3,000)= Rs.9,000
Buying cost (125 Units x Rs.150) = Rs.18,750
Total Cost = Rs.2,31,750

This is better than manufacturing the entire 1,065 units in 2nd shift as it saves a cost of Rs.1,250 (Rs.2,23,000
– Rs.2,31,750).

Alternatively,

1) Benefit → Savings in fixed cost by not producing 125 units = Rs.3,000


2) Cost → Additional variable cost due to buying 125 units = 125 Units x (Rs.150 – Rs.136) = Rs.1,750
Net Saving = Rs.3,000 – Rs.1,750 = Rs.1,250

Indifference point:
Particulars Manufacture (Rs.) Buy (Rs.)
Variable Cost 136 150
Fixed Cost 3,000 -
Difference in Fixed Cost 3,000−0
Indifference point = Difference in variable cost per unit = 150−136 = 214 Units (or) 428 Hours

It will be worth to commit Rs.3,000 fixed cost only when we have work load of at least 428 hours (or) 214
units. Since, we had only 125 Units workflow (or) 250 hours work load we deiced to buy it.

7.5.4. Limiting factor with Specific Fixed Cost

Question no 18: Bloom ltd. makes 3 products A, B and C. The following information is available:

E M Reddy Page | 262


AMA-Notes

Particulars Figures in Rs. /Units


A B C
Selling price (peak season) 550 630 690
Selling price (off season) 550 604 690
Material Cost 230 260 290
Labour (peak season) 110 120 150
Labour (off season) 100 99 149
Variable production overhead 100 120 130
Variable selling overhead (only for peak season) 10 20 15
Labour hours required for one unit of production 8 11 7
Material cost and variable production overheads are the same for the peak season and off season.
Variable selling overheads are not incurred in the off season. Fixed costs amount to Rs.26,780 for
each season of which Rs.2,000 is towards salary for special technician incurred only for product ,
and Rs.4,780 is the amount that will be incurred on after sales warranty and free maintenance of
only product C to match competition.
Labour force can be interchangeably used for all the product. During peak season there is labour
shortage and the maximum labour hours available are 1,617 hours. During off season labour is
freely available but demand is limited to 100 units of A, 115 units of B and 135 units of C with
production facility being limited to 215 units of A, B and C put together.
Required:
(i) Advise the company about the best production mix during peak season for maximum profit.
(ii) What will be the maximum profit for the off season?

Solution:

Step 1: Peak Season ranking of products


Items A (Rs.) B (Rs.) C (Rs.)
Selling Price 550 630 690
Less: Variable Cost
Direct Materials 230 260 290
Direct Labour 110 120 150
Variable overhead – Production 100 120 130
Variable overhead - Selling 10 20 15
Total Variable Cost 450 520 585
Contribution per unit 100 110 105
Hours per unit 8 11 7
Contribution per hour 12.5 10 15
Rank 2 3 1

Step 2: Analysis of fixed cost

Total Fixed Cost = Rs.26,780


Specific fixed cost of ‘B’ = Rs.2,000
Specific fixed cost of ‘C' = Rs.4,780
General fixed cost = Rs.20,000

Step 3: Allocation of 1,617 labour hours

E M Reddy Page | 263


AMA-Notes

1) Allocating it to product ‘C’


Contribution (1,617 Hours x Rs.15) = Rs.24,255
Less: Specific fixed cost = (Rs.24,780)
Profit/ (Loss) = (Rs.525)
2) Allocating the hours to product ‘A’
Contribution (1,617 Hours x Rs.12.5) = Rs.20,213
Less: Specific fixed cost = (Rs.20,000)
Profit/ (Loss) = Rs.213
3) Allocating it to product ‘B’
Contribution (1,617 Hours x Rs.10) = Rs.16,170
Less: Specific fixed cost = (Rs.22,000)
Profit/ (Loss) = (Rs.5,830)

It is recommended to allocated the 1,617 hours to product ‘A’ because it gives the highest profit.

Notes:

1) Allocating the 1,617 hours to product ‘C’ (1st rank product) gives us the maximum contribution.
2) When contribution is maximized the profit also will be maximized. That is why we allocate limiting
factor generally to high contribution products.
3) However, when there exists specific fixed cost, it is not necessary that high contribution gives high
profit. In this problem product ‘C’ gives high contribution but due to it’s specific fixed cost it results in
loss.
4) When we have specific fixed cost the limiting factor should be allocated to maximize profit and not
contribution.

Alternatively, the solution can be presented as follows:


Items Unties Possible Required to Break-even Decision
C 1,617/7 = 231 Units 24,780/105 = 236 Units Do not produce
A 1,617/8 = 202 Units 20,000/100 = 200 Units Produce
B 1,617/11 = 142 Units 22,000/110 = 200 Units Do not produce

1,617 hours should be allocated to product ‘A’ because product ‘B’ and product ‘C’ does not break-even.

Step 4: Off season ranking the products


Items A (Rs.) B (Rs.) C (Rs.)
Selling Price 550 604 690
Less: Variable Cost
Direct Materials 230 260 290
Direct Labour 100 99 149
Variable overhead – Production 100 120 130
Total Variable Cost 430 479 569
Contribution per unit 120 125 121
Rank 3 1 2
Max demand (Units) 100 115 135
Overall limit (Units) 215 Units

E M Reddy Page | 264


AMA-Notes

Step 5: Profitability under different options


Items A B C Total Fixed Cost Profit/(Loss)
Contribution per unit 120 125 121
Option 1:
Units - 115 100 215
Amount - 14,375 12,100 26,475 26,780 (305)
Option 2:
Units 100 115 - 215
Amount 12,000 14,375 26,375 22,000 22,000 4,375
Option 3:
Units 80 - 135 215
Amount 9,600 - 16,335 25,935 24,780 1,155

Select option since the profit is highest.

Question no 19: Question no 27

Solution:

Step 1: Calculation of net contribution required to earn profit of Rs.1,800

Profit = Rs.1,800
General Fixed Cost = Rs.4,000
Net contribution required = Rs.5,800

Step 2: Calculation of net contribution given by 50,000 leaflets of A & B


Products Units Contribution/1000 Contribution Specific Fixed Net
(‘000s) Cost Contribution
A 50 60 3,000 2,400 600
B 50 150 7,500 4,000 3,500
Total Net Contribution 4,100

Step 3: Number of units ‘C’ required to earn a profit of Rs.1,800

Additional net contribution required = Rs.5800 – Rs.4,100 = Rs.1,700


Add: Specific Fixed Cost = Rs.9,500
Contribution required from C = Rs.11,200
Contribution/1000 leaflets of C = Rs.320
Rs.11,200
No.of units of C = = 35,000 leaflets
Rs.320

We required to print 35,000 leaflets of C to earn a profit of Rs.1,800.

Notes:

1) When we have specific fixed cost we should understand that there exists two levels of contribution.

E M Reddy Page | 265


AMA-Notes

i. Gross Contribution → Contributes towards (a) Specific Fixed Cost (b) General Fixed Cost and
then (c) Profit
ii. Net Contribution → Gross Contribution – Specific Fixed Cost → It contributes to the common
pool towards general fixed cost recovery and profit.

Step 4: Calculation of contribution per pack and ranking the leaflets


Product Contribution/1,000 Packs per 1,000 leaflets Contribution /pack Rank
A 60 2 30 1
B 150 6 25 2
C 320 16 20 3

Step 5: Allocation of 170 packs


Product Qty.in (‘000s) No. of packs/1,000 Requirement ∑ 𝐑𝐞𝐪𝐮𝐢𝐫𝐞𝐦𝐞𝐧𝐭
A 10 2 20 20
B 10 6 60 80
C 5 16 90 160

After this allocation there remains 10 unutilized packs and unsatisfied demand of ‘C’ to the extent of 5,000.

Step 6: Alternative Allocation

We can try compromising the limiting factor ranking in order to use full capacity. We can sacrifice 1,000
leaflets of ‘B’ and release 6 packs which along with the 10 idle packs can be used to produce 1 more 1,000
leaflets of ‘C’.
Alternative 1 Alternative 2
Product Contribution/1,000 Numbers Gross Contribution Numbers Gross Contribution
A 60 10 600 10 600
B 150 10 1,500 9 1,350
C 320 5 1,600 6 1,920
3,700 3,870

Since 2nd alternative gives Rs.170 extra contribution it should be preferred.

Alternatively,

Benefit: Contribution from ‘C’ = Rs.320


Cost: Contribution lost from ‘B’ =Rs.150
Net Benefit = Rs.320 – Rs.150 = Rs.170

Notes:

1) In the previous sum it was observed that when specific fixed cost exists our aim is not to earn more
contribution but to earn more profit.
2) In limiting factor allocation specific fixed costs is also considered.
3) In this sum while allocating 170 packs we have ignored the specific fixed costs and try to maximize
contribution. Why?

E M Reddy Page | 266


AMA-Notes

Answer: Specific fixed costs is irrelevant because we have already committed to produce 50,000 leaflets
of each product. Specific fixed cost becomes relevant only when we have an option to abandon a
product line.

Step 7: Commenting on profitability of the products as stand alone


Particulars A B C
Selling Price 100 220 450
Less: Variable Cost 40 70 130
Contribution 60 150 220
Fixed Cost
Specific 2,400 4,000 9,500
General 4,000 4,000 4,000
Total Fixed Cost 6,400 8,000 13,500
BEP 6,400/60 = 107 leaflets 8,000/150 = 54 leaflets 13,500/220 = 43 leaflets
Maximum Demand 60 leaflets 60 leaflets 60 leaflets

Summary table:
Product BEP (Standalone) Rank (in standalone status)
A 107 Not Possible
B 54 2
C 43 1

Notes:

1) Standalone product means when the demand for other products becomes ‘0’ can this product survive?
2) When a product becomes standalone it should recover it’s specific fixed cost and entire general fixed
cost to break even.
3) ‘A’ can never be a standalone product because even at it’s maximum demand it cannot break-even. It
can only co-exists with ‘B’ & ‘C’.
4) ‘C’ is a better standalone product because it’s break-even sooner and has high margin of safety.

Step 8: Commenting on the profitability of the products as member of the group


Product Specific Fixed Cost Contribution/1000 leaflets BEP (‘000 Ranking
(Rs.) (Rs.) leaflets)
A 2,400 60 40 III
B 4,000 150 27 I
C 9,500 320 30 II

Notes:

1) Product ‘B’ breaks-even sooner than minimum demand and thus surely contributes towards the general
fixed cost. It is an asset to the group.
2) ‘C’ breaks-even exactly at minimum demand. It will surely recover it’s specific fixed cost and hence it
will never be liability to the group.
3) ‘A’ should be printed only when we are certain that the demand will be more than 40,000 leaflets.
Otherwise ‘B’ & ‘C’ will be made to recover ‘A’s fixed cost also.

E M Reddy Page | 267


AMA-Notes

7.6. Shut down (or) Continue decisions

1) During the periods of lean demand management may consider temporary shutdown of operations. Such
a decision is called “Shutdown Decision”.
2) The shutdown decision should be made after doing cost – benefit analysis.
3) Shutdown Decision
i. Benefit → Avoidable Fixed Cost
ii. Cost → Shut Down cost & Contribution lost on possible shut down period demand
4) Avoidable fixed cost are those fixed costs that can be avoided when the plant is shut down and
shutdown costs are costs specifically incurred during the shutdown period to maintain the factory in
working condition.
5) If the benefit exceeds cost shutdown else continue.
6) The shutdown decision greatly depends on the possible shutdown period. If the shutdown period
demand is high continue, if it is low shutdown.
7) The volume at which the company is indifferent between shutting down or continuing is called ‘Shut
Down Point’. It can be calculated as follows:
Avoidable Fixed Cost−Shutdown Cost
i. In Units → Contribution per unit
Avoidable Fixed Cost−Shutdown Cost
ii. In Value → PV Ratio

Question no 20:
3(a) A firm incurs a fixed cost of Rs.1,20,000 at 60% capacity. AT 60% capacity, fixed cost is only
Rs.40,000. If its VC Ratio is 80%, find out the shutdown point.
3(b) A paint manufacturing company manufactures 2,00,000 per annum medium – sized tins of
“Spray Lac Paint” when working at normal capacity. It incurs the following costs of manufacturing
per unit:

Solution:

a) Total Fixed Cost = Rs.1,20,000


Less: Unavoidable Fixed Cost = (Rs.40,000)
Avoidable Fixed Cost = Rs.80,000
PV Ratio = 100% - Variable Cost Ratio = 100% - 80% = 20%
Avoidable Fixed Cost−Shutdown Cost 1,20,000−40,000
Shut down point = = = Rs.4,00,000
PV Ratio 20%

Conclusion:
Sales Range Decision
Less than Rs.4,00,000 Shutdown
At Rs.4,00,000 Shutdown (or) Continue
Greater than Rs.4,00,000 Continue

b) Step 1: Contribution per unit


Selling Price Rs.21
Less: Variable Cost
Direct Materials Rs.7.8

E M Reddy Page | 268


AMA-Notes

Direct Labour Rs.2.1


Variable Overhead Rs.2.5
Variable selling & administration cost Rs.0.6 Rs.13
Contribution per unit R.s.8

Step 2: Avoidable fixed cost


Fixed Cost per annum = Normal Capacity x Standard Rate
= 2,00,000 x Rs.4 = Rs.8,00,000
Fixed Cost per quarter = Rs.8,00,000 x 3/12 = Rs.2,00,000
Unavoidable fixed cost = Rs.74,000
Avoidable fixed cost = Rs.2,00,000 – Rs.74,000 = Rs.1,26,000

Step 3: Shutdown Cost


Shutdown cost (Given) = Rs.14,000

Step 4: Shutdown Point

Avoidable Fixed Cost−Shutdown Cost


Shutdown Point = Contribution per unit
Rs.1,26,000−Rs.14,000
= = 14,000 Units
Rs.8
Step 5: Conclusion
Shutdown period Demand Decision
Less than 14,000 Units Shutdown
At 14,000 Units Shutdown (or) Continue
Greater than 14,000 Units Continue

Since the marketing team believes that in next quarter only 10,000 units could be sold it is
recommended that plant should be shut down.

Question no 21: Question no 2

Solution:

a) PV Ratio (or) Contribution to Sales Ratio:


Change in Profit 4,000−2,000
PV Ratio = = 30,000−20,000 = 20%
Change in Sales
Change in Cost 26,000−18,000
Variable cost Ratio = Change in Sales = 30,000−20,000 = 80%
b) Breakeven Sales:

Particulars 2002 2003


Contribution (Sales x PV Ratio) 4,000 (20,000 x 20%) 3,000 (30,000 x 20%)
Less: Profit (20000) (4000)
Fixed Cost 2,000 2,000
Fixed Cost Rs.2,000
Breakeven Sales = PV Ratio = 20%
= Rs.10,000
c) Sales to earn Rs.6,000 profit

E M Reddy Page | 269


AMA-Notes

Profit = Rs.6,000
Add: Fixed Cost = Rs.2,000
Contribution = Rs.8,000
Contribution Rs.8,000
Sales = = = Rs.40,000
PV Ratio 20%
Alternatively,
Profit Rs.6,000
Margin of Safety = PV Ratio = 20% = Rs.30,000
Sales = Margin of Safety + Breakeven Sales = Rs.30,000 + Rs.10,000 = Rs.40,000
d) Profit when the sales is Rs.12,000
Contribution (Sales x PV Ratio) = Rs.12,000 x 20% = Rs.2,400
Less: Fixed Cost = Rs.2,000
Profit = Rs.400
Alternatively,
Margin of Safety = Sales – Breakeven Sales = Rs.12,000 – Rs.10,000 = Rs.2,000
Profit = Margin of Safety x PV Ratio = Rs.2,000 x 20% = Rs.400

7.7. Differential Costing

Question no 22: Question no 18

Solution:

Part A: Determination of optimum output:


Sales Selling price/Unit (Rs.) Sales Incremental Revenue Incremental Cost Net
60,000 0.9 54,000 - - -
70,000 0.8 56,000 2,000 1,500 (10,000 x 15) 500
80,000 0.75 60,000 4,000 1,500 (10,000 x 15) 2,500
90,000 0.67 60,300 300 1,500 (10,000 x 15) (1,200)
1,00,000 0.61 61,000 700 1,500 (10,000 x 15) (800)

Notes:

1) Cost accountant assumes linearity in sales and cost function.


2) When linearity is assumed it means selling price is constant, variable cost per unit is constant and fixed
cost in total is constant at all output levels.
3) In case of linearity the optimum output is the maximum output.
4) In this problem the linearity is assumption is broken due to price elasticity of demand.
5) Advantage of low volume is higher selling price and high volume is more units. Optimum output is the
output where the profit is highest. It need not be maximum output.
6) In such scenario compare marginal revenue with marginal cost (Incremental revenue with incremental
cost/Differential revenue with differential cost). Keep increasing the output as long as the marginal
revenue exceeds the marginal cost. In this problem the optimum output level is 80,000 units beyond
which the marginal cost exceeds marginal revenue.

Part B: Acceptance of export order

E M Reddy Page | 270


AMA-Notes

Selling price = Rs.0.50


Less: Variable cost = Rs.0.15
Contribution per unit = Rs.0.35
Contribution from the order = 20,000 Units x Rs.0.35 = Rs.7,000

Instead of keeping the capacity idle it is better to earn Rs.7,000 contribution.

Notes:

1) We had rejected a selling price of Rs.0.61. Then how come we accept a selling price of Rs.0.50?
Answer: It is improper to compare the two selling prices. The selling price of Rs.0.61 is for cumulative
1,00,000 units but the selling price of Rs.0.50 is only for incremental 20,000 units i.e. we can sell locally
80,000 units at Rs.0.75 (Optimum output) and in addition sell 20,000 units at Rs.0.50

Question no 23: Question no 19

Solution:
Units Additional Cost per Total Cost Incremental Cost Incremental cost per
Units unit (Rs.) (Rs.) (Rs.) unit (Rs.)
70,000 - 97 67,90,000 - -
80,000 10,000 92 73,90,000 5,70,000 57
90,000 10,000 87 78,30,000 4,70,000 47
1,00,000 10,000 82 82,00,000 3,70,000 37

The orders will be independently rejected because the selling price is less than incremental cost per unit.
May be accepting all the three together will push the factory to different capacity level which may be
feasible.
Order Units Operating Capacity Incremental Cost Offer price Conclusion
A 5,000 70% - 80% 57 55 Reject
B 10,000 70% - 80% 57 52 Reject
C 10,000 70% - 80% 57 51 Reject
All 25,000 90% - 100% 12,25,000 (WN – 1) 13,05,000 (WN – 2) Accept

WN – 1: 5,70,000 + 4,70,000 + ½ of 3,70,000 = 12,25,000


WN – 2: (5,000 x 55) + (10,000 x 52) + (10,000 x 51) = 13,05,000

Question no 24: Question no 32

Solution:

Step 1: Calculation of contribution per unit


Particulars Area 1 (Rs.) Area 2 (Rs.) Area 3 (Rs.) Area 4 (Rs.) Area 5 (Rs.)
Selling Price 100 100 100 100 100
Less: Production Cost 80 80 80 80 80
Less: Distribution cost 10 8 6 4 2
Contribution per unit 10 12 14 16 18

E M Reddy Page | 271


AMA-Notes

Step 2: Analysis of 2001 profits


Area Units Contribution per unit (Rs.) Contribution (Rs.)
1 6,500 10 65,000
2 6,500 12 78,000
3 6,500 14 91,000
4 6,500 16 1,04,000
5 6,500 18 1,17,000
Total Contribution 4,55,000
Less: Fixed Cost (Sales Man Salary) (35 Men x Rs.8,000) 2,80,00
Profit 1,70,000

Step 3: Analysis of 2002 profits


Area Units Contribution per unit (Rs.) Contribution (Rs.)
1 5,000 10 50,000
2 5,000 12 60,000
3 5,000 14 70,000
4 7,800 16 1,24,800
5 7,800 18 1,40,400
Total Contribution 4,45,200
Less: Fixed Cost (Sales Man Salary) (35 Men x Rs.8,000) 2,80,00
Profit 1,65,200

Notes:

1) Instead of giving equal sales man to all five areas the company gave minimum penetration of 5 sales
men to all 5 areas and the balance 10 they gave to two high contributing areas ‘A4’ and ‘A5’. Instead of
improving the profit the strategy resulted in drop in profits. Why?
2) The volume achieved with 7 sales in each men area was 32,500 units but in the new strategy in 2002 the
volume dropped to 30,600 units resulting in 1,900 units drop.
3) Cost-Benefit analysis of this strategy as follows:
i. Benefit
A4 – 1,300 Units x Rs.16
A5 – 1,300 Units x Rs.16
Contribution gained = Rs.44,200
ii. Cost
A1 – 1,500 Units x Rs.10
A2 – 1,500 Units x Rs.12
A3 – 1,500 Units x Rs.14
Contribution lost = Rs.54,000
4) The volume lost from A1, A2, A3 is more than the volume gained in A4 and A5. The sales force is not
reduced but only transferred. Then why there is a volume drops?
5) The volume drops because the sales man and penetration relationship is non-linear which means when
more sales men is allocated to an area the volume increases but at decreasing rate.

Step 4: Calculation of contribution and marginal contribution for all this areas at different levels

E M Reddy Page | 272


AMA-Notes

Men Units A1 A2 A3 A4 A5
Cn MCn Cn MCn Cn MCn Cn MCn Cn MCn
5 5,000 50,000 - 60,000 - 70,000 - 80,000 - 90,000 -
6 5,800 58,000 8,000 69,600 9,600 81,200 11,200 92,800 12,800 1,04,400 14,400
7 6,500 65,000 7,000 78,000 8,400 91,000 9,800 1,04,000 11,200 1,17,000 12,600
8 7,100 71,000 6,000 85,200 7,200 99,400 8,400 1,13,600 9,600 1,27,800 10,800
9 7,600 76,000 5,000 91,200 6,000 1,06,400 7,000 1,21,600 8,000 1,36,800 9,000
10 7,800 78,000 2,000 93,600 2,400 1,09,200 2,800 1,24,800 3,200 1,40,400 3,600
11 8,000 80,000 2,000 96,000 2,400 1,12,000 2,800 1,28,000 3,200 1,44,000 3,600

Step 5: Allocation of the additional 10 sales men


Sales Men Area
26th Sales Men A5
27th Sales Men A4
28th Sales Men A5
29th Sales Men A3
30th Sales Men A4
31st Sales Men A5
32nd Sales Men A3
33rd Sales Men A2
34th Sales Men A4
35th Sales Men A5

Summary of Sales Men allocated to each area:


Area Sales Men Allocated
A1 5
A2 6
A3 7
A4 8
A5 9
Total 35

Step 6: Calculation of the maximum profit with 35 sales men


Area Units Contribution/Unit (Rs.) Contribution (Rs.)
A1 5,000 10 50,000
A2 5,800 12 69,600
A3 6,500 14 91,000
A4 7,100 16 1,13,600
A5 7,600 18 1,36,800
Total Contribution 4,61,000
Less: Fixed Cost 2,80,000
Profit 1,81,000

Notes: When we have asked to allocate the limiting factor for a non-linear data where the rankings
frequently change do the allocation stage by stage using tick (√) technique.

E M Reddy Page | 273


AMA-Notes

Question no 24: Question no 5, pricing chapter, page no.56

Solution:

Step 1: Calculation of Variable cost per unit

Fixed Cost = 200% of Variable Cost. Therefore, FC:VC = 2:1


Therefore Variable Cost = 1/3 of total cost
Product Variable cost per unit
A 6 (1/3 x 18)
B 8 (1/3 x 24)
C 10 (1/3 x 30)

Step 2: Calculation of Contribution and Marginal Contribution of product ‘A’


Units Contribution/Unit Contribution Incremental Contribution/2000 Hours
2,000 18.5 37,000 37,000
4,000 17.5 70,000 33,000
6,000 16.5 99,000 29,000
8,000 15.5 1,24,000 25,000
10,000 14.5 1,45,000 21,000
12,000 13.5 1,62,000 17,000
14,000 12.5 1,75,000 13,000

Step 3: Calculation of Contribution and Marginal Contribution of product ‘B’


Units Contribution/Unit Contribution Incremental Contribution/2000 Hours
2,000 26 52,000 52,000
4,000 25 1,00,000 48,000
6,000 24 1,44,000 44,000
8,000 23 1,84,000 40,000
10,000 22 2,20,000 36,000
12,000 21 2,52,000 32,000
14,000 20 2,80,000 28,000

Step 4: Calculation of Contribution and Marginal Contribution of product ‘C’


Units Contribution/Unit Contribution Incremental Incremental
Contribution/4000 Contribution/2000
Hours Hours
2,000 29 58,000 58,000 29,000
4,000 28 1,12,000 54,000 27,000
6,000 27 1,62,000 50,000 25,000
8,000 26 2,08,000 46,000 23,000
10,000 25 2,50,000 42,000 21,000
12,000 24 2,88,000 38,000 19,000
14,000 23 3,22,000 34,000 17,000

Step 5: Allocation of limiting factor

E M Reddy Page | 274


AMA-Notes

Hours A B C Select
2,000 37,000 52,000 29,000 B
4,000 37,000 48,000 29,000 B
6,000 37,000 44,000 29,000 B
8,000 37,000 40,000 29,000 B
10,000 37,000 36,000 29,000 A
12,000 33,000 36,000 29,000 B
13,500 33,000 32,000 29,000 A

Conclusion: The 13,500 hours should be allocated as follows:

A = 3,500 Hours → 3,500 Units → Selling price = Rs.23.50


B = 10,000 Hours → 10,000 Units → Selling Price = Rs.30.00

Step 6: Calculation of net profit for the above plan


Products Units Contribution per unit (Rs.) Contribution (Rs.)
A 3,500 17.5 61,250
B 10,000 22 2,20,000
Total Contribution 2,81,250
Less: Fixed Cost (WN) (1,83,000)
Profit 98,250

WN: Calculation of fixed cost

A: 2/3 x Rs.18 = Rs.12 x 6,000 Units = Rs.72,000


B: 2/3 x Rs.24 = Rs.16 x 6,000 Units = Rs.96,000
C: 2/3 x Rs.30 = Rs.20 x 750 Units = 15,000

Total Fixed Cost = Rs.72,000 + Rs.96,000 + Rs.15,000 = Rs.1,83,000

Question no 25: Question no 12, Relevant Costing, page no.56

Solution:

Step 1: Analysis of production is strike is allowed

Total Production Capacity = 27,500 Machines


- 46 Weeks – 25,300 Machines
- 4 Weeks – 2,200 Machines
- Demand lost due to competition – 650 Machines
- OT Production – 1,550 Machines

Step 2: Profitability when the strike is allowed and when not allowed
Particulars Not Allow Strike (‘000s) Allow Strike (‘000s)
Sales 66,000 (27,500 Machines x Rs.2,400) 64,440 (26,850 Machines x Rs.2,400)
(-) Wages 21,186 (WN – 1) 21,079.80 (WN – 1)
(-) Other Production Cost 29,700 (27,500 Machines x Rs.1,080) 28,998 (26,850 Machines x Rs.1,080)

E M Reddy Page | 275


AMA-Notes

(-) Distribution Cost 2,750 (27,500 Machines x Rs.100) 2,685 (26,850 Machines x Rs.100)
(-) Overhaul Expenses 100 25
(-) Additional Fixed Cost - 10
Profit 12,264 11,642.20

WN – 1: Wages Calculation

Variable Production Cost = Rs.1,800 per Machine


- Wages = Rs.1,800 x 40% = Rs.720 per Machine
- Other Production expenses = Rs.1,080 per Machine

Wages when strike is allowed = Rs.720 x 107% x27,500 Machines = Rs.21,186,000

Wages when the strike is allowed:

Normal time wages = Rs.720 x 105% x 25,300 Machines = Rs.19,126,800


Overtime Wages = Rs.720 x 105% x 150% x 1,550 Machines/90% = Rs.19,53,000
Total Wages = Rs.19,126,800 + Rs.19,53,000 = Rs.21,079,800

If we decide to allow the strike the profit is likely to drop by Rs.6,21,800 (Rs.1,22,64,000 – Rs.1,16,42,200).
Hence recommended not to allow strike.

Step 3: Analysis of production after strike takes place

Total Production Capacity = 27,500 Machines


- 47 Weeks – 25,850 Machines
- 3 Weeks – 1,650 Machines
- Demand lost due to competition – 650 Machines
- OT Production – 1,000 Machines

Step 4: Profitability after the strike takes place


Particulars Computation Amount (‘000s)
Sales 26,850 Machines x Rs.2,400 64,440
(-) Wages Normal Rs.720 x 106% x 25,850 Machines 19,728.72
(-) Wages Normal Rs.720 x 106% x 150%x 1,000 Machines/90% 1,272
(-) Other Production Cost 26,850 Machines x Rs.1,080 28,998
(-) Distribution Cost 26,850 Machines x Rs.100 2,685
(-) Overhaul Expenses 25
(-) Additional Fixed Cost 10
Profit 11,721.28

Due to allowing strike the profit lost by the management is Rs.5,42,720 (Rs.1,22,64,000 – Rs.1,17,21,280)

Step 5: Checking the viability of Overtime


Particulars Amount (Rs.)
Selling Price 2,400
(-) Other Production Cost 1,080

E M Reddy Page | 276


AMA-Notes

(-) Distribution Cost 100


Contribution excluding wage cost 1,220

Benefit: Contribution = 100 Machines x Rs.1,220 = Rs.12,20,000


Cost:
Wages = Rs.720 x 106% x 150 x 1,000 Machines/90% = Rs.12,72,000
Additional Fixed Cost = Rs.10,000

Net Benefit/(Cost) = Rs.12,20,000 – Rs.12,82,000 = (Rs.62,000)

It is not viable to work overtime because the cost exceeds benefit by Rs.62,000

Conclusion: The cost of strike earlier calculated of Rs.5,42,720 is wrong because included in it is Rs.62,000
lost due to the wrong decision of working overtime. The real cost of strike is Rs.4,80,720 (Rs.5,42,720 –
Rs.62,000)

Question no 26: Question no 29 (1:02:00)

Solution:

Step 1: Types of Cost

1) Material
i. Company to contractor 110% (100 + 10) → Company get 10%
ii. Contractor to customer 137.5% = 110 + (25% of 110) (Charge by contractor) → Contractor
gets 27.5%
2) Labour
i. Maintenance 90% of list price is contractor share → Company gets 10%
ii. Ad-hoc 85% of list price is contractor share → Company gets 15%

Step 2: Calculation of incremental income if the entire work is done on own


Particulars Income to Company Bill Income to Additional
company when get % Amount company when income (Rs.)
sub-contract (Rs.) own (Rs.)
(Rs.)
Labour – 30,000 10% 3,00,000 3,00,000 2,70,000
Maintenance
Labour – Ad- 12,000 15% 80,000 80,000 68,000
hoc
Material- 18,000 10%/137.5% 2,47,500 67,500 (2,47,500 x 49,500
Maintenance 37.5/137.5)
Material-Ad- 6,000 10%/137.5% 82,500 22,500 (82,500 x 16,500
hoc 37.5/137.5)
Total Additional Income 4,04,000

If the company does the entire work on its own it will earn an additional income of Rs.4,04,000 (without
considering specific fixed cost for different options).

E M Reddy Page | 277


AMA-Notes

Note: Labour cost is included in fixed cost. So, total bill amount is considered for calculating incremental
revenue.

Step 3: Evaluation of different options


Option 1 Option 2 Option 3
Own 40% 60% 100%
Contract 60% 40% -
Increase in income (Rs.4,04,000 x Own%) Rs.1,61,600 Rs.2,42,400 Rs.4,04,000
Cost (Includes labour cost) (Rs.1,48,000) (Rs.2,85,000) (Rs.3,85,000)
Net Benefit/(Cost) Rs.13,600 (Rs.42,600) Rs.19,000

It is recommended to choose option 3 i.e. fully do on our own because that gives us the maximum net
benefit.

E M Reddy Page | 278


AMA-Notes

8. SIMULATION

1) There are various models that supports decision making.


2) Most of the models has underlying assumptions and the model will work only when the underlying
assumptions are satisfied in real world situation.
3) Where no model can be applied in a decision making situation, then we should make decision based on
chance. The scientific way of making decision based on chance is called ‘Simulation’.
4) Simulation is a process through which we build a model and test it through random numbers to
understand how it works.
5) Random numbers represent the chance factor and is closely related to probabilities of events.
6) In any simulation problem there are two steps:
(i) Random Number Coding → Each event is coded with a range of Random Numbers based on
its probability.
(ii) Simulation Work Sheet (or) Fitting Random Numbers → The Random Numbers selected is
fitted to the code to identify the decision.
7) The above process is popularly called “Monti Carlo Simulation”.

Question no 1: The occurrence of rain in a day is dependent upon whether it rained in the previous
day. If it rained in the previous day, the rain distribution is given by:
Event Probability
No rain 0.50
1 cm 0.25
2 cm 0.15
3 cm 0.05
4 cm 0.03
5 cm 0.02
If it did not rain in the previous day, the rain distribution is given by:
Event Probability
No rain 0.75
1 cm 0.15
2 cm 0.06
3 cm 0.04
Simulate the city’s whether for 10 days and determine by simulation the total days without rain as
well as the total rainfall during the period. Use the following random numbers:
76 78 84 75 02 86 02 78 07 63
Assume that for the first day of simulation it has not rained on the previous day.

Solution:

Step 1: Random Number Coding

Table 1: Random Number Coding (Rained Previous Day)


Event Probability Cumulative Probability Random Number Interval
No rain 0.50 0.50 00 – 49
1 cm 0.25 0.75 50 – 74
2 cm 0.15 0.90 75 – 89

E M Reddy Page | 279


AMA-Notes

3 cm 0.05 0.95 90 – 94
4 cm 0.03 0.98 95 – 97
5 cm 0.02 1.00 98 – 99

Table 2: Random Number Coding (No Rain Previous Day)


Event Probability Cumulative Probability Random Number Interval
No rain 0.75 0.75 00 – 74
1 cm 0.15 0.90 74 - 89
2 cm 0.06 0.96 90 – 95
3 cm 0.04 1.00 96 – 99

Step 2: Fitting Random Numbers (or) Simulation Worksheet


Day Random Number Table Reference Rainfall (Cm)
1 76 Table 2 1 Cm
2 78 Table 1 2 Cm
3 84 Table 1 2 Cm
4 75 Table 1 2 Cm
5 02 Table 1 No rain
6 86 Table 2 1 Cm
7 02 Table 1 No rain
8 78 Table 2 1 Cm
9 07 Table 1 No rain
10 63 Table 2 No rain

Conclusion: Total days without rain are 4 days and total rain is 9 Cm.

Question no 2: Dr. Strong is a dentist who schedules all her patients for 30 minutes appointments.
Some of the patients take more or less than 30 minutes depending on the type of dental work to be
done. The following summary shows the various categories of the work, their probabilities and
time requires completing them:
Category Filling Crown Cleaning Extraction Check Up
Time Required (Minutes) 45 60 15 45 15
Probability 0.40 0.15 0.15 0.10 0.20
Simulate the dentist clinic for 4 hours and determine the average waiting for the patients as well as
idleness of the doctor. Arrival time of 1st patient is 8 AM. Random numbers are as follows:
40 82 11 34 25 66 17 79

Solution:

Step 1: Random Number Coding


Types Time (Minutes) Probability Cumulative Probability Random Number Interval
Filling 45 0.40 0.40 00 – 39
Crown 60 0.15 0.55 40 – 54
Cleaning 15 0.15 0.70 55 – 69
Extraction 45 0.10 0.80 70 – 79
Check Up 15 0.20 1.00 80 – 99

E M Reddy Page | 280


AMA-Notes

Step 2: Fitting Random Numbers (or) Simulation Worksheet


Simulation Random Type Time Arrival Waiting Entry Exit
Run Number Required Time Time Time Time
1 40 Crown 60 Minutes 8:00 0 8:00 9:00
2 82 Checkup 15 Minutes 8:30 30 Minutes 9:00 9:15
3 11 Filling 45 Minutes 9:00 15 Minutes 9:15 10:00
4 34 Filling 45 Minutes 9:30 30 Minutes 10:00 10:45
5 25 Filling 45 Minutes 10:00 45 Minutes 10:45 11:30
6 66 Cleaning 15 Minutes 10:30 60 Minutes 11:30 11:45
7 17 Filling 45 Minutes 11:00 45 Minutes 11:45 12:30
8 79 Extraction 45 Minutes 11:30 60 Minutes 12:30 01:15
Total Waiting time 285 Minutes

The average waiting time per patient = 285/8 = 35.62 Minutes. The doctor is never idle during the 4 hours
period.

Question no 3: Arial ltd. trades in a perishable commodity, each day it receives supplies of the
goods from a wholesaler but the quantity supplied random variable, as is subsequent retail
customer demand for commodity. Both supply and demand are expressed in batches 50 units and
over the past working year (consider 300 days) company has kept records of supplies and demands.
The rates are given in the following table:
Wholesaler No.of days Customer Demand No.of days occurring
50 60 50 60
100 90 100 60
150 90 150 150
200 60 200 30

Buys the commodity Rs.6 p.u. and sells at Rs.10 p.u. at present there are no storage facilities and
unsold units at the end of the day are worthless. Arial estimates that each unit of unsatisfied
demand on any day costs them Rs.2.
Use the following random number for supply – 8,4,8,0,3,3, and for demand 4,7,9,6,1,5.
Simulate six days trading and estimate annual profit, return the exercise to estimate value of
storage facilities.

Solution:

Step 1: Random Number Coding


Purchase/ Purchase Sales Purchases Sales
Sales in Probability Probability ∑ 𝐏𝐫𝐨𝐛𝐚𝐛𝐢𝐥𝐢𝐭𝐲 Random ∑ 𝐏𝐫𝐨𝐛𝐚𝐛𝐢𝐥𝐢𝐭𝐲 Random
Batches Number Number
1 60/300=0.2 60/300=0.2 0.2 0–1 0.2 0–1
2 90/300=0.3 60/300=0.2 0.5 2–4 0.4 2–3
3 90/300=0.3 150/300=0.5 0.8 5–7 0.9 4–8
4 60/300=0.2 30/300=0.1 1.0 8–9 1.0 9–9

E M Reddy Page | 281


AMA-Notes

Step 2: Fitting Random Numbers (or) Simulation Work Sheet


Simulation Run Purchases Random Number Purchases Sales Random Number Sales
1 8 4 4 3
2 4 2 7 3
3 8 4 9 4
4 0 1 6 3
5 3 2 1 1
6 3 2 5 3

Step 3: Understanding the financial details

There are 3 situations:


- Satisfied Demand → Profit per batch = (Rs.10 – Rs.6) x 50 Units = Rs.200
- Excess Stock → Cost per Batch = Rs.6 x 50 Units = Rs300
- Stock Out → Cost per batch = Rs.2 x 50 Units = Rs.100

Step 4: Calculation of weekly profit and annual profit without storage facility
Day Purchase Sales Excess Stock – Short Stock – Profit producing sales (Demands
loss Loss satisfied)
1 4 3 1 - 3
2 2 3 - 1 2
3 4 4 - - 4
4 1 3 - 2 1
5 2 1 1 - 1
6 2 3 - 1 2
Total 2 4 13

Profitability statement – 6 Days Cycle:

Profit (13 Batches x Rs.200) = Rs.2,600


Less: Stock out cost (4 Batches x Rs.100) = (Rs.400)
Less: Excess stock loss (2 Batches x Rs.300) = (Rs.600)
Profit/(Loss) = Rs.1,600
Rs.1,600
Estimated Annual profit = x 300 Days = Rs.80,000
6 Days

Step 5: Calculation of weekly and annual profits with storage facilities


Day Opening Stock Purchases Sales Closing Stock Sales Lost Demand Satisfied
1 - 4 3 1 3
2 1 2 3 - 3
3 - 4 4 - 4
4 - 1 3 but 1 - 2 1
5 - 2 1 1 1
6 1 2 3 - 2
Total 2 15

E M Reddy Page | 282


AMA-Notes

Profitability statement – 6 Days Cycle:

Profit (15 Batches x Rs.200) = Rs.3,000


Less: Stock out cost (2 Batches x Rs.100) = (Rs.200)
Profit/(Loss) = Rs.2,800
Rs.2,800
Estimated Annual profit = x 300 Days = Rs.1,40,000
6 Days

Conclusion: Due to storage facility the company could earn extra profit of Rs.60,000 (Rs.1,40,000 –
Rs.80,000). This is the value of storage facilities. The company can maximum pay Rs.60,000 per annum
towards the storage facilities.

**Question no 4: A plant has a large number of similar machines. The machines breakdown
random and the breakdowns are independent of each other. Once a machine breaks down, it has to
be taken out of production till the time it is repaired. On the basis of the past data, the following
distributions have been constructed.
No.of breakdowns per Probability No.of hours required for repair per Probability
hour breakdown
0 0.900 1 0.100
1 0.085 2 0.240
2 0.012 3 0.450
3 0.03 4 0.165
5 0.040
6 0.005
Each hour that a machine remains idle due to being, or waiting to be repaired, it costs the plant,
Rs.80 per hour by way of lost production. If a repairman is paid at Rs.8 per hour, how many
repairmen should be hired by the company to service the machine breakdowns? For the purpose,
simulate the system for a 50-hour period and use the following random numbers, reading row-wise
starting with the NW corner.
For Breakdowns:
100 375 084 990 128 660 310 852 635 737
985 118 834 886 995 654 801 743 699 098
914 803 441 125 636 611 154 945 424 235
044 005 353 598 460 321 692 195 451 948
980 331 809 797 185 740 541 116 483 690
For Repair times:
765 648 196 093 801 340 455 020 053 035
672 121 099 195 981 783 389 421 125 623

Solution:

Step 1: Random Number Coding (Number of Breakdowns)


Number of Breakdowns Probability Cumulative Probability Class Intervals
0 0.900 0.900 000 – 899
1 0.085 0.985 900 – 984
2 0.012 0.997 985 – 996
3 0.03 1.000 997 – 999

E M Reddy Page | 283


AMA-Notes

Step 2: Random Number Coding (Repair time)


Number of hours taken for repair Probability Cumulative Probability Class Intervals
1 0.100 0.100 000 – 099
2 0.240 0.340 100 – 339
3 0.450 0.790 340 – 789
4 0.165 0.955 790 – 954
5 0.040 0.995 955 – 994
6 0.005 1.000 995 – 999

Step 3: Simulation Worksheet

Breakdown Hours:
Random Numbers No.of Breakdowns Hour of Breakdown
990 2 4
985 2 11
995 2 15
914 1 21
945 1 28
948 1 40
980 1 41
10

Repair time (Only one repair man):


Breakdown Hour of Random Time Entry time Finish Waiting
Numbers Breakdown no.of repairs Taken for repair time time
B1 4 765 3 4 7 Nil
B2 4 648 3 7 10 3
B3 11 196 2 11 13 Nil
B4 11 093 1 13 14 2
B5 15 801 4 15 19 Nil
B6 15 340 3 19 22 4
B7 21 455 3 22 25 1
B8 28 020 1 28 29 Nil
B9 40 053 1 40 41 Nil
B10 41 035 1 41 42 Nil

Step 4:

1 Repair Man:

Repairman Cost (Rs.8 x 50 Hours) = Rs.400


Idle time cost (Rs.80 x 10 Hours) = Rs.800
Total = Rs.1,200

E M Reddy Page | 284


AMA-Notes

2 Repair Men:

Repairmen Cost (Rs.8 x 50 Hours x 2) = Rs.800


Idle time cost = Nil
Total = Rs.800

Notes:

1) With respect to number of breakdowns all Radom numbers below 900 represents ‘0’ breakdowns.
Hence those numbers are ignored.
2) We didn’t consider 3 repair men option because as per the indication given by random number the
maximum number of simultaneous breakdown is ‘2’. Hence the 3rd repair man will always remain idle.
3) The company loses Rs.80 per hour on hour’s lost due to repairing and waiting to be repaired time. We
have ignored the opportunity cost of repairing time. Why?
Answer: This is because the number of repair men decision will not alter the cost.

Question no 5: The tit-fit scientific laboratories is engaged in producing different types of High-
class equipment’s for use in science labs. The company has two difference assembly line to
produce its popular product ‘P’.
Processing time (min) 10 11 12 13 14
Assembly A1 0.10 0.15 0.40 0.25 0.10
Assembly A2 0.20 0.40 0.20 0.14 0.05
Use the following random numbers, generate data on the process times for 15 units of the item and
compute the expected process time for the product.
4134 8343 3602 7505 7428
7476 1185 9445 0089 3424
4943 1915 5415 0880 9390

Solution:

Step 1: Random Number Coding


Time Assembly A1 Assembly A2
Probability ∑ 𝐏𝐫𝐨𝐛𝐚𝐛𝐢𝐥𝐢𝐭𝐲 Random Probability ∑ 𝐏𝐫𝐨𝐛𝐚𝐛𝐢𝐥𝐢𝐭𝐲 Random
Numbers Numbers
10 0.10 0.10 0000 – 0999 0.20 0.20 0000 – 1999
11 0.15 0.25 1000 – 2499 0.40 0.60 2000 – 5999
12 0.40 0.65 2500 – 6499 0.20 0.80 6000 – 7999
13 0.25 0.90 6500 – 8999 0.15 0.95 8000 – 9499
14 0.10 1.00 9000 – 9999 0.05 1.00 9500 – 9999

Step 2: Simulation Worksheet (or) Fitting Random Numbers


Simulation Run Random Number Assembly A1 Time Assembly A2 Time
1 4134 12 11
2 8343 13 13
3 3602 12 11
4 7505 13 12

E M Reddy Page | 285


AMA-Notes

5 7428 13 12
6 7476 13 12
7 1185 11 10
8 9445 14 13
9 0089 10 10
10 3424 12 11
11 4943 12 11
12 1915 11 10
13 5415 12 11
14 0880 10 10
15 9390 14 13

Step 3: Expected process time for the product


Simulation Run Time Required
Simulation Run Assembly A1 Time ∑ 𝐓𝐢𝐦𝐞 Assembly A2 Time ∑ 𝐓𝐢𝐦𝐞
1 12 12 11 12 + 11 = 23
2 13 25 13 25 + 13 = 38
3 12 37 11 38 + 11 = 49
4 13 50 12 50 + 12 = 62
5 13 63 12 63 + 12 = 75
6 13 76 12 76 +12 = 88
7 11 87 10 88 + 10 = 98
8 14 101 13 101 + 13 = 114
9 10 111 10 114 + 10 = 124
10 12 123 11 124 + 11 = 135
11 12 135 11 135 + 11= 146
12 11 146 10 146 + 10 = 156
13 12 158 11 158 + 11 = 169
14 10 168 10 169 + 10 = 179
15 14 182 13 182 + 13 = 195
Total Time 195 Minutes
The expected average time per unit = No.of Units = 15
= 13 Minutes per unit

Note: The minimum spent by a unit in each assembly is 10 minutes. Therefore, to produce one unit we will
minimum take 20 minutes. Hence, for 15 units the minimum time should have been 300 minutes. Then
how it is completed in 195 minutes?
Answer: This is because of simultaneous activates i.e. when we are processing the 10th unit of A2 we will
simultaneously process 11th unit’s A1.

Question no 6: Process involves the production of a particular component, which is then installed
into an end product. Past observation has indicated that the average production time for the
component is 4 minutes but fluctuations about the average do occur. The following probability
distribution has been derived:
Production time (min) 2 3 4 5 6 7
Probability 0.10 0.25 0.40 0.10 0.10 0.05

E M Reddy Page | 286


AMA-Notes

Average time taken to install a component is 3 minutes but it also fluctuations and the following
probability distribution has been derived:
Production time (min) 2 3 4 5
Probability 0.30 0.45 0.15 0.10
The current system uses an operative for installation but the company is considering employing
another operative for installation but the company is considering employing another operative on
the installation process.
Simulate 10 times the current system, using the following 2 digit random numbers:
20, 74, 94, 22, 93, 45, 44, 16, 04, 32 and 03, 62, 61, 89, 01, 27, 49, 50, 90, 98.

Solution:

Step 1: Random Number Coding (Production Time)


Production Time Probability ∑ 𝐏𝐫𝐨𝐛𝐚𝐛𝐢𝐥𝐢𝐭𝐲 Random Number
2 0.10 0.10 00 – 09
3 0.25 0.35 10 – 34
4 0.40 0.75 35 – 74
5 0.10 0.85 75 – 84
6 0.10 0.95 85 – 94
7 0.05 1.00 99 – 99

Step 2: Random Number Coding (Installation Time)


Production Time Probability ∑ 𝐏𝐫𝐨𝐛𝐚𝐛𝐢𝐥𝐢𝐭𝐲 Random Number
2 0.30 0.30 00 – 29
3 0.45 0.75 30 – 74
4 0.15 0.90 75 – 89
5 0.10 1.00 90 – 99

Step 3: Simulation Worksheet (or) Fitting Random Numbers [Installation time]


Simulation Run Random Number Installation time
1 03 2
2 62 3
3 61 3
4 89 4
5 01 2
6 27 2
7 49 3
8 50 3
9 90 5
10 98 5

E M Reddy Page | 287


AMA-Notes

Step 4: Simulation Worksheet (or) Fitting Random Numbers


Simulation Random Production ∑ 𝐏𝐫𝐨𝐝𝐮𝐜𝐭𝐢𝐨𝐧 𝐭𝐢𝐦𝐞 Installation Cumulative
Run Number time time time
1 20 3 3 2 3+2=5
2 74 4 7 3 7 + 3 = 10
3 94 6 13 3 13 + 3 = 16
4 22 3 16 4 16 + 4 = 20
5 93 6 22 2 22 + 2 = 24
6 45 4 26 2 26 + 2 = 28
7 44 4 30 3 30 + 3 = 33
8 16 3 33 3 33 + 3 = 36
9 04 2 35 5 36 + 5 = 41
10 32 3 38 5 41 + 5 = 46

Conclusion: Out of 10 simulation runs only 2 times the production was over and the units are waiting to
get installed due to installation machine being busy. In remaining 8 times the installation was completed and
the machine is waiting for the next unit to get produced. Hence, it is recommended not to have one more
operative in installation department.

E M Reddy Page | 288


AMA-Notes

9. MATERIALS REQUIREMENT PLANNING (MRP)

9.1. Introduction

9.2. Planning Order Release

Question no 1: The product structure and the lead times for a finished product ‘X’ are given in
figure below.
If 100 units of ‘X’ are required in week 12 and if none of the components, sub-assembles and the
end product are either on hand or on order. Compute the amounts and sub-assemblies. Assume
that there is no particulars order size and therefore all the order quantities are lot for lot.

E M Reddy Page | 289


AMA-Notes

Solution:

Summary:
Product Planned Order Release Units
X 10th Week 100
6th Week 40
P
7th Week 100
Q 9th Week 200
3rd Week 1,200
R
4th Week 300
3rd Week 800
S
4th Week 200

E M Reddy Page | 290


AMA-Notes

9.3. Construction of Product Tree

Question no 2: The manufacture of product ‘x’, requires the assembly of modules ‘a’, ‘b’ and ‘c’.
Two modules, each of ‘a’ & ‘c’ and only one module of ‘b’ is needed to make one unit of ‘x’.
Module ‘a’ is made from components ‘i’, ‘j’ and ‘k’. To make one sub-assembly of ‘d’, two
components each of ‘j’ and ‘i’ are required and 1 component of ‘k’. Sub-assembly ‘f’ needs
components ‘i’ and ‘m’ (one each). Module ‘c’ needs sub-modules of ‘g’ and ‘h’ in quantities of two
units and one unit, respectively. Sub-module ‘g’ is, in turn, assembled from five units each of
components ‘i’ and ‘j’. Item ‘i’ need 1 unit each of components ‘n’ and ‘o’.
Draw the product structure based on the above information.
If 100 units of x are to be produced, what are the requirements at the various levels of the product?
Write an indented bill of materials and calculate the requirements of materials at the various
intervals?
Calculate the net requirements if the quantities on hand and/or on order are as shown below. A
safety stock of ‘i’ of 400 is seen as essential as it is used sometimes in another product ‘y’ whose
demand is not all that predictable.
Item On Hand On Order
D 70 -
E - 100
F 50 100
I 500 500

Solution:

Step 1: Product Structure tree

E M Reddy Page | 291


AMA-Notes

Step 2: Planning Order Release

9.4. Preparation of MRP

Question no 3: Given the following information, how many units are on hand at the end of week 9?
Which are the weeks in which the orders may be accepted?
Order Quantity = 200 Week
Lead Time = 2 Weeks 1 2 3 4 5 6 7 8 9
Requirements 90 10 140 55 5 15 115 95 100
Scheduled Receipts
On the hand at the end of the period 110
Planned Order Release

Solution:
Particulars Week
0 1 2 3 4 5 6 7 8 9
Requirements - 90 10 140 55 5 15 115 95 100
Scheduled Receipts - - - 200 - - 200 - 200 -
Closing Stock 110 20 10 70 15 10 195 80 185 85
Planned Order Release - 200 - - 200 - 200 - - -

9.5. Preparation of MRP with safety stock

Question no 4: Geetha industries uses MRP for its production materials planning. The table below
provides the information about a particular component X. The demand for this component is
somewhat uncertain and in order to take care of a sudden spurt in the demand, a safety stock of 50
items is recommended.

E M Reddy Page | 292


AMA-Notes

Order Quantity = 250 Week


Lead Time = 3 Weeks 1 2 3 4 5 6 7 8 9
Requirements 40 100 70 150 20 20 50 100 70
Scheduled Receipts 250
On the hand at the end of the period 150
Planned Order Release

Solution:
Particulars Week
0 1 2 3 4 5 6 7 8 9
Requirements - 40 100 70 150 20 20 50 100 70
Scheduled Receipts - - 250 - 250 - - - - 250
Closing Stock 150 110 260 190 290 270 250 200 100 280
Planned Order Release - 250 - - - - 250 - - -

9.6. Material Purchase Budget and Economic Order Quantity [EOQ]

Question no 5: A company is engaged in manufacturing two products A and B. Product A uses one
unit of component X and two units of component Y. Product B uses two units of component X, one
unit of component Y and two units of component Z. Component Z which is assembled in the
factory uses one unit of component Y. Components X and Y are purchased from the market. The
company has prepared the following forecast of sales and inventory for the next year:
Particulars Product A [Units] Product B [Units]
Sales 80,000 1,50,000
Stock at the end of the year 10,000 20,000
Stock at the beginning of the year 30,000 50,000

The production of both the products and the assembling of the component Z will be spread out
uniformly throughout the year. The company at present orders its inventory of X and Y in
quantities equivalent to 3 months’ production. The company has compiled the following data
related to the two components.
Particulars Product A [Units] Product B [Units]
Price per unit (Rs.) 20 8
Order placing cost per order (Rs.) 1,500 1,500
Carrying cost per annum 20% 20%

Required:
(i) Prepare a budget for production and requirements of components for the year.
(ii) Suggest the optimal order quantity of components X and Y.

E M Reddy Page | 293


AMA-Notes

Solution:

Step 1: Product Structure Tree

Step 2: Production
Particulars A B
Sales 80,000 1,50,000
Add: Closing Stock 10,000 20,000
Less: Opening Stock (30,000) (50,000)
Production 60,000 1,20,000

Step 3: Annual Consumption of ‘X’

A: 60,000 Units x 1 = 60,000 Units


B: 1,20,000 Units x 2 = 2,40,000 Units
Total Units of ‘X’ required = 60,000 Units + 2,40,000 Units = 3,00,000 Units

Step 4: Annual Consumption of ‘Y’

A: 60,000 Units x 2 = 1,20,000 Units


B: 1,20,000 Units x 1 = 1,20,000 Units
Z: 2,40,000 Units x 2 = 2,40,000 Units
Total Units of ‘X’ required = 1,20,000 Units + 1,20,000 Units + 2,40,000 Units = 4,80,000 Units

Step 5: Calculation of EOQ of ‘X’

A (Annual Consumption) = 3,00,000 Units


B (Ordering Cost) = Rs.1,500
C (Carrying Cost) = Rs.20% x Rs.20 = Rs.4 per unit per annum

2AB 2 x 3,00,000 x 1,500


EOQ = √ =√ = 15,000 Units
C 4

Annual Consumption 3,00,000


Ordering cost = x Ordering Cost = x Rs.1,500 = Rs.30,000 per annum
EOQ 15,000

E M Reddy Page | 294


AMA-Notes

1 1
Carrying Cost = 2 x EOQ x Carrying Cost = 2 x 15,000 Units x Rs.4 = Rs.30,000 per annum

Step 6: Calculation of EOQ of ‘Y’

A (Annual Consumption) = 4,80,000 Units


B (Ordering Cost) = Rs.1,500
C (Carrying Cost) = Rs.20% x Rs.8 = Rs.1.6 per unit per annum

2AB 2 x 4,80,000 x 1,500


EOQ = √ =√ = 30,000 Units
C 1.6

Annual Consumption 4,80,000


Ordering cost = x Ordering Cost = x Rs.1,500 = Rs.24,000 per annum
EOQ 30,000

1 1
Carrying Cost = 2 x EOQ x Carrying Cost = 2 x 30,000 Units x Rs.1.6 = Rs.30,000 per annum

E M Reddy Page | 295


AMA-Notes

10. NETWORK ANALYSIS

10.1. Learning Objectives

1) Understanding some basic terms used in Network


2) Learning to draw a network diagram and appropriately use dummy activates
3) Forward Pass and Backward Pass Procedure
4) Calculating EST, LST, EFT and LFT of each activity
5) Total float, free float and independent float
6) Identification of critical activates and critical path
7) Program evaluation review technique
8) Network Crashing
9) Resource allocation with loading chart
10) Resource leveling or smoothing with time scale network
11) Network updating

10.2. Introduction

A. Projects involves huge investments, complex activities and a longer time frame.
B. To avoid time & cost over-run it is necessary to properly plan & control the project implementation.
C. One of the popular techniques used in project planning and control is “Network Analysis”.
D. In Network Analysis we,
a) Break the projects into number of activities
b) Sequence the activities
c) Present the same in the form of a network diagram which is a pictorial representation of the
entire project.
d) Estimate the time and resource required for each activity and fit it into the network diagram.
e) Then use techniques like crashing, resource allocation etc., to manage the network.

10.3. Understanding some basic terms used in Network

1) Activities
2) Events
3) Types of Activities
4) Types of Events
5) Errors in drawing a Network
6) Conventions in Network Diagram

10.3.1. Activities & Events

1)
a) Activity is something that consumes time and resource.
b) It is represented as a straight line in the Network.
c) Here Activity ‘A’ is covered by 2 circles ① and ②. They are called “Events”.

E M Reddy Page | 296


AMA-Notes

d) Event ① is the starting event of Activity ‘A’ and called “Tail Event” and event ② is the ending
event of an activity and called “Head Event”.
2) Types of Activities:

The diagram shows Activity ‘A’ should be done first and when Activity ‘A’ is over Activity ‘B’ and Activity
‘C’ can start. Here there are 3 types of activities:
a) Preceding Activities → For Activity ‘B’ and Activity ‘C’, Activity ‘A’ is proceeding.
b) Succeeding Activities → For Activity ‘A’, Activity ‘B’ and Activity ‘C’ are succeeding.
c) Simultaneous Activities → For Activity ‘B’ Activity ‘C’ is simultaneous and vice-versa.
3) Events:

a) When an event is a starting event for more than one activity it is called “Burst Event”. Here
Event ② is starting event for Activity ‘B’ and Activity ‘C’.
b) If an event is ending event of more than one activity it is called “Merge Event”. Event ⑤
represents completion of Activity ‘D’ and Activity ‘E’.

10.3.2. Errors in Networking

1) Looping Error
2) Dangling Error
3) Mistake in Succeeding, Preceding relationship

10.3.2.1. Looping Error

1) When Activity ‘A’ is over Activity ‘B’ will starts, when Activity ‘B’ is over Activity ‘C’ starts, on
completion of Activity ‘C’ again Activity ‘B’ starts. Thus a loop is formed.
2) This project will never end.
3) In real world there are situations where finite loops can be formed which is supported by flowcharting
and programming techniques. However, in network looping is not possible because with loops we
cannot perform the Forward and Backward Pass procedures.

E M Reddy Page | 297


AMA-Notes

10.3.2.2. Dangling Error

1) A project can have only one starting event and one ending event.
2) In the above diagram if we call event ③ as the completing event of the project then Activity ‘B’
becomes irrelevant for the project completion and vice-versa.
3) This error is called Dangling error. Every activity should either be connected to next Activity or to the
last event.
4) The error can be removed by using ‘Dummy Activity’.
5) Dummy activity is an activity that consumes ‘0’ time and ‘0’ resource. It is represented by dotted lines.

6) Event ④ has now become merge event. It represents the completion of activities ‘B’ and dummy. Since
dummy takes ‘0’ time it gets completed as soon as Activity ‘C’ is completed. Hence event ④ represents
completion of Activity ‘B’ and Activity ‘C’.

10.3.2.3. Mistake in Preceding, Succeeding relationship

Example: A = 1st activity. B & C can start once A is completed. D can start once B & C is over. ‘E’ can
start once B is over.
1) Wrong Diagram:

There are two mistakes in this diagram.


i) Two Activities can have a common tail event or can have common head event but cannot
have the same head and tail events as it creates problems in Forward, Backward pass
computations.
ii) For staring of Activity ‘D’, Activity ‘B’ and Activity ‘C’ should merge and for starting of
activity ‘E’ activity ‘B’ alone should be completed.
The above can be resolved using dummy activity.

E M Reddy Page | 298


AMA-Notes

10.3.3. Conventions in Network Drawing

1) Time moves from left to right.


2) Activities are represented as straight lines.
3) Head event number should be greater than tail event number.
4) Try to avoid crossing of lines while drawing network. If necessary, use gates.

10.4. Floats, Forward Pass, Backward Pass and Critical Path

Question no 1: With the help of activities given below draw a network and find out: -
(a) Earliest start time (b) Earliest finish time (c) Latest start time (d) Latest finish time (e) Total
float (f) Free float (g) Independent float.
The following are the activities and their duration:
Activity Duration
1–2 6
2–3 8
2–4 10
3–4 0
3–5 6
4–5 20
5–6 16

Solution:

Step 1: Network Diagram

E M Reddy Page | 299


AMA-Notes

Step 2: Forward Pass Procedure (Earliest Start Time – EST)

Step 3: Backward Pass Procedure (Latest Finish Time – LFT)

Forward Pass Procedure:

1) Forward Pass is a procedure through which we find out earliest start time for every activity.
2) It is done as follows:
a. Put “E” of event 1 as “0”.
b. “E” of head event = “E” of tail event + Duration
c. Where the head event is merge event it will have multiple tails. In such case it is “E” of each tail
event + Duration whichever is higher.

E M Reddy Page | 300


AMA-Notes

Backward Pass Procedure:

1) Through this procedure we find out latest finish time of each activity.
2) It is done as follows:
a. Assign “L” of the last event as “E” of last event.
b. “L” of tail event = “L” of Head event – Duration
c. If the tail event is burst event, it will have multiple heads. In such case it is “L” of each head
event – Duration whichever is lower.

Step 4: Calculation of EST, EFT, LST, LFT and Total Float


Activity 𝐭 𝐧 – Duration Start Finish Total Float
E L E L
1–2 6 0 0 6 6 0
2–3 8 6 8 14 16 2
2–4 10 6 6 16 16 0
3–4 0 14 16 14 16 2
3–5 6 14 30 20 36 16
4–5 20 16 16 36 36 0
5–6 16 36 36 52 52 0

Calculation of time estimates:

1) EST = “E” of Tail Event


2) LFT = “L” of Head Event
3) LST = “LFT – Duration
4) EFT = EST + Duration

Step 5: Network with Critical Paths

E M Reddy Page | 301


AMA-Notes

Total Float, Critical Activity and Critical Path:

1) Total Float = LFT – EFT (or) LST – EST


2) Total Float indicates by how many days we can postpone an activity without affecting the project
duration.
3) The total float of activity ‘2 – 3’ days. This means instead of starting activity ‘2 – 3’ on 6th day if we start
on 8th day, still we can complete on time.
4) Those activities having total float as ‘0’ cannot be postponed and are referred as “Critical Activity”.
5) The path in which all the activities are critical activates is called “Critical Path”.
6) Critical path is the longest path in the Network and all the activities in the path are critical activates.
Paths Duration
1–2–3–5–6 6 + 8 + 6 + 16 = 36
1 – 2 – 3 – 4 – 5 – 6 6 + 8 + 0 + 20 + 16 = 50
1–2–4–5–6 6 + 10 + 20 + 16 = 52 Critical Path

Step 6: Calculation of Free Float and Independent Float


Activity Total Free Float (Total Float – Head Independent Float (Free Float – Tale
Float event slack) event slack)
1–2 0 0–0=0 0–0=0
2–3 2 2–2=0 0–0=0
2–4 0 0–0=0 0–0=0
3–4 2 2 – 0= 0 2–2=0
3–5 16 16 – 0 = 16 16 – 2 = 14
4–5 0 0–0=0 0–0=0
5–6 0 0–0=0 0–0=0

Notes:

1) Concept of free float:


a) Free float = Total Float – Head Event Slack
b) It is that part of total float which does not affect the float of succeeding activity.
c) For example, activity ‘2 – 3’ is having total float of 2 days and activity ‘3 – 5’ is having a total
float f 16 days. If we postpone activity ‘2 – 3’ by 2 days, we can postpone activity ‘3 – 5’ only by
14 days. That means using activity ‘2 – 3’ floats affect the float of succeeding activity ‘3 – 5’.
Thus activity ‘2 – 3’ has ‘0’ free float.
2) Concept of independent float:
a) Independent Float = Free Float – Tail Event Slack
b) It is that part of that total float which does not affect the float of succeeding and preceding
activity.
c) For example, activity ‘3 – 5’ has a total float of 16 days Postponing it by 16 days results in
reduction of float of activity ‘2 – 3’ but if we postpone by 14 days (Independent Float), it does
not affect the float of activity ‘2 – 3’.

E M Reddy Page | 302


AMA-Notes

10.5. Network Crashing

1) Crashing is a process through which we try to reduce duration of a project.


2) The network crashing affects the project cost in two ways:
(i) Benefit → Savings in Overhead Cost → Overhead costs are those costs which are linked to
project duration
(ii) Cost → To reduce the project duration we should reduce activity duration for which we should
employ more resources in the activities. The cost involved is called “Crash Cost”.
3) Keep Crashing the network as long as the benefit of crashing exceeds the cost of crashing. When this
reverses stop crashing.
4) The project length (duration) at which the total cost is minimum is called “Optimum Project
Length/Duration”.
5) If the problem asks “Minimum Project Length/Shortest Project Duration”, continue crashing till we
reach a stage where further crashing not possible.

Question no 2: A small maintenance project consists of jobs in the table below. With each job is
listed its normal time and a minimum or crash time in days. The cost in Rs. Per day of each job is
also given:
Job (i – j) Normal Days Crash Days Crash Cost per Day
1–2 9 6 20
1–3 8 5 25
1–4 15 10 30
2–4 5 3 10
3–4 10 6 15
4–5 2 1 40
a) What is the normal project length and minimum project length?
b) Determine the minimum crashing cost of schedule ranging from normal length down to, and
including, the minimum length schedule.
c) Overhead costs total Rs.60 per day. What is the optimum length schedule in terms of both
crashing and overhead cost?

Solution:

Step 1: Network Diagram

E M Reddy Page | 303


AMA-Notes

Step 2: Network Crashing

Network Diagram:

Paths Table:
Paths 0 3 4 5 7 8
1–2–4–5 16 16 15 15 13 12
1–4–5 17 17 16 15 13 12
1–3–4–5 20 17 16 15 13 12

Slash Table:
Activity Crash Days Possible Crash Cost/Day
1–2 3/2 20
1–3 3/1/0 25
1–4 5/4/2/1 30
2–4 2/0 10
3–4 4/1/0 15
4–5 1/0 40

Cost Table:
Duration Crash Cost Overhead @ 60 per Day Total Cost
20 Nil 1,200 1,200
17 45 1,020 1,065
16 85 960 1,045
15 130 900 1,030
13 260 780 1,040
12 335 720 1,055

Evaluation Table:
Stages Activities Possible Remarks Crash Cost ∑ 𝐂𝐫𝐚𝐬𝐡 𝐂𝐨𝐬𝐭
(Rs.)
A 1 – 3, 3 – 4, 4 – 5 Crash ‘3 – 4’ by 3 days Rs.15 x 3 Days = 45
45

E M Reddy Page | 304


AMA-Notes

B [1 – 3, 1 – 4, 3 – 4]; [1 – Crash ‘4 – 5’ by 1 day Rs.40 x 1 Day = 85


4, 4 – 5] Rs.40
C [1 – 3, 1 – 4, 3 – 4]; [1 – Crash ‘3 – 4’ & ‘1 – 4’ by Rs.45 x 1 Day = 130
4, 4 – 5] 1 day Rs.45
D 1 – 2, 2 – 4, 1 – 4, 1 – 3 Crash 2 – 4, 1- 4, 1-3 by Rs.65 x 2 Days = 260
1 day Rs.130
E 1 – 2, 1 – 4, 1 – 3 Crash 1 – 2, 1 – 4, 1 – 3 Rs.75 x 1 Day = 335
by 1 day Rs.75
F 1 – 2, 1 – 4, Not possible to crash

Final Solution:
Duration Days Cost (Rs.)
Normal 20 Days 1,200
Optimum 15 Days 1,030
Shortest 12 Days 1,055

Notes:

1) When decision regarding crashing is made in the remarks column in evaluation table.
2) There are two aspects to be decide:
(i) What activity to be crashed
(ii) How many days to crash.
3) Always crash only critical activities because crashing non-critical activities does not result in reduction of
project duration. It is a wasteful expenditure.
4) While selecting select least cost critical activity.
5) If there are more than one critical paths, all the paths should be simultaneously crashed. Here we have
two options:
(i) Crash an activity common to all the parts
(ii) Crash one activity each from every critical path
That option that gives lowest crash cost should be selected.
6) In deciding the number of days to be crashed we should consider 2 tables:
(i) Slash Table → To see how many crash days are available
(ii) Paths Table → To ensure that the path that being is crashed retains it’s criticality. For example,
in stage A we decided to crash ‘3 – 4’ by 3 days even though ‘3 – 4’ days available to crash?
Answer: This is because if we crash ‘3 – 4’ by 4 days the critical path ‘1 – 3 – 4 – 5’ becomes 16
days while ‘1 – 4- 5’ continues to be 17 days. Inspire of 4 days crashing the project duration gets
reduced only 3 days making the 4th day expenditure wasteful. The rule is “A path once critical
should always be critical”.
7) Till 15 days crashing we can see the total cost getting reduced beyond which it begins to increase. Hence
the optimum duration is 15 days.
8) Continue crashing till we reach a stage where we don’t have crash days available in any one of the critical
path. For example, in stage F there are 3 critical paths but we do not have any critical activity to crash in
the critical path ‘1 – 3 – 4 – 5’.
9) Situation 1: If the problem requires us to find out normal and shortest duration without
requiring any cost detail the question can be answered simply as follows:
Answer:

E M Reddy Page | 305


AMA-Notes

Paths Normal Duration Shortest Duration


1–2–4–5 16 10
1–4–5 17 11
1–3–4–5 ⑳ ⑫

10) Situation 2: Network Crashing when the problem requires us to calculate normal and shortest
duration with their associated costs
Step 1: Draw Network with Normal & Shortest Duration

Step 2: Prepare Slash Table


Activity Crash Days Possible Crash Cost/Day
1–2 3 20
1–3 3 25
1–4 5 30
2–4 2 10
3–4 4 15
4–5 1 40

Step 3: Crashing Table


Path Normal Shortest Activities Crash Days Crash Cost
Duration Duration Available Days
1 – 2 – 4 – 5 16 10 1–2 3 1 1 x 20 = 20
2–4 2 2 2 x 10 =20
4–5 1 1 -
1–4–5 17 11 1–4 5 4 4 x 30 = 120
4–5 1 1 -
1 – 3 – 4 – 5 20 12 1–3 3 3 3 x 25 = 75
3–4 4 4 4 x 15 =60
4–5 1 1 1 x 40 = 40
Total Crash Days Cost 335

E M Reddy Page | 306


AMA-Notes

(i) The Critical Path for the shortest duration is ‘1 – 3 – 4 – 5’. Thus we can understand that, that path is
crashed fully i.e. all the activities in that path has been crashed to the extent of crash days available.
(ii) The path ‘1 – 4 – 5’ can be crashed by 6 days and ‘1 – 2 – 4 – 5’ also by 6 days but we have decided to
crash them by 5 days & 4 days respectively. Why?
Answer: Any effort to crash these paths below 12 days is wasteful expenditure as it will have no impact
on project duration.
(iii) When we crash ‘4 – 5’ by 1 day, the other two paths also get crashed by 1 day because ‘4 – 5’ is present
in those paths also.
(iv) Thus the second path ‘1 – 4 – 5’ should now be crashed by 4 days which we do it by crashing path ‘1 –
4’.
(v) Similarly, the path ‘1 – 2 – 4 – 5’ should now being crashed by 3 days for which we first crash path ‘2 –
4’ due it’s low crash cost and next path ‘1 – 2’.
(vi) Final Solution:
Time Duration Overhead Cost @ 60 /day Crash Cost Total Cost
Normal 20 Days Rs.1,200 - Rs.1,200
Shortest 12 Days Rs.720 Rs.335 Rs.1,055

Question no 3: The following table shows for each activity needed to complete the project the
normal time, the shortest time in which the activity can be completed of a building contract and
the cost per day for reducing the time of activity. The contract includes a penalty clause of Rs.100
per day over 17 days. The overhead cost per day is Rs.160.

Activity Normal time (in days) Shortest time (in days) Cost of reduction per day
1–2 6 4 80
1–3 8 4 90
1–4 5 3 30
2–4 3 3 -
2–5 5 3 40
3–6 12 8 200
4–6 8 5 50
5–6 6 6 -

The cost of completing the eight activities in normal time is Rs.6,500.

1) Calculate the normal duration of the project, its cost and the critical path.
2) Calculate and plot on graph the cost time function for the project and state.
(i) The lowest cost and associated time.
(ii) The shortest time and associated cost.

E M Reddy Page | 307


AMA-Notes

Solution:

Step 1: Network Diagram

Step 2: Network Crashing

Network Diagram:

Paths Table:
Paths 0 3 4 5 7
1–2–5–6 17 17 16 15 13
1–2–4–6 17 17 16 15 13
1–4–6 13 13 13 13 11
1–3–6 20 17 16 15 13

E M Reddy Page | 308


AMA-Notes

Slash Table:
Activity Crash Days Possible Crash Cost/Day
1–2 2/1/0 80
1–3 4/1/0 90
1–4 2 30
2–4 0 -
2–5 2/0 40
3–6 4/3/1 200
4–6 3/1 50
5–6 0 -

Cost Table:
Duration Normal Cost Overhead @ Rs.160 per Day Crash Cost Penalty Cost Total Cost
20 6,500 3,200 - 300 10,000
17 6,500 2,720 270 - 9,490
16 6,500 2,560 440 - 9,500
15 6,500 2,400 720 - 9,620
13 6,500 2,080 1,300 - 9,880

Evaluation Table:
Stages Activities Possible Remarks Crash Cost ∑ 𝐂𝐫𝐚𝐬𝐡 𝐂𝐨𝐬𝐭
(Rs.)
A 1 – 3, 3 – 6 Crash ‘1 – 3’ by 3 days Rs.90 x 3 Days = 270
Rs.270
B [1 – 3, 3 – 6], [1 – 2, 2 – 5], Crash ‘1 – 2’ & ‘1 – 3’ by Rs.170 x 1 day = 440
[1 – 2, 4 – 6] 1 day Rs.170
C [3 – 6], [1 – 2, 2 – 5], [1 – 2, Crash ‘1 – 2’ & ‘3 – 6’ by Rs.280 x 1 day = 720
4 – 6] 1 day Rs.280
D [3 – 6], [2 – 5], [4 – 6] Crash ‘3 – 6’, ‘2 – 5’, ‘4 – Rs.290 x 2 days 1,300
6’ by 2 days = Rs.580

Final Solution:
Duration Days Associated Cost (Rs.)
Normal 20 Days 10,000
Optimum 17 Days 9,490
Shortest 13 Days 9,880

E M Reddy Page | 309


AMA-Notes

Graphical Representation:

10.6. Program Evaluation Review Technique

1) When the activity times for a project could not be estimated with certainty, Project Evaluation Review
Technique should be used.
2) In Project Evaluation Review Technique 3 time estimates are made for every activity namely Optimistic,
Pessimistic and Most likely time.
3) These estimates are assumed to fall into Normal Probability Distribution.
4) For Drawing the network which time should be taken? Is it optimistic, pessimistic or most likely?
Answer: Neither of the 3. We should take expected time for the network.
5) Expected time is the average time where the weights for optimistic and pessimistic is ‘1’ and most likely
‘4’.
to +4tm +tp
te = where
6
t e = Expected time
t o = Optimistic time
t m = Most likely time
t p = Pessimistic time
6) When the time is expected (Mean) there should be a variance around mean. The variance of expected
tp −to 2
time = ( )
6

**Question no 4: A small project network is composed of 7 activities whose time estimates are
listed in the table below.
a) Draw the project network and identify all the paths through it.
b) Find the expected duration and variance for each activity. What is the expected project length?
c) Calculate the variance and the standard deviation of project length. What is the probability that

E M Reddy Page | 310


AMA-Notes

the project will be completed


i) At least 3 weeks earlier than expected
ii) Not more than 3 weeks later than expected
d) If the project due date is 18 weeks what is the probability of not meeting it?
e) What due date has about 90% chance of being met?
f) Find probability of reaching event – 5 in 9 weeks
g) Also find the event variances.
Activities (I – J) 𝐭 𝐨 (Optimistic Time) 𝐭 𝐦 (Most Likely Time) 𝐓𝐏 (Pessimistic Time)
1–2 1 1 7
1–3 1 4 7
1–4 2 2 8
2–5 1 1 1
3–5 2 5 14
4–6 2 5 8
5–6 3 6 15

Solution:

Step 1: Calculation of expected time and variance for each activity


Activities (I – J) 𝐭 𝐞 (Expected Time) Variance
1–2 1+(4 x 1)+7
=2
7−1
( 6 )2 = 1
6
1–3 1+(4 x 4)+7
=4 (
7−1 2
) =1
6 6
1–4 2+(4 x 2)+8
=3
8−2 2
( 6 ) =1
6
2–5 1+(4 x 1)+1
=1
1−1
( 6 )2 = 0
6
3–5 2+(4 x 5)+14
=6
14−2
( 6 )2 = 4
6
4–6 2+(4 x 5)+8
=5
8−2
( 6 )2 = 1
6
5–6 3+(4 x 6)+15
=7
15−3
( 6 )2 = 4
6

Step 2: Drawing Network using expected times

E M Reddy Page | 311


AMA-Notes

Step 3: Identification of the critical path using the expected projected length
Paths Duration
1–2–5–6 2 + 1 + 7 = 10
1–3–5–6 4 + 6 + 7 = 17 Critical Path
1–4–6 3+5=8

Expected project length = 17 Weeks

Step 4: Variance and standard deviation of project length

1) Variance of project length is the total of variances of all the activities in the critical path of the project.
2) Standard Deviation if the square root of the variance.
Activities Duration Variance
1–3 4 1
3–5 6 4
5–6 7 4
17 9

Standard Deviation of project = √9 = 3

Step 5: Probability of completing the project within 14 weeks

X−X 14−17
a) Z = σ = 3 = -1
b) Normal Table (Z) = NT (1) = 0.3413
Note: 0.3413 means 34.13% chance of completing the project between 14 and 17 weeks.
c) Probability = 0.5 – 0.3413 = 0.1587. This means there is 15.87% chance of completing the project
within 14 weeks.

Step 6: Probability of completing the project within 20 weeks

E M Reddy Page | 312


AMA-Notes

X−X 20−17
a) Z = σ = 3 = 1
b) Normal Table (Z) = NT (1) = 0.3413
c) Probability = 0.5 + 0.3413 = 0.8413. This means there is 84.13% chance of completing the project
within 20 weeks.

Step 7: Probability of not meeting the due date of 18 weeks

X−X 18−17
a) Z = σ = 3 = 0.33
b) Normal Table (Z) = NT (0.33) = 0.1293
c) Probability = 0.5 – 0.1293 = 0.3707. This means there is 37.07% chance of exceeding the due date of
18 weeks.

Step 8: The due date that has 90% chance of being met

0.40 is the normal table value of Z which is 1.28.


X−X
Z = σ
X−17
1.28 = 3
X = 3.84 + 17 = 20.84
20.84 weeks has got chance of 90% being met.

Step 9: Probability of reaching event 5 in 9 weeks


The longest leading to event 5 is ‘1 – 3 – 5’. The expected time in reaching event 5 is 10 weeks (4 weeks + 6
weeks). The variance of this expected time is the total of the variances of the longest path leading to this
event.
Activity Variance
1–3 1
3–5 4
Variance 5

Standard Deviation = √5 = 2.24

E M Reddy Page | 313


AMA-Notes

X−X 9−10
a) Z = σ = 2.24 = - 0.45
b) Normal Table (Z) = NT (0.45) = 0.1736
c) Probability = 0.5 – 0.1736 = 0.3264. This means there is 32.64% chance of reaching event-5 in 9 weeks.

Step 10: Calculation of event variances


Event Longest Path Computation Variance
6 1–3–5–6 1+4+4 9
5 1–3–5 1+4 5
4 1–4 1 1
3 1–3 1 1
2 1–2 1 1
1 Nil 0 0

10.7. Resource Allocation

Question no 5: Find out the time required to complete the project. No. of persons 4.
Job Time Men
1–2 10 1
1–3 6 2
1–5 5 3
2–3 0 0
2–6 8 1
3–4 10 2
4–7 10 3
5–6 7 1
6–7 5 2

Solution:

Step 1: Network Diagram

E M Reddy Page | 314


AMA-Notes

Step 2: Calculation of EST, LFT, LST, LFT and Total Float


Activity Duration Start Finish Total Float
E L E L
1–2 10 0 0 10 10 0
1–3 6 0 4 6 10 4
1–5 5 0 13 5 18 13
2–3 0 10 10 10 10 0
2–6 8 10 17 18 25 7
3–4 10 10 10 20 20 0
4–7 10 20 20 30 30 0
5–6 7 5 18 12 25 13
6–7 5 18 25 23 30 7

Step 3: Resource Allocation table


Halt Resources Activities Total Men x Ra Allocated Idle
Time Available Available Float Days nk Men
0 R1, R2, R3 & R4 1–2 0 1 x 10 1 R1
1–3 4 2x6 2 R2 & R3
1–5 13 3x5 3 Not Possible R4
6 R2, R3 & R4 1–5 7 3x5 1 R2, R3, R4 -
10 R1 2–3 0 Dummy
2–6 7 1x8 2 R1
3–4 0 2 x 10 1 Not Possible -
11 R2, R3 & R4 3–4 0 2 x 10 1 R2 & R3
5–6 13 1x7 2 R4 -

E M Reddy Page | 315


AMA-Notes

18 R1 & R4 6–7 7 2x5 1 R1 & R4 -


21 R2 & R3 4–7 0 3 x 10 1 Not Possible R2 & R3
23 R1, R2, R3 & R4 4–7 0 3 x 10 1 R1, R2 & R3 R4

The Project can be completed with 4 persons only in 33 days. Two times we postponed critical activities,
first ‘3 – 4’ by 1 day and next ‘3 – 4’ by 2 days.

Step 4: Loading Chart (or) Gantt Chart

1) Loading Chart indicates at different points of time how the resources are loaded and what resource is
idle.
2) The loading chart is also called “Gantt Chart”.

10.8. Resource Leveling (or) Resource Smoothing

Question no 6:
Job Time Men
1–2 4 3
1–3 6 6
3–5 5 7
2–4 5 5
3–6 4 5
4–7 5 4
5–7 3 3
6–7 7 4

Calculate the minimum number of men required to complete the above project in time.

E M Reddy Page | 316


AMA-Notes

Solution:

Step 1: Network Diagram

Step 2: Calculation of EST, LFT, LST, LFT and Total Float


Activity Duration Start Finish Total Float Free Float
E L E L
1–2 4 0 3 4 7 3 0
1–3 6 0 0 6 6 0 0
3–5 5 6 9 11 14 3 0
2–4 5 4 7 9 12 3 0
3–6 4 6 6 10 10 0 0
4–7 5 9 12 14 17 3 3
5–7 3 11 14 14 17 3 3
6–7 7 10 10 17 17 0 0

Step 3: Before resource smoothening


Halting time In Out Strength
0 3+6 0 9
4 5 -3 11
6 7+5 -6 17
9 4 -5 16
10 4 -5 15
11 3 -7 11
14 - -4-3 4
17 - -4 -

E M Reddy Page | 317


AMA-Notes

From the above table it can be seen that we require 17 persons on a single day for doing the project. It
occurs during day 6.

Step 4: Post pone 3 – 5 by 3 days


Halting time In Out Strength
0 3+6 0 9
4 5 -3 11
6 5 -6 10
9 4+7 -5 16
10 4 -5 15
14 3 -7-4 7
17 - -3-4 -

Notes:

1) 17 persons were required on day 6 when we are starting ‘3 – 5’ & ‘3 – 6’ activities.


2) ‘3 – 6’ cannot be postponed as it is a critical activity. Hence we postponed ‘3 – 5’ in order to level peak
requirement.
3) By postponing ‘3 – 5’ by 3 days we are able to complete the project with 16 persons.
4) The peak requirement of 16 occurs on day 9. On that day we are starting activities ‘3 – 5’ & ‘4 – 7’.
5) ‘3 – 5’ cannot be postponed because it’s float is fully used but ‘4 – 7’ can be postponed by 3 days i.e.
start on 12th day.

Step 5: Post-pone ‘4 – 7’ by 3 days


Halting time In Out Strength
0 3+6 0 9
4 5 -3 11
6 5 -6 10
9 7 -5 12
10 4 -5 11
12 4 0 15
14 3 -7 11
17 - -4-3-4 -

Notes:

1) Due to postponing ‘4 – 7’ by 3 days we are able to the project with 15 persons.


2) On 12th day when the peak requirement happens we are starting activity ‘4 – 7’ which cannot be further
postponed. Hence the resource leveling stops.

Step 6: Time scale diagram

Before Resource Smoothening

Dotted line represents “Float”.

E M Reddy Page | 318


AMA-Notes

After Resource Smoothening:

How to draw time scale network diagram:

1) In a normal network diagram there is no link between the length of the line and the duration of the
activity.
2) In a time-scale diagram, the network will be drawn on a time scale where activities with longer duration
will have longer lines and vice-versa. In other words, the length of the line is proportioned to duration
of the activity.
3) First we should draw the longest path i.e. critical path around which other paths should be drawn.
4) The other paths will obviously complete before the critical path and then wait till for the critical path to
complete. The difference is called “Float” and is represented by dotted lines.
5) The activity ‘4 – 7’ is completed on day 14 but event 7 occurs only on day 17.

E M Reddy Page | 319


AMA-Notes

6) Instead of finishing early and waiting we used the float and started late and then completed on time due
to which we are able to level the resources. It is called resource leveling (or) resource smoothening.
Same logic for ‘3 – 5’ also.

10.9. Understanding to draw a network & use dummy activities

Situation 1: P and Q has the same tail event and also are joint preceding activity
Condition To be done
a) P is not repeated as preceding activity and Q also is not Dummy from P to Q or Q to P
repeated as preceding activity
b) P is repeated as preceding activity and not Q Dummy from P to Q
c) Q is repeated as preceding activity and not P Dummy from Q to P
c) P is repeated and Q also repeated Two dummies. P to a new event and
Q to a new event

Example 1:
Activity Preceding
A -
P A
Q A
R P, Q
S R

Network Diagram:

Example 2:
Activity Preceding
A -
P A
Q A
R P, Q
S P

E M Reddy Page | 320


AMA-Notes

Network Diagram:

Example 3:
Activity Preceding
A -
P A
Q A
R P, Q
S Q

Network Diagram:

Example 4:
Activity Preceding
A -
P A
Q A
R P, Q
S P
T Q

E M Reddy Page | 321


AMA-Notes

Network Diagram:

Situation 2: P and Q has different tail events and are joint preceding activity
Condition To be done
a) P is not repeated as preceding activity and Q also is not No dummy. Merge P and Q to start
repeated as preceding activity next activity
b) P is repeated as preceding activity and not Q Dummy from P to Q
c) Q is repeated as preceding activity and not P Dummy from Q to P
c) P is repeated and Q also repeated Two dummies. P to a new event and
Q to a new event

Example 1:
Activity Preceding
A -
B -
P A
Q B
R P, Q
S R

Network Diagram:

E M Reddy Page | 322


AMA-Notes

Example 2:
Activity Preceding
A -
B -
P A
Q B
R P, Q
S P

Network Diagram:

Example 3:
Activity Preceding
A -
B -
P A
Q B
R P, Q
S Q

Network Diagram:

Example 4:
Activity Preceding
A -
B -

E M Reddy Page | 323


AMA-Notes

P A
Q B
R P, Q
S P
T Q

Network Diagram:

Question no 7: Draw a network for the following data:


Task Immediate predecessor
A --
B A
C A
D B
E A
F B, E
G C
H D, F
I G
J H, I

Solution:

Question no 8: Draw a network for the following data:


Task Immediate predecessor
A --
B --
C B
D B

E M Reddy Page | 324


AMA-Notes

E B
F E
G A, D, C

Solution:

**Question no 9: The production manager at Gemini Machines ltd. has been asked to present
information about the times and costs for the development of a new machine that the company
may choose to manufacture. The Managing Director requires accurate time and cost estimates
since the project will involve a fixed fee contract offering no provision for later re-generation, even
in the event of modifications.
Activity Preceding Activities Duration (Weeks) Cost Rs.’000
A Obtain engineering quotes I 1 4
B Sub-contract specifications A, J 4 8
C Purchase of raw materials -- 3 24
D Construct prototype I 5 15
E Final Drawings I 2 6
F Fabrication H 6 30
G Special Machine Study -- 4 12
H Sub-contract work B, E 8 40
I Preliminary design G 2 8
J Vendor evaluation C, D 3 3

The production manager has been asked to identify the critical activities, to determine the shortest
project duration and to provide a week-by-week cost schedule.
Required:

E M Reddy Page | 325


AMA-Notes

(a) Draw a network to represent the inter-relationships between the activities indicated, and insert
earliest and latest event times throughout.
(b) Determine the critical path and the shortest possible duration of the project.
(c) Assuming each activity commences at the earliest start date and that for each activity the cost
is incurred evenly over its duration construct a week-by-week schedule of cash flows.
(d) The project is to financed by Rs.50,000 available initially, a further Rs.50,000 available at the
start of week 9 and the final Rs.50,000 available from week 20. Identify any particular problems
and suggest solutions.
Solution:
Step 1: Network Diagram

Step 2: Identification of the critical path


Paths Duration
1–4–5–6–7–8 3 + 3 + 4 + 8 + 6 = 24
1–2–3–4–5–6–7–8 4 + 2+ 5 + 3 + 4 + 8 + 6 = 32
1–2–3–5–6–7–8 4 + 2 + 1 + 4 + 8 + 6 = 25
1–2–3–6–7–8 4 + 2 + 2 + 8 + 6 = 22

Step 3: Calculation of cost per week


Activity Duration (Weeks) Cost Rs.’000 Cost Rs.’000/week
A Obtain engineering quotes 1 4 4
B Sub-contract specifications 4 8 2
C Purchase of raw materials 3 24 8
D Construct prototype 5 15 3
E Final Drawings 2 6 3
F Fabrication 6 30 5
G Special Machine Study 4 12 3
H Sub-contract work 8 40 5
I Preliminary design 2 8 4
J Vendor evaluation 3 3 1

E M Reddy Page | 326


AMA-Notes

Step 4: Calculation of EST, EFT, LST, LFT and Total Floats


Activities Activity Duration EST LST EFT LFT Total Float
1–2 G 4 0 0 4 4 0
1–4 C 3 0 8 3 11 8
2–3 I 2 4 4 6 6 0
3–4 D 5 6 6 11 11 0
3–5 A 1 6 13 7 14 7
3–6 E 2 6 16 8 18 10
4–5 J 3 11 11 14 14 0
5–6 B 4 14 14 18 18 0
6–7 H 8 18 18 26 26 0
7–8 F 6 26 26 32 32 0

Step 5: Cash requirement schedule


Halting time In Out Balance Activity
0 8+3 -- 11 C, G
3 -- 8 3 G
4 4 3 4 I
6 4+3+3 4 10 A, D, E
7 -- 4 6 D, E
8 -- 3 3 D
11 1 3 1 J
14 2 1 2 B
18 5 2 5 H
26 5 5 5 F

Step 6: Week by week schedule of cash flow


Week 1 2 3 4 5 6 7 8 9 10 11 12 13 14 15 16
Cash 11 11 11 3 4 4 10 6 3 3 3 1 1 1 2 2
Week 17 18 19 20 21 22 23 24 25 26 27 28 29 30 31 32
Cash 2 2 5 5 5 5 5 5 5 5 5 5 5 5 5 5

Step 7: Cash flow Management

1) Availability and Requirement:

2) It is evident that we do not have enough cash to do all the activities planned in 1st 8 weeks. Hence we
should post-pone some activities to the next slot so that a cost of Rs.10,000 can be post-poned.

E M Reddy Page | 327


AMA-Notes

3) The activities we are doing in the first 8 weeks are C, G, I, A, D, E. In these activities we cannot post-
pone G, D, I because these are critical activities.
4)

Question no 10:

Solution:

(i) Simple Critical Path Method (CPM)


(ii) Critical Path Method with Crashing
(iii) Program Evaluation Revaluation Technique
(iv) Critical Path Method with resource allocation
(v) Program Evaluation Review Technique

Question no 11: A project consists of 7 activities. The time for performance of each of the activity s
as follows:
Activity Immediate Time Probability
A -- 3 0.2
4 0.6
5 0.2
B -- 4 1.0
C A 1 1.0
D B, C 4 0.8
5 0.2
E D 3 0.1
4 0.3
5 0.3
6 0.3
F D 5 0.2
7 0.8
G E, F 2 0.5
3 0.5

E M Reddy Page | 328


AMA-Notes

a) Draw a network and identify critical path using expected time.


b) Simulate the project for 5 times using random number and find the critical paths.
68 13 09 20 73 07 72
99 93 18 24 22 07 29
57 33 49 65 92 98 00
57 12 31 96 85 92 91
77 37 34 11 27 10 59

Solution:

Step 1: Network Diagram

From the above diagram we can generalize the critical path as follows:
{‘A + C’ or B} → D → {E or F} → G

Step 2: Calculation of expected time


Activity Computation Expected Time
A (0.2 x 3) + (0.6 x 4) + (0.2 x 5) 4
B 1x4 4
C 1x1 1
D (0.8 x 4) + (0.2 x 5) 4.2
E (0.1 x 3) + (0.3 x 4) + (0.3 x 5) + (0.3 x 6) 4.8
F (0.2 x 5) +(0.8 x 7) 6.6
G (0.5 x 2) + (0.5 x 3) 2.5

Critical Path = A → C → D → F → G
Duration = 4 + 4 + 1 + 4.2 + 4.8 + 6.6 + 2.5 = 18.3

E M Reddy Page | 329


AMA-Notes

Step 3: Random Number Coding


Activity Time Probability Cumulative Probability Random Numbers
A 3 0.2 0.2 00 – 19
4 0.6 0.8 20 – 79
5 0.2 1.0 80 – 99
B 4 1.0 1.0 00 – 99
C 1 1.0 1.0 00 – 99
D 4 0.8 0.8 00 – 79
5 0.2 1.0 80 – 99
E 3 0.1 0.1 00 – 99
4 0.3 0.4 10 – 39
5 0.3 0.7 40 – 69
6 0.3 1.0 70 – 99
F 5 0.2 0.2 00 – 19
7 0.8 1.0 20 – 99
G 2 0.5 0.5 00 – 49
3 0.5 1.0 50 – 99

Step 4: Simulation Worksheet


Run A B C D E F G
No. Time No. Time No. Time No. Time No. Time No. Time No. Time
1 68 4 13 4 09 1 20 4 73 6 07 5 72 3
2 99 5 93 4 18 1 24 4 22 4 07 5 29 2
3 57 4 33 4 49 1 65 4 92 6 98 7 00 2
4 57 4 12 4 31 1 96 5 85 6 92 7 91 3
5 77 4 37 4 34 1 11 4 27 4 10 5 59 3

Simulation Run Critical Path Duration


1 A→C→D→E→G 4 + 1 + 4 + 6 + 3 = 18
2 A→C→D→F→G 5 + 1 + 4 + 5 + 2 = 17
3 A→C→D→F→G 4 + 1 + 4 + 7 + 2 = 18
4 A→C→D→F→G 4 + 1 + 5 + 7 + 3 = 20
5 A→C→D→E→G 4 + 1 + 4 + 5 + 3 = 17

10.10. Network Updation

Question no 12: After 15 days of working the following progress is notes for the network of an
erection job.
a) Activity 1 – 2, 1 – 3 and 1 – 4 completed as per original schedule.
b) Activity 2 – 4 is in progress and will be completed in 3 more days.
c) Activity 3 – 6 is in progress and will need 18 days more for completion.
d) Activity 6 – 7 appears to present some problem and its new estimated time of completion is 12
days.
e) Activity 6 – 8 can be completed in 5 days instead of originally planned for 7 days.

E M Reddy Page | 330


AMA-Notes

You are required to:


(i) Update the above diagram after 15 days of the start of work based on the assumptions given
above.
(ii) Write down the critical path with total project duration.
Solution:
Paths Duration (Days)
1–2–5–7–8 9 + 18 + 8 + 6 = 41
1–2–4–7–8 9 + 9 + 20 + 6 = 44
1–4–7–8 6 + 20 + 6 = 32
1–3–4–7–8 10 + 5 + 20 + 6 = 41
1–3–6–7–8 10 + 23 + 12 + 6 = 51
1–3–6–8 10 + 23 + 5 = 38

Critical Path is ‘1 – 3 – 6 – 7 – 8’

E M Reddy Page | 331


AMA-Notes

11. TRANSFER PRICING

11.1. Introduction

1) In a decentralized form of organization, the companies will be having divisions which can be
categorized as follows:

2)

3) Division X manufactures Intermediate Product (IP) and transfers to Division Y which further process it
and sell to outsiders as final product.
4) Basically the supplying division is a cost center. However, to promote inter-divisional competition and
improve efficiency companies may treat the supplying division as profit center.
5) The transfer of intermediate product will be deemed to be sales of the supplying division and the
transfer is made at a price higher than cost which is called “Transfer Price”.
6) Transfer price is selling price to supplying division, purchase cost to receiving division and neither
selling price nor purchase cost for the company.
7) Thus transfer price will affect divisional profits but prima facie does not affect company’s profit.
8) Transfer price affects the company’s profits through output decision of divisions.

11.2. Transfer Price - Variable Cost

Question no 1: A company with two manufacturing divisions is organized on profit center basis.
Division 'A' is the only source for the supply of a component that is used in Division ‘B’ in the
manufacture of a product KLIM. One such part is used in each unit of the product KLIM. As the
demand for the product is not steady, division ‘B’ can obtain order in increased quantities only by
spending more on sale promotion and by reducing the selling prices. The Manager of Division B
has accordingly prepared the following forecast of sales quantities and selling prices.

E M Reddy Page | 332


AMA-Notes

Sales in units per day Average selling price per unit of Klim (Rs.)
1,000 5.25
2,000 3.98
3,000 3.30
4,000 2.78
5,000 2.40
6,000 2.01
The manufacturing cost of KLIM in Division ‘B’ is Rs.3,750 for first 1000 units and Rs.750 per 1000
units in excess of 1000 units. Division A incurs a total cost of Rs.500 per day for an output up to
1000 components and the total costs will increase by Rs.900 per day for every additional 1000
components manufactured.
The manager of Division A state that the operating results of his division will be optimized if the
transfer price of Component is set at Rs.1.20 per unit and he has accordingly set the aforesaid
transfer price for his supplies of the component to Division B.
Required:
A) Prepare a schedule showing the profitability at each level of output of Division A & Division B.
B) Find the Profitability of the Company as a whole at the output level at which:
I. Division A's net profit is maximum; II. Division B's net profit is maximum.
C) If the Company is not organized on profit center basis, what level of output will be chosen to
yield the maximum profit?

Solution:

Step 1: Statement of profitability – Division A (Supplying Division)


Units Sales (Rs.) Cost (Rs.) Profit(Rs.)
1,000 1,200 1,500 - 300
2,000 2,400 2,400 --
3,000 3,600 3,300 300
4,000 4,800 4,200 600
5,000 6,000 5,100 900
6,000 7,200 6,000 1,200

Step 2: Statement of profitability – Division B (Receiving Division)


Units Sales (Rs.) IP Purchase Cost (Rs.) Further Processing Cost (Rs.) Profit(Rs.)
1,000 5,250 1,200 3,750 300
2,000 7,960 2,400 4,500 1,060
3,000 9,900 3,600 5,250 1,050
4,000 11,120 4,800 6,000 320
5,000 12,000 6,000 6,750 -750
6,000 12,060 7,200 7,500 -2640

Step 3: Statement of profitability – Company


Units Sales (Rs.) IP Production Cost (Rs.) Further Processing Cost (Rs.) Profit(Rs.)
1,000 5,250 1,500 3,750 0
2,000 7,960 2,400 4,500 1,060

E M Reddy Page | 333


AMA-Notes

3,000 9,900 3,300 5,250 1,350


4,000 11,120 4,200 6,000 920
5,000 12,000 5,100 6,750 150
6,000 12,060 6,000 7,500 -1,440

Notes:

1) The profit of Division ‘A’ is maximum at an output level of 6,000 Units.


2) The profit of Division ‘B’ is maximum at an output level of 2,000 Units.
3) The Company profit is maximum at an output level of 3,000 Units.
4) Even though Division ‘A’ wants to transfer 6,000 Units, it can transfer only what the Division ‘B’ is
willing to take i.e. 2,000 Units.
5) Thus the Division ‘B’ determines the output of the company which is against the company’s optimum
output level.
6) Thus there exists a conflict in goals between the Company and Division ‘B’ which is due to Transfer
Price.
7) Can the company order the Division ‘B’ to produce 3,000 units?
Answer: No, because Divisional autonomy should be protected. No decision whether output or
transfer price should be thrust on a division.
8) A goods transfer price should ensure the following:
i. Avoid Goal Conflict
ii. Ensure Divisional Autonomy
iii. Evaluates fairly the Divisional performance
9) The Transfer Price of Rs.1.2 is incorrect because it creates Goal Conflict

Step 4: Optimum Output – Organization View


Units Revenue Further Net Net Marginal Marginal Cost of
Processing Cost Revenue Revenue Intermediary Product
1,000 5,250 3,750 1,500 -- --
2,000 7,960 4,500 3,460 1,960 900
3,000 9,900 5,250 4,650 1,190 900
4,000 11,120 6,000 5,120 470 900
5,000 12,000 6,750 5,250 130 900
6,000 12,060 7,500 4,560 - 690 900

Fixation of Transfer Price:

Since the transfer price is fixed as Rs.1.2 it resulted in Goal Conflict.

E M Reddy Page | 334


AMA-Notes

Question no 2: A company has two divisions. South division manufactures an intermediate product
for which there is no immediate external market. North division incorporates this intermediate
product into a final product, which it sells. One unit of the intermediate product is used in the
production of the final product. The expected units of the final product, which North division
estimates it can sell at various selling prices, are as follows:
Net Selling Price (Rs.) Quantity Sold (Units)
100 1,000
90 2,000
80 3,000
70 4,000
60 5,000
50 6,000
The costs of each division are as follows:
Particulars South Division (Rs.) North Division (Rs.)
Variable cost per unit 11 7
Fixed costs per annum 60,000 90,000
The transfer price is Rs.35 for the intermediate products, and is determined on a full cost-plus
basis.
You are required to:
(a) Prepare profit statements for each division and the company as a whole for the various selling
prices.
(b) State which selling price maximizes the profit of north division and the company as a whole
and comment on why the latter selling price is not selected by north division.
(c) State which transfer pricing policy will maximize the company's profit under a divisional
organization. Assume that there is no capacity constraint.
(d) State the implications of transfer pricing policy in (c) above on south division's profitability.

Solution:

Step 1: Statement of profitability – South Division (Supplying Division)


Units Sales (Rs.) Cost (Rs.) Profit(Rs.)
1,000 35,000 11,000 24,000
2,000 70,000 22,000 48,000
3,000 1,05,000 33,000 72,000
4,000 1,40,000 44,000 96,000
5,000 1,75,000 55,000 1,20,000
6,000 2,10,000 66,000 1,44,000

Step 2: Statement of profitability – North Division (Receiving Division)


Units Sales (Rs.) IP Purchase Cost (Rs.) Further Processing Cost (Rs.) Profit(Rs.)
1,000 1,00,000 35,000 7,000 58,000
2,000 1,80,000 70,000 14,000 96,000
3,000 2,40,000 1,05,000 21,000 1,14,000
4,000 2,80,000 1,40,000 28,000 1,12,000
5,000 3,00,000 1,75,000 35,000 90,000
6,000 3,00,000 2,10,000 42,000 48,000

E M Reddy Page | 335


AMA-Notes

Step 3: Statement of profitability – Company


Units Sales (Rs.) IP Production Cost (Rs.) Further Processing Cost (FPC) (Rs.) Profit(Rs.)
1,000 1,00,000 11,000 7,000 82,000
2,000 1,80,000 22,000 14,000 1,44,000
3,000 2,40,000 33,000 21,000 1,86,000
4,000 2,80,000 44,000 28,000 2,08,000
5,000 3,00,000 55,000 35,000 2,10,000
6,000 3,00,000 66,000 42,000 1,92,000

Step 4: Optimum output level – Organization view


Units Revenue (Rs.) FPC (Rs.) Net Revenue Net Marginal Revenue Marginal Cost (Rs.)
1,000 1,00,000 7,000 93,000 -- -
2,000 1,80,000 14,000 1,66,000 73,000 11,000
3,000 2,40,000 21,000 2,19,000 53,000 11,000
4,000 2,80,000 28,000 2,52,000 33,000 11,000
5,000 3,00,000 35,000 2,65,000 13,000 11,000
6,000 3,00,000 42,000 2,58,000 -7,000 11,000
The profit Can be understood as follows:
Units Computation Profit (Rs.)
1,000 82,000
2,000 82,000 + 73,000 – 11,000 1,44,000
3,000 1,44,000 + 53,000 – 11,000 1,86,000
4,000 1,86,000 + 33,000 – 11,000 2,08,000
5,000 2,08,000 + 13,000 – 11,000 2,10,000
6,000 2,10,000 – 7,000 – 11,000 1,92,000

Notes:

1) The Optimum output for the company is 5,000 units because till that output level the marginal revenue
exceeds the marginal cost.
2) For division North the statement looks similar except that the marginal cost is Rs.35,000 per 1,000 units
instead of Rs.11,000. This made North division stop at 3,000 Units output level.
3) Thus it is evident that transfer price is the reason for Goal Conflict.
4) If the transfer price is fixed as variable cost of intermediate product i.e. Rs.11, then the north &
company will select the same output level. Is this transfer price is correct?
Answer: Wrong, because the supplying division will not have any motivation to transfer since it just
recovers it’s variable cost. Therefore, transfer price should be more than Rs.11.
5) The transfer price should not exceed Rs.13. If it exceeds the north division will not be selecting 5,000
units as it’s output. For the north division to go from 4,000 units to 5,000 units can spend only
maximum Rs.13,000 towards the Intermediate Product (IP).
6) Transfer price Range:

E M Reddy Page | 336


AMA-Notes

Question no 3: Division ‘A’ of a large divisionalised organization manufactures a single


standardized product. Some of the output is sold externally whilst the remainder is transferred to
Division ‘B’ where it is a sub-assembly in the manufacture of that division's product. The unit
costs of Division A's product are as follows:
Particulars Rs.
Direct Material 4
Direct Labour 2
Direct Expense 2
Variable Manufacturing overheads 2
Fixed Manufacturing overheads 4
Selling and packaging expenses – variable 1
Total 15
Annually 10,000 units of the product are sold externally at the standard price of Rs.30.
In addition to the external sales, 5,000 units are transferred annually to Division B it an internal
transfer charge of Rs. 29 per unit. This transfer price is obtained by deducting variable selling and
packing expense from the external price since this expense is not incurred for internal transfers.
Division B incorporates the transferred in goods into a more advanced product. The unit costs of
this product are as follows:
Particulars Rs.
Transferred in item (from Division A) 29
Direct Material and components 23
Direct Labour 3
Variable overheads 12
Fixed overheads 12
Selling and packaging expenses – variable 1
Total 80
Division B's manager disagrees with the basis used to set the transfer price. He argues that the
transfers should be made at variable cost plus an agreed (minimal) mark-up since he claims that
his division is taking output that division ‘A’ would be unable to sell at the price of Rs. 30.
Partly because of this disagreement, the company's sales director has recently make a study of the
relationship between selling price and demand for each division. The resulting report contains the
following table:
Customer demand at various selling prices:

E M Reddy Page | 337


AMA-Notes

Division A:
Selling Price Rs.20 Rs.30 Rs.40
Demand 15,000 10,000 5,000
Division B:
Selling Price Rs.80 Rs.90 Rs.100
Demand 7,200 5,000 2,800
The manager of Division B claims that this study supports his case. He suggests that a transfer
price of Rs.12 would give Division ‘A’ a reasonable contribution to its fixed overheads while
allowing Division B to earn a reasonable profit. He also believes that it would lead to an increase of
output and an improvement in the overall level of company profits:
You are required:
(a) To calculate the effect that the transfer pricing system has had on the company's profits, and
(b) To establish the likely effect on profit of adopting the suggestion by the manager of Division B
of a transfer price of Rs. 12.

Solution:

Step 1: Optimum output for Division A for external Sales


Units Revenue Cost @ Rs.11 Marginal Revenue Marginal Cost
5,000 2,00,000 55,000 -- --
10,000 3,00,000 1,10,000 1,00,000 55,000
15,000 3,00,000 1,65,000 -- 55,000

Step 2: Optimum output for Division B


Units Revenue FPC @ Rs.39 Net Revenue Marginal Revenue Marginal Cost @ Rs.29
2,800 2,80,000 1,09,200 1,70,800 -- --
5,000 4,50,000 1,95,000 2,55,000 84,200 63,800
7,200 5,76,000 2,80,800 2,95,200 40,200 63,800

Step 3: Optimum output for company


Units Revenue FPC @ Rs.39 Net Revenue Marginal Revenue Marginal Cost @ Rs.29
2,800 2,80,000 1,09,200 1,70,800 -- --
5,000 4,50,000 1,95,000 2,55,000 84,200 22,000
7,200 5,76,000 2,80,800 2,95,200 40,200 22,000

Notes:

1) Reason for Division ‘A’ fixing Rs.29 as transfer price:


The Market pays Rs.30 for the intermediary product and internal transfers does not have selling cost of
Rs.1 hence Division ‘B’ should pay Rs.29.
2) Is this Rs.29 transfer justified?
No, because
a. Division ‘A’ transfers only those units which it is unable to sell in the external market at Rs.30.
b. Further this transfer price creates Goal Conflict as it makes Division ‘B’ select 5,000 units as it’s
output while for the company the optimum output is 7,200 units.

E M Reddy Page | 338


AMA-Notes

3) What should be the transfer price?


The transfer price should be within the following range:

4) Can the Rs.12 recommended transfer price be accepted?


Yes, because it falls within the range. It gives Division ‘A’ a Rs.2 contributions for the capacity which
otherwise would remain idle and for Division ‘B’ since it is within Rs.18.27 makes it select 7,200 units
output level inconformity with the company.

11.3. Transfer Price – Specific Fixed Cost

***Question no 4: A group of highly integrated divisions wishes to be advised as to how it should


set transfer prices for the following interdivisional transactions:
Division L sells all its output of product LX to Division M. To 1 Kg of LX, Division M adds other
direct materials and processes it to produce 2 Kg of product MX that it sells outside the group.
The price of MX is influenced by volume offered and the following cost and revenue data are
available:
Division L:
The variable costs of LX are (per kg) at 50,000 Kg.
Direct Materials 4.00
Direct Labour 2.00
Total 6.00
The following cost increases are expected at different levels of production per annum:
Direct Materials At 60,000 kg p.a. increases Rs.5.00 per kg
At 90,000 kg p.a. increases Rs.5.50 per kg
At 1,00,000 kg p.a. increases Rs.6.00 per kg
Direct Labour At 80,000 kg p.a. increases Rs.2.50 per kg
At 1,00,000 kg p.a. increases Rs.3.00 per kg

Fixed Overhead Under 70,000 kg 2,10,000 p.a.


70,000 – 79,999 Kg 2,60,000 p.a.
80,000 – 89,999 kg 2,80,000 p.a.
90,000 or more kg 3,10,000 p.a.
Division M:
To produce 1 kg of product MX, the following variable cost incurred for each 0.5 kg of LX used (at
1,00,000 kg of MX):

E M Reddy Page | 339


AMA-Notes

Other Direct Materials Rs.1.50


Processing Cost Rs.3.50
The following cost increases are expected at different levels of production of MX per annum:
Other Direct Materials At 1,40,000 kg p.a. increases Rs.1.75 per kg
At 1,60,000 kg p.a. increases Rs.2.00 per kg
At 1,80,000 kg p.a. increases Rs.4.00 per kg

Fixed Overhead: Under 1,20,000 Kg 2,50,000 p.a.


1,20,000 – 1,39,999 Kg 2,80,000 p.a.
1,40,000 – 1,59,999 Kg 2,90,000 p.a.
1,60,000 – 1,99,999 Kg 3,20,000 p.a.
2,00,000 or more kg 3,60,000 p.a.
Selling Price: Up to 1,99,999 kg? Rs.16.00 per kg
2,00,000 or more kg Rs.15.50 per kg
You are required to:
Recommend, with supporting calculations and explanations, the most appropriate narrow range of
transfer price per kg for product LX as between the two divisions; assume that any changes in
output are steps of 10,000 kg of product LX and 20,000 kg of product MX.

Solution:

Step 1: Calculation of ‘LX’ cost


Output Direct Material Direct Labour Fixed Overhead Total Cost
50,000 2,00,000 1,00,000 2,10,000 5,10,000
60,000 3,00,000 1,20,000 2,10,000 6,30,000
70,000 3,50,000 1,40,000 2,60,000 7,50,000
80,000 4,00,000 2,00,000 2,80,000 8,80,000
90,000 4,95,000 2,25,000 3,10,000 10,30,000
1,00,000 6,00,000 3,00,000 3,10,000 12,10,000

Step 2: Calculation of further processing cost of ‘MX’


Output Other Direct Material Processing Cost Fixed Overhead Total Cost
1,00,000 1,50,000 3,50,000 2,50,000 7,50,000
1,20,000 1,80,000 4,20,000 2,80,000 8,80,000
1,40,000 2,45,000 4,90,000 2,90,000 10,25,000
1,60,000 3,20,000 5,60,000 3,20,000 12,00,000
1,80,000 3,60,000 7,20,000 3,20,000 14,00,000
2,00,000 4,00,000 8,00,000 3,60,000 15,60,000

Step 3: Determination of Optimum Output Level


LX MX
Units Total Cost Marginal Cost Units Revenue FPC Net Revenue NMR
50,000 5,10,000 -- 1,00,000 16,00,000 7,50,000 8,50,000 --
60,000 6,30,000 1,20,000 1,20,000 19,20,000 8,80,000 10,40,000 1,90,000
70,000 7,50,000 1,20,000 1,40,000 22,40,000 10,25,000 12,15,000 1,75,000
80,000 8,80,000 1,30,000 1,60,000 25,60,000 12,00,000 13,60,000 1,45,000

E M Reddy Page | 340


AMA-Notes

90,000 10,30,000 1,50,000 1,80,000 28,80,000 14,00,000 14,80,000 1,20,000


1,00,000 12,10,000 1,80,000 2,00,000 31,00,000 15,60,000 15,40,000 60,000

The transfer price should be fixed in such a way that both the division selects the Optimum Output Level.

Step 4: Transfer Price Fixation

A. From Receiving Division’s View Point

Notes:
1) What happens if transfer price more than Rs.14.5 say Rs.15?
a) If transfer price is Rs.15, for every 10,000 Kgs of ‘LX’ Division M has to pay Rs.1,50,000.
b) The Net Marginal Revenue (NMR) exceeds Rs.1,50,000 only up to 70,000 Kgs of further
processing ‘LX’.
c) Thus anything above Rs.14.5 will make Division M stop before the optimum output level of
80,000 Kgs.
2) What happens if transfer price is below Rs.12 say Rs.11?
a) For every 10,000 Kgs of ‘LX’ the receiving division has to pay Rs.1,10,000.
b) The Net Marginal Revenue (NMR) exceeds the Marginal Cost of Rs.1,10,00 up to 90,000
Kgs of further processing ‘LX’.
c) To prevent Division ‘M’ in further processing up to 90,000 Kgs the transfer price should be
more than Rs.12.
B. Transfer Price from Supplying Division View Point

Notes:
1) What happens if transfer price is less than Rs13 say Rs.12.5?
a) When transfer price is Rs.12.5 ‘LX’ earns Rs.1,25,000 for every 10,000 Kgs transferred.
b) This revenue of Rs.1,25,000 exceeds the marginal cost only up to 70,000 Kgs. Beyond
which the cost is more than revenue.

E M Reddy Page | 341


AMA-Notes

c) To make ‘LX’ reach optimum output of 80,000 Kgs transfer price should be more than
Rs.13.
2) What happens if transfer price exceeds Rs.15 say Rs.16?
a) For every 10,000 Kgs transferred ‘LX’ earns Rs.1,60,000.
b) Up to 90,000 Kgs of transfer this Rs.1,60,000 exceeds the marginal cost.
c) To prevent ‘LX’ from crossing 80,000 units the transfer price should be less than Rs.15.
C. Narrow Range
‘MX’ Range : Rs.12 < Transfer Price < Rs.14.5
‘LX’ Range : Rs.13 < Transfer Price < Rs.15
Narrow Range : Rs.13 < Transfer Price < Rs.14.5

11.4. Linear Programming Method of Transfer Price fixation

Question no 5: Black and Brown are two divisions in a group of companies and both require
intermediate products Alpha and Beta which are available from Divisions A and B respectively.
Black and Brown divisions convert the intermediate products into products Blackalls and
Brownalls respectively. The market demand for Blackalls and Brownalls considerably exceeds the
production possible, because of the limited availability of intermediate products Alpha and Beta.
No external market exists for Alpha and Beta and no other intermediate product market is
available to Black and Brown divisions.
Other data are as follows:
Black Division:
Blackalls Selling price per unit Rs.45
Processing cost per unit Rs.12
Intermediate products required per unit:
Alpha: 3 Units
Beta: 2 Units
Brown Division:
Brownalls: Selling price per unit Rs.54
Processing cost per unit Rs.14
Intermediate products required per unit:
Alpha: 2 Units
Beta: 4 Units
A Division:
Alpha Variable cost per unit Rs.6
Maximum production capacity 1,200 Units
B Division:
Beta Variable cost per unit Rs.4
Maximum production capacity 1,600 Units
The solution to a linear programing model of the situation shows that the imputed scarcity value
(shadow price) of Alpha and Beta is Rs.0.50 and Rs.2.75 per unit respectively and indicates that the
intermediate products be transferred such that 200 units of Blackalls and 300 units of Brownalls are
produced and sold.
Required:
(a) Calculate the contribution earned by the group if the sales pattern indicated by the linear
programming model is implemented.

E M Reddy Page | 342


AMA-Notes

(b) Where the transfer prices are set on the basis variable cost plus shadow price, show detailed
calculations for:
(i) The contribution per unit of intermediate product earned y divisions A and B and
(ii) The contribution per unit of final product produced by Black and Brown divisions.
(c) Comment on the results derived in (b) and on the possible attitude of management of the
various divisions to the proposed transfer pricing and product deployment policy.
(d) In the following year the capacities of divisions A and B have each doubled and the following
changes have taken place.
(i) Alpha: There is still no external market for the product, but A division has a large
demand for other products which could use the capacity and earn contribution of 5%
over cost. Variable cost per unit for the other products would be the same as that for
Alpha and such products would use the capacity at the same rate as Alpha.
(ii) Beta: An intermediate market for this product now exists and beta can be bought and
sold in unlimited amounts at Rs.7.50 per unit. External sales of beta would incur
additional transport costs of Rs.0.50 per unit, which are not incurred in inter-division
transfers.
The market demand for Blackalls and Brownalls will still exceed the production availability of
Alpha and Beta.
(i) Calculate the transfer prices at which Alpha and Beta should now be offered to Black
and Brown divisions in order that the transfer policy implemented will lead to the
maximization of group profit.
(ii) Determine the production and sales pattern for Alpha, Beta, Blackalls and Brownalls,
which will now maximize the given contribution and calculate the group contribution
that could be achieved. It may be assumed that divisions will make decisions consistent
with the financial data available.

Solution:

Step 1: Profit per unit of Black and Brown (Company Level)


Particulars Black (Rs.) Brown (Rs.)
Selling Price 45 54
Less: Variable Cost
Alpha 18 12
Beta 8 16
Further Processing Cost 12 14
Total Variable Cost 38 42
Contribution per unit 7 12

Step 2: Formulating Situation into LPP to find out optimum output

Let X1 = Number of Units of Blackalls & X2 = Number of Units of Brownalls


Maximize Z = 7X1 + 12X2
Subject to
3X1 + 2X2 < 1,200
2X1 + 4X2 < 1,600
Where X1 & X2 > 0

E M Reddy Page | 343


AMA-Notes

Step 3: Final Solution to LPP


FR – Fixed Ratio Program Quantity X1 X2 S1 S1 RR – Replacement Ratio
X1 200 1 0
X2 300 0 1
Cj 7 12 0 0
Zj 7 12 0.5 2.75
NER (Cj − Zj ) 0 0 -0.5 -2.75

Notes:

1) Through the simplex we determined the optimum output for the company. It is 200 units of Blackalls
and 300 units of Brownalls.
2) The shadow cost of Alpha and Beta is 0.5 and 2.75. It is nothing but S1, S2 in the NER (Net Evaluation
Row) in the final simplex table where S1 reprints unused Alpha Capacity and S2 unused Beta capacity.
3) When we introduce S1 as basic variable (Plan to keep 1 unit of Alpha unused), then the profit drops by
Rs.0.50 i.e. every alpha is capable of generating Rs.0.50 profit. Same for Beta also.

Step 4: Calculation of the Division’s profitability when transfer price is equal to variable cost
Items A (Rs.) B (Rs.) Black (Rs.) Brown (Rs.)
Selling Price 6 4 45 54
Less: Variable Cost
Alpha - - 18 12
Beta - - 8 16
Processing Cost 6 4 12 14
Contribution per unit - - 7 12
Units Sold 1,200 1,600 200 300
Contribution - - 1,400 3,600

Step 5: Division’s profitability when transfer price is equal to variable cost plus Shadow Cost

Alpha = Rs.6 + Rs.0.5 = Rs.6.5


Beta = Rs.4 + Rs.2.75 = Rs.6.75
Items A (Rs.) B (Rs.) Black (Rs.) Brown (Rs.)
Selling Price 6.5 6.75 45 54
Less: Variable Cost
Alpha - - 19.5 13
Beta - - 13.5 27
Processing Cost 6 4 12 14
Contribution per unit 0.5 2.75 - -
Units Sold 1,200 1,600 200 300
Contribution 600 4,400 - -

Notes:

1) The Company’s profit of Rs.5,000 represents efforts of all 4 divisions.

E M Reddy Page | 344


AMA-Notes

2) At the first instance the profits are receiving division because they are the profit centers.
3) The profit gets transferred from receiving divisions to supplying divisions through transfer price i.e.
always transfer price acts as between divisions.
4) If we fix transfer price as variable cost of Intermediate product then no profit is transferred between
receiving and supplying divisions and entire profit is cornered by receiving divisions.
5) If transfer price = Variable Cost + Opportunity cost, then the entire profit through the opportunity cost
is transferred from receiving to supplying divisions.
6) Thus the transfer price should be between variable cost and ‘Variable Cost + Opportunity cost’

Transfer Price

Alpha Beta

Rs.6 < Transfer Price < Rs.6.5 Rs.4 < Transfer Price < Rs.6.75

Step 6: Fixation of Transfer Price for Alpha and Beta in the changed scenario

1) Capacity constraint continues and alternative uses emerges. In this case transfer price should not be a
range but a single price.
2)
Transfer Price

Alpha Beta

Variable Cost + Opportunity Cost External Market Price – Selling Cost

Rs.6 + (Rs.6 x 5%) = Rs.6.30 Rs.7.5 – Rs.0.5 = Rs.7

Variable Cost + Opportunity Cost

Rs.4 + (Rs.7.5 – Rs0.5 – Rs.4)

Rs.4 + Rs.3 = Rs.7


3) When Black and Brown uses Alpha it has the duty to recover 2 things for the company:
(i) It’s variable manufacturing cost of Rs.6

E M Reddy Page | 345


AMA-Notes

(ii) The contribution the company loses from other revenues due to use of Alpha for Black and
Brown i.e. Rs.0.30
If Black and Brown selling price is unable to recover Rs.6.3 it is better to Alpha for other purposes.
4) If we fix transfer price more than Rs.6.3 there is a risk of Goal Conflict. For example, the Black can
afford recovery up to Rs.6.32 and when transfer price is fixed as Rs.6.35 Black division may incur losses
but still from organization’s view point it is a better product compared to other uses as it gives Rs.0.32
above Rs.6. This results in Goal Conflict.
5) Similarly, when Beta is used internally the company loses external price of Rs.7.50 but saves a selling
cost of Rs.0.50 resulting in Rs.7 loss which Black and Brown should recover.
6) An interim chart for transfer price fixation:

Step 7: Profitability of Black and Brown in the revised scenario


Items Black (Rs.) Brown (Rs.)
Selling Price 45 54
Less: Variable Cost
Alpha 18.9 12.6
Beta 14 29
Processing Cost 12 14
Total Variable Cost 44.9 54.6
Contribution per unit 0.10 -0.60

Notes:

1) We should transfer Alpha & Beta for Black production because it gives Rs.0.10 extra contribution
competed to the next alternative. This can be understood as follows:
Contribution earned = Rs.7
Contribution lost:

E M Reddy Page | 346


AMA-Notes

Alpha (3 Units x Rs.0.3) = Rs.0.9


Beta (2 Units x Rs.3) = Rs.6
Incremental Contribution = Rs.0.10
2) By using Alpha and Beta for Brown we earned Rs.0.60 less than the next possible alternative which can
be understood as follows:
Contribution earned = Rs.12
Contribution lost:
Alpha (2 Units x Rs.0.3) = Rs.0.6
Beta (4 Units x Rs.3) = Rs.12
Decrease in Contribution = Rs.0.60
3) Conclusion if Black should be produced and Brown should be abandoned.

Step 8: Optimum production plan for the company


Particulars Alpha Beta
Available 2,400 Units 3,200 Units
Units Required for 1 Blackalls 3 Units 2 Units
Possible Blackalls production 800 Units 1,600 Units
Output can be produced 800 Units

Beta Balance = 3,200 Units – (800 Units x 2) = 1,600 Units. Sell this unused of 1,600 units to outside
market.
The contribution from the above plan is as follows:
Black: 800 Units x Rs.7 = Rs.5,600
Beta: 1,600 Units x Rs.3 = Rs.4,800
Profit = Rs.5,600 + Rs.4,800 = Rs.10,400

11.5. Limiting Factor and Transfer Pricing

**Question no 6: Question no 11

Solution:

Step 1: Contribution per limiting factor


Particulars A B C D
Selling Price (Rs.) 150 146 140 130
Less: Variable Cost (Rs.) 130 100 90 85
Contribution per units (Rs.0 20 46 50 45
Hours per unit 3 4 2 3
Contribution per hour (Rs.) 6.67 11.5 25 15
Rank IV III I II

Step 2: Allocation of 20,000 labour hours


Product Units Hours/Unit Hours ∑ 𝐇𝐨𝐮𝐫𝐬
C 2,300 2 4,600 4,600
D 1,600 3 4,800 9,400

E M Reddy Page | 347


AMA-Notes

B 2,500 4 10,000 19,400


A 200 3 600 600

Step 3: Allocation of 30,000 hours


Product Units Hours/Unit Hours ∑ 𝐇𝐨𝐮𝐫𝐬
C 2,300 2 4,600 4,600
D 1,600 3 4,800 9,400
B 2,500 4 10,000 19,400
A 2,800 3 8,400 27,800
Idle Capacity - - 2,200 30,000

Step 4: Transfer Price fixation when capacity is 20,000 hours

 Division Z is having capacity constraint. It does not have enough hours to manufacture units both for
external sales and internal transfer.
 Producing 2,500 units of D for internal transfer requires 2,500 Units x 3 Hours = 7,500 Hours.
 These 7,500 hours can be obtained only by releasing 600 hours from A and 6,900 hours from B.
 Transfer price = Variable Cost + Opportunity Cost of Department Z.
Variable Cost 2,500 Units x Rs.85 2,12,500
Add: Opportunity Cost
A 600 Hours x Rs.6.67 4,000
B 6,900 Hours x Rs.11.5 79,350
Total Cost 2,95,850
Rs.2,95,850
Transfer price = = Rs.118.34/Unit
2,500 Units

Step 5: Transfer price fixation when capacity is 30,000 hours

 Division Z is having capacity constraint. It does not have enough hours to manufacture units both for
external sales and internal transfer.
 Producing 2,500 units of D for internal transfer requires 2,500 Units x 3 Hours = 7,500 Hours.
 These 7,500 hours can be obtained only by using 2,200 hours of idle time and releasing 5,300 hours
from A.
 Opportunity cost of idle time is nil and balance 5,300 hours, it is the contribution lost from product A.
 Transfer price = Variable Cost + Opportunity Cost of Department Z.
Variable Cost 2,500 Units x Rs.85 2,12,500
Add: Opportunity Cost
Idle 2,200 Hours x Rs.0 0
A 5,300 Hours x Rs.6.67 35,350
Total Cost 2,47,850
Rs.2,47,850
Transfer price = 2,500 Units = Rs.99.14/Unit

Notes:

E M Reddy Page | 348


AMA-Notes

1) When supplying division is a single product division having capacity constraint the transfer price is
“Variable Cost + Opportunity Cost of Intermediate Product”.
2) If the supplying division is a multi-product division having capacity constraint, then transfer price is
equal to “Variable Cost + Opportunity Cost of Supplying Division”. In this problem for internally
transferring ‘D’ we don’t lose D’s external demand but lose the demand of Demand of ‘A’ & ‘B’ based
on limiting factor ranking. Hence it is not D’s contribution lost but supplying division contribution lost
which is the opportunity cost.
3) The Intermediate Product can be purchased at Rs.125. Here we can have 2 situations:
(i) “Variable Cost + Opportunity Cost” > Rs.125 – Do not Transfer as it cheaper to Buy. Hence
no need to fix transfer price.
(ii) “Variable Cost + Opportunity Cost” < Rs.125 – Transfer is good from the company’s view
point. Ensure that transfer price is not greater than Rs.125 as it will make receiving division
purchase from outside creating Goal Conflict.

Question no 7: Question no 12

Solution:

Step 1: Calculation of contribution per unit


Items X Y Z
Selling Price (Rs.) 800 160 1,450
Less: Variable Cost (Rs.) 620 140 1,320
Contribution per unit (Rs.) 180 20 130
Hours Per unit 4 1 --
Contribution per hour 45 20 --
Rank I II --

Step 2: Allocation of 96,000 Hours


Product Units Hours/Unit Hours ∑ 𝐇𝐨𝐮𝐫𝐬
X 15,000 4 60,000 60,000
Y 36,000 1 36,000 96,000

Step 3: Calculation of Divisional profit and Company profit when component imported

A. Division A Profit
Particulars Computation Amount (Rs.)
Contribution from ‘X’ 15,000 Units x Rs.180 27,00,000
Contribution from ‘Y’ 36,000 Units x Rs.20 7,20,000
Total Contribution 34,20,000
Less: Fixed Cost (30,00,000)
Profit 4,20,000

B. Division B Profit
Particulars Computation Amount (Rs.)
Contribution from ‘Z’ 5,000 Units x Rs.130 6,50,000

E M Reddy Page | 349


AMA-Notes

Less: Fixed Cost (5,00,000)


Profit 1,50,000

C. Company Profit

Company Profit = Rs.4,20,000 + Rs.1,50,000 = Rs.5,70,000

Step 4: Calculation of Divisional profit and Company profit when division B purchases at Rs.800 from
division A

A. Division A Profit
Particulars Computation Amount (Rs.)
Contribution from ‘X’ 20,000 Units x Rs.180 36,00,000
Contribution from ‘Y’ 16,000 Units x Rs.20 3,20,000
Total Contribution 39,20,000
Less: Fixed Cost (30,00,000)
Profit 9,20,000

B. Division B Profit
Particulars Computation Amount (Rs.)
Sales 5,000 Units x Rs.1,450 72,50,000
Less: Variable Cost (Excluding transfer price) 5,000 Units x Rs.520 26,00,000
Less: Transfer Price 5,000 Units x Rs.800 40,00,000
Less: Modification Cost 5,000 Units x Rs.80 4,00,000
Contribution 2,50,000
Less: Fixed Cost (5,00,000)
Profit/(Loss) (2,50,000)

C. Company Profit

Company Profit = Rs.9,20,000 – Rs.2,50,000 = Rs.6,70,000

Notes:

1) The import substitution increases the company’s profits by Rs.1,00,000. Hence, it is a good decision.
2) How the company profit increased by Rs.1,00,000?
Cost of ‘X’ manufactured and transferred:
Variable Cost = Rs.620
Opportunity cost of Division A = Rs.80 (20,000 Units x Rs.20/5,000 Units)
Modification Cost = Rs.80
Total Cost = Rs.780
Cost of Import = Rs.800
Savings per unit = Rs.20
Increase in profit (20,000 Units x Rs.20) = Rs.1,00,000
3) Import substitution decrease division B profit by Rs.4,00,000 (Rs.1,50,000 – (-Rs.2,50,000). Why the
profit has decreased?
a) Transfer price paid to division A = Rs.800

E M Reddy Page | 350


AMA-Notes

Modification Cost = Rs.80


Cost of Using ‘X’ = Rs.880
Imported Cost = Rs.800
Extra cost per unit = Rs.80
Decrease in profit (5,000 Units x Rs.80) = Rs.4,00,000
4) Is the transfer price of Rs.800 correct?
Answer: No, because it creates Goal Conflict. Company decides to substitutes import while division B
prefers imports.

Step 5: Calculation of Divisional profits and company profit when division B purchase @ Rs.720 from
division A

A. Division A Profit
Particulars Computation Amount (Rs.)
Contribution from ‘X’ [15,000 Units x Rs.180] + 32,00,000
[5,000 Units x Rs.100]
Contribution from ‘Y’ 16,000 Units x Rs.20 3,20,000
Total Contribution 35,20,000
Less: Fixed Cost (30,00,000)
Profit 5,20,000

B. Division B Profit
Particulars Computation Amount (Rs.)
Sales 5,000 Units x Rs.1,450 72,50,000
Less: Variable Cost (Excluding transfer price) 5,000 Units x Rs.520 26,00,000
Less: Transfer Price 5,000 Units x Rs.720 36,00,000
Less: Modification Cost 5,000 Units x Rs.80 4,00,000
Contribution 6,50,000
Less: Fixed Cost (5,00,000)
Profit/(Loss) 1,50,000

C. Company Profit

Company Profit = Rs.5,20,000 + Rs.1,50,000 = Rs.6,70,000

Notes:

1) Transfer price of Rs.720 is also wrong because the incremental profit due to import substitution is fully
cornered by division A.
2) Division B manager may feel that it can earn the same Rs.1,50,000 profit by importing and avoid
modification work.
3) The transfer price should be fixed as follows:

E M Reddy Page | 351


AMA-Notes

Transfer Price

Minimum Maximum

VC + OC of Supplying Division Purchase Cost – Modification Cost

Rs.620 + Rs.80 = Rs.700 Rs.800 – Rs.80 = Rs.720

11.6. Multi-Division range fixations

***Question no 8: Question no 13

Solution:

Step 1: Transfer Price fixation for Product ‘X’

Capacity 20,000 Units

External Demand Idle Capacity – 6,000


– 14,000 Units Units

TP = EMP – SC Min TP = VC
110 – 15 = 95 Rs.40 + Rs.30 = Rs.70

No Transfer Max TP = EP –
Delivery Cost
Rs.85 – Rs.6 = Rs.79

Rs.70 < TP < Rs.79

Notes:

1) When external demand of ‘X’ is sacrificed and transfer made the company loses Rs.110 selling price but
saves Rs.15 agent’s commission. The net loss to the company is Rs.95.
2) If the company purchases the ‘X’ from outside it has to pay only Rs.85 purchase cost. It is better to pay
Rs.85 than lose Rs.95. Hence we should not transfer by sacrificing external demand. When there is no
transfer no need to fix transfer price.
3) The real cost of transferring is not Rs.95 but Rs.101 i.e. we lose Rs.95 and also incurring Rs.6 pickup
cost. Buying is cheaper as long as it is less than Rs.101.

E M Reddy Page | 352


AMA-Notes

4) Regarding idle capacity there is no opportunity cost. The transfer price should minimum recover the
variable cost of intermediate product.
5) Transfer price should not exceed Rs.79 (Rs.85 – Rs.6) because in that case division Y will prefer to buy
from outside which creates Goal Conflict.
6) The Goal Conflict is because in company’s interest using idle capacity costs only Rs.76 but buying cost
is Rs.85. Company would like to transfer while division Y would like to purchase if Transfer price
exceeds Rs.79.

Step 2: Transfer Price fixation for product Y


Particulars When TP = Rs.70 When TP = Rs.79
Product X Rs.70 Rs.79
Pick up cost of X Rs.6 Rs.6
Labour Rs.50 Rs.50
Delivery Cost – Internal Rs.8 Rs.8
Total Variable Cost for Internal transfer Rs.134 Rs.143

Capacity 30,000 Units

External Demand Idle Capacity – 4,000


– 26,000 Units Units

TP = EMP – SC – Min TP = VC
Extra Delivery Cost = Rs.134
170 – 15 – 2 = 153
Max TP = EP
= Rs.135
No Transfer

Rs.134 < TP < Rs.135

Notes:

1) When we sacrifice external demand of ‘Y’ and transfer to ‘Z’ the company loses a selling price of Rs.170
and saves & extra delivery cost of Rs.15 & Rs.2 (Rs.10 – Rs.8) respectively thereby losing net Rs.153.
2) It is better for the company to buy ‘Y’ from outside at Rs.135 rather than lose Rs.153 on internal
transfer. Thus no transfer should be made by sacrificing external demand. Since no transfer made, no
need to fix transfer price.
3) In company’s interest idle capacity should be used and transfer should be made and the variable cost of
‘Y’ is Rs.134 which should be minimum transfer price.
4) Any transfer price above Rs.135 will make division ‘Z’ purchase ‘Y’ which creates Goal Conflict.
5) Hence transfer price for ‘Y’ should be between Rs.134 and Rs.135. To have this range ‘X’ transfer price
should be between Rs.70 and Rs.71 (Rs.135 – Rs.8 – Rs.50 – Rs.6)

E M Reddy Page | 353


AMA-Notes

6) Produce 6,000 units of ‘X’ using the idle capacity and transfer to ‘Y’ at a transfer price of Rs.70 to Rs.71
and produce 4,000 units of ‘Y’ using idle capacity and transfer to ‘Z’ at a transfer price of Rs.134 to
Rs.135.
7) It should be observed that manufacturing ‘Y’ and transferring to ‘Z’ is feasible only when ‘X’ is
manufactured and transferred to ‘Y’. Buying ‘X’ and manufacturing ‘Y’ costs the company Rs.143 which
is more than Y’s buying cost of Rs.135.
8) We can conclude that Y to Z transfer can happen only for so many units transferred from ‘X’ to ‘Y’.
Suppose ‘X’ idle capacity is 4,000 units and ‘Y’ idle capacity 6,000 units, in this case ‘Y’ to ‘Z’ we can
transfer only 4,000 units keeping 2,000 units idle.

11.7. Lump Sum Consideration Method

Question no 9: Question no 4

Solution:

Step 1: Calculation of ROCE before recommendation


Particulars Division A Division B
A. Revenues
External Sales 30,00,000 (1,00,000 Units x Rs.30) 12,50,000 (25,000 Units x Rs.50)
Internal Transfer 7,50,000 (25,000 Units x Rs.30) --
Total Revenues 37,50,000 12,50,000
B. Cost
Own Variable Cost 18,75,000 (1,25,000 Units x Rs.15) 2,50,000 (25,000 Units x Rs.10)
Own Fixed Cost 5,00,000 2,25,000
Transfer Price -- 7,50,000 (25,000 Units x Rs.30)
Total Cost 23,75,000 12,25,000
C. Profit (A – B) 13,75,000 25,000
D. Capital Employed 66,25,000 12,50,000
E. ROCE (C/D) 20.75% 2%

Notes:

1) Division B reports a very low ROCE which it blames on transfer price.


2) Division A states that transfer price of Rs.30 is market determined, hence correct. Is this correct?
Answer: No, because A transfers only those units which it is unable to sell in outside market at Rs.30.
3) The transfer price should recover variable cost of intermediary product and contributes some towards
fixed cost.
4) Fixed Cost of Division A = Rs.5,00,000
1,00,000
a) Share attributable to outsiders – Rs.4,00,000 (Rs.5,00,000 x 1,25,000)
20,000
b) Share attributable to Division ‘B’ – Rs.1,00,000 (Rs.5,00,000 x 1,25,000)
5) Transfer price = Rs.15 per unit + A lump sum of Rs.1,00,000.

Step 2: Calculation of ROCE after recommendation


Particulars Division A Division B

E M Reddy Page | 354


AMA-Notes

A. Revenues
External Sales 30,00,000 (1,00,000 Units x Rs.30) 12,50,000 (25,000 Units x Rs.50)
Internal Transfer 4,75,000 (25,000 Units x Rs.15 + Rs.1,00,000) --
Total Revenues 34,75,000 12,50,000
B. Cost
Own Variable Cost 18,75,000 (1,25,000 Units x Rs.15) 2,50,000 (25,000 Units x Rs.10)
Own Fixed Cost 5,00,000 2,25,000
Transfer Price -- 4,75,000
Total Cost 23,75,000 9,50,000
C. Profit (A – B) 11,00,000 3,00,000
D. Capital Employed 66,25,000 12,50,000
E. ROCE (C/D) 16.60% 24%

Notes:

1) This method is good it contributes Rs.1,00,000 to division A for using it’s idle capacity and also
improves ROCE of division B.
2) If the division B actually creates a demand of 30,000 units should the lump sum consideration increase?
If the demand falls to 10,000 units should the lump sum consideration decrease?
Answer: In both the cases Rs.1,00,000 should not change. ‘B’ has paid Rs.1,00,000 for capacity of
25,000 units. If it overuses the capacity, it is favourable fixed overhead volume variable and if it under
uses the capacity it results in adverse fixed overhead volume variance. When we change the
consideration then it becomes variable.
3) If the demand for B exceeds 30,000 units, then capacity constraint arises in division A. For those excess
units transfer price should be ‘Variable Cost + Opportunity Cost’ or External Market Price of Rs.30 in
this case.

*****Question no 10: Question no 9

Solution:
Eshwar (Rs. in ‘000s) Brahma (Rs. in ‘000s)
Units Revenue Cost MR MC Revenue FPC NR NMR
100 204 115 204 115 703 570 133 133
200 362 185 158 70 1,375 1,120 255 122
300 486 261 124 76 2,036 1,670 366 111
400 598 344 112 83 2,676 2,220 456 90
500 703 435 105 91 3,305 2,770 535 79
600 803 535 100 100 3,923 3,320 603 68
700 898 645 95 110 4,530 3,870 660 57
800 988 766 90 121 5,126 4,420 706 46

Units MR NMR MC Yes/No Internal/External


100 204 133 115 Y E
200 158 133 70 Y E
300 124 133 76 Y I
400 124 122 83 Y E

E M Reddy Page | 355


AMA-Notes

500 112 122 91 Y I


600 112 111 100 Y E
700 105 111 110 Y I
800 105 90 121 N --

Conclusion: Produce 700 engines and sell as engines 400 & sell as dories 300. Optimum output for brahma
300 units, Eshwar 700 units.

Fixation of transfer price:

A. Receiving Division Range (Brahma):

B. Supplying Division Range (Eshwar):

C. Narrow Range:

Receiving Division: Rs.900 < Transfer Price < Rs.1,110


Receiving Division: Rs.1,100 < Transfer Price < Rs.1,210
Narrow Range: Rs.1,100 < Transfer Price < Rs.1,110

Question no 11: Question no 5

Solution:

Step 1: Real Cost of Company B

E M Reddy Page | 356


AMA-Notes

Step 2: Real Cost of ‘RS’

Real Cost of RS quotation = Rs.3,000 + Rs.11,000 + Rs.9,100 + Rs.6,300 = Rs.29,300

E M Reddy Page | 357


AMA-Notes

Conclusion: RT should purchase it’s component from RS in the group interest. The transfer price should
be fixed in such a way that there is no Goal Conflict.

Working Note 1: Own Cost of RS for Company B Contract

RS wants to earn 25% profit on it’s own cost.


Own Cost = Rs.5,500/125 x 100 = Rs.4,400
Profit = Rs.5,500 x 25/125 = Rs.1,100

Cost of Company B Contract = Rs.22,000 + Rs.,7,500 + Rs.3,080 = Rs.32,580

Working Note 2: Calculation of profit in RT’s price

Profit = 20% on total Cost.


Selling Price = 100% + 20% (100 x 20%) = 120%
Profit = 30,000 x 20/120 = Rs.5,000
Own Cost = Rs.19,000 – Rs.5,000 = Rs.4,000

Working Note 3: Calculation of Own Cost and Profit of ‘RS’

Transfer Price = Rs.48,000 – Total Cost


= Rs.48,000 – Rs.42,000
= Rs.6,000
Own Cost = Total Cost – Cost of Bought in Components from RR & RT
= Rs.42,000 – Rs.33,000
= Rs.9,000

11.8. Transfer Pricing Fixation Chart

1)

2)

3)

E M Reddy Page | 358


AMA-Notes

E M Reddy Page | 359


AMA-Notes

12. PRICING

12.1. Pricing Using Calculus

Question no 1: Question no 1

Solution:

Step 1: Current Period Cost at Previous period price level

Current Period Cost = Rs.1077.44


Increase in price level during the period = 4%
Current period cost at previous period price level = 1077.44/1.04 = 1036

Step 2: Segregation into variable and fixed at previous period price level
Particulars Previous Year Current Year
Units (‘000) 100 106
Cost (‘000) 1000 1036

Variable Cost = Change in Cost/Change in units


= (1036 – 1000)/(106 – 100) = 6 per unit
Fixed Cost = Total Cost – Variable Cost
= 1000 – (6 x 100) = 400

Step 3: Variable and fixed cost at next year price level


Variable Cost = 6 x 104% x 106% = 6.6144 per unit
Fixed Cost = 400 x 104% x 106% = 440.96
Part A: Budgeted position when Selling price is Rs.13
Particulars Computation Amount
Sales 1,06,000 Units x 109% x Rs.13 Rs.15,02,020
Less: Variable Cost 1,06,000 Units x 109% x Rs.6.6144 Rs.7,64,228
Contribution Rs.7,37,792
Less: Fixed Cost Rs.4,40,960
Profit Rs.2,96,832

Part B: Budgeted position when Selling price is Rs.13.78 (Increased by 6%)


Particulars Computation Amount
Sales 1,06,000 Units x Rs.13 Rs.14,60,680
Less: Variable Cost 1,06,000 Units x Rs.6.6144 Rs.7,01,126
Contribution Rs.7,59,554
Less: Fixed Cost Rs.4,40,960
Profit Rs.3,18,594

Part C:

E M Reddy Page | 360


AMA-Notes

The option of increase in selling price is recommended because it gives the highest budget profit. In the
option 1, keeping selling price constant helps to increase the volume but the volume increase also increases
the variable cost.

Part D: Assumptions

1) Price is the sole factor influencing the demand.


2) The price elasticity is accurately estimated.
3) Both variable and fixed cost has the same rate of inflation.
4) Decision is made purely on the basis of economic factors.
Question no 2: Question no 2
Solution:
Part A: Calculation of Profit at Current Selling Price
Particulars Computation Amount (Rs.)
Sales 50 Units x Rs.150 7,500
Less: Cost (Rs.0.5 x 502) + (Rs.20 x 50 Units) + Rs.4,000 6,250
Profit 1,250

Part B: Optimum Output and Best Selling Price

Step 1: Revenue & Cost Functions

Selling Price = 300 – 3Q


Revenue = Units Sold x Selling Price = Q x (300 – 3Q) = 300Q – 3Q2
Cost = 0.5Q2 + 20Q + 4,000 (Given)

Step 2: Marginal Revenue & Marginal Cost


dR dR
Marginal Revenue (dQ) = dQ (300Q − 3Q2 ) = 300 – 6Q
dC dC
Marginal Cost (dQ) = dQ (0.5Q2 + 20Q + 4,000) = Q + 20

Step 3: Optimum Output

Optimum Output is the level at which marginal revenue equals marginal cost.

300 – 6Q = Q + 20 → 7Q = 280 → Q = 40 Units

Step 4: Best Selling Price

Selling Price = 300 – 3Q → 300 – (3 x 40) = Rs.180

Step 5: Profit at Optimum Output


Particulars Computation Amount (Rs.)
Sales 40 Units x Rs.180 7,200

E M Reddy Page | 361


AMA-Notes

Less: Cost (Rs.0.5 x 402) + (Rs.20 x 40 Units) + Rs.4,000 5,600


Profit 1,600

Notes:

1) Understanding the first derivative:


a) Variable Cost = Rs.5 per unit and Fixed Cost = Rs.10,000. The Cost function is Y = 5X +
10,000.
dY dY
b) = 5X 0 = 5. This means when ‘X’ changes by ‘1’ unit the ‘Y’ changes by Rs.5. The dX means
dX
change in ‘Y’ with respect to ‘X’.
2) When price is elastic to demand then the concept of optimum output will emerge.
3) The optimum output is the output at which marginal revenue equals marginal cost.
4) The first step is to construct a price function. Selling Price = Selling Price at ‘0’ demand – Elasticity
times quantity.
5) Next step is to construct revenue function by multiplying price with quantity.
6) Differentiate with respect to quantity which is called Marginal Revenue and cost with respect to quantity
which is called ‘Marginal Cost.
7) Equate Marginal Revenue and Marginal Cost to identify optimum output.
8) Determine the best selling price using the optimum output.

***Question no 3: Question no 3

Solution:

Part A: Optimum Output and Best Selling Price when there is no tax

Step 1: Revenue & Cost function

Selling Price = 157 – 3X


Revenue = (157 – 3X) X = 157X - 3X 2
Cost = 1064 + 5X + 0.04X 2

Step 2: Marginal Revenue & Marginal Cost


dR dR
Marginal Revenue (dQ) = dQ (157X − 3X 2 ) = 157 – 6X
dC dC
Marginal Cost (dQ) = dQ (1064 + 5X + 0.04X 2 ) = 5 + 0.08X

Step 3: Optimum Output

Optimum Output is the level at which marginal revenue equals marginal cost.

157 – 6X = 5 + 0.08X → 6.08X = 152 → X = 25 Units

Step 4: Fixation of Selling Price

Selling Price = 157 – 3X = 157 – 3 x 25 = 82 i.e. Optimum selling price is Rs.82,000.

E M Reddy Page | 362


AMA-Notes

Step 5: Maximum Profit


Particulars Computation Amount (Rs.)(‘000s)
Sales 25 Units x 82 2,050
Less: Cost 1,064 + (5x 25 Units) + (0.04 x 252) 1,214
Profit 836

The profit is Rs.8,36,000

Part B: Optimum Output and Selling Price with tax

Step 1: Revenue & Cost function

Revenue function will not change but the cost function changes as follows:

Selling Price = 157 – 3X


Revenue = (157 – 3X) X = 157X - 3X 2
Cost = 1064 + 5X + 0.04X 2 + tX

Step 2: Marginal Revenue & Marginal Cost


dR dR
Marginal Revenue (dQ) = dQ (157X − 3X 2 ) = 157 – 6X
dC dC
Marginal Cost (dQ) = dQ (1064 + 5X + 0.04X 2 + tx) = 5 + 0.08X + t

Step 3: Optimum Output

Optimum Output is the level at which marginal revenue equals marginal cost.

157 – 6X = 5 + 0.08X + t → 6.08X = 152 – t → X = 25 – 0.1645t

Step 4: Fixation of Selling Price

Selling Price = 157 – 3X = 157 – 3 x (25 – 0.1645t) = 157 – 75 + 0.4935t → 82 + 0.4935t

Part C:

Without tax the selling price is Rs.82 and when we have tax we increase the selling price by Rs.0.4935t
which means 49.35% of tax is passed on to the customer.

Part D: Profit when tax is Rs.4,000

Step 1: Revenue & Cost function

Selling Price = 157 – 3X


Revenue = (157 – 3X) X = 157X - 3X 2
Cost = 1064 + 5X + 0.04X 2 + 4X

Step 2: Marginal Revenue & Marginal Cost

E M Reddy Page | 363


AMA-Notes

dR dR
Marginal Revenue (dQ) = dQ (157X − 3X 2 ) = 157 – 6X
dC dC
Marginal Cost (dQ) = dQ (1064 + 5X + 0.04X 2 + 4x) = 5 + 0.08X + 4 = 9 + 0.08X

Step 3: Optimum Output

Optimum Output is the level at which marginal revenue equals marginal cost.

157 – 6X = 9 + 0.08X → 6.08X = 148 → X = 24.34

Step 4: Fixation of Selling Price

Selling Price = 157 – 3X = 157 – 3 x (24.34) = 157 – 73 = Rs.84

Step 5: Maximum Profit


Particulars Computation Amount (Rs.)(‘000s)
Sales 24.34 Units x 84 2,044.56
Less: Cost 1,064 + (5 x 24.34 Units) + (0.04 x 24.342) + (4 x 24.34) 1306.76
Profit 737.8

The profit is Rs.7,37,800

12.2. Pricing Under Uncertainty and Expected Value of Perfect Information (EVPI)

Question no 4: Question no 8, pg. no.61

Solution:

Step 1: Calculation of Material Cost under all Options


Demand Selling Price = Rs.15 Selling Price = Rs.20
Option 1 Option 2 Option 3 Option 4 Option 5 Option 6
Optimistic (Rs.) 3,24,000 2,97,500 2,70,000 2,52,000 2,31,000 2,10,000
Most Likely (Rs.) 2,52,000 2,31,000 2,10,000 2,07,000 1,89,750 1,74,000(W-3)
Pessimistic (Rs.) 1,62,000 1,48,500 1,59,000 (W-1) 1,17,000 1,26,500 (W-2) 1,44,000 (W-4)

WN – 1: Material Cost when selling price is Rs.15, demand is Pessimistic and Purchase Price is Rs.2.5

Consumption of Materials (18,000 Units x 3 Kgs) = 54,000 Kgs


Purchase Cost (70,000 Kgs x Rs.2.5) = Rs.1,75,000
Less: Realizable value of unused materials (16,000 x 1) = Rs.16,000
Net Material Cost (Rs.1,75,000 – Rs.16,000) = Rs.1,59,000

WN – 2: Material Cost when selling price is Rs.20, demand is Pessimistic and Purchase Price is Rs.2.75

Consumption of Materials (13,000 Units x 3 Kgs) = 39,000 Kgs


Purchase Cost (39,000 Kgs x Rs.2.75) = Rs.1,37,500

E M Reddy Page | 364


AMA-Notes

Less: Realizable value of unused materials (11,000 x 1) = Rs.11,000


Net Material Cost (Rs.1,37,500 – Rs.11,000) = Rs.1,26,500

WN – 3: Material Cost when selling price is Rs.20 and demand is Most likely and Purchase Price is Rs.2.5

Consumption of Materials (23,000 Units x 3 Kgs) = 69,000 Kgs


Purchase Cost (70,000 Kgs x Rs.2.5) = Rs.1,75,000
Less: Realizable value of unused materials (1,000 x 1) = Rs.1,000
Net Material Cost (Rs.1,75,000 – Rs.1,000) = Rs.1,74000

WN – 4: Material Cost when selling price is Rs.20 and demand is Pessimistic and Purchase Price is Rs.2.5

Consumption of Materials (13,000 Units x 3 Kgs) = 39,000 Kgs


Purchase Cost (70,000 Kgs x Rs.2.5) = Rs.1,75,000
Less: Realizable value of unused materials (31,000 x 1) = Rs.31,000
Net Material Cost (Rs.1,75,000 – Rs.31,000) = Rs.1,44,000

Step 2: Profit Table


Demand Probability
Selling Price = Rs.15 Selling Price = Rs.20
Option 1 Option 2 Option 3 Option 4 Option 5 Option 6
Optimistic (Rs.) 0.3 83,000 1,10,000 1,37,000 1,28,000 1,49,000 1,70,000
Most Likely (Rs.) 0.5 59,000 80,000 1,01,000 88,000 1,05,250 1,21,000
Pessimistic (Rs.) 0.2 29,000 42,500 32,000 8,000 -1,500 -19,000
Expected Profit (Weighted Avg.) 60,200 81,500 98,000 84,000 97,025 1,07,700

Conclusion: Option 6 gives highest expected profit. Hence select option 6 and fix selling price as Rs.20 and
negotiate for a purchase price of Rs.2.5 per kg with a minimum 70 Kgs quantity.

Step 3: Expected Value of Perfect Information

1) Expected Profit with perfect information:


Optimistic = Rs.1,70,000 x 0.3
Most Likely = Rs.1,21,000 x 0.50
Pessimistic = Rs.42,500 x 0.20
Expected Profit = Rs.1,20,000
2) Expected Profit Without Perfect Information is Rs.1,07,700
3) Expected Value of perfect Information (1 – 2) = Rs.1,20,000 – Rs.1,07,700 = Rs.12,300

This is the price we can pay to get perfect information about the demand.

Question no 5: Question no 6, pg. no.61

Solution:

Step 1: Identification of Maximum production part A

E M Reddy Page | 365


AMA-Notes

Line S = 4000 Hours/0.6 = 6,666 Units


Line T = 4,500 Hours/0.5 = 9,000 Units
Materials = 13,000 Kilos/1.6 = 8,125 Units

The company can produce 6,666 units of part A and the limiting factor is Line S.

Step 2: Identification of Maximum production part A

Line S = 4000 Hours/0.25 = 16,000 Units


Line T = 4,500 Hours/0.55 = 8,181 Units
Materials = 13,000 Kilos/1.6 = 8,125 Units

The company can produce 8,125 units of part B and the limiting factor is Materials.

Part A: Which part to Manufacture


Particulars Computation Amount (Rs.)
Part A:
Sales 6,666 Units x Rs.145 9,66,570
- Materials 6,666 Units x 1.6 Kgs x Rs.12.5 -1,33,320
-Line S 6,666 Units x 0.6 Hours x Rs.80 -3,19,968
-Line T 6,666 Units x 0.5 Hours x Rs.100 -3,33,300
Contribution 1,79,982
Part B:
Sales 8,125 Units x Rs.115 9,34,375
-Materials 8,125 Units x 1.6 Kgs x Rs.12.5 -1,62,500
-Line S 8,125 Units x 0.25 Hours x Rs.80 -1,62,500
-Line T 8,125 Units x 0.55 Hours x Rs.100 -4,46,875
Contribution 1,62,500

Part A should be manufactured as it gives highest contribution.

Part B:

The company earns the maximum profit of Rs.1,79,982 and it could not meet the maximum call off unit
because of non-availability of Line S hours.

Part C:
Particulars Computation Amount (Rs.)
Part A:
Sales Rs.9,66,570 x 90% 8,69,913
-Cost Rs.1,33,320+Rs.3,19,968+Rs.3,33,300 -7,86588
Contribution 83,325
+ Payment for unused hours Line S: Nil
Line T: [4,500 – 6,666 x 0.5] x Rs.60 70,020
Total Contribution 1,53,345
Part B:
Sales Rs.9,34,375 x 90% 8,40,937

E M Reddy Page | 366


AMA-Notes

-Cost Rs.1,62,500+ Rs.1,62,500+Rs.4,46,875 -7,71,875


Contribution
+ Payment for unused hours Line S: [4,000 – 8,125 x 0.25] x Rs.60 1,18,125
Line T: [4,500 – 8,125 x 0.55] x Rs.60 1,875
Contribution 1,89,062
In case of this offer the company should manufacture and sell Part B.

12.3. ROI (Return on Investment) Pricing

Question no 6: Question no 10

Solution:

Step 1: Cost of Production per unit of Product M


Particulars Computation Amount (Rs.)
Direct Materials Dept A: 4 x Rs.6 = Rs.24 44
Dept B: 8 x Rs.2.5 = Rs.20
Direct Labour Dept A: 2 x Rs.4 = Rs.8 17
Dept B: 3 x Rs.3 = Rs.9
Variable Overhead Dept A: Rs.24 x Rs.0.8 = Rs.19.2 25.20
Dept B: 3 Hours x Rs.2 = Rs.6
Total Variable Cost 86.20
Fixed Overheads Dept A: Rs.24 x Rs.2.2 = Rs.19.2
Dept B: 3 Hours x Rs.2 = Rs.6

Question no 7: Question no 11

Solution:

Question no 8: Question no 14

Solution:

E M Reddy Page | 367


AMA-Notes

13. DEVELOPMENTS IN COST ACCOUNTING (STRAGETIC COST MANAGEMENT

13.1. Learning Objectives

1) Throughput Costing
2) Theory of Constraints (TOC)
3) Just-In-Time System (JIT)
4) Backflush Costing System
5) Total Quality Management (TQM)
6) Life Cycle Costing (LCC)
7) Target Costing
8) Activity Based Costing (ABC)
9) Value Chain Analysis
10) Balance Score Card

13.2. Through Put Costing

Question no 1: Emerson corporations, which uses throughput costing, just completed its first year
of operations, planned and actual production equaled 10,000 units, and sales totaled 9,600 units at
Rs.72 per unit. Cost data for the year are as follows:
Direct Material (per unit) Rs.12
Conversion Cost:
Direct Labour Rs.45,000
Variable Manufacturing Overhead Rs.65,000
Fixed Manufacturing Overhead Rs.2,20,000
Selling and administrative costs:
Variable (per unit) Rs.8
Fixed Rs.1,18,000
The company classified only direct material as a throughput cost. Required:
1. Compute the company’s total cost for the year assuming that variable manufacturing costs are
driven by the number of units produced, and variable selling and administrative costs are
driven by the number of units sold.
2. How much of this costs would be held in year-end inventory under (a) absorption costing (b)
Variable Costing, and (c) Throughput costing
3. Prepare Emerson’s throughput costing income statement.
Solution:
Part 1: Calculation of total cost
Particulars Computation Amount (Rs.)
Direct Materials 10,000 Units x Rs.12 1,20,000
Direct Wages Given 45,000
Variable Manufacturing Overhead Given 65,000
Fixed Manufacturing Overhead Given 2,20,000
Variable Selling Overheads 9,600 Units x Rs.8 76,800
Fixed Selling Overheads Given 1,18,000
Total Cost 6,44,800

E M Reddy Page | 368


AMA-Notes

Part 2: Stock Valuation under three systems


Items Absorption Costing Marginal Costing Throughput Costing
Direct Materials 1,20,000 1,20,000 1,20,000
Direct Wages 45,000 45,000 --
Variable Manufacturing Overhead 65,000 65,000 --
Fixed Manufacturing Overhead 2,20,000 -- --
Product Cost 4,50,000 2,30,000 1,20,000
Production 10,000 10,000 10,000
Cost per unit 45 23 12
Closing Stock 400 400 400
Closing Stock Value 18,000 9,200 4,800

Part 3: Income statement under three systems


Items Absorption Costing Marginal Costing Throughput Costing
Sales (9,600 Units x Rs.72) 6,91,200 6,91,200 6,91,200
Less: CGS/ VCGS/ TCGS 4,32,000 (9,600 x 45) 2,20,800 (9,600 x 23) 1,15,200 (9,600 x 12)
GP/GC/GTC 2,59,200 4,70,400 5,76,000
Less: Period Cost:
Direct Wages -- -- 45,000
Variable Manufacturing Overhead -- -- 65,000
Fixed Manufacturing Overhead -- 2,20,000 2,20,000
Variable Selling Overhead 76,800 76,800 76,800
Fixed Selling Overhead 1,18,000 1,18,000 1,18,000
Total Period Cost 1,94,800 4,14,800 5,24,800
Profit 64,400 55,600 51,200

Part 4: Reconciling Profits


Particulars Computation Amount (Rs.)
Throughput Profit Rs.51,200
Add: Difference stock in value (Marginal vs. Throughput) (Rs.9,200 – Rs.4,800) Rs.4,800
Marginal Costing in Profit Rs.55,600
Add: Difference stock in value (Absorption vs. Marginal) (Rs.180200 – Rs.9,200) Rs.8,800
Absorption Costing Profit Rs.64,400

Part 5: Understanding Expired & Unexpired Cost


Particulars Absorption Costing Marginal Costing Throughput Costing
Total Cost 6,44,800 6,44,800 6,44,800
Less: Unexpired Cost 18,000 9,200 4,800
(Closing Stock Value)
Expired Cost 6,26,800 6,35,600 6,40,000

CGS/VCGS/TCGS 4,32,000 2,20,800 1,15,200


Add: Period Cost 1,94,800 4,14,800 5,24,800
Expired Cost 6,26,800 6,35,600 6,40,000

E M Reddy Page | 369


AMA-Notes

Notes:

1) Product costs are those costs considered for stock valuation and period costs are ignored for stock
valuation.
2) There are 3 types of costing systems:

Absorption Costing System Marginal Costing System Throughput Costing

Values Stock at Full Values Stock at Raw Material


Values Stock at Variable
Manufacturing Cost including Cost
Manufacturing Cost
unitized fixed cost
3) It was earlier discussed that Absorption Costing hides inefficiency by allowing fixed manufacturing
overhead to go to next year through stock (for more details refer Marginal Costing). Fixed Cost will
anyhow be incurred whether or not the production take place. Hence should not go into stock
valuation.
4) Throughput costing states that wages and variable overheads also should not enter stock value What is
tangible transferred through stock is only raw materials. We cannot transfer a labour time or a power
consumption from one period to another period.
5) Even if the finished good is unsalable we can still scrap the raw materials but not labour or variable
overhead.
6) Contribution = Sales – Variable Cost under Marginal Costing. In throughput costing we used a term
‘Throughput Contribution’ which is ‘Sales – Material Cost of Goods Sold’.
7) CGS = Cost of Goods Sold, VCGS = Variable Cost of Goods Sold, TCGS = Throughput Cost of
Goods Sold
8) GP = Gross Profit, GC = Gross Contribution, GTC = Gross Throughput Contribution

13.3. Theory of Constraints

**Question no 2: The Wellesley corporation makes printed cloth in two operations, weaving and
printing. Direct Material costs are Wellesley’s only variable costs. It sells at Rs.1,250 per roll to a
distributor who markets, distributes and provided customer service for the product.
Weaving Printing
Monthly Capacity 10,000 rolls 15,000 rolls
Monthly Production 9,500 rolls 8,550 rolls
Direct Materials cost per roll of cloth processed at each operation Rs.500 Rs.100
Fixed operating costs Rs.28,50,000 Rs.4,27,500
Fixed operating cost per roll (Rs.28,50,000/9,500; Rs.4,27,500/8,550) Rs.300/roll Rs.50/roll
Wellesley can start only 10,000 rolls of cloth in the weaving department because of capacity
constraints of the weaving machines. If the weaving operation produces defective cloth, the cloth
must be scrapped and yields zero net revenue. Of the 10,000 rolls of cloth started at the weaving
operation, 500 rolls (5%) are scrapped. Scrap costs per roll, based on total (fixed and variable)
manufacturing costs per roll incurred up to the end of the weaving operation, equal Rs.785 per roll
as follows:
Direct materials cost per roll (variable) Rs.500

E M Reddy Page | 370


AMA-Notes

Fixed operating costs per roll (Rs.28,50,000/10,000 rolls Rs.285


Total manufacturing costs per roll in weaving department Rs.785
The good rolls from the weaving department (called grey cloth) are sent to the printing
department. Of the 9,500 good rolls started at the printing operation, 950 rolls (10%) are scrapped
and yield Zero net revenue. Scrap costs, based on total (fixed and variable) manufacturing costs per
unit up to the end of the printing decisions, equal Rs.930 per roll calculated as follows:
Total manufacturing costs per roll in weaving department Rs.785
Printing department manufacturing costs
Direct materials costs per roll (variable) Rs.100
Fixed operating costs per roll (Rs.4,27,500/9,500) Rs.45 Rs.145
Total manufacturing costs per roll Rs.930
Wellesley corporation’s total monthly sales of printed cloth equals the printing department’s
output.
Required:
1. The printing department is considering buying 5,000 additional rolls of grey cloth from an
outside supplier at Rs.900 per roll. The printing department manager is concerned that the cost
of purchasing the grey cloth is much higher than Wellesley’s cost of manufacturing the grey
cloth. The quality of the grey cloth acquired from outside is very similar to that manufactured
in-house. The printing department expects that 10% of the rolls obtained from the outside
supplier will be scrapped. Should the printing department buy the grey cloth from the outside
supplier? Show your calculations.
2. Wellesley’s engineers have developed a method that would lower the printing department’s
scrap rate to 6% at the printing operations. Implementing the new method would cost
Rs.3,50,000 per month. Should Wellesley implement the change? Show your calculations.
3. The design engineering team has proposed a modification that would lower the weaving
department’s scrap rate to 3%. The modification would cost the company Rs.1,75,000 per
month. Should Wellesley implement the change? Show your calculations.
Solution:
Part 1: Viability of Purchasing 5,000 rolls of Gray cloth from outside
Particulars Computation Amount (Rs.)
Sales 5,000 x 90% x Rs.1,250 56,25,000
Less: Purchase Cost 5,000 x Rs.900 (45,00,000)
Less: Further Processing Cost 5,000 x Rs.100 (5,00,000)
Incremental Profit 6,25,000
This suggestion is viable as it increase profit.

Part 2: Feasibility of scrap reduction in printing department


Particulars Computation Amount
Existing Scrap 9,500 x 10% 950 rolls
Scrap after new method 9,500 x 10% 570 rolls
Reduction of Scrap 380 rolls
Sales 380 x Rs.1250 Rs.4,75,000
Less: New Method Cost (Rs.3,50,000)
Incremental Profit Rs.1,25,000

E M Reddy Page | 371


AMA-Notes

Scrap reduction is feasible as it improves the profit.

Part 3: Feasibility of scrap reduction in weaving department


Particulars Computation Amount
Existing Scrap 10,000 x 5% 500 rolls
Scrap after new method 10,000 x 3% 300 rolls
Reduction of Scrap 200 rolls
Sales 200 x 90% x Rs.1250 Rs.2,25,000
Less: FPC & New Method Cost [200 x Rs.100] + Rs.1,75,000 (Rs.1,95,000)
Incremental Profit Rs.30,000
This option is also good as it improves profit.
Notes:

1) The Output of the factory is determined by the speed of the slowest Machine or Operation. That
Machine (or) Operation is called Bottle Neck Machine (or) Bottle Neck Operation.
2) TOC (Theory of Constraints) is all about managing Bottle Necks.
3) The following things should be kept in mind while managing Bottle Necks:
a) The Production of all the operations should be sub-ordinated to Bottle Neck Operation.
b) For example, M1 can produce 10,000 units per day but the production should take place only
for 8,000 units because if the full capacity is operated it leads to build up of work-in-progress
inventories which involves holding costs.
c) The Company should strive to take measures to remove bottle necks.
d) Some measures that can be undertaken to remove bottle necks are:
i) Sub-contract portion of Bottleneck Operation
ii) Improve the capacity of the Bottleneck Operation by adding additional machines.
iii) Upgrade the technology to improve the speed of Bottleneck Machines
iv) Try for methods to reduce scrap (Both Bottleneck & Non-Bottleneck Machines)
4) The company should not spend any money that improves the output of non-bottle neck operations
because it is a wasteful expenditure having no impact on the factory output. Similar to crashing a non-
critical activity.

E M Reddy Page | 372


AMA-Notes

13.3.1. Theory of Constraints Measures (TOC Measures)

Throughput Operating Cost


Investments
Contribution (Money going
(Money tied up)
(Money Into) out)

Sum of materials in direct All operating costs


Sales – Material cost materials, WIP and FG, excluding materials like
of goods sold R&D, and cost of wages, VOH, rent,
equipments and buildings utilities, deprecation etc.

Notes:
1) There are 3 cash flows while running a business:
a) Income → Money Coming into business
b) Expense → Money going out of business
c) Asset → Money tied in the business
2) Company should spend (or) tie up money only when if it improves throughput contribution. Otherwise
such expenses and investments should be avoided.
3) For example, investment in R&D to improve efficiency of Bottleneck operations increases throughput
contribution. Hence, it is a value added investment.
4) On the other hand, incurring expenses like power, wages etc., to run non-bottleneck operations at full
capacity is wasteful expenditure. Investment in stocks unproductive investment as these will not
improve ‘Throughput contribution’.
5) When we manage Bottleneck ensure that it does not get shifted to another operation.
Question no 3: Vikram Ltd produces 4 products using 3 different machines. Machine capacity is
limited to 3,000 hours for each machine. The following information is available for February 2009:
Products A B C D
Contribution (sales-direct material) Rs. 1,500 1,200 1,000 600
Machine Hours required/unit:
Machine 1 10 6 2 1
Machine 2 10 9 3 1.5
Machine 3 10 3 1 0.5
Estimated demand (units) 200 200 200 200
Required: From the above information identify the bottleneck activity and allocate the machine
time.
Solution:
Step 1: Calculation of requirement of machine hours
Machine A B C D Total
Machine 1 (Hours) 2,000 1,200 400 200 3,800
Machine 2 (Hours) 2,000 1,800 600 300 4,700
Machine 3 (Hours) 2,000 600 200 100 2,900

E M Reddy Page | 373


AMA-Notes

Step 2: Calculation of Throughput Accounting Ratio (TAR)


Machine Requirement Availability TAR
Machine 1 (Hours) 3,800 3,000 126.67%
Machine 2 (Hours) 4,700 3,000 156.67%
Machine 3 (Hours) 2,900 3,000 96.67%
Requirement
1) Throughput Accounting Ratio = Avaibility
2) That Machine which is having the highest throughput accounting ratio is called “Bottleneck Machine.
When we allocate the bottleneck machine all the other machines stands automatically allocated.
Step 3: Calculation of throughput contribution per bottleneck hour
Particulars A B C D
Throughput Contribution (Rs.) 1,500 1,200 1,000 600
Machine 2 Hours 10 9 3 1.5
Contribution per hour (Rs.) 150 133.33 333.33 400
Rank III IV II I

Step 4: Allocation of Machine 2 hours


Product Units Hours per unit Hours Cumulative Hours
D 200 1.5 300 300
C 200 3 600 900
A 200 10 2000 2900
B 11 9 100 3000

Question no 4: A company produces three products A, B and C. The following information is


available for a period:
Particulars A B C
Contribution (Rs. p.u.) (Sales – Direct Materials) 30 25 15
Machine hours required per unit of production:
Particulars Hours Throughput
A B C Accounting Ratio
Machine 1 10 2 4 133.33%
Machine 2 15 3 6 200%
Machine 3 5 1 2 66.67%
Estimated sales demand for A, B and C is 500 units each and machine capacity is limited to 6,000
hours for each machine.
Required:
(a) Analyze the above information and apply theory of constraints process to remove the
constraints.
(b) How many units of each product will be made?
Solution:
Step 1: Calculation of requirement of machine hours
Machine A B C Total

E M Reddy Page | 374


AMA-Notes

Machine 1 (Hours) 5,000 1,000 2,000 8,000


Machine 2 (Hours) 7,500 1,500 3,000 12,000
Machine 3 (Hours) 2,500 500 1,000 4,000

Step 2: Calculation of Throughput Accounting Ratio (TAR)


Machine Requirement Availability TAR
Machine 1 (Hours) 8,000 6,000 133.33%
Machine 2 (Hours) 12,000 6,000 200%
Machine 3 (Hours) 4,000 6,000 66.67%
Machine 2 has highest throughput accounting ratio and hence it is bottleneck machine.

Step 3: Calculation of throughput contribution per bottleneck hour


Particulars A B C
Throughput Contribution (Rs.) 30 25 15
Machine 2 Hours 15 3 6
Contribution per hour (Rs.) 2 8.33 2.5
Rank III I II

Step 4: Allocation of Machine 2 hours


Product Units Hours per unit Hours Cumulative Hours
B 500 3 1,500 1,500
C 500 6 3,000 4,500
A 100 15 1,500 6,000

Question no 5: Guptha ltd produces 4 products P, Q, R and S by using 3 different machines X, Y


and Z. Each machine capacity is limited to 6,000 hours per month. The details given below are for
July 2013:
Particulars P Q R S
Selling price p.u. 10,000 8,000 6,000 4,000
Variable cost p.u. 7,000 5,600 4,000 2,800
Machine hours p.u.
X 20 12 4 2
Y 20 18 6 3
Z 20 6
Expected demand (units) 200 200 200 200
Required:
 Find out bottleneck activity
 Allocate the machine hours on the basis of bottleneck
 Ascertain the profit expected in the month if the monthly fixed cost amounts to Rs.9,50,000.
 Calculate the unused spare hours of each machine.
Solution:
Step 1: Calculation of requirement of machine hours
Machine P Q R S Total

E M Reddy Page | 375


AMA-Notes

X 4,000 2,400 800 400 7,600


Y 4,000 3,600 1,200 600 9,400
Z 4,000 1,200 400 200 5,800

Step 2: Calculation of Throughput Accounting Ratio (TAR)


Machine Requirement Availability TAR
X 7,600 6,000 126.67%
Y 9,400 6,000 156.67%
Z 5,800 6,000 96.67%

Machine Y is bottleneck machine as it has highest throughput accounting ratio.

Step 3: Calculation of contribution per bottleneck hour


Particulars P Q R S
Throughput Contribution (Rs.) 3,000 2,400 2,000 1,200
Machine Y Hours 20 18 6 3
Contribution per hour (Rs.) 150 133.33 333.3 400
Rank III IV II I

Step 4: Allocation of Machine 2 hours


Product Units Hours per unit Hours Cumulative Hours
S 200 3 600 600
R 200 6 1,200 1,800
P 200 20 4,000 5,800
Q 11 18 200 6,000

Step 5: Calculation of Profit


Product Units Contribution per unit (Rs.) Contribution (Rs.)
P 200 3,000 6,00,000
Q 11 2,400 2,64,00
R 200 2,000 4,00,000
S 200 1,200 2,40,000
Total Contribution 12,66,400
Less: Fixed Cost (9,50,000)
Profit 3,16,400

Step 6: Unused capacity of X and Z


Machine P Q R S Total
Units 200 11 200 200
X (Hours) 4,000 132 800 400 5,332
Z (Hours) 4,000 66 400 200 4,666
Machine Available Used Idle
X (Hours) 6,000 5,332 668
Z (Hours) 6,000 4,666 1,334

E M Reddy Page | 376


AMA-Notes

13.4. Just-in-Time System (JIT)

1) Push Approach:

a) The stores purchases Raw Material on reaching re-order level (or) the purchasing time. The Raw
Material is issued to the factory when it is required for production. The factory produces at it’s
full capacity and transfers the finished goods to the warehouse where it is sold when the demand
arises.
b) In this approach we don’t purchase Raw Material for requirement and we don’t produce for the
demand due to which inventories build up in different stages.
c) The Money tied up on these inventories does not increase the throughput. Hence are non value
added investments.

2) Pull Approach (JIT System):

a) Here customer orders finished goods which triggers production in the factory which results in
Raw Material Purchase.
b) As soon as materials are purchased it gets consumed and as soon as production is complete it
gets sold.
c) Hence, there is no buildup of inventories in the system.
3) Pre-requisites for a JIT System:
a) Reliable Supplier – Supplier should be willing to delivery any quantity ordered at the right quality
on right time.
b) Order and Receiving Facility – Ordering Procedure and Receiving Procedure should be simple
and fast (automated) (or) EDI (Electronic Data Interchange) with the supplier. Delivery should
be straight away to Machine instead of stores from the supplier.
c) Flexible Manufacturing System – The setup time & setup cost should be ‘0’. The cost for
producing even ’1’ unit and ‘10,000’ units should be same.
d) Production flow based on bottleneck operation (Refer Theory of Constraints_
e) Kanban Card or Machine Cells – ‘Kanban Card’ is a request from Upstream Department to
Downstream department to produce and send goods. ‘Machine Cells’ are those which contains
the all type of machines for production of a goods. Suppose a product has to be passed through
3 big machines (30,000 capacity) to become finished good, we can have 3 cells with 3 machines
(10,000 capacity). This helps in fixation of responsibility very easily.
f) Task force rearrangement – People should be capable of doing multi-tasking

E M Reddy Page | 377


AMA-Notes

g) Change in accounting system – We should maintain less books of accounts instead of so many
cost control accounts (Backflush Costing).
4) Impact of JIT on Costs:
Reduction in Overhead Material Handling, inspection and facilities
Most overhead cost becomes Depreciation
direct cost due to Machine Cells Electricity
Material Handling
Consumable and Supplies
Repairs and Maintenance
Supervision
Reduction in inventory carrying Warehouse Rent
cost Insurance
Obsolete Stores
Cost of stores equipments
Employee cost of stores department
Reduction in capital investment Number of Small machines in place of big machines
Reduction in working capital
Simplified accounting procedures Backflush Costing

5) Performance Measures in JIT:


What are not performance Machine Utilization
measures Piece work wages schemes
Some performance measures Inventory Turnover Ratio (COGS/Stock) (Should be very
minimum)
Setup time reduction (Should be low)
Customer complaints (Should be low)
Scrap (Should be reduced)
Cost of Quality
Ideas Generated

Question no 6: The management of Alliance Enterprises recently decided to adopt a just-In-time


inventory policy to curb steadily rising costs and free up cash for purposes of investment. The
company anticipates that Inventory will decrease from Rs.36,00,000 to Rs.6,00,000, with the
released funds to be invested at a 12 percent return for the firm. Additional data are as follows:
• Reduced Inventories should produce savings in Insurance and property taxes of Rs.27,000.
• Alliance will lease 75% of an existing warehouse to another firm for Rs.2 per square foot. The
warehouse has 30,000 square feet.
• Because of the need to handle an increased number of small shipments from suppliers, alliance
will remodel production and receiving - dock facilities at a cost of Rs.6,00,000. The construction
costs will be depreciated over a 10-year life.
• A shift in suppliers is expected to result in the purchase and use of more expensive raw materials.
However, these materials should give rise to fewer warranty and repair a net savings for the firm of
Rs.25,000.
• Three employees who currently earn Rs.30,000 each will be directly affected by the just-in-time
adoption decision. Two employees will be transferred to other positions with Alliance; one will be

E M Reddy Page | 378


AMA-Notes

terminated.
• Reduced raw material inventory levels and accompanying stock outs will cost Alliance Rs.70,000.
Required:
Compute the annual financial impact of Alliance's decision to adopt a just-in-time inventory
system.

Solution:

Cost – Benefit Analysis:


Particulars Computation Amount (Rs.)
A) Benefit:
Interest on fund released due to (Rs.36,00,000 – Rs.6,00,000) x 12% 3,60,000
reduction of inventories
Rental income on warehouse released 30,000 Sq. feet x 75% x Rs.2 45,000
Savings in Insurance and property tax Given 27,000
Reduction in Materials Cost due to supplier Given 25,000
change
Savings in wages 3 employees x Rs.30,000 90,000
Total Benefit 5,47,000
B) Cost:
Construction cost Depreciation Rs.6,00,000/10 Years 60,000
Stock Out cost Given 70,000
Total Cost 1,30,000
C) Net Benefit (A – B) 4,70,000

Since net benefit is positive Just-in-Time should be implemented.

Question no 7: Steel Tech ltd., is an automotive supplier that uses automatic screw machines to
manufacture precision parts from steel bars. Steel Tech's Inventory of raw steel averages
Rs.6,00,000 with a turnover rate of four times per year. John, president of Steel Tech, is concerned
about the costs of carrying Inventory. He is considering the adoption of just-in-time inventory
procedures in order to eliminate the need to carry any raw steel Inventory. John asked the
company's financial controller, to evaluate the feasibility of JIT for the corporation. He has
identified the following effects of adopting JIT.
• Without scheduling any overtime, lost sales due to stock outs would increase by 35,000 units per
year. However, by incurring overtime premiums of Rs.40,000 per year, the increase in lost sales
could be reduced to 20,000 units. This would be the maximum amount of overtime that would be
feasible for Steel Tech.
• Two warehouses presently used for steel bar storage would no longer be needed. Steel Tech rents
one warehouse from another company at an annual cost of Rs.60,000. The other warehouse is
owned by Steel Tech and contains 12,000 square feet. Three-forth of the space in the owned
warehouse could be rented out for Rs.1.50 per square foot per year.
• Insurance totaling Rs.14,000 per year would be eliminated. Steel Tech's projected operating result
for 2008 are as follows. Long - term capital investments by Steel Tech are expected to produce a
rate of return of 20 percent before taxes.

E M Reddy Page | 379


AMA-Notes

STEEL TECH, INC.


Budgeted Income Statement
For the year ended December 31, 2008 (in thousands)
Sales (9,00,000 units) Rs.10,800
Cost of goods sold:
Variable Rs.4,050
Fixed Rs.1,450 Rs.5,500
Gross Margin Rs.5,300
Selling and administrative expenses:
Variable Rs.900
Fixed Rs.1,500 Rs.2,400
Income before interest and taxes Rs.2,900
Interest expenses Rs.900
Income before taxed Rs.2,000
Required:
Calculate the estimated savings or loss for Steel Tech, Ltd that would result in 2008 from the
adoption of just-in-time inventory methods. Ignore income taxes.
Solution:
Step 1: Calculation of contribution per unit
Particulars Amount (Rs.)
Sales 1,08,00,000
-Variable cost of goods sold -40,50,000
-Variable selling and administrative expenses -15,00,000
Contribution 58,50,000
Rs.58,50,000
Contribution per unit = 9,00,000 Units = Rs.6.5

Step 2: Viability of Overtime

Recommended to work overtime because benefit exceeds costs.

Step 3: Just-in-Time Cost Benefit Analysis


Particulars Computation Amount (Rs.)
A) Benefit:
Interest saved due to reduction in inventory Rs.6,00,000 x 20% 1,20,000

E M Reddy Page | 380


AMA-Notes

Rent Saved in Warehouse Given 60,000


Rental Income on Warehouse space released 12,000 Sq. feet x 3/4 x Rs.1.5 13,500
Savings in Insurance Given 14,000
Total Benefit 2,07,500
B) Cost:
Stock Out Cost 25,000 Units x Rs.6.5 1,30,000
Overtime Premium Given 40,000
Total Cost 1,70,000
C) Net Benefit (A – B) 37,500

It is recommended to implement Just-In-Time system and also work to overtime to reduce stock outs
arising due to Just-in-Time.

13.5. Backflush Costing

13.5.1. Backflush Costing – Version 1

Question no 8: Road warrior corporation, assembles hand-held computers that have scaled-down
capabilities of laptop computers. Each hand-held computer takes 6 hours to assemble. Road
warrior uses a JIT production system and a back flush costing system with three trigger points:
 Purchase of direct (Raw) materials
 Completion of finished units of product
 Sale of finished goods

There are no beginning inventories of materials or finished goods. The following data are for
August 2008:
Direct (raw) materials purchased Rs.27,54,000
Direct (raw) materials used Rs.27,33,600
Conversion cost incurred Rs.7,23,600
Conversion cost allocated Rs.7,50,400
Road warrior records direct materials purchased and conversion costs incurred at actual costs.
When finished goods are sold, the back flush costing system “pulls through” standard direct
materials costs (Rs.102 per unit) and standard conversion costs (Rs.28 per unit). It produced 26,800
finished goods units in Augusts 2008 and sold 26,400 units. The actual direct material cost per unit
in August 2008 was Rs.102 while the actual conversion costs per unit was Rs.27.
Required:
Prepare necessary ledger accounts for August 2008.
Solution:
Raw Material and In-process Control A/c
Particulars Amount (Rs.) Particulars Amount (Rs.)
To Cash 27,54,000 By FG A/c (26,800 Units x 27,33,600
Rs.102)
By Closing Balance 20,400
Total 27,54,000 Total 27,54,000
Finished Goods Control A/c
Particulars Amount (Rs.) Particulars Amount (Rs.)

E M Reddy Page | 381


AMA-Notes

To RM A/c (26,800 Units x Rs.102) 27,33,600 By Cost of Sales a/c 34,32,000


To CC A/c (26,800 Units x Rs.28) 7,50,400 By Closing Balance 52,000
Total 34,84,000 Total 34,84,000
Cost of Sales A/c
Particulars Amount (Rs.) Particulars Amount (Rs.)
To FG A/c (26,400 Units x Rs.130) 34,32,000 By Costing P&L a/c 34,32,000
Total 34,32,000 Total 34,32,000
Conversion Cost A/c (Wages & Overheads)
Particulars Amount (Rs.) Particulars Amount (Rs.)
To Cash 7,23,600 By FG A/c (26,800 Units x 7,50,400
Rs.28)
To Costing P&L a/c 26,800
Total 7,50,400 Total 7,50,400

13.5.2. Backflush Costing – Version 2

Assume the same facts in question no 5, except for the following change. Road Warrior
Corporation, now used a back flush costing system with the following two trigger points:

 Purchase of direct (raw) materials


 Sale of finished goods.
The inventory control account here will include direct materials purchased but not yet in
production, materials in work in process, and materials in finished goods but not sold. No
conversion costs are inventoried. Any under-or over allocated conversion costs are written off
monthly to costing P&L account.
Required:
Prepare necessary ledger accounts for August 2008.
Solution:
Raw Material and In-process Control A/c
Particulars Amount (Rs.) Particulars Amount (Rs.)
To Cash 27,54,000 By COS A/c (26,400 Units x 26,92,800
Rs.102)
By Closing Balance 61,200
Total 27,54,000 Total 27,54,000
Cost of Sales A/c
Particulars Amount (Rs.) Particulars Amount (Rs.)
To RM A/c (26,400 Units x Rs.102) 26,92,800 By Costing P&L a/c 34,32,000
To CC A/c (26,400 Units x Rs.102) 7,39,200
Total 34,32,000 Total 34,32,000
Conversion Cost A/c (Wages & Overheads)
Particulars Amount (Rs.) Particulars Amount (Rs.)
To Cash 7,23,600 By COS A/c (26,400 Units x 7,39,200
Rs.28)
To Costing P&L a/c 15,600

E M Reddy Page | 382


AMA-Notes

Total 7,50,400 Total 7,50,400

13.5.3. Backflush Costing – Version 3

Assume the same facts in question no 5, except now Road Warrior uses only two trigger points, the
completion of finished unit of product and the sale of finished goods. Any under or over allocated
costs are written off monthly to costing P&L account.
Required:
Prepare necessary ledger accounts for August 2008.
Solution:

Finished Goods Control A/c


Particulars Amount (Rs.) Particulars Amount (Rs.)
To Cash A/c (26,800 Units x 27,33,600 By Cost of Sales a/c 34,32,000
Rs.102)
To CC A/c (26,800 Units x Rs.28) 7,50,400 By Closing Balance (400 52,000
Units x Rs.130)
Total 34,84,000 Total 34,84,000

Cost of Sales A/c


Particulars Amount (Rs.) Particulars Amount (Rs.)
To FG A/c (26,400 Units x Rs.130) 34,32,000 By Costing P&L a/c 34,32,000
Total 34,32,000 Total 34,32,000
Conversion Cost A/c (Wages & Overheads)
Particulars Amount (Rs.) Particulars Amount (Rs.)
To Cash 7,23,600 By FG A/c (26,800 Units x 7,50,400
Rs.28)
To Costing P&L a/c 26,800
Total 7,50,400 Total 7,50,400

Notes:

1) Traditional Costing System meticulously traces the various cost items into inventories. The system
ensures high degree of inventory control.
2) The Traditional Costing System operates with 4 trigger points. A trigger point is an event which makes
cost accounting system to pass journal entries.
3) The 4 trigger points are:
a) Purchase of Raw Materials
b) Issue for Production
c) Completion of Finished Goods
d) Sale of Finished Goods
The accounts maintained are Raw Material Control A/c, WIP Control A/c, Finished Goods Control
A/c and Cost of Sales A/c.

E M Reddy Page | 383


AMA-Notes

4) Companies operating Just-In-Time (JIT) system ideally will have no inventories (or) very negligible
inventory. Hence there is no necessity to have an elaborate cost accounting system that traces costs to
the stocks and control stocks. A simplified is called ‘Backflush Costing System’.
5) There are 3 versions in Backflush Costing System:

Version 1 Version 2 Version 3

3 Trigger Points:
2 Trigger Points: 2 Trigger Points:
a) Purchase of Raw Materials
a) Purchase of Raw Materials a) Completion of Finished Goods
b) Completion of Finished Goods
b) Sale of Finished Goods b) Sale of Finished Goods
c) Sale of Finished Goods

Accounts: Accounts: Accounts:


a) Raw Material & In process A/c a) Raw Material & In process A/c a) Finished Goods A/c
b) Finished Goods A/c c) Cost of Sales A/c c) Cost of Sales A/c
c) Cost of Sales A/c There is no Finished Goods A/c There is no Raw Material A/c &
There is no WIP A/c & WIP A/c WIP A/c
6) In Version 1 Consumption of Raw Materials and charging of Conversion Cost happens not during
Production bure recognized only on completion of goods. The completed goods pulls the Standard
Cost of Raw Materials Consumed & Conversion Cost into Production.
7) In an ideal Just-In-Time System there should be no Price Variance because negotiation with supplier
happens at the beginning of the period itself. There should also be no usage variance because in Just-In-
Time System there is no time available to re-work Scrap & Defectives. Still if it arises a separate Scrap
Reporting System should capture it and transfer to Abnormal Loss Account.
8) In Version 3 the suppliers are paid only for standard consumption. Any excess consumption reported in
scrap reporting system should be analyzed with reasons. If it is due to poor quality Raw Material the
supplier will not be paid and if it is due to other reasons they will be paid and the cost booked to
Abnormal Loss a/c.
9) In Version 1 the Rs.20,400 closing balance of Raw Material represents “Raw Material as Raw Material &
Raw Material in Work-in-Progress”. In Version 2 the Rs.61,200 Raw Material closing balance represents
“Raw Material as Raw Material, Raw Material in Work-in-Progress and Raw Material in Finished
Goods”.

10) Conversion Cost treatment differs between the versions:


a) Version 1 & Version 3 → Treated as Product Cost → Expires only to the extent of Cost of
Goods Sold
b) Version 2 → Treated as Period Cost → Expires Fully

E M Reddy Page | 384


AMA-Notes

11) The above can be understood as follows (Version 1 & Version 3):

In Version 1 & Version 3, the conversion cost debited net to costing P&L is Rs.7,12,400 (Rs.7,39,200 –
Rs.26,800). The balance transferred to next year.

12) The above can be understood as follows (Version 2):

Thus the entire Rs.7,23,600 gets debited in current year’s costing P&L and no part of it is inventoried
and transferred to next year.

Question no 9: Little field company uses a backflush costing system with three trigger points:
 Purchase of Direct Material
 Completion of good finished units of product
 Sale of finished goods
There are no beginning inventories. Information for March 2008 is:
Direct Material Purchased Rs.4,40,000 Conversion Cost Allocated Rs.2,00,000
Direct Material Used Rs.4,25,000 Cost transferred to finished goods Rs.6,25,000
Conversion Cost Incurred Rs.2,11,000 Cost of goods sold Rs.5,95,000
Required:
1. Prepare journal entries for April [without disposing of under allocated or over allocated
conversion cost]. Assume there are no direct materials variances.
2. Under an ideal JIT production system, how would the amounts in your journal entries differ for
the journal entries in requirement 1.

Solution:

Part 1:

E M Reddy Page | 385


AMA-Notes

Raw Material and In-process Control A/c


Particulars Amount (Rs.) Particulars Amount (Rs.)
To Cash 4,40,000 By Finished Goods a/c 4,25,000
By Closing Balance 15,000
Total 4,40,000 Total 4,40,000
Finished Goods Control A/c
Particulars Amount (Rs.) Particulars Amount (Rs.)
To RM & In process control a/c 4,25,000 By Cost of Sales a/c 5,95,000
To Conversion Cost a/c 2,00,000 By Closing Balance 30,000
Total 6,25,000 Total 6,25,000
Cost of Sales A/c
Particulars Amount (Rs.) Particulars Amount (Rs.)
To Finished Goods a/c 5,95,000 By Costing P&L a/c 5,95,000
Total 5,95,000 Total 5,95,000
Conversion Cost A/c (Wages & Overheads)
Particulars Amount (Rs.) Particulars Amount (Rs.)
To Cash 2,11,000 By Finished Goods a/c 2,00,000
By Closing Balance (Under 11,000
absorption)
Total 2,10,000 Total 2,10,000

Part 2: Journal entries under an ideal Just-in-Time System

Step 1: Calculation of Cost of Production and materials purchased

1) Ideal Just-in-time system will have no inventories (Raw Material or Finished Goods).
2) If finished goods inventory has to be ‘nil’ then cost of finished goods produced should be equal to cost
of goods sold.
Cost of Finished Goods Produced = Rs.5,95,000
Rs.4,25,000
a) Raw Material Used → Rs.5,95,000 x Rs.6,25,000 = Rs.4,04,600
Rs.2,00,000
b) Conversion Cost Allocated → Rs.5,95,000 x Rs.6,25,000 = Rs.1,90,400
3) When Raw Material stock has to be ‘nil’ then Raw Material used should be equal to Raw Material
purchased i.e. Rs.4,04,600

Step 2: Preparation of Ledger Accounts


Raw Material and In-process Control A/c
Particulars Amount (Rs.) Particulars Amount (Rs.)
To Cash 4,04,600 By Finished Goods a/c 4,04,600
Total 4,40,000 Total 4,40,000
Finished Goods Control A/c
Particulars Amount (Rs.) Particulars Amount (Rs.)
To RM & In process control a/c 4,04,600 By Cost of Sales a/c 5,95,000
To Conversion Cost a/c 1,90,400
Total 5,95,000 Total 5,95,000

E M Reddy Page | 386


AMA-Notes

Cost of Sales A/c


Particulars Amount (Rs.) Particulars Amount (Rs.)
To Finished Goods a/c 5,95,000 By Costing P&L a/c 5,95,000
Total 5,95,000 Total 5,95,000
Conversion Cost A/c (Wages & Overheads)
Particulars Amount (Rs.) Particulars Amount (Rs.)
To Cash 2,11,000 By Finished Goods a/c 1,90,4000
By Closing Balance (Under 20,600
absorption)
Total 2,10,000 Total 2,10,000

13.6. Total Quality Management (TQM)

1) Quality is what differentiates our product from others.


2) “Quality in Design” & “Quality in Performance” gives total quality of a given product (or) service.
3) Quality is a commitment given by a chance and the commitment is not compulsory. The quality
commitment given once should be adhered.
4) Quality costs are those costs which are incurred by the company to adhere to it’s quality commitment.
5) There are 4 types of quality costs:
a) Preventive Cost → Prevention costs are those costs incurred to prevent below quality goods
from being purchased or produced. E.g.; - Quality training program given to employees
b) Appraisal Cost → Appraisal cost is the cost spent to check whether the units produced (or)
units purchased to adhered (or) confirmed the quality. E.g.; - Inspection Cost
c) Internal failure Cost → Internal failure cost if the cost of producing a defective product before
it reaches the customers. E.g.; - Rectification Cost → Curative in Nature
d) External failure Cost → External failure cost if the cost of producing a defective product after it
reaches the customers. E.g.; - Warranty Cost → Curative in Nature

Question no 10: Calton Ltd. Makes and sell a single product. The existing product unit
specifications are as follows:
Direct Material X: 8 sq. meters at Rs.4 per sp. Meter
Machine time: 0.6 running hours
Machine cost per gross hour: Rs.40
Selling Price: Rs.100
Calton Ltd., require fulfilling orders for 5,000 product units per period. There are no stocks of
product units at the beginning or end of the period under review. The stock level of material X
remains unchanged throughout the period. The following additional information affects the costs
and revenues:
1. 5% of incoming material from suppliers is scrapped due to poor receipt and storage
organization.
2. 4% of material X input to the machine process is wasted due to processing problems.
3. Inspection and storage of material X costs Rs.0.10 paisa per sq. meter purchased.
4. Inspection during the production cycle, calibration checks on inspection equipment, vendor
rating and other checks costs Rs.25,000 per period.

E M Reddy Page | 387


AMA-Notes

5. Production quantity is increased to allow for the downgrading of 12.5% of product units at the
final inspection stage. Downgraded units are sold as 'second quality' units at a discount of 30%
on the standard selling price.
6. Production quantity is increased to allow for returns from customers which are replaced free of
charge. Returns are due to specification failure and account for 5% of units initially delivered to
customers. Replacement units incur a delivery cost of Rs.8 per unit. 80% of the returns from
customers are rectified using 0.2 hours of machine running time per unit and are re-sold as
‘third quality’ products at a discount of 50% on the standard selling price. The remaining
returned units are sold as scrap for Rs.5 per unit.
7. Product liability and other claims by customers is estimated at 3% of sales revenue from
standard product sales.
8. Machine idle time is 20% of gross machine hours used (i.e. running hours = 80% of gross
hours).
9. Sundry costs of administration, selling and distribution total Rs.60,000 per period.
10. Calton Ltd is aware of the problem of excess costs and currently spends Rs.20,000 per period in
efforts to prevent a number of such problems from occurring.
Calton Ltd. is planning a quality management programme which will increase its excess cost
prevention expenditure from Rs.20,000 to Rs.60,000 per period. It is estimated that this will have
the following impact.
1. A reduction in stores losses or material X to 3% of incoming material.
2. A reduction in the downgrading of product units at Inspection to 7.5% of units inspected.
3. A reduction in material X losses in process to 2.5% of input to the machine process.
4. A reduction in returns of products from customers to 2.5% of units delivered.
5. A reduction in machine idle time to 12.5% of gross hours used.
6. A reduction in product liability and other claims to 1% of sales revenue from standard product
sales.
7. A reduction in inspection, calibration, vendor rating and other checks by 40% of existing figure.
8. A reduction in sundry administration, selling and distribution costs by 10% of the existing
figure.
9. A reduction in machine running time required per product unit to 0.5 hours.
Required:
(a) Prepare summaries showing the calculation of (i) total production units (pre-Inspection), (ii)
purchases of material X (sq. meters), (iii) gross machine hours. In each the figures are required
for the situation both before and after the implementation of the additional quality
management programme, in order that the orders for 5,000 product units may be fulfilled.
(b) Prepare profit and loss account for Calton Ltd for the period showing the profit earned both
before and after the implementation of the additional quality management programme.
(c) Comment on the relevance of a quality management programme and explain the meaning of
the terms Internal failure costs, appraisal costs and prevention costs giving examples for each,
taken where possible from the information in the question.

Solution:

Step 1: Calculation of Production Units


Particulars Before (Units) After (Units)

E M Reddy Page | 388


AMA-Notes

Demand 5,000 5,000


Add: Returns 250 (5,000 x 5%) 125 (5,000 x 2.5%)
Post Inspection Production 5,250 5,125
Add: Downgrading 750 (5,250/87.5x12.5) 416 (5,125/92.5x7.5)
Pre Inspection Production 6,000 5,541

Step 2: Raw Materials Purchases in Meters


Particulars Before (Sq. Mts.) After (Sq. Mts.)
Requirements 48,000 (6,000 x 8) 44,328 (5,541 x 8)
Add: Process Loss 2,000 (48,000/96x4) 1,137 (44,328/97.5x2.5)
Input 50,000 45,465
Add: Loss on receipt 2,632 (50,000/95x5) 1,406 (45,465/97x3)
Purchase of Raw Materials 52,632 46,871

Step 3: Gross Machine Hours


Particulars Before (Hours) After (Hours)
Running Hours 3,600 (6,000 x 0.6) 2,771 (5,541 x 0.5)
Add: Rectification time 40 (250 x 80% x 0.2) 20 (125 x 80% x 0.2)
Total Running Hours 3,640 2,791
Add: idle time 910 (3,640/80x20) 399 (2,791/87.5x12.5)
Gross Machine Hours 4,550 3,190

Step 4: Income before and after the production


Particulars Before (Rs.) After (Rs.)
A. Revenues
First Quality 5,00,000 (5,000 x 100) 5,00,000 (5,000 x 100)
Second Quality 52,500 (750 x 70) 29,120 (416 x 70)
Third Quality 10,000 (250 x 80% x 50) 5,000 (125 x 80% x 50)
Scrap Sales 250 (250 x 20% x 5) 125 (125 x 20% x 5)
Total 5,62,750 5,34,245
B. Cost
Material 2,10,528 (52,632 x 4) 1,87,484 (46,871 x 4)
Machine running cost 1,82,000 (4,550 x 40) 1,27,600 (3,190 x 40)
Inspection Cost 5,263 (52,632 x 0.10) 4,687 (46,871 x 0.10)
Vendor Rating 25,000 15,000 (25,000 x 60%)
Delivery Cost 2,000 (250 x 8) 1,000 (125 x 8)
Product liability claims 15,000 (5,00,000 x 3%) 5,000 (5,00,000 x 1%)
Administration & Other expenses 60,000 54,000 (60,000 x 90%)
Quality Program Cost 20,000 60,000
Total Cost 5,19,791 4,54,771
C. Profit 42,959 79,474

It is recommended to go for enhanced quality control program.

Step 4: Examples for different quality costs

E M Reddy Page | 389


AMA-Notes

Quality Costs Example


Prevention Cost Quality Program Cost
Appraisal Cost Inspection Cost
Internal Failure Cost Cost spent on downgraded units
External Failure Cost Liability Claims

Question no 11: TQ Ltd implemented a quality programme and had the following results:
Particulars 2007 2008
Rs. in ‘000s
Sales 6,000 6,000
Scrap 600 300
Rework 500 400
Production Inspection 200 240
Product Warranty 300 150
Quality Training 75 150
Material Inspection 80 60
You are required to:
1. Classify the quality costs as prevention, appraisal, internal failure and external failure and
express each class as a percentage of sales.
2. Compute the amount of increase in proms due to quality Improvement.

Solution:
Particulars 2007 2008
% Amount % Amount
A. Sales 100 6,000 100 6,000
B. Cost
1. Prevention Cost
Quality Training 1.25 75 2.5 150
2. Appraisal Cost
Production Inspection 3.33 200 4 240
Material Inspection 1.33 80 1 60
Total 4.66 280 5 300
3. Internal failure Cost
Scrap 10 600 5 300
Rework 8.33 500 6.67 400
Total 18.33 1,100 11.67 700
4. External failure Cost
Product Warranty 5 300 2.5 150
Total Cost (1 + 2 + 3 + 4) 29.25 1,755 21.67 1,300
C. Profit 70.75 4,245 78.33 4,700

1) In 2007 the company spent 5.91% in prevention and appraisal cost resulting in a spending of 23.33% on
failure costs.
2) In 2008 it increased the spending on prevention and appraisal cost to 7.5% resulting in reduction of
failure costs to 14.17%.

E M Reddy Page | 390


AMA-Notes

Question no 12: X video company sells package of blank video tapes to its customers. It purchases
video tapes from Y Tape Company @ Rs.140 a packet. Y Tape Company pays all freight to X video
company. No incoming inspection is necessary because Y Tape Company has a superb reputation
for delivery of quality merchandise. Annual demand of X video company is 13,000 packages. X
video company requires 15% annual return on investment. The purchase order lead time is two
weeks. The purchase order is passed through internet and it costs Rs.2 per order. The relevant
material handling, insurance cost etc. is Rs.3.10 per package per year. X video company has to
decide whether or not to shift to JIT purchasing. Y Tape Company agrees to deliver 100 packages
of video tapes 130 times per year (5 times every two weeks) instead of existing delivery system of
1,000 packages 13 times a year with additional amount of Rs.0.02 per package. X video company
incurs no stock out under its current purchasing policy. It is estimated X video company incurs
stock out cost on 50 video tape packages under a JIT purchasing policy. In the event of stock out X
video company has to rush order tape packages which costs Rs.4 per package.
Required: Comment whether X video company should implement JIT purchasing system.

Z Co. also supplies video tapes. It agrees to supply at Rs.136 per package under JIT delivery
system. If video tape purchased from Z Co. relevant carrying cost would be Rs.3 per package
against Rs.3.10 in case of purchasing from Y tape company. However, Z Co. does not enjoy so
sterling reputation for quality. X video Co. anticipates the following negative aspects of purchasing
tapes from Z Co.
(a) To incur additional inspection cost of 5 paisa per package. Average stock out of 360 tapes
package per year would occur largely resulting from late deliveries. Z Co. cannot rush order at
short notice. X Video Co. anticipates lost contribution margin per package of Rs.8 from stock
out.
(b) Customer would likely return 2% of all packages due to poor quality of the tape and to handle
this return an additional cost of Rs.25 per package.
Required: Comment whether X Video Co. places order to Z Co.

Solution:

Part 1: Existing vs. Just-in-Time System

Step 1: Total Annual Cost under existing system


Particulars Computation Amount (Rs.)
Purchase Cost 13,000 Packs x Rs.140 18,20,000
Ordering Cost 13 Orders x Rs.2 26
Interest Cost 1,000 Packs x ½ x Rs.140 x 15% 10,500
Material Handling Cost 1,000 Packs x ½ x Rs.3.10 1,550
Total Cost 18,32,076

Step 2: Total Annual Cost under Just-in-Time system


Particulars Computation Amount (Rs.)
Purchase Cost 13,000 Packs x Rs.140.02 18,20,260
Ordering Cost 130 Orders x Rs.2 260
Interest Cost 100 Packs x ½ x Rs.140.02 x 15% 1,050
Material Handling Cost 100 Packs x ½ x Rs.3.10 155

E M Reddy Page | 391


AMA-Notes

Stock out Costs 50 Packs x Rs.4 200


Total Cost 18,21,925

It is recommended to shift to Just-in-Time purchasing since there is a reduction in cost.

Part 2: Calculation of annual cost due to Just-in-Time purchase with Z


Particulars Computation Amount (Rs.)
Purchase Cost 13,000 Packs x Rs.136 17,68,000
Ordering Cost 130 Orders x Rs.2 260
Interest Cost 100 Packs x ½ x Rs.136 x 15% 1,020
Material Handling Cost 100 Packs x ½ x Rs.3 150
Stock out Costs 360 Packs x Rs.8 2,880
Inspection Cost 13,000 Packs x Rs.0.05 1,650
Return Cost 13,000 Packs x 2% x Rs.25 6,500
Total Cost 17,79,460

It is recommended to go for supplier Z in spite of poor quality due to reduction in cost. However non-
financial aspect like brand value erosion etc., should be considered while making the decision.

Question no 13: Classify the following items under appropriate categories of quality cost viz.
Prevention Costs, Appraisal Costs, Internal Failure Costs and External Failure Costs:
(i) Rework
(ii) Disposal of scrap
(iii) Warranty Repairs
(iv) Revenue loss
(v) Repair to manufacturing equipment
(vi) Discount on defective sale
(vii) Raw material inspection
(viii) Finished product Inspection
(ix) Establishment of quality circles
(x) Packaging inspection

Solution:
Particulars Classification
(i) Rework Internal Failure Cost
(ii) Disposal of Scrap Internal Failure Cost
(iii) Warranty Repairs External Failure Cost
(iv) Revenue Loss Internal Failure Cost
(v) Repair to Manufacturing Equipment Internal Failure Cost
(vi) Discount on Defective Sale Internal Failure Cost
(vii) Raw Material Inspection Appraisal Cost
(viii) Finished Product Inspection Appraisal Cost
(ix) Establishment of Quality Circles Prevention Cost
(x) Packaging Inspection Appraisal Cost

E M Reddy Page | 392


AMA-Notes

13.7. Life Cycle Costing (LCC)

1) The Product life cycle is classified as follows:


a) Infinite Phase
b) Growth Phase
c) Maturity Phase
d) Decline Phase
2) Infinite Phase
a) It is the phase where the product is getting introduced (or) developed.
b) Revenue will be very less in this phase when compared to cost.
c) In this phase Fixed Overhead Cost & R&D cost are more and variable costs are less.
d) The Product will be reporting losses at this infant phase.
3) Growth Phase
a) It is the phase where the market share of a product rapidly increases.
b) Revenue growth is high.
c) Marketing cost and variable costs are very high and fixed cost is recovered.
4) Maturity Phase
a) In this phase the product marker share grows at declining rate.
b) Company builds-up loyal customers and company will have stable market share.
c) Costs are mostly heavy manufacturing costs and moderate marketing cost.
d) This is the longest phase in most of the products life cycles.
5) Decline Phase
a) In this phase the company share gets reduced
b) The company should try to extend the product life through some cost reduction techniques.

Another way of understanding the product life cycle:

1) Stages in Product Life Cycle


i) Market Research
Product is a bundle of features that customer wants. Company should do the market research
and should understand the customer needs.
ii) Design
Based on the customer requirements the company should design the product. The company
should redesign the product to make it feasible for manufacturing.
iii) Development
Based on the design the product is developed and the design is freeze at the end of development
stage.
iv) Prototype
Prototype means a model of the product and do simulation, testing under real conditions by
doing marketing etc.,
v) Commissioning
Once the prototype is working effectively, all the infrastructure costs are incurred (Pre-
Manufacturing Costs).
vi) Manufacturing
vii) Marketing
viii) Distribution

E M Reddy Page | 393


AMA-Notes

ix) Customer Support


x) Decommissioning
Costs incurred to exit the market.

Question no 14: Destin Products makes digital watches. Destin is preparing a product life-cycle
budget for a new watch, MX3. Development on the new watch is to start shortly. Estimates about
MX3 are as follows:
Life-cycle units manufactured and sold 4,00,000
Selling price per watch Rs.40
Life-Cycle Costs:
R&D and design costs Rs.10,00,000
Manufacturing:
Variable costs per watch Rs.15
Variable costs per batch Rs.600
Watches per batch 500
Fixed Costs Rs.18,00,000
Marketing:
Variable costs per batch Rs.3.20
Fixed Costs Rs.10,00,000
Distribution:
Variable cost per batch Rs.280
Watches per batch 160
Fixed Costs Rs.7,20,000
Customer-service costs per watch Rs.1.50
Ignore the time value of money.
Required:
1) Calculate the budgeted life-cycle operating income for the new watch.
2) What percentage of the budgeted total product life-cycle costs at the end of the R&D and
design stages will incur?
3) An analysis reveals that 80% of the budgeted total product life-cycle costs of the new watch will
be locked in at the end of the R & D and design stages. What implications do these findings
have for managing MX3's costs?
4) Destin's Market Research Department estimates that reducing MX3's price by Rs.3 will
increase life-cycle unit sales by 10 percent. If unit sales increase by 10%, Destin plans to
increase manufacturing and distribution batch sizes by 10% as well. Assume that all variable
costs per watch, variable costs per batch, and fixed costs will remain the same. Should Destin
reduce MX3's price by Rs.3? Show your calculations.

Solution:

Part 1: Budgeted Life Cycle Income Statement


Particulars Computation Amount (Rs.)
A. Revenues
Sales 4,00,000 Watches x Rs.40 1,60,00,000
B. Cost
R&D Design Cost Given 10,00,000

E M Reddy Page | 394


AMA-Notes

Variable Manufacturing Cost (Unit Level) 4,00,000 Watches x Rs.15 60,00,000


Variable Manufacturing Cost (Batch Level) 4,00,000 Watches/500 x Rs.600 4,80,000
Fixed Manufacturing Costs Given 18,00,000
Variable Marketing Cost (Unit Level) 4,00,000 Watches x Rs.3.20 12,80,000
Fixed Marketing Cost Given 10,00,000
Variable Distribution Cost (Batch Level) 4,00,000 Watches/160 x Rs.280 7,00,000
Fixed Distribution Cost Given 7,20,000
Customer Service Cost (Unit level) 4,00,000 Watches x Rs.1.50 6,00,000
Total Cost 1,35,80,000
C. Budgeted Life Cycle Income (A – B) 24,20,000

Part 2: Concept of Locked in Cost


Rs.10,00,000
% of R&D Cost to total cost = = 7.36%
Rs.1,35,80,000

1) R&D cost constitutes just 7.36% of the total life cycle cost. Hence it is not a major activity like
manufacturing, marketing etc., Is it true?
2) It is not, because at the end of R&D the company commits itself to a given type of production facility, a
required labour time, a raw material of a given quality and a particular marketing strategy.
3) All these costs may be spent tomorrow but is locked at the end of R&D stage itself. In this problem
80% of the cost is locked.
4) A poor R&D activity will result in negligible cost reduction possibility for the remaining 80% of cost.

13.8. Target Costing

Question no 15: For many years, Leno Corporation has used a straightforward cost-plus pricing
system, marking its goods up approximately 25 percent of total cost. The company has been
profitable; however, it has recently lost considerable business to foreign compellers that have
become very aggressive in the marketplace.
These firms appear to be using target costing.
An example of Leno's problem is typified by Item no. 8976, which has the following unit-cost
characteristics:
Direct Material Rs.30
Direct Labour 75
Manufacturing Overhead 50
Selling and administrative expenses 25
The going market price of an Identical product of comparable quality is Rs.195, which is
significantly below what Leno is charging.
Required:
1) Contrast cost-plus pricing and target costing. Which of the two approaches could be aptly
labeled price-led costing? Why?
2) What is Leno's current selling price of item no.8976
3) If Leno used target costing for item no.8976, what must happen to costs if the company desires
to meet the market price and maintain its current rate of profit on sales? By how much?
4) Would the identification of value-added and non-value added costs assist Leno in this
situation? Briefly explain.

E M Reddy Page | 395


AMA-Notes

5) Suppose that by previous cost-cutting drives, costs had already been "pared to the bone" on
Item no.8976. What might Leno be forced to do with its markup on cost to remain competitive?
By how much?

Solution:

Step 1: Calculation of Leno’s current selling price


Particulars Amount (Rs.)
Product Cost 180
Add: Markup (Rs.180 x 25%) 45
Selling Price 225

Step 2: Target Cost

1) If Leno adopts cost plus pricing and sells the product at Rs.225 the following things happens:
a) The Demand reduces because of higher price
b) The unit cost increases because the same fixed cost is shared by fewer units.
c) When the unit cost increases the selling price increases due to cost plus pricing.
d) Which further triggers demand drop and this becomes vicious cycle.
e) After some time, the product will be pushed out of the market.
2) An alternative is to go for target costing where the equation becomes Target Cost = Selling Price –
Profit
3) This is not change in equation but the change in attitude. Target costing recopies that selling price is
market determined and company has no control over it but what can be really controlled is cost.
4) Cost plus pricing is cost led pricing but target costing is price led costing.
Particulars Amount (Rs.)
5) Market determined price 195
Less: Markup (Rs.195 x 20%) 39
Target Cost 156

Step 3: Value Engineering

1) The company should try to reduce the current cost of Rs.180 to Rs.156 i.e. the target cost reduction is
Rs.24 (Rs.180 – Rs.156).
2) To do this it should start value engineering program. Value engineering is a process through which the
costs are classified as value added and non-value added.
3) Value engineering process has 2 aspects:
a) Design engineering → A design of a product is nothing but bundle of features which can be
classified into 2:
i. Function features → Defined the utility of the product and is always value added.
ii. Aesthetic features → Adds appeal to the product and is value added only when
customer is willing to pay for it.
b) Process engineering → Once design is finalized analyses the production and selling process and
eliminate the non-value added process. This can be done when the company continuously seeks
to improve it’s process which is called “Kaizen Approach”.

E M Reddy Page | 396


AMA-Notes

4) When non-value added features are eliminated the activities required to have those features can be
avoided thereby saving Activity Based Cost (ABC). To know how much, we saved we required Activity
Based Costing (ABC) System. Same for process engineering also.
5) Even after value engineering i.e. after the cost is “pared to the bone”, still target cost not achieved then
the company has to compromise on margin. Even after that if target cost could not be achieved the
company should quit the market but never should be increased the selling price, otherwise it would be
pushed out of the market.

Question no 16: Danish Furniture (DF) manufactures easy-to-assemble wooden furniture for home
and office. The firm is considering modification of a table to make it more attractive to individuals
and businesses. The table is small, can be used to hold a computer printer or a fax machine, and
has several shelves for storage.
The company's marketing department surveyed potential buyers of the table regarding five
proposed modifications. The 200 survey participants were asked to evaluate the modifications by
using a five-point scale that ranged from 1 (strongly disagree) to 5 (strongly agree). Their
responses, along DF's related unit costs for the modifications, follow.
1 2 3 4 5
Strongly Dis- Neutral Agree Strongly
Disagree agree Agree
Add cabinet doors in storage (Rs.6.00) 10 20 30 60 80
Expand storage area (Rs.2.50) 10 40 70 50 30
Add security lock to storage area (Rs.1.65) 30 60 50 40 20
Give table top a more rich, marble appearance 10 20 50 60 60
(Rs.4.25)
Extend warranty to five years (Rs.1.50) 40 70 30 35 25
The table currently costs Rs.64 to produce and distribute and DFs selling price for this unit
averages Rs.80. The current selling price for these tables with all or some or the aforesaid features
averages Rs.95.
Required:
1. Why is there a need in target costing to (a) focus on the customer and (b) have a marketing
team become involved with product design?
2. DF's marketing team will evaluate the survey responses by computing a weighted-average
rating of each of the modifications. This will be accomplished by weighting (multiplying) the
point values (1, 2, etc.) by the frequency of responses, summing the results, and dividing by 200.
Rank the popularity of the five modifications using this approach.
3. Management desires to earn approximately the same rate of profit on sales that is being earned
with the current design.
a. If DF uses target costing and desires to meet the current competitive selling price, what
is the maximum cost of the modified table?
b. Which of the modifications should DF consider?
4. Assume that DF wanted to add a modification or two that you excluded in your answer to
requirement 3(b). What process might management adopt to allow the company to make its
target profit for the table? Briefly explain.
Solution:
Step 1: Calculation of the desired profit margin for DF

E M Reddy Page | 397


AMA-Notes

Current Selling Price = Rs.80


Less: Cost = Rs.64
Profit = Rs.16
16
% of profit on Selling Price = 80 = 20%
16
% of profit on Cost = 64 = 25%

Step 2: Calculation of target cost of modified table


Market determined selling price = Rs.95
Less: Desired margin @ 20% = Rs.19
Target cost of modified table = Rs.76
Step 3: Ranking the modifications based on the customer preferences
1) Current cost is Rs.64 and target cost is Rs.76. Hence the company can maximum spent Rs.12 towards
modification.
2) It is not possible to have all modifications within Rs.12. Hence only some of them can be added.
3) What should be selected depends on the customer preferences obtained through the survey of 200
customers.
4) Ranking the customer preferences based on weights:
Modification Computation Weighted Point Rank
Cabinet Doors (10x1)+(20x2)+(30x3)+(60x4)+(80x5) 3.9 I
200
Storage Area (10x1)+(40x2)+(70x3)+(50x4)+(30x5) 3.25 III
200
Security Lock (30x1)+(60x2)+(50x3)+(40x4)+(20x5) 2.8 IV
200
Rich Appearance (10x1)+(20x2)+(50x3)+(60x4)+(60x5) 3.7 II
200
Extended Warranty (40x1)+(70x2)+(30x3)+(35x4)+(25x5) 2.67 V
200

Step 4: Selection of modification within available Rs.12 cost


Modification Cost (Rs.) Cumulative Cost (Rs.)
Cabinet Doors 6.00 6.00
Rich Appearance 4.25 10.25
Storage Area Not Possible 10.25
Security Lock 1.65 11.9

The company should select the following features namely Cabinet Doors, Rich Appearance and Security
Lock.

Step 5: Strategy to include all features

1) To include the remaining 2 features, we need to spend Rs.7.6 but we already have Rs.0.1 and needs to
reduce the cost by Rs.7.5.
2) This can be done by value engineering. (See previous question for notes)
3) Still if target cost reduction is not achieved compromise on margin. Never should selling price be
increased.

E M Reddy Page | 398


AMA-Notes

Question no 20: Vijay associates, a small structural design firm prepares architectural drawings for
various clients to ensure the structural safety of buildings. The architectural plans are then
submitted to local government departments for approval. Vijay’s income statement for 2001 follows:
Revenues Rs.6,80,000
Salaries of professional staff (8,000 Hours x Rs.50 per hour) Rs.4,00,000
Travel Rs.18,000
Administration and support Rs.1,60,000
Total Costs Rs.5,78,000
Operating Income Rs.1,02,000
An analysis of the parentage of time spent by professional staff on various activities gives this data:
Doing calculations and preparing drawings for clients 75%
Checking calculations and drawings 4%
Correcting errors found in drawings (not billed to clients) 7%
Making changes in response to client requests (billed to clients) 6%
Correction errors to meet government building code requirements (not billed to clients) 8%
Total 100%
Assume administration and support costs vary with professional labour costs.
Required:
Consider each requirement independently. There is no connection between the requirements.
1. How much of the total costs in 2001 are value-added, non-value-added, or in the gray area in
between? Explain your answers briefly what act ions can Vijay take to reduce its costs?
2. Suppose Vijay continued to check all calculations and drawings but, could eliminate all errors
so that it did not need to spend any time making corrections and, as a result could
proportionately reduce professional labour costs. Calculate Vijay's operating Income.
3. Now suppose Vijay could take on as much business as it could get done, but it could not add
more professional staff. Assume, as in requirement 2, that Vijay could eliminate all errors so
that it does not need to spend any time making corrections. Suppose Vijay could use the time
saved to increase revenues proportionately. Assume travel costs will remain at Rs.18,000.
Calculate Vijay's operating Income.
Solution:
Step 1: Analyzing the proportion of time to Value Added, Non-value added and Gray Area
Particulars Value Added Gray Area Non-value added
Doing calculations and preparing drawings for clients 75% -- --
Checking calculations and drawings -- 4% --
Correcting errors found in drawings -- -- 7%
Making changes on client’s requests 6% -- --
Correction errors to meet government code -- -- 8%
Total 81% 4% 15%

Step 2: Analyzing the total cost of Rs.5,78,000 in to value added, non-value added and Gray area
Particulars Value Added Gray Area Non-value added
Salaries of professional staff 3,24,000 16,000 60,000
Travel Costs 18,000 -- --
Administration and support (same % as salary) 1,29,600 6,400 24,000
Total 4,71,600 22,400 84,000

E M Reddy Page | 399


AMA-Notes

Step 3: Cost savings

When Vijay associates eliminates all errors it could save the non-value added hours to the extent of 1,200
(8,000 Hours x 15%). Due to this the non-value added cost of Rs.84,000 could be saved and the operating
income becomes Rs.1,86,000 (Rs.1,02,000 + Rs.84,000).

Step 4: Increasing profit through increased revenue

Revenue = Rs.6,80,000
Productive Hours = 8,000 Hours – 1,200 Hours = 6,800 Hours
Rs.6,80,000
Revenue/Productive Hour = 6,800 Hours = Rs.100 per hour
Increase in productive hours due to elimination of corrections = 1,200 Hours
Increase in revenue = Rs.100 x 1,200 Hours = Rs.1,20,000

The above Rs.1,20,000 is both increase in revenue as well as increase in profit because cost does not
increase.
Operating Profit = Rs.1,02,000 + Rs.1,20,000 = Rs.2,22,000

13.9. Activity Based Costing (ABC)

13.9.1. Activity Based Costing (ABC) - Introduction

1) For what we collect cost that is called cost object. E.g.: Product, Service, division etc.,
2) Costs are classified into 2 types:
a. Direct Costs → If a costs is solely for a cost object that is called Direct Cost.
b. Indirect Cost → A cost incurred for common cost objects is called Indirect Cost.
3) The cost object which generates revenue is called Ultimate Object.

Example:
Overhead Cost = Rs.1,00,000
Machine Hours = 10,000 Hours
Product A = 5,000 Hours; 200 Units
Product B = 5,000 Hours; 100 Units
Calculate Overhead cost per unit of A and B.

Solution:

Overhead Rs.10,00,000
Machine Hour Rate = = = Rs.10 per Machine Hour
Machine Hours 10,000 Hours
Overhead = Rs.1,00,000
Product A = 5,000 Hours x Rs.10 = Rs.50,000 → Overhead/Unit = Rs.250
Product B = 5,000 Hours x Rs.10 = Rs.50,000 → Overhead/Unit = Rs.500

Notes:

1) When we use machine hour rate to charge overheads to the products we assume that all the overheads
vary with machine hours.

E M Reddy Page | 400


AMA-Notes

2) This means the cost driver is machine hours and overheads are apportioned in the ratio of this cost
driver.

More facts:

Machine Running activity = Rs.60,000


Setup Activity = Rs.15,000
Inspection activity = Rs.25,00
Total Overhead = Rs.1,00,000
Cost Drivers A B
Machine hours 5,000 5,000
Setup Hours 3,000 2,000
Inspection time 1,500 3,500

Activity Based Costing System:

Step 1: Calculation of Cost driver rate


Activity Cost Pool Cost Driver Name Cost Driver Quantity Cost Driver Rate
Machine hours 60,000 Machine hours 10,000 6
Setup Hours 15,000 Setup Hours 5,000 3
Inspection time 25,000 Inspection time 5,000 5

Step 2: Apportionment of Overhead Costs


Activity Product A Product B
Cost Driver Quantity Amount (Rs.) Cost Driver Quantity Amount (Rs.)
Machine hours 5,000 30,000 5,000 30,000
Setup Hours 3,000 9,000 2,000 6,000
Inspection time 1,500 7,500 3,500 17,500
Total 46,500 53,500
Units 200 100
Overhead/Unit 232.50 535

Question no 21: Having attended a CIMA course on activity-based costing (ABC) you decide an
experiment by applying the principles of ABC to the four products currently made and sold by your
company. Details of the four products and relevant information are given below for one period:
Product A B C D
Output in units 120 100 80 120
Costs per unit: (Rs.) (Rs.) (Rs.) (Rs.)
Direct Material 40 50 30 60
Direct Labour 28 21 14 21
Machine hours (per unit) 4 3 2 3
The four products are similar and are usually produced in production runs of 20 units and sold in
batches of 10 units.
Using a machine hour rate currently absorbs the production overhead, and the total of the
production overhead for the period has been analyzed as follows:
(Rs.)

E M Reddy Page | 401


AMA-Notes

Machine department costs (rent, depreciation and supervision) 10,430


Setup Costs 5,250
Stores receiving 3,600
Inspection/Quality Control 2,100
Materials handling and dispatch 4,620
Total 26,000
You have ascertained that the cost drivers to be used are as listed below for the overhead cost
shown:
Cost Cost Driver
Setup Costs Number of production runs
Stores Receiving Requisition raised
Inspection/Quality Control Number of production runs
Material Handling and dispatch Orders executed
The number of requisition raised on the stores was 20 for each product and the number of orders
executed was 42, each orders being for a batch of 10 of a product. You are required:
(a) To calculate the total costs for each product if all overhead costs are absorbed on a machine
hour basis;
(b) To calculate the total costs for each product, using activity based costing;
(c) To calculate and list the unit product cost from your figures in (a) and (b) above, to show the
differences and to comment briefly on any conclusions, which may be drawn which could have
pricing and profit implications.
Solution:
Part 1: Calculation of product cost using Traditional Costing System
Particulars A B C D
Direct Materials 40 50 30 60
Direct Labour 28 21 14 21
Prime Cost 68 71 44 81
Overheads (WN – 1) 80 60 40 60
Total Cost 148 131 84 141

Part 2: Activity Based Costing System

Step 1: Calculation of Cost driver rate


Activity Cost Pool Cost Driver Name Cost Driver Quantity Cost Driver Rate (Rs.)
Machine Running 10,430 Machine hours 1,300 8.02/ Hour
Setup 5,250 Production runs 21 (WN – 2) 250/Production Run
Stores Receding 3,600 Requisitions raised 80 (20 x 4) 45/Requisition
Inspection 2,100 Production runs 21 (WN – 2) 100/Production Run
Material Handling 4,620 Orders executed 42 (WN – 3) 110/Order

Step 2: Apportionment of Overhead Costs


Activity A B C D
CDQ Amount CDQ Amount CDQ Amount CDQ Amount
Machine Running 480 3,850 300 2,406 160 1,283 360 2,891

E M Reddy Page | 402


AMA-Notes

Setup 6 1,500 5 1,250 4 1,000 6 1,500


Stores Receding 20 900 20 900 20 900 20 900
Inspection 6 600 5 500 4 400 6 600
Material Handling 12 1,320 10 1,100 8 880 12 1,320
Total Cost 8,170 6,156 4,463 7,211
Units 120 100 80 120
Overhead/Unit 68.08 61.56 55.79 60.09

Step 3: Calculation of total cost under Activity Based Costing System (ABC)
Particulars A B C D
Prime Cost 68 71 44 81
Overheads 68.08 61.56 55.79 60.09
Total Cost 136.08 132.56 99.79 141.09

WN 1: Calculation of Machine hour rate


Items Units Hours/Unit Hours
A 120 4 480
B 100 3 300
C 80 2 160
D 120 3 360
Total Hours 1,300
Overhead Cost Rs.26,000
Machine Hour rate = Machine Hours = = Rs.20/Hour
1,300

WN 2: Calculation of production runs


Items Units Computation Runs
A 120 120/20 6
B 100 100/20 5
C 80 80/20 4
D 120 120/20 6
Total Production Runs 21

WN 3: Calculation of Number of orders executed


Items Units Computation Orders
A 120 120/10 12
B 100 100/10 10
C 80 80/10 8
D 120 120/10 12
Total Production Runs 42

Question no 22: KL currently manufactures over 100 products of varying levels of design
complexity. A single, plant-wide overhead absorption rate (OAR), based on direct labour hour is
used to absorb overhead costs.
In the quarter-ended march, KL's manufacturing overhead costs were:

E M Reddy Page | 403


AMA-Notes

(Rs.000)
Equipment operation expenses 125
Equipment maintenance expenses 25
Wages paid to technicians 85
Wages paid to stores men 35
Wages paid to dispatch staff 40
Total 310
During the quarter, RAM Management Consultants were engaged to conduct a review of KL's cost
accounting systems. RAM report includes the following statement:
In KL's circumstances, absorbing overhead costs individual products on a labour hour absorption
basis is meaningless. Overhead costs should be attributed to products using an activity based
costing (ABC) system. We have identified the following as being the most significant activities:
(1) Receiving component consignments from suppliers
(2) Setting up equipment for production runs
(3) Quality inspections
(4) Dispatching goods orders to customers.
Our research has indicated that, in the short term, KL's overheads are 40% fixed costs and 60%
variable. Approximately half the variable overheads vary in relation to direct labour hours worked
and half vary in relation to the number of quality inspections. This model applies only to relatively
small changes in the level of output during a period of two years or less.
Equipments operation and maintenance expenses are apportionable as follows:
 Component stores (15%), manufacturing (70%) and goods dispatch (15%).
 Technician wages are apportionable as follows:
Equipment maintenance (30%), setting up equipment for production runs (40%) and quality
inspection (30%).
During the quarter
 A total of 2,000 direct labour hours were worked (paid at Rs.12 per hour),
 980 component consignments were received from suppliers,
 1,020 production runs were set up,
 640 quality inspections were carried out, and
 420 goods orders were dispatched to customers.
KL's production during the quarter included components R, S and T. the following information is
available:
Component R Component S Component T
Direct labour hours worked 25 480 50
Direct material costs Rs.1,200 Rs.2,900 Rs.1,800
Component consignments received 42 24 28
Production runs 16 18 12
Quality inspections 10 8 18
Goods orders dispatched 22 85 46
Quality produced 560 12,000 2,400
In April 2008 a potential customer asked KL to quote for the supply of a new component (Z) to a
given specification. 1,000 units of Z are to be supplied each quarter for a two-year period. They will
be paid for in equal installments on the last day of each quarter. The job will involve an initial
design cost of Rs.40,000 and production will involve 80 direct labour hours, Rs.2,000 materials, 20
component consignments, 15 production runs, 30 quality inspections and 4 goods dispatches per

E M Reddy Page | 404


AMA-Notes

quarter.
KL's Sales Director comments:
Now we have a modern ABC system, we can quote selling prices with confidence. The quarterly
charge we quote should be the forecast ABC production cost of the units plus the design cost of the
Z depreciated on a straight-line basis over the two years of the job-to which we should add a 25%
mark-up for profit. We can base our forecast on costs experienced in the quarter-ended march.
Requirements:
(a) Calculate the unit cost of components R, S and T using KL's existing cost accounting system
(single factory labour OAR).
(b) Explain how an ABC system would be developed using the information given. Calculate the
unit cost of components R, S and T, using this ABC system.
(c) Calculate the charge per quarter that should be quoted for supply of components Z in a
manner consistent with the Sales Director's comments. Advise KL's management on the
merits of this selling price, having regard to factor you consider relevant.
Note: KL’s cost of capital is 3% per quarter.

Solution:

Part 1: Calculation of cost per unit using traditional costing system

Step 1: Calculation of overhead absorption rate

Absorption rate used ‘labour hour rate used’.


Overheads Rs.3,10,000
Labour Rate = = 2,000 Hours = Rs.155/Labour Hour
Hours

Step 2: Calculation of cost per unit


Particulars Component R (Rs.) Component S (Rs.) Component T (Rs.)
Direct Material Costs 1,200 2,900 1,800
Add: Direct Labour @ 12 per hour 300 (25 x 12) 5,760 (480 x 12) 600 (50 x 12)
Prime Cost 1500 8,660 2,400
Add: Overheads @ 155 per hour 3,875 (25 x 155) 74,400 (480 x 155) 7,750 (50 x 155)
Total Cost 5,375 83,060 10,150
Units Produced 560 12,000 2,400
Cost per unit 9.60 6.92 4.23

Part 2: Activity Based Costing System

Step 1: Calculation of Cost driver rate


Activity Cost Pool Cost Driver Name Cost Driver Quantity Cost Driver Rate (Rs.)
Manufacturing 1,22,850 Labour Hours 2,000 61.425/Hour
Stores Receiving 61,325 Consignments 980 62.576/Consignment
Setup 34,000 Production Runs 1,020 33.33/Production Run
Quality Inspection 25,500 Inspections 640 39.84/Inspection
Dispatch 66,325 Orders executed 420 157.92/Order

E M Reddy Page | 405


AMA-Notes

Step 2: Cost per unit f R, S and T


Activity Component R Component S Component T
CDQ Amount CDQ Amount CDQ Amount
Direct Material 1,200 2,900 1,800
Direct labour @ 12 300 5,760 2,400
Prime Cost 1,500 8,660 2,400
Add: Overheads
Manufacturing @ Rs.61.425 25 1,536 480 29,484 50 3,072
Stores Receiving @ Rs.62.576 42 2,628 24 1,502 28 1,752
Setup @ Rs.33.33 16 533 18 600 12 400
Quality Inspection @ Rs.39.84 10 398 8 319 18 717
Dispatch @ Rs.157.92 22 3,474 85 13,423 46 7,264
Total Cost 10,069 53,988 15,605
Units 560 12,000 2,400
Cost/Unit 18 4.5 6.5

WN 1: Technician Wages

Rs.85,000

Equipment Inspection
Setup (40%)
Maintenance(30 (30%) =
= Rs.34,000
%) = Rs.25,500 Rs.25,500

Overhead Cost Activity Cost

WN 2: Equipment Operation and maintenance expenses

Rs.1,25,000 + Rs.25,000 +
Rs.25,500 = Rs.1,75,500

Manufacturin Dispatch
Stores (15%) =
g (70%) = (15%) =
Rs.26,325
Rs.1,22,850 Rs.26,325

WN 3: Constructing activity cost pool


Items Manufacturing Stores Setup Inspection Dispatch
Equipment Operation & Maintenance 1,22,850 26,325 -- -- 26,325
Technician Wages -- -- 34,000 25,500 --
Stores Wages -- 35,000 -- -- --

E M Reddy Page | 406


AMA-Notes

Dispatch Wages -- -- -- -- 40,000


Total 1,22,850 61,325 34,000 25,500 66,325

Part 3: Accept Z order or not

Step 1: Fixation of Selling Price


Particulars Computation Amount (Rs.)
Direct Materials Given 2,000
Direct Wages 80 Hours x Rs.12 960
Prime Cost 2,960
Add: Overheads
Manufacturing @ Rs.61.425 80 Hours x Rs.61.425 4,915
Stores Receiving @ Rs.62.576 20 x Rs.62.576 1,252
Setup @ Rs.33.33 15 Runs x s.33.33 500
Quality Inspection @ Rs.39.84 30 x Rs.39.84 1,195
Dispatch @ Rs.157.92 4 x Rs.157.92 632
Depreciation 40,000/8 5,000
Total Cost 16,454
Add: Profit Rs.16,454 x 20% 4,114
Sales 20,568
Selling Price 20,568/1,000 20.568

Every quarter the cash flow from this order is Rs.20,568 – Rs.11,454 = 9,114

Step 2: Calculation of NPV


Period Cash flow (Rs.) PVF@3% Discounted Cash flow (Rs.)
1–8 9,114 7.0197 63,977
Less: Discounted cash outflow -40,000
Net Present Value 23,977

Recommended to accept the order since NPV is positive.

Question no 23: Repack Ltd is a Warehousing and Distribution Company, which receives products
from customers, stores the products and then re-packs them for distribution as required. There are
three customers for whom the services are provided - John ltd, George Ltd and Paul Ltd. The
products from all three customers are similar in nature but have varying degrees of fragility. Basic
budget information has been gathered for the year to 30 June and is shown in the following table:
Product handled (cubic meters)
John Ltd 30,000
George Ltd 45,000
Paul Ltd 25,000
Cost
Packaging materials (see note 1) 19,50,000
Labour – Basic 3,50,000
– Overtime 30,000
Occupancy 5,00,000
Administration and management 60,000

E M Reddy Page | 407


AMA-Notes

Note 1: Packing materials are used in re-packing each cubic meter of product for John ltd, George
ltd and Paul Ltd in the ratio 1:2:3 respectively. This ratio is linked to the relative fragility of the
good for each customer.
Additional information has been obtained in order to enable unit costs to be prepared for each of
the three customers using an activity-based costing approach. The additional information for the
year to 30 June has been estimated as follows:
(i) Labour and overhead costs have been identified as attributable to each of three work centers-
receipts and Inspection, storage and packing as follows:
Cost allocation proportions
Rectification & Inspection Storage Packing
% % %
Labour – Basic 15 10 75
– Overtime 50 15 35
Occupancy 20 60 20
Administration and Management 40 10 50
(ii) Studies have revealed that the fragility of different goods affects the receipt and Inspection time
needed for the products for each customer. Storage required is related to the average size of the
basic incoming product units from each customer. The re-packing of goods for distribution is
related to the complexity of packaging required by each customer. The relevant requirements per
cubic meter of product for each customer have been evaluated as follows:
John ltd. George ltd. Paul ltd.
Receipt and inspection (minutes) 5 9 15
Storage (square meters) 0.3 0.3 0.2
Packing (minutes) 36 45 60
Required
Calculate the budgeted average cost per cubic meter of packaged products for each customer each
of the following two circumstances:
(i) Where only the basic budget information is to be used,
(ii) Where the additional information enables an activity-based costing approach to be applied.

Solution:

Part 1: Cost per cubic meter using basic budgeted information (Traditional Costing System)
Particulars John Ltd (Rs.) George Ltd (Rs.) Paul ltd (Rs.)
Packaging Material Cost 3,00,000 9,00,000 7,50,000
Labour – Basic 1,05,000 1,57,500 87,500
Labour – Overtime 9,000 13,500 7,500
Occupancy 1,50,000 2,25,000 1,25,000
Administration 18,000 27,000 15,000
Total Cost 5,82,000 13,23,000 9,85,000
Cubic Meters 30,000 45,000 25,000
Cost per cubic meter 19.4 29.4 39.4

E M Reddy Page | 408


AMA-Notes

Part 2: Cost per cubic meter using basic budgeted information (Traditional Costing System)

Step 1: Calculation of Cost driver rate


Activity Cost Cost Driver Cost Driver Cost Driver Rate
Pool Name Quantity (Rs.)
Receipt and 1,91,500 Inspection time 15,500 12.3548/Hour
inspection
Storage 3,45,500 Square meters 27,500 12.5636/Sq. mtr.
Packing 4,03,000 Packing time 76,750 5.2508/Hour

Step 2: Apportionment of overhead cost


Activity John Ltd George Ltd Paul Ltd
CDQ Amount (Rs.) CDQ Amount (Rs.) CDQ Amount (Rs.)
Receipt and inspection 2,500 30,887 6,750 83,395 6,250 77,218
Storage 9,000 1,13,072 13,500 1,69,609 5,000 62,818
Packing 18,000 94,515 33,750 1,77,215 25,000 1,31,270
Total Cost 2,38,474 4,30,219 2,71,306

Step 3: Calculation of cost per cubic meter


Particulars John Ltd George Ltd Paul Ltd
Indirect cost 2,38,474 4,30,219 2,71,306
Add: Packing 3,00,000 9,00,000 7,50,000
Total Cost 5,38,474 13,30,219 10,21,306
Cubic Meters 30,000 45,000 25,000
Cost/Cubic Meter 17.95 29.56 40.85

WN 1: Packaging Material Cost


Particulars John Ltd George Ltd Paul ltd
Ratio 1 2 3
Cubic Meters 30,000 45,000 25,000
Weighted Ratio 30,000 90,000 75,000
30,000
John Ltd: Rs.19,50,000 x 1,95,000 = Rs.3,00,000
90,000
George Ltd: Rs.19,50,000 x 1,95,000 = Rs.9,00,000
75,000
Paul Ltd: Rs.19,50,000 x = Rs.7,50,000
1,95,000

WN 2: Other Cost
Customer Labour – Basic Labour – Occupancy Administration
Overtime
John ltd Rs.3,50,000 x Rs.30,000 x 30,000
Rs.5,00,000 x 1,05,000
30,000
Rs.60,000 x 1,05,000 =
30,000 30,000
= = Rs.9,000 = Rs.1,50,000 Rs.18,000
1,05,000 1,05,000
Rs.1,05,000

E M Reddy Page | 409


AMA-Notes

George Rs.3,50,000 x Rs.30,000 x 45,000


Rs.5,00,000 x 1,05,000
45,000
Rs.60,000 x 1,05,000 =
ltd 45,000
=
45,000
=
1,05,000 1,05,000 = Rs.2,25,000 Rs.27,000
Rs.1,57,500 Rs.13,500
Paul ltd Rs.3,50,000 x Rs.30,000 x 25,000
Rs.5,00,000 x 1,05,000
25,000
Rs.60,000 x 1,05,000 =
25,000 25,000
= Rs.87,500 = Rs.7,500 = Rs.1,25,0500 Rs.15,000
1,05,000 1,05,000

WN 3: Construction of cost pool


Cost Receipt Storage Packing
% Amount % Amount % Amount
Labour – Basic 15 52,500 10 35,000 75 2,62,500
Labour – Overtime 50 15,000 15 4,500 35 10,500
Occupancy 20 1,00,000 60 3,00,000 20 1,00,000
Administration and Management 40 24,000 10 6,000 50 30,000
Cost Pool Total 1,91,500 3,45,500 4,03,000

WN 4: Calculation of Receipts and Inspection time


Customer Computation Time in minutes Time in hours
John ltd 30,000 x 5 Minutes 1,50,000 2,500
George ltd 45,000 x 9 Minutes 4,05,000 6,750
Paul ltd 25,000 x 15 Minutes 3,75,000 6,250
Total Inspection time 15,500

WN 5: Calculation of Storage Space


Customer Computation Square Meters
John ltd 30,000 x 0.3 Square Meters 9,000
George ltd 45,000 x 0.3 Square Meters 13,500
Paul ltd 25,000 x 0.2 Square Meters 5,000
Total Storage Space 27,500

WN 6: Calculation of packing time


Customer Computation Time in minutes Time in hours
John ltd 30,000 x 36 Minutes 10,80,000 18,000
George ltd 45,000 x 45 Minutes 20,25,000 33,750
Paul ltd 25,000 x 60 Minutes 15,00,000 25,000
Total Inspection time 76,750

Question no 24: Tropicana Ltd.· has decided to increase the size of its store. It wants information
about the profitability of individual product lines: Orange juice, Apple juice and Mango juice.
Tropicana ltd. provides the following data for 2008 for each product line:
Particulars Orange Juice Apple Juice Mango Juice
Revenues Rs.3,17,400 Rs.8,40,240 Rs.4,83,960
Cost of goods sold Rs.2,40,000 Rs.6,00,000 Rs.3,60,000
Cost of bottles returned Rs.4,800 Rs.0 Rs.0
Number of purchase orders placed 144 336 144
Number of deliveries received 120 876 264

E M Reddy Page | 410


AMA-Notes

Hours of shelf-stocking time 216 2,160 1,080


Items sold 50,400 4,41,600 1,22,400
Tropicana ltd. also provides the following information for 2008:
Sr.No. Activity (1) Description of Activity (2) Total Costs Cost allocation
(3) base (4)
1 Bottle Returning of empty bottles Rs.4,800 Direct tracking to
returns soft drink line
2 Ordering Placing of orders for purchases Rs.62,400 624 purchase orders
3 Delivery Physical delivery and receipts of Rs.1,00,800 1,260 deliveries
merchandise
4 Shelf- Stocking of merchandise on store Rs.69,120 3,456 hours of shelf-
Stocking shelves and ongoing restocking stocking time
5 Customer Assistance provided to customers Rs.1,22,880 6,14,400 items sold
Support including bagging and checkout
Total Rs.3,60,000
Required:
(a) Tropicana Ltd. currently allocates store support costs (all costs other than cost of goods sold)
to product lines on the basis of cost of goods sold of each product. Calculate operating income
and operating income as a percentage of revenues for each product line.
(b) If Tropicana Ltd. allocates store support costs (all costs other than cost of goods sold) to
product lines using ABC system, calculate operating income and operating income as a
percentage of revenues for each product line.
(c) Compare both the systems.

Solution:

Part 1: Product line profitability under existing system


Particulars Orange Juice (Rs.) Apple Juice (Rs.) Mango Juice (Rs.)
Revenues 3,17,400 8,40,240 4,83,960
Less: Cost of goods sold 2,40,000 6,00,000 3,60,000
Less: Stores Support 76,800 (72,000 + 4,800) 1,80,000 1,08,000
Profit 600 60,240 15,960
Profit as % of sales 0.19% 7.17% 3.29%

Part 2: Product line profitability under ABC System

Step 1: Calculation of Cost-Driver Rate


Activity Cost Pool Cost Driver Name Cost Driver Cost Driver
(Rs.) Quantity Rate (Rs.)
Ordering 62,400 Placing of orders for purchases 624 100/Order
Delivery 1,00,800 Physical delivery and receipts of 1,260 80/Deliver
merchandise
Shelf- 69,120 Stocking of merchandise on store 3,456 20/Stocking time
Stocking shelves and ongoing restocking
Customer 1,22,880 Assistance provided to customers 6,14,400 0.2/Unit
Support including bagging and checkout

E M Reddy Page | 411


AMA-Notes

Step 2: Apportionment of Overhead


Activity Orange Juice Apple Juice Mango Juice
CDQ Amount (Rs.) CDQ Amount (Rs.) CDQ Amount (Rs.)
Bottle return -- 4,800 -- -- -- --
Ordering 144 14,400 336 33,600 144 14,400
Delivery 120 9,600 876 70,080 264 21,120
Shelf-Stocking 216 4,320 2,160 43,200 1,080 21,600
Customer Support 50,400 10,080 4,41,600 88,320 1,22,400 24,480
Total Cost 43,200 2,35,000 81,600

Step 3: Product line profitability


Particulars Orange Juice (Rs.) Apple Juice (Rs.) Mango Juice (Rs.)
Revenues 3,17,400 8,40,240 4,83,960
Less: Cost of goods sold 2,40,000 6,00,000 3,60,000
Less: Stores Support 43,200 2,35,000 81,600
Profit 34,200 5,240 42,360
Profit as % of sales 10.78% 0.62% 8.75%

WN 1: Apportionment of stores support cost


Rs.3,60,000
% of indirect cost to cost of goods sold = Rs.12,00,000 = 30%
Orange Juice = Rs.2,40,000 x 30% = Rs.72,000
Apple Juice = Rs.6,00,000 x 30% = Rs.1,80,000
Mango Juice = Rs.3,60,000 x 30% = Rs.1,08,000

13.9.2. Activity Based Costing (ABC) & Target Costing

Question no 25: Cauvery ltd. manufactures two component parts for the television industry:
• T: Annual production and sales of 50,000 units at a selling price of Rs.40.60 per unit.
• Premia: Annual production and sales of 25,000 units at a selling price of Rs.60 per unit.
Cauvery includes all R&D and design costs in engineering costs. Assume that Cauvery has no
marketing, distribution, or customer-service costs. The direct and indirect costs incurred by
Cauvery on T and Premia are as follows:
T Premia Total
Direct Material Costs (Variable) 8,50,000 6,00,000 14,50,000
Direct Manufacturing Labour Costs (Variable) 3,00,000 2,00,000 5,00,000
Direct Machining Costs (Fixed) 1,50,000 1,00,000 2,50,000
Indirect Manufacturing Costs:
Machine Setup Costs 86,250
Testing Costs 4,87,500
Engineering Costs 4,50,000
Total Indirect Manufacturing Costs 10,23,750
Total Costs 32,23,750
Cauvery's management Identifies the following activity cost pools, cost drivers for each activity,
and the cost per unit of cost driver for each overhead cost pool:

E M Reddy Page | 412


AMA-Notes

Manufacturing Description of Activity Cost Driver Cost per unit of


Activity cost driver
1. Setup Preparing machine to manufacture Setup hours Rs.25 per setup
a new batch of products hours
2. Testing Testing components and final Testing Hours Rs.2 per testing
product (Cauvery tests each unit of hour
T and Premia individually)
3. Engineering Designing products and processes Complexity of Cost assigned to
and ensuring their smooth product and products by
functioning process special study
Over a long-run horizon, Cauvery's management views direct materials costs and direct
manufacturing labour costs as variable with respect to the units of T and Premia produced, and
overhead costs as variable with respect to their chosen cost drivers.
For example, setup costs vary with the number of setup-hours. Direct machining costs represent
the cost of machine capacity dedicated to the production of each product (50,000 hours at Rs.3 per
hour for T). These costs are fixed and are not expected to vary over the long run horizon.
Additional Information is as follows:
T Premia
1. Production batch sizes 500 Units 200 Units
2. Setup time per batch 12 Hours 18 Hours
3. Testing and inspection time per unit of product produced 2.5 Hours 4.75 Hours
4. Engineering cost incurred on each product Rs.1,70,000 Rs.2,80,000
Cauvery is facing competitive pressure to reduce the price of T and has set a target price of
Rs.34.80, well below its current price of Rs.40.60. The challenge of Cauvery is to reduce the cost of
T. Cauvery’s engineers have proposed new product design and process improvements for the
"New T" to replace T. The new design would improve product quality, and reduce scrap and
waste. The reduction in prices will not enable Cauvery to increase its current unit sales. (However,
if Cauvery does not reduce prices, it will lose sales). The expected effect of new design relative to T
as follows:
a. Direct materials costs for New T are expected to decrease by Rs.2.00 per unit.
b. Direct manufacturing labour costs for New T are expected to decrease by Rs.0.50 per unit.
c. Machining time required to make New T is expected to decrease by 20 minutes. It currently
takes 1 hour to manufacture 1 unit of T. The machines will be dedicated to the production
of New T.
d. New T will take 7 setup-hours for each setup.
e. Time required for testing each unit of New T is expected to be reduced by 0.5 hour.
f. Engineering costs will be unchanged.
Assume that the batch sizes are the same for New T as for T. If Cauvery required additional
resources to implement the new design, it can acquire these additional resources in the quantities
needed. Further assume the costs per unit of cost drivers for the New T are the same as those for
T.
Required:
1. Calculate the full cost per unit for T and Premia using activity based costing.
2. What is the markup on the full cost per unit for T?
3. What is Cauvery's target cost per unit for New T if it is to maintain the same markup
percentage on the full cost per unit as it had for T?

E M Reddy Page | 413


AMA-Notes

4. Will the New T design achieve the cost reduction targets that Cauvery has set? Explain.
5. What price will Cauvery charge for New T if it uses the same markup percentage on the full
cost per unit for New T as it did for T?
6. What price should Cauvery charge for New T? Specify any other management action that
Cauvery should take regarding New T.
Solution:
Part 1: Full Cost of T and Premia under ABC System
Step 1: Apportionment of Overhead Costs
Activities T Premia
Cost Driver Quantity Amount Cost Driver Quantity Amount
Setup 1,200 Hours 30,000 2,250 Hours 56,250
Testing 1,25,000 Hours 2,50,000 1,18,750 Hours 2,37,500
Engineering -- 1,70,000 -- 2,80,000
Total Overhead Cost 4,50,000 5,73,750

Step 2: Calculation of total cost


Particulars T (Rs.) Premia (Rs.)
Direct Material Costs 8,50,000 6,00,000
Direct Manufacturing Labour Costs 3,00,000 2,00,000
Direct Machining Costs 1,50,000 1,00,000
Indirect Manufacturing Costs 4,50,000 5,73,750
Total Cost 17,50,000 14,73,750
Units 50,000 Units 25,000 Units
Cost per unit 35 58.75

Part 2: Markup on full cost of T


Selling Price Rs.40.60
Less: Total Cost Rs.35
Markup Rs.5.60
% Markup on Selling Price Rs.5.6/Rs.40.6 = 13.79%
% Markup on Cost Rs.5.6/Rs.35 = 16%

Part 3: Target Cost for “New T”


Selling Price (Market Determined) Rs.34.8
Less: Markup @ 13.79% (Rs.34.8 x 13.79%) Rs.4.80
Target Cost Rs.30

Part 4: Calculation of “New T” Cost


Particulars Computation Cost (Rs.)
Direct Material Costs (8,50,000/50,000 – 2) x 50,000 7,50,000
Direct Manufacturing Labour Costs (3,00,000/50,000 – 0.5) x 50,000 2,75,000
Direct Machining Costs 1,50,000
Setup Cost 100 Batches x 7 Hours x Rs.25 17,500

E M Reddy Page | 414


AMA-Notes

Testing Cost 50,000 Units x 2 Hours x Rs.2 2,00,000


Engineering Cost 1,70,000
Total Cost 15,62,500
Units 50,000
Cost per unit Rs.15,62,500/50,000 Units 31.25

The new design has reduced cost but fall short of target cost by Rs.1.25 (Rs.31.25 – Rs.30).

Part 5: Selling price using the Same markup

Cost of New T = Rs.31.25


Add: Markup @ 16% = Rs.5 (Rs.31.25 x 16%)
Selling Price = Rs.36.25

Part 6:

We should not increase the selling price beyond Rs.34.8. For reasons see target costing introduction notes.
The strategy is to compromise on margin. If it is not possible, exit from the market.

WN 1: Calculation of Setup Hours


Particulars T Premia
Production 50,000 Units 25,000 Units
Batch Size 500 Units 200 Units
No. of batches 100 Batches 125 Batches
Setup time per batch 12 Hours 14 Hours
Setup Hours 1,200 Hours 2,250 Hours

WN 2: Calculation of Testing Hours


Particulars T Premia
Production 50,000 Units 25,000 Units
Testing hours per unit 2.5 Hours 4.75 Hours
Total Hours 1,25,000 Hours 1,18,750 Hours

Notes:
1) Direct Machining time has reduced by 20 Minutes but has no impact on cost due to following reasons:
a) The Machine cost if fixed for which the cost driver is capacity and not the actual hours. Hence it
will not reduce.
b) With reduced time can’t we increase volume and decrease per unit cost?
Answer: No, because there is no demand. It is better to keep capacity idle than build up
inventory
c) Can’t the idle capacity be diverted to Premia which has demand?
d) Answer: No, because the machine is specific to “New T”.
2) Activities of an organization can be classified into 4 types:
a) Unit Level Activity
b) Batch Level Activity
c) Product Level Activity

E M Reddy Page | 415


AMA-Notes

d) Facility Level Activity

Activity

Unit Level Activity Batch Level Activity Product Level Activity Facility Level Activity

The activities which are


The activities which are The activities which are
performed for each batch/lot of
performed for each unit are performed specially for one The activities which are
units are called Batch level
called Unit level activities and product are called Product level performed for entire company
activities and cost drivers are
cost drivers are called Unit level activities. E.g. Marketing (or) all products are Facility level
called Batch level cost drivers.
cost drivers. E.g.: Testing Activity on introduction of a activities. E.g.: Brand Building
E.g.: Setup Activity in above
Activity in above problem new product
problem

Overhead allocation is similar Overhead allocation is similar


Overhead allocation is different under both ABC System & Traditional
under both ABC System & under both ABC System &
Costing System. ABC system is superior in overhead allocation
Traditional Costing System Traditional Costing System

Question no 26: State with a brief reason whether you would recommend an activity based system
of costing in each of the following Independent situations:
 Company K produces one product. The overhead costs mainly consist of depredation.
 Company L produces 5 different products using different production facilities.
 A consultancy firm consisting of lawyers accountants and computer engineers provides
management consultancy services to clients.
 Company S produces two different labour intensive products. The contribution per unit In both
products is very high. The SEP Is very low. All the work Is carried on efficiently to meet the
target costs.
Solution:
 Only one product – ABC System is not required
 Most of the costs are directly traceable as different products are manufactured using different facilities –
ABC System is not required.
 ABC System is required
 Breakeven Point low means variable costs are low and variable costs are high. Variable costs high means
most costs are unit level activity costs – ABC System is not required
Question no 27: State whether each of the following independent activities is value-added or non-
value-added:
 Polishing of furniture used by systems engineer in a software for a banking company.
 Maintenance by a software company of receivables management software for a banking
company
 Painting of pencils manufactured by a pencil factory.
 Cleaning of customer's computer key boards by a computer repair center.
 Providing, brake adjustments in cars received for service by a car service station.

E M Reddy Page | 416


AMA-Notes

Solution:
 Non-Value Added
 Value Added
 Value Added
 Value Added
 Value Added

13.9.3. Activity Based Costing (ABC) with Variance Analysis

Question no 28: X uses traditional standard costing system. The inspection and setup costs are
actually Rs.1,760 against a budget of Rs.2,000.
ABC system is being implemented and accordingly, the number of batches is identified as the cost
driver for inspection and setup costs. The budgeted production is 10,000 units in batches of 1,000
units, whereas actually, 8,800 units were produced in 11 batches.
(i) Find the volume and total fixed overhead variance under the traditional standard costing
system.
(ii) Find the total fixed overhead cost variance under the ABC system.

Solution:

Part 1: Calculation of fixed overhead variances under traditional costing system

Step 1: Calculation of Standard Rate


BFO Rs.2,000
SR/ Unit = = 10,000 Units = Rs.0.2/Unit
BO

Step 2: Computation table


[1] [2] [3]
AO x SR AFO BFO
8,800 Units x Rs.0.2 Rs.1,760 Rs.2,000
Rs.1,760 Rs.1,760 Rs.2,000

Step 3: Variance Computation

Fixed Overhead
Cost Variance (1 –
2) = Rs.1,760 –
Rs.1,760 = Rs.0

Fixed Overhead Expenditure Fixed Overhead Volume


Variance (3 – 2) = Rs.2,000 – Variance (1 – 3) =
Rs.1,760 = Rs.240 Rs.1,760 – Rs.2,000 =
(Favourable) Rs.240 (Adverse)

E M Reddy Page | 417


AMA-Notes

Part 2: Calculation of variances under ABC system

Step 1: Calculation of Standard Rate

Budgeted Output = 10,000 Units; Batch Size = 1,000 Units


10,000 Units
Budgeted Batches = = 10 Batches
1,000 Units
Budgeted Fixed Overheads Rs.2,000
Standard Rate/Batch = = 10 Batches = Rs.200/Batch
Budgeted Batches

Step 2: Computation table


[1] [2] [3] [4]
`SB x SR AFO BFO AB x SR
9 Batches x Rs.200 Rs.1,760 Rs.2,000 11 Batches x Rs.200
Rs.1,800 Rs.1,760 Rs.2,000 Rs.2,200

WN: Standard Batch (Standard Batch for actual output


Input Output
1 Batch 1,000 Units
9 Batches 8,800 Units

Step 3: Variance Calculation

Fixed Overhead Cost


Variance (1 – 2) =
Rs.1,800 – Rs.1,760 =
Rs.40 (Favourable)

Fixed Overhead Expenditure Fixed Overhead Volume


Variance (3 – 2) = Rs.2,000 – Variance (1 – 3) =
Rs.1,760 = Rs.240 Rs.1,800 – Rs.2,000 =
(Favourable) Rs.200 (Adverse)

Fixed Overhead
Fixed Overhead Capacity
Expenditure Variance (1
Variance (4 – 3) =
– 4) = Rs.1,800 –
Rs.2,200 – Rs.2,000 =
Rs.2,200 = Rs.400
Rs.200 (Favourable)
(Adverse)

Notes:

1) Understanding Capacity Variance:


Plan to setup 10 batches but actually setup 11 batches resulting in over absorption of 1 Batch x Rs.200
= Rs.200 (Favourable)

E M Reddy Page | 418


AMA-Notes

2) Understanding efficiency variance:


3) For the output of 8,800 units the batches allowed is 9 batches but actually dine in 11 batches. Hence
there is adverse efficiency of 2 batches x Rs.200 = Rs.400 (Adverse)

Question no 26: Toy master ltd produces a plastic toy car, TGC in batches. To manufacture a batch
of TGCs, Toy master must set up the machines. Setup costs are batch-level costs. A separate setup
department is responsible for setting up machines for TGC.
Setup overhead costs consists of some costs that are variable and some that are fixed with respect
to the number of set-up hours. The following information pertains to 2007.
Static-Budget Amount Actual Amounts
Units of TGC produced and sold 30,000 22,500
Batch size (number of units per batch) 250 225
Setup hours per batch 5 5.25
Variable overhead cost per setup hour Rs.250 Rs.240
Total fixed setup overhead costs Rs.1,80,000 Rs.1,75,350
Required:
1. For variable setup overhead costs, compute the efficiency and spending variances. Comment
on the results.
2. For fixed setup overhead costs, compute the spending and the production-volume variances.
Comment on the results.
Solution:
Step 1: Calculation of Standard Rate
Standard Rate/Batch = Rs.250 x 5 Hours = Rs.1,250/Batch
Standard Rate/Setup Hour = Rs.250
Step 2: Computation Table
[1] [2] [3]
SH x SR (or) SB x SR AVO AH x SR
450 Hours x Rs.250 (or) 90 Batches x Rs.1,250 525 Hours x Rs.240 525 Hours x Rs.250
Rs.1,12,500 Rs.1,26,000 Rs.1,31,250

WN 1: Standard Hours (Standard Setup hours for actual output)


Input Output
1 Batch 250 Units
90 Batches 22,500 Units

Standard Hours = 90 Batches x 5 Hours = 450 Hours

WN 2: Calculation of AVO
Input Output
1 Batch 225 Units
100 Batches 22,500 Units

Actual Hours = 100 Batches x 5.25 Hours = 525 Hours

E M Reddy Page | 419


AMA-Notes

Step 3: Variance Calculation

Variable Overhead Cost


Variance (1 – 2) =
Rs.1,12,500 – Rs.1,26,060
= Rs.13,500 (Adverse)

Fixed Overhead Expenditure Fixed Overhead Efficiency


Variance (3 – 2) = Variance (1 – 3) =
Rs.1,31,250 – Rs.1,26,060 = Rs.1,12,500 – Rs.1,31,250 =
Rs.5,250 (Favourable) Rs.18,750 (Adverse)

Due to Batch Size Due to Setup time


= (100 Batches – 90 Batches) = (5.25 Hours – 5 Hours) x
x Rs.1,250 = Rs.12,500 100 Batches x Rs.250 =
(Adverse) Rs.6,250

E M Reddy Page | 420

Anda mungkin juga menyukai